You are on page 1of 185

TARGET PT 2020 3

TEST
DAY - 1

Time Allowed: 30 mins Maximum Marks: 50

1. Consider the following statements 3. Which of the following statements


regarding the Preamble of the Indian is correct regarding the powers of
Constitution: Parliament under Article 3?
1. It is based on the ‘Objectives Resolution’, (a) Parliament can change the name of the
drafted by Dr. B. R. Ambedkar. state even without taking the view of
2. It is a part of the Indian Constitution. the State Legislature.
3. Since the time of adoption, it has been (b) Parliament can create a new state out
amended two times. of an existing state even against the
4. The words Socialist, Secular, and view of State Legislature.
Republic were added by the 42nd
(c) Parliament cannot change the
amendment.
boundary of any state.
Which of the above statements is/are (d) All of the above
correct?
(a) 1 and 3 only 4. Consider the following statements
with respect to Indian Constitution?
(b) 2 only
1. The term ‘Judicial Review’ is mentioned
(c) 1, 3 and 4 only in Article 31B of the Constitution.
(d) 1, 2, 3 and 4 2. Ninth schedule was added by the 42nd
Constitutional Amendment Act.
2. Consider the following statements
with respect to the term ‘Republic 3. In I.R. Coelho case (2007), the Supreme
’mentioned in the Preamble of the Court removed the immunity of the
Indian Constitution: Ninth Schedule from Judicial Review.
1. In a Republican system, the Head of Which of the above statements is/are
the State is elected. correct?
2. Every democratic country is also a
(a) 1 only
republic.
3. In the Presidential form of Government, (b) 3 only
the Head of the State is indirectly (c) 1 and 3 only
elected while in a Republic, the Head
of the State is directly elected. (d) 2 and 3 only

Which of the above statements is/are 5. Which of the following is not a criterion
correct? for acquiring Citizenship in India?
(a) 1 only (a) Age of an applicant
(b) 1 and 2 only (b) Years of stay in India
(c) 2 and 3 only (c) Birthplace of an applicant
(d) 1, 2 and 3 (d) Origin country of a person
4 TARGET PT 2020

6. Consider the following statements 1. The Act transferred the powers of the
regarding the Indian Constitution: East India Company to the British
1. Being a sovereign state, India can Crown.
neither acquire a foreign territory nor 2. It ended the system of double
cede a part of its territory in favour of government by abolishing the Board of
a foreign state. Control and Court of Directors.
2. The liberty conceived by the Preamble 3. It created a new office of the Viceroy of
or Fundamental Rights is not absolute India, who was a member of the British
but qualified. Cabinet.
Which of the above statements are Which of the above statements are correct?
incorrect?
(a) 1 and 2 only
(a) 1 only
(b) 1 and 3 only
(b) 2 only
(c) 2 and 3 only
(c) Both 1 and 2
(d) 1, 2 and 3
(d) Neither 1 nor 2
10. Who among the following were the
7. Consider the following statements
members of the drafting committee of
regarding parliamentary systems of
the Constitution?
India and Britain:
1. N. Gopalaswami
1. Britain has the system of legal
responsibility of the minister while 2. Alladi Krishnaswami Ayyar
India has not. 3. D.P. Khaitan.
2. Indian system is not based on 4. Sardar Patel
the doctrine of the sovereignty of
Parliament. Select the correct answer using the codes
given below:
Which of the above statements is/are correct?
(a) 1 and 2 only
(a) 1 only
(b) 1, 2, and 3 only
(b) 2 only
(c) 3 and 4 only
(c) Both 1 and 2
(d) 1, 2, 3, and 4
(d) Neither 1 nor 2
11. Consider the following provisions:
8. Consider the following statements
with respect to the Government of 1. It abolished the office of the Secretary
India Act 1935: of State for India and transferred his
functions to the Secretary of State for
1. It introduced dyarchy in the provinces.
Commonwealth Affairs.
2. It further extended the principle of
2. It designated the Governor-General
communal representation.
of India and the provincial governors
3. It abolished the Council of India and to act on the advice of their respective
the post of secretary of state. Council of Ministers in all matters.
Which of the above statements are 3. It discontinued the appointment to
incorrect? civil services and reservation of posts
by the Secretary of State for India.
(a) 1 and 2 only
(b) 2 and 3 only Which of the following Acts have the above
provisions?
(c) 1 and 3 only
(a) Government of India Act, 1935
(d) 1, 2 and 3
(b) Cabinet Mission
9. Consider the following statements
(c) Indian Independence Act of 1947
with respect to the ‘Act for the Good
Government of India’: (d) Cripps Mission
TARGET PT 2020 5

12. Which of the following are the unitary 16. Which of the following is considered as
features of the Indian Constitution? the first written document relating to
1. Written Constitution the Fundamental Rights of citizens?
(a) US Constitution
2. Flexibility of the Constitution
3. Single Citizenship (b) French Constitution

4. All India Services (c) Spanish Constitution


(d) Magna Carta
Select the correct answer using the code
given below: 17. “Basic Features” of the Constitutions
was first mentioned by the Supreme
(a) 1 and 2 only
Court in which of the following cases?
(b) 1, 2 and 3 only
(a) GolakNath V State of Punjab
(c) 2, 3 and 4 only (b) KesavanandaBharati V State of
(d) 1, 2, 3 and 4 Kerala
(c) Sajjan Singh V State of Rajasthan
13. Which of the following statements is/
are correct regarding Union Territories (d) KihotoHollohan V Zachillhu
(UT) in India?
18. Consider the following statements
1. All the UTs have their representation regarding B N Rau:
in Rajya Sabha at the center.
1. He was the legal advisor to the
2. For a UT to become a state, a Constituent Assembly.
constitutional amendment under 2. He prepared the draft of the
Article 368 is needed. Constitution and presented it to the
3. Delhi is the only UT having a Legislative Drafting Committee.
Assembly. 3. He was the first Indian to become a
judge at the International Court of
Select the correct option using the code Justice.
given below:
Which of the above statement is/are
(a) 1 only
correct?
(b) 2 and 3 only (a) 1 only
(c) 3 only (b) 1 and 3 only
(d) None of the above (c) 2 and 3 only
14. Which of the following had proposed (d) 1, 2 and 3
the idea of the Constituent Assembly
19. Consider the following pairs:
for the first time?
1. Advisory Committee on Minority:
(a) Motilal Nehru Sardar Patel
(b) Mahatma Gandhi 2. Provincial Constitution Committee:
(c) Khan AbdurGaffar Khan Jawaharlal Nehru
3. Union Powers Committee: J.B.
(d) Manbendra Nath Roy
Kripalani
15. The national flag was adopted by Which of the above pairs is/are correctly
Constituent Assembly on___ matched?
(a) 22 July 1947 (a) 1 only
(b) 22 August 1947 (b) 2 only
(c) 26 November 1947 (c) 2 and 3 only
(d) 15 August 1947 (d) 1, 2 and 3
6 TARGET PT 2020

20. Match List-I with List-II: (a) 1 only

List-I List-II (b) 2 only


(Features) (Source) (c) Both 1 and 2
A. Concurrent List 1. Russia (d) Neither 1 nor 2
B. Ideal of justice 2. France 23. Consider the following statements
C. Judicial review 3. USA regarding the recent HDI report 2019:
1. This year’s title is “Beyond income,
D. Rule of Law 4. Britain beyond averages, beyond today”.
Select the correct answer using the code 2. There are four indicators to calculate
given below: HDI.

A B C D 3. India is in second place amongst


SAARC members.
(a) 2 4 3 1
(b) 3 1 2 4 Which of the above statements is/are
correct?
(c) 2 1 4 3
(a) 1 only
(d) 1 2 4 3
(b) 1 and 2 only
21. Consider the following statements
(c) 3 only
regarding the Citizenship (Amendment)
Bill, 2019: (d) 1, 2, and 3
1. The Act allows the specified class of
24. Consider the following statements:
illegal migrants to apply for citizenship
by naturalization after the five years of 1. Both Hong Kong and Macau returned
residency in India. to Chinese control from Britain.
2. The provisions of the Bill are applicable 2. Their mini-Constitutions would remain
across the country. valid for 50 years i.e. till 2047.
3. The cutoff date for the immigration is
Which of the above statements is/are
on or before December 31, 2014.
incorrect?
Which of the above statements is/are (a) 1 only
correct?
(b) 2 only
(a) 1 only
(c) Both 1 and 2
(b) 2 only
(d) Neither 1 nor 2
(c) 2 and 3 only
(d) 1 and 3 only 25. Recently, Skills Build platform
has been launched by which of the
22. Consider the following statements: following pairs?
1. The total of two seats are reserved for (a) Ministry of Skill Development &
the Anglo-Indians in both Lok Sabha
Entrepreneurship, IBM
and State Legislatures.
2. This reservation was added to the (b) Ministry of Skill Development &
Constitution by the First Amendment Entrepreneurship, IIT Bombay
Act, 1951. (c) IIT Bombay, IBM
Which of the above statements is/are (d) IIT Gandhi Nagar, Ministry of
incorrect? Finance

**********
TARGET PT 2020 7

ANSWER HINTS
DAY - 1

1. Correct Option: (b)  Therefore, the term ‘Republic’ in our


Preamble indicates that India has an
Explanation:
elected head called the President. He is
Preamble of Indian Constitution elected indirectly for a fixed period of five
years.
It is based on the ‘Objectives Resolution’,
drafted by Jawaharlal Nehru.  A Republic also means two more things:
one, vesting of political sovereignty in
 It has been amended only once, in 1976,
by the 42nd Amendment Act. the people and not in a single individual
like a king; second, the absence of any
 In the Berubari Union case, 1960, the privileged class and hence all public offices
Supreme Court stated that Preamble is not
being opened to every citizen without any
a part of the Constitution.
discrimination.
 But, in 1973, in the Kesavananda
Bharati case, the Supreme Court held 3. Correct Option: (b)
that Preamble is a part of the Indian
Constitution. Hence, the current opinion Explanation:
held by the Supreme Court that the
Preamble is a part of the Constitution is in Article 3
consonance with the opinion of the founding  As per the article, Parliament may by law
fathers of the Constitution.
! form a new state by separation of
 However, two things should be noted-
territory from any state or by uniting
! The Preamble is neither a source of two or more states or parts of states
power to legislature nor a prohibition or by uniting any territory to a part of
upon the powers of legislature.
any State
! It is non-justiciable, that is, its
! Increase the area of any state
provisions are not enforceable in
courts of law. ! Diminish the area of any state

2. Correct Option: (a) ! Alter the boundaries of any state

Explanation: ! Alter the name of any state


 A bill contemplating the above changes
Republic
can be introduced in the Parliament only
 A democratic polity can be classified into two with the prior recommendation of the
categories viz. Monarchy and Republic. President.
 In a Monarchy, the head of the state
 Before recommending the bill, the President
(usually king or queen) enjoys a hereditary
has to refer the same to the state legislature
position, that is, he comes into office
through succession, e.g. Britain. concerned for expressing its views within a
specified period.
 In a Republic, on the other hand, the head
of the State is always elected directly or  However, the Parliament is not bound
indirectly for a fixed period, e.g. The USA. on the views of the State Legislature.
8 TARGET PT 2020

4. Correct Option: (b) parliamentary system. However, it never


became a replica of the British system and
Explanation: differs in the following respects:
Judicial Review ! India has a republican system in place
 The provisions of several Articles explicitly of the British monarchical system. In
confer the power of judicial review on other words, the Head of the State in
the Supreme Court and the High Courts, India (that is, President) is elected,
but not mentioned anywhere in the while the Head of the State in Britain
constitution. (that is, King or Queen) enjoys a
hereditary position.
 In a significant judgment delivered in I.R.
Coelho case (2007), the Supreme Court ! The British system is based on
ruled that there could not be any blanket the doctrine of the sovereignty of
immunity from judicial review of laws Parliament, while the Parliament
included in the Ninth Schedule. is not supreme in India and enjoys
 The Ninth Schedule was added by the limited and restricted powers due to a
1st Constitutional Amendment Act of written Constitution, federal system,
1951. judicial review, and Fundamental
Rights.
5. Correct Option: (a) ! Britain has the system of legal
Explanation: responsibility of the minister
while India has no such system.
Citizenship in India Unlike in Britain, the ministers in
India are not required to countersign
 The age of an applicant is not a criterion
for acquiring citizenship in India either the official acts of the Head of the
under the Indian Constitution or under State.
Citizenship Act 1955 for acquiring ! ‘Shadow cabinet’ is a unique
citizenship in India. institution of the British cabinet
 It can be acquired by birth, registration, system. It is formed by the opposition
descent, and Naturalization. party to balance the ruling cabinet
and to prepare its members for future
6. Correct Option: (a) ministerial office. There is no such
institution in India.
Explanation:
Indian Constitution
8. Correct Option: (d)

 Liberty means the absence of restraints Explanation:


on the activities of individuals, at the
same time providing opportunities for the Government of India Act 1935
development of individual personalities.  It provided for the establishment of an All-
 it doesn’t mean license to do what one likes India Federation consisting of provinces
and has to be enjoyed within the limitations and princely states as units. However, the
mentioned in the constitution itself. The federation never came into being as the
liberty conceived by the preamble or princely states did not join it.
fundamental rights is not absolute but  It abolished dyarchy in the provinces
qualified.
and introduced ‘provincial autonomy’ in its
 India is neither a dependency nor a place.
dominion of any other nation, but an
 It further extended the principle
independent state. Being a sovereign state,
India can either acquire a foreign territory of communal representation by
or cede a part of its territory in favour of a providing separate electorates for
foreign state. depressed classes (scheduled castes),
women and labour (workers).
7. Correct Option: (c)  It abolished the Council of India,
Explanation: established by the Government of India Act
of 1858. The secretary of state for India
Parliamentary System of India and was provided with a team of advisors
Britain but did not get abolished.
 The parliamentary system of government  It extended franchise. About 10 percent of
in India is largely based on the British the total population got the voting right.
TARGET PT 2020 9

 It provided for the establishment of a  It was set up on 29 August 1947.


Reserve Bank of India.
 At its first meeting on 30th August 1947,
 It provided for the establishment of not only the Drafting Committee elected Dr. B.R
a Federal Public Service Commission but Ambedkar as its Chairman.
also a Provincial Public Service Commission
and Joint Public Service Commission for 11. Correct Option: (c)
two or more provinces.
Explanation:
 It provided for the establishment of a
Federal Court, which was set up in 1937. Indian Independence Act, 1947
 It introduced bicameralism in six out of  It ended the British rule in India and
eleven provinces. declared India as an independent and
 It provided for the adoption of dyarchy at sovereign state from August 15, 1947.
the Centre. However, this provision of the  It provided for the partition of India and
Act did not come into operation at all. creation of two independent dominions of
India and Pakistan with the right to secede
9. Correct Option: (a) from the British Commonwealth.
Explanation:  It abolished the office of viceroy and
provided, for each dominion, a governor-
Government of India Act, 1858 (Act for the
general, who was to be appointed by the
Good Government of India)
British King on the advice of the Dominion
 The act transferred the powers of East cabinet. His Majesty’s Government in
India Company to the British Crown. Britain was to have no responsibility with
respect to the Government of India or
 It provided that India henceforth was to
Pakistan.
be governed by, and in the name of, Her
Majesty. It changed the designation of  It empowered the Constituent Assemblies
the Governor-General of India to that of the two dominions to frame and adopt
of Viceroy of India. any constitution for their respective
 Viceroy was the direct representative of nations and to repeal any act of the British
the British Crown in India. Lord Canning Parliament, including the Independence
thus became the first Viceroy of India. act itself.

 It ended the system of double  It empowered the Constituent Assemblies


government by abolishing the Board of both the dominions to legislate for
of Control and Court of Directors. their respective territories till the new
constitutions were drafted and enforced.
 It created a new office, Secretary of State No Act of the British Parliament passed
for India, vested with complete authority after August 15, 1947, was to extend to
and control over Indian administration.
either of the new dominions unless it was
The secretary of state was a member of
extended thereto by a law of the legislature
the British cabinet and was responsible
of the dominion.
ultimately to the British Parliament.
 It abolished the office of the Secretary of
 It established a 15-member Council of India
State for India and transferred his functions
to assist the secretary of state for India.
to the secretary of state for Commonwealth
The council was an advisory body. The
secretary of state was made the chairman Affairs.
of the council.  It proclaimed the lapse of British
 It constituted the secretary of state-in- paramountcy over the Indian princely
council as a body corporate, capable of suing states and treaty relations with tribal
and being sued in India and in England. areas from August 15, 1947.
 It granted freedom to the Indian princely
10. Correct Option: (b) states either to join the Dominion of India
Explanation: or Dominion of Pakistan or to remain
independent.
Drafting committee:  It provided for the governance of each
 The Drafting Committee had seven of the dominions and the provinces by
members: Alladi Krishnaswami Ayyar, the Government of India Act of 1935, till
N. Gopalaswami; B.R. Ambedkar, K.M the new Constitutions were framed. The
Munshi, Mohammad Saadulla, B.L. dominions were however authorized to
Mitter and D.P. Khaitan. make modifications in the Act.
10 TARGET PT 2020

 It designated the Governor-General of 14. Correct Option: (d)


India and the provincial governors to act
on the advice of the respective Council of Explanation:
Ministers in all matters.
Constituent Assembly
 It discontinued the appointment to civil
 The idea of a constituent assembly for India
services and reservation of posts by the
Secretary of State for India. was proposed by ManabendraNath Roy, a
founder-member of the Communist Party
12. Correct Option: (c) of India, in the year 1934.
 The proposal became a demand of the
Explanation:
Indian National Congress (INC) in 1935.
Features of Indian Constitution The British accepted the proposal in 1940.

Unitary features 15. Correct Option: (a)


 Strong Centre Explanation
 Single Constitution.
Important dates in the Constituent
 Flexibility of Constitution Assembly
 Integrated judiciary  The National Flag of India was adopted
during the fourth session of the Constituent
 Single citizenship
Assembly on 22 July 1947.
 Appointment of state governor by the
Centre  It ratified the India’s membership of the
Commonwealth in May 1949.
 All-India Services,
 It adopted the national anthem on January
 Emergency provisions, 24, 1950.
 Veto over State Bills,  It adopted the national song on January
 Parliament’s Authority over State List, 24, 1950.
etc.  It elected Dr. Rajendra Prasad as the first
President of India on January 24, 1950.
Federal features:
 Division of powers (Schedule 7) 16. Correct Option: (d)
 Independent Judiciary Explanation:
 Bicameralism
Magna Carta
 Rigidity of the constitution
 ‘Magna Carta’ is the Charter of Rights
 Supremacy of the Constitution issued by King John of England in 1215
under pressure from the barons.
 Written Constitution, etc.
 This is the first written document relating
13. Correct Option: (d) to the Fundamental Rights of citizens.
Explanation:
17. Correct Option: (c)
Union Territories in India Explanation:
 Out of the nine union territories, only
three (Delhi, J&K, and Puducherry) Evolution of the basic features
have representation in Rajya Sabha. The  The “basic features” principle was first
population of the other UTs are too small expounded in 1953, by Justice J.R.
to have any representation in the Rajya Mudholkar in his dissent, in the case
Sabha. of Sajjan Singh v. State of Rajasthan,
 As union territories are placed under Article 1964.
239-241, to confer statehood it requires a  Justice Mudholkar in his minority
normal amendment of constitution under judgment observed that the Constitutional
Article 2 to create a new state. Amendment be excluded from the definition
 Presently the union territories of J&K, of law under Article 13 and he also gave
Puducherry, and Delhi are provided with an argument that every Constitution has
the Legislative Assembly and a council of certain basic principles which could not be
ministers headed by a chief minister. changed.
TARGET PT 2020 11

 However, the Doctrine of the Basic 20. Correct Option: (c)


Structure was brought into limelight in the
majority judgment(7:6) of the 13-Judges Explanation:
bench of the KesavanandBharti V State of
Features borrowed in the Constitution
Kerala.
 The Indian Constitution borrowed various
18. Correct Option: (d) features from other constitutions such as-
Explanation: ! Government of India Act of
1935: Federal Scheme, Office of
Benegal Narsing Rau governor, Judiciary, Public Service
 Sir Benegal Narsing Rau was one of Commissions, Emergency provisions
the foremost Indian jurists of his time and administrative details.
who played a key role in drafting the ! British Constitution: Parliamentary
Constitution of India. government, Rule of Law, legislative
 Actually, the initial draft of the text of the procedure, single citizenship,
Constitution of India prepared by him. cabinet system, prerogative writs,
parliamentary privileges, and
 Rau helped draft the constitution of Burma
bicameralism.
in 1947 also.
 From February 1952 until his death(30 ! US Constitution: Fundamental
November 1953), he was a judge of the rights, independence of judiciary,
Permanent Court of International Justice, judicial review, impeachment of the
The Hague. president, removal of Supreme Court
and high court judges and post of vice
 He was also India’s representative to the president.
United Nations Security Council from 1950
to 1952. ! Irish Constitution: Directive Principles
of State Policy, nomination of members
19. Correct Option: (a) to Rajya Sabha and method of election
of president.
Explanation:
! Canadian Constitution: Federation
Major Committees of Constituent with a strong Centre, vesting of
Assembly residuary powers in the Centre,
 There were eight major committees under appointment of state governors by the
the Constituent Assembly. Centre, and advisory jurisdiction of
the Supreme Court.
 These were—
! Australian Constitution: Concurrent
! Union Powers Committee headed List, freedom of trade, commerce and
by Jawaharlal Nehru. intercourse, and joint sitting of the
! Union Constitution Committee headed two Houses of Parliament.
by Jawaharlal Nehru.
! Weimar Constitution of Germany:
! Provincial Constitution Suspension of Fundamental Rights
Committee headed by Sardar during Emergency.
Patel.
! Soviet Constitution (USSR, now
! Drafting Committee headed by Dr. Russia): Fundamental duties and the
B.R. Ambedkar. ideal of justice (social, economic and
! Advisory Committee on political) in the Preamble.
Fundamental Rights, Minorities ! French Constitution: Republic and
and Tribal and Excluded Areas the ideals of liberty, equality, and
headed by Sardar Patel. fraternity in the Preamble, Concurrent
! Rules of Procedure Committee headed list
by Dr. Rajendra Prasad. ! South African Constitution: Procedure
! States Committee headed by for amendment of the Constitution and
Jawaharlal Nehru. election of members of Rajya Sabha.
! Steering Committee headed by Dr. ! Japanese Constitution: Procedure
Rajendra Prasad. established by Law.
12 TARGET PT 2020

21. Correct Option: (d)  This year’s title is “Beyond income,


beyond averages, beyond today”.
Explanation:
 A total of 5 indices were published in this
Citizenship (Amendment) Bill, 2019 report viz. Human Development Index (HDI),
Inequality-adjusted Human Development
 Recently, the Bill has been passed by the Index (IHDI), Gender Development Index
both Houses of the Parliament. (GDI), Gender Inequality Index (GII), and
 The Bill proposes that the specified class Multidimensional Poverty Index (MPI).
(communities) of illegal migrants from  For HDI, a total of 4 indicators viz. life
the three countries will not be treated as expectancy at birth, expected years of
illegal migrants, making them eligible for schooling, mean years of schooling, and
citizenship by Naturalization. per capita gross national income (PPP) of 3
dimensions are used.
 These communities viz. Hindu, Sikh,
Buddhist, Jain, Parsi, or Christian are the  India is at 129th place (one position gain
minorities of the three neighbor countries since last year).
viz. Afghanistan, Bangladesh, or Pakistan.
 Amongst SAARC nations, India is
 For the specified class of illegal migrants, in 3rd place after Srilanka(71) and
the number of years of residency has been Maldives(104).
relaxed from 11 years to 5 years.  Norway, Switzerland, Ireland occupied the
 The Bill is not applicable to Schedule-VI top three positions.
areas of Assam, Tripura, Mizoram and
Meghalaya. 24. Correct Option: (c)
 The Bill is also not applicable to the Explanation:
Inner Line Permit states viz. Arunachal
Macau and Hong Kong
Pradesh, Mizoram, and Nagaland
 Both Hong Kong and Macau became Special
 The cutoff date for the immigration is
Administrative Regions of China.
on or before December 31, 2014.
 Hong Kong was returned to Chinese
22. Correct Option: (c) control on July 1, 1997, whereas,
Macau was transferred to China from
Explanation: Portuguese control on December 20,
1999.
Reservation for Anglo-Indians
 The regions would have their own
 Under Article 331, the President of India currencies, economic and legal systems, but
is authorized to nominate 2 members defense and diplomacy would be decided by
of the Anglo Indian community in Lok Beijing.
Sabha, in case of under-representation.
 Their mini-Constitutions would remain
 In the same way, under Article 332, the valid for 50 years i.e. till 2047 for Hong
governor of the state is authorized to Kong and 2049 for Macau.
nominate 1 Anglo Indian in the lower
house of the State Legislature, in case of 25. Correct Option: (a)
under-representation. Explanation:
 This reservation was provided by the
Skills Build platform
original Constitution for 70 years.
 Directorate General of Training (DGT),
 Recently, the Government has decided to
under the aegis of the Ministry of Skill
introduce a bill to amend the Constitution Development & Entrepreneurship
seeking to not to extend the provision (MSDE) together with IBM announced
the launch of the Skills Build platform.
23. Correct Option: (b)
 As part of the program, a two-year
Explanation: advanced diploma in IT, networking and
cloud computing, co-created and designed
HDI report 2019 by IBM, will be offered at the Industrial
 The 2019 Human Development Report is Training Institutes (ITIs) & National Skill
the latest in the series of global Human Training Institutes (NSTIs).
Development Reports published by the  The platform will be extended to train ITI &
United Nations Development Programme NSTI faculty on building skills in Artificial
(UNDP) since 1990. Intelligence (AI).
TARGET PT 2020 13

TEST
DAY - 2

Time Allowed: 30 mins Maximum Marks: 50

1. Which of the following statements 1. Only Supreme Court has been


is/are correct regarding the Right empowered with Writ Jurisdiction.
to Education as provided under the
2. Parliament by law can extend power to
Indian Constitution?
issue Writ to any other courts.
1. It is the only provision in the
3. The Writ system has been adopted
Constitution for free and compulsory
from the US Presidential system.
education for children.
2. It has been added by the 86th 4. Writs are extra-constitutional
Constitutional Amendment Act, 2002. rights given to the Judiciary by the
Parliament.
3. It is available to both citizens as well
as foreign nationals. Select the correct answer using the code
given below:
Select the correct answer using the code
given below: (a) 3 and 4 only
(a) 2 and 3 only (b) 1, 2 and 4 only
(b) 1 and 3 only (c) 2 only
(c) 2 only (d) 1, 2, 3 and 4
(d) 1, 2 and 3
4. Which of the following provisions
2. The definition of ‘State’, as provided in impose limitations upon the
Article 12 of the Indian Constitution, enforcement of the Fundamental
includes which of the following Rights?
authorities? 1. State Legislatures shall have the
1. The Parliament of India power to modify the application of the
2. The Legislature of each of the States Fundamental Rights to the members
of the Armed Forces.
3. LIC and ONGC
2. The Parliament may enact a law
4. District Boards
stating that certain jobs can only be
Select the correct answer using the code filled by applicants who are domiciled
given below: in the area.
(a) 1 only Select the correct answer using the code
(b) 1, 2 and 3 only given below:

(c) 2, 3 and 4 only (a) 1 only

(d) 1, 2, 3 and 4 (b) 2 only


(c) Both 1 and 2
3. Which of the following statements is/are
correct regarding Writ jurisdiction? (d) Neither 1 nor 2
14 TARGET PT 2020

5. Which of the following items doesn’t 9. How many types of educational


come under the ambit of the Right institutions are recognized by Article
to Freedom of Religion provided in 28?
Indian Constitution? (a) 3
1. Practicing religion (b) 4
2. Professing the religion (c) 5
3. Propagating the religion (d) 2
Select the correct answer using the code 10. Consider the following cases:
given below:
1. MP Sharma vs Satish Chandra, 1954
(a) 1 and 2 only 2. Justice K.S. Puttaswamy (Retd) vs
(b) 2 and 3 only Union of India, 2017
3. Kharak Singh vs State of Uttar
(c) 1 and 3 only
Pradesh, 1962
(d) None of the above 4. Sajjan Singh vs State of Rajasthan,
1964
6. Under which of the following Article,
the Supreme Court has held that Which of the following cases is/are related
hoisting the National Flag atop the to the Right to Privacy?
private buildings is a fundamental (a) 2 only
right of every citizen?
(b) 1, 2 and 3 only
(a) Article 14 of the Constitution
(c) 1 and 3 only
(b) Article 19(1)(a) of the Constitution
(d) 1, 2, 3, and 4
(c) Article 21 of the Constitution
11. Consider the following Articles:
(d) Article 25 of the Constitution
1. Article 31A
7. The writ of Mandamus cannot be 2. Article 31B
issued against____ 3. Article 31C
1. Private individuals 4. Article 31D
2. The Chief Justice of the High Court
Which of the above articles limit(s) the scope
3. Governors of operation of the fundamental rights?
Select the correct option using the codes (a) 4 only
given below: (b) 1 and 2 only
(a) 1 only (c) 1, 2, and 3 only
(b) 3 only (d) 2, 3, and 4 only
(c) 2 and 3 only 12. Which of the following statements
(d) 1, 2 and 3 regarding fundamental rights is/are
incorrect?
8. Which of the following Articles is 1. All of the fundamental rights are
automatically suspended during the negative in character.
national emergency? 2. All of them are directly enforceable.
(a) Article 32 3. They are defended and guaranteed by
(b) Article 14 the Supreme Court only.
4. They strike a balance between the
(c) Article 21
rights of the individual and those of
(d) Article 19 society as a whole.
TARGET PT 2020 15

Select the correct option using the codes 1. Institutions wholly maintained by the
given below: State
(a) 4 only 2. Institutions administered by the State
but established under any endowment
(b) 2 and 3 only
or trust
(c) 1, 2 and 3 only
3. Institutions recognized by the State
(d) 1, 2, 3, and 4 4. Institutions receiving aid from the
13. Consider the following lists: State

1. Ordinances In which of the above institution(s), religious


2. Delegated legislation instruction is completely prohibited?
3. Customs (a) 1 only
4. Bye-laws (b) 1 and 2 only

Which of the above lists come(s) under the (c) 2, 3, and 4 only
purview of ‘law’ in Article 13? (d) 1, 2, 3, and 4
(a) 1 and 4 only
17. Consider the following statements:
(b) 1, 2 and 3 only
1. Article 29 grants protection of language,
(c) 2 and 3 only script or culture to religious minorities
(d) 1, 2, 3, and 4 as well as linguistic minorities only.
2. The political speeches or promises
14. The fundamental rights under which made for the conservation of the
of the following articles are available language amount to corrupt practice
only to the Indian citizens? under the Representation of the People
1. Article 14 Act, 1951.
2. Article 15
Which of the above statements is/are
3. Article 19 correct?
4. Article 20 (a) 1 only
5. Article 30
(b) 2 only
Select the correct option using the codes (c) Both 1 and 2
given below:
(d) Neither 1 nor 2
(a) 1, 2 and 3 only
(b) 2, 3 and 4 only 18. Consider the following statements
regarding the RTI (Amendment) Act,
(c) 2, 3 and 5 only 2019:
(d) 1, 2, 3, and 5 only 1. The Bill mandates that the Chief
Information Commissioner (CIC) will
15. The Supreme Court in which of the
hold office for a term of five years.
following cases held that the protection
under Article 21 is available only 2. The salary of the CIC will be equivalent
against arbitrary executive action and to the salary paid to the Chief Election
not from arbitrary legislative action? Commissioner.
(a) A.K. Gopalan Case Which of the above statements is/are
(b) Shankari Prasad Case incorrect?
(c) Menaka Gandhi Case (a) 1 only
(d) ADM Jabalpur Case (b) 2 only
(c) Both 1 and 2
16. Consider the four types of educational
institutions under Article 28: (d) Neither 1 nor 2
16 TARGET PT 2020

19. The prevention of the following are (c) HRIDAY Scheme


included in which Fundamental
(d) Vidyanjali Scheme
Rights?
1. Human trafficking 22. “Hausla 2018” has been launched by
2. Forced labour which Ministry?

3. Child labour (a) Ministry of Women and Child


Development
Select the correct answer using the code
(b) Ministry of Home Affairs
given below:
(c) Ministry of Defense
(a) Right to Equality
(d) Ministry of External Affairs
(b) Right to Freedom
(c) Right against Exploitation 23. India’s first owl festival was held in
which state?
(d) Right to Constitutional Remedies
(a) Manipur
20. Which of the following statements is/
(b) Madhya Pradesh
are correct?
1. The Constitutional remedies under (c) Maharashtra
both Article 32 and Article 226 are (d) Jharkhand
mandatory.
2. The writ jurisdiction of High Courts 24. Which of the following state has not
under Article 226 is broader than those achieved 100% electrification under
of the Supreme Court. Saubhagya Scheme?
(a) Uttar Pradesh
Select the correct option using the codes
given below: (b) Bihar
(a) 1 only (c) Jammu and Kashmir
(b) 2 only (d) Madhya Pradesh
(c) Both 1 and 2 25. ‘COPE INDIA 2019’ is a joint exercise
(d) Neither 1 nor 2 between India and which country?
(a) Japan
21. Bhasha Sangam has been launched
under which scheme? (b) Russia
(a) Know India Campaign (c) UAE
(b) Ek Bharat Shreshtha Bharat (d) USA

**********
TARGET PT 2020 17

ANSWER HINTS
DAY - 2

1. Correct Option: (a) that is, executive and legislative


organs of State governments.
Explanation:
! All other authorities, that is, statutory
Article 21A: Right to Education or non-statutory authorities like LIC,
ONGC, SAIL, etc.
 Article 21 A declares that the State shall
provide free and compulsory education to ! All local authorities, that is,
all children of the age of six to fourteen municipalities, Panchayats, district
years in such a manner as the State may boards, improvement trusts, etc.
determine.
 Thus, State has been defined in a wider
 Thus, this provision makes only elementary sense so as to include all its agencies. It is
education a Fundamental Right and not the actions of these agencies that can be
higher or professional education. challenged in the courts as violating the
 This provision was added by the 86th Fundamental Rights.
Constitutional Amendment Act of  According to the Supreme Court, even a
2002. private body or an agency working as an
! This amendment is a major milestone instrument of the State falls within the
in the country’s aim to achieve meaning of the ‘State’ under Article 12.
‘Education for All’. The government
described this step as ‘the dawn of the 3. Correct Option: (c)
second Revolution in the chapter of Explanation:
citizens’ rights’.
! This amendment also added a new Writ jurisdiction of Indian Constitution
Fundamental Duty under Article  Article 32 confers the right to remedies
51A that reads —‘It shall be the duty for the enforcement of the Fundamental
of every citizen of India to provide Rights of an aggrieved citizen. In other
opportunities for education to his words, the right to get the Fundamental
child or ward between the age of six Rights protected is in itself a Fundamental
and fourteen years’. Right.
 It is available to both citizens as well as  The origin of writs is from the English
foreigners. Judicial system.

2. Correct Option: (d)  Dr. Ambedkar called Article 32 as the most


important article of the Constitution.
Explanation:
 The Supreme Court has ruled that Article
Definition of State 32 is a basic feature of the Constitution.
 The term ‘State’ has been used in different  The Supreme Court shall have the power
provisions concerning the Fundamental to issue directions or orders or writs for the
Rights. Hence, Article 12 has defined the enforcement of any of the Fundamental
term for the purposes of Part III. According Rights. The writs issued may include:
to it, the State includes the following:
! Habeas corpus
! Government and Parliament of India, ! Mandamus
that is, executive and legislative
organs of the Union government. ! Prohibition
! Government and Legislature of states, ! Certiorari
18 TARGET PT 2020

! Quo-warranto 7. Correct Option: (d)


 Parliament can empower any other Explanation:
court to issue directions, orders, and
writs of all kinds. However, this can be Writ of Mandamus
done without prejudice to the above powers
conferred on the Supreme Court. Any other  The term ‘mandamus’ literally means
court here does not include High Courts ‘we command’. It is a command issued by
because Article 226 has already the court to a public official asking him
conferred these powers on the High to perform the official duties that he has
Courts. failed or refused to perform.
 It can also be issued against any public body,
4. Correct Option: (b) a corporation, an inferior court, a tribunal
Explanation: or government for the same purpose.

Fundamental Rights  However, the writ of mandamus cannot be


issued against-
 The power to make laws under Article 33
is conferred only on Parliament and ! private individual or body
not on State Legislatures. Any such law ! to enforce departmental instruction
made by Parliament cannot be challenged that does not possess statutory force
in any court on the ground of contravention
of any of the Fundamental Rights. ! when the duty is discretionary and
not mandatory
 Article 35 lays down that the power to
make laws, to give effect to certain specified ! to enforce a contractual obligation
Fundamental Rights shall vest only in
the Parliament and not in the State ! the president of India or the state
Legislatures. This provision ensures governors
that there is uniformity throughout ! the chief justice of a high court
India with regard to the nature of those acting in a judicial capacity.
Fundamental Rights and punishment for
their infringement. 8. Correct Option: (d)
 The Parliament can prescribe residence
as a condition for certain employments or Explanation:
appointments in a state or union territory
Suspension of Fundamental Rights
or local authority or other authority
(Article 16).  After the 44th Amendment Act, 1978, Article
358 provides that when the proclamation of
5. Correct option: (d) emergency is made by the president under
Article 352, the freedoms guaranteed
Explanation:
by Article19(1) are automatically
Right to Freedom of Religion suspended for the period of emergency on
the ground of war or external aggression.
 Article 25 says that every citizen of the
State has the right to not only practice  Articles 20 and 21 cannot be suspended
but also profess and propagate the even during the National Emergency.
religion of his/her choice.
9. Correct Option: (b)
6. Correct Option: (b)
Explanation:
Explanation:
Article 28
Fundamental rights
 Article 28 distinguishes between four
 According to the Supreme Court of India, types of educational institutions.
hoisting the national Flag atop the private
buildings is a fundamental right of every  These are (i) Institutions wholly maintained
citizen under Article 19 (1)(a) of the by the State, (ii) Institutions administered
constitution. by the State but established under any
endowment or trust, (iii) Institutions
 Article 14 guarantees equality before the recognized by the State, and (iv) Institutions
law or the equal protection of the laws. receiving aid from the State.
 Article 21 deals with the right to life and
 In (i) religious instruction is completely
personal liberty. prohibited while in (ii), religious instruction
 Article 25 provides for freedom of is permitted. In (iii) and (iv), religious
conscience and free profession, practice instruction is permitted on a voluntary
and propagation of religion. basis.
TARGET PT 2020 19

10. Correct Option: (b)  Some of them are negative in character


(Rights to Equality), that is, place
Explanation: limitations on the authority of the State,
while others are positive in nature,
Right to Privacy
conferring certain privileges on the persons
 The right to privacy in India has developed (Rights to Equal Opportunity).
through a series of decisions over the past
 They are defended and guaranteed by both
60 years.
the Supreme Court and High Courts.
 In MP Sharma vs Satish Chandra in
 They are not sacrosanct or permanent. The
1954 and Kharak Singh vs State of
Parliament can curtail or repeal them but
Uttar Pradesh in 1962 cases, Supreme
only by a constitutional amendment act
Court held that privacy was not a
without affecting the ‘basic structure’ of
fundamental right.
the Constitution.
 But in the Justice K.S. Puttaswamy
(Retd) vs Union of India 2017, SC ruled  Most of them are directly enforceable
that the right to privacy is protected (self-executory) while a few of them can
as an intrinsic part of the right to life be enforced on the basis of a law made
and personal liberty under Article 21 and for giving effect to them by Parliament only
as a part of the freedoms guaranteed by under Article 35.
Part III of the Constitution.
13. Correct Option: (d)
11. Correct Option: (c) Explanation:
Explanation: Article 13
Limitation on the Fundamental rights  The term ‘law’ in Article 13 has been given
 Article 31A and 31B were added to the a wide connotation so as to include the
constitution by the First Amendment following:
Act, 1951. Article 31C was by the 25th ! Permanent laws enacted by the
Amendment Act, 1971. All of these articles Parliament or the state legislatures;
limit the scope of the Fundamental Rights.
! Temporary laws like ordinances
 The Fundamental Rights can be suspended issued by the President or the State
during the operation of a National Governors;
Emergency except the rights guaranteed
by Articles 20 and 21. ! Statutory instruments in the nature
of delegated legislation (executive
 Further, the six rights guaranteed by legislation) like order, bye-law, rule,
Article 19 are suspended naturally when regulation or notification; and
the emergency is declared on the grounds
of war or external aggression. ! Non-legislative sources of law, that is,
custom or usage having the force of
 Furthermore, Article 33 and Article 34 too, law.
limit their scope.
 Thus, not only legislation but any of the
12. Correct Option: (b) above can be challenged in the courts as
violating a Fundamental Right and hence
Explanation: can be declared as void.
Nature of Fundamental rights  Further, the Supreme Court held in the
Kesavananda Bharati case (1973) that
 Some of them are available only to the a Constitutional amendment can be
citizens while others are available to all challenged on the ground that it violates a
persons whether citizens, foreigners or legal fundamental right that forms a part of the
persons like corporations or companies. ‘basic structure’ of the Constitution.
 They are not absolute but qualified. The
state can impose reasonable restrictions on 14. Correct Option: (c)
them. However, whether such restrictions
are reasonable or not is to be decided by
Explanation:
the courts. Thus, they strike a balance FR available only to citizens and not to
between the rights of the individual foreigners
and those of society as a whole.
 Prohibition of discrimination on grounds
 Most of them are available against the of religion, race, caste, sex or place of birth
arbitrary action of the State, with a few
(Article 15).
exceptions like those against the State’s
action and against the action of private  Equality of opportunity in matters of public
individuals. employment (Article 16).
20 TARGET PT 2020

 Protection of six rights regarding freedom of  In (i) religious instruction is completely


(i) speech and expression, (ii) assembly, (iii) prohibited while in (ii), religious instruction
association, (iv) movement, (v) residence, is permitted. In (iii) and (iv), religious
and (vi) profession (Article 19). instruction is permitted on a voluntary
 Protection of language, script, and culture basis.
of minorities (Article 29).
 Right of minorities to establish and 17. Correct Option: (d)
administer educational institutions
Explanation:
(Article 30).
Article 29
15. Correct Option: (a)
 Article 29 provides that any section of the
Explanation:
citizens residing in any part of India having
Right to life a distinct language, script or culture of its
own, shall have the right to conserve the
 Article 21 declares that no person shall be
same.
deprived of his life or personal liberty except
according to the procedure established by  However, the Supreme Court held that the
law. scope of this article is not necessarily
 In the A.K. Gopalan Case 1950, the Supreme restricted to minorities only, as it is
Court has taken a narrow interpretation commonly assumed to be. This is because
of Article 21. It held that the protection of the use of the words ‘section of citizens’
under Article 21 is available only against in the Article that includes minorities as
arbitrary executive action and not from well as the majority.
arbitrary legislative action.
 This means that the State can deprive  The Supreme Court also held that the
the right to life and personal liberty of a right to conserve the language includes
person based on a law. This is because of the right to agitate for the protection of the
the expression ‘procedure established by language. Hence, the political speeches
law’ in Article 21. or promises made for the conservation
 But in the Menaka Gandhi Case 1978, the of the language of a section of the
Supreme Court overruled its judgement citizens does not amount to corrupt
in the Gopalan case by taking a wider practice under the Representation of
interpretation of Article 21. the People Act, 1951.
 It ruled that the right to life and personal
liberty of a person can be deprived by law 18. Correct Option: (c)
provided the procedure prescribed by that
law is reasonable, fair and just. Thus, Explanation:
it introduced the ‘due process of law’ in
Article 21. RTI (Amendment) Act, 2019
 The Amendment Act changes the terms
16. Correct Option: (a) and conditions of service of the CIC and
Explanation: Information Commissioners at the center
and in states which was provided by the
Article 28 original Act of 2005.
 Under Article 28, no religious instruction  The Amendment Act of 2019 states that
shall be provided in any educational the central government will notify the
institution wholly maintained out of State
term of office for the CIC and the ICs
funds.
against the fixed term of five years,
 Article 28 distinguishes between four types provided by the 2005 Act.
of educational institutions viz.
 It also states that the salaries,
! Institutions wholly maintained by the
allowances, and other terms and
State
conditions of service of the central and
! Institutions administered by the State state CIC and ICs will be determined
but established under any endowment by the central government against the
or trust
equivalence of the salary paid to the
! Institutions recognized by the State Election Commissioners and the Chief
and Secretary to the state government,
! Institutions receiving aid from the respectively, as provided by the
State original Act.
TARGET PT 2020 21

 As per the Government, the CIC is a  Bhasha Sangam is a programme for


statutory body and Election Commission schools and educational institutions to
is a Constitutional body hence, the provide multilingual exposure to students
mandate of the Election Commission of in Indian languages.
India and Central and State Information
Commissions are different and needed to  Another objective of Bhasha Sangam is to
be determined accordingly. enhance linguistic tolerance and respect
and promote national integration.
19. Correct Option: (c)
22. Correct Option: (a)
Explanation:
Explanation:
Right against exploitation
 The National Festival for Children of
 The Constitution of India has special
Child Care Institutions (CCIs)- “Hausla
provision mentioned right under the
2018” of the Ministry of Women and Child
columns of Fundamental Rights to prevent
and prohibit human trafficking, forced and Development (MWCD) was inaugurated in
child labour. New Delhi.

 The fundamental right which deals with  The inter-Child Care Institution festival
the same is known as “Right against is witnessing the participation of more
Exploitation”. Through this Right, the than 600 Children drawn from CCIs from
Constitution of India prohibits human 18 states in various events like painting
trafficking, forced labor, as well as Child competition, athletics meet, football, chess
labour, which means the children who are competition and speech writing as part of
working under 14 years if their age. the Festival.

20. Correct Option: (b) 23. Correct Option: (c)


Explanation: Explanation:
Articles 32 and 226  The Indian Owl Festival, the country’s
 A remedy under Article 32 is in itself first owl fest, was held at Pingori village in
a Fundamental Right and hence, the Purandar taluka of Pune, Maharashtra.
Supreme Court shall not refuse to  The two-day festival, organised by Ela
exercise its writ jurisdiction.
Foundation, an NGO working towards
 On the contrary, the remedy under nature education and conservation, will
Article 226 is discretionary and hence, give information on owl conservation and
a high court may refuse to exercise its writ feature art forms like pictures, paintings,
jurisdiction. lanterns, lampshades, posters, origami,
 The Supreme Court is thus constituted as a stitched articles, poems and stories on
defender and guarantor of the fundamental owls.
rights.
 Unlike Article 32, Article 226 confers
24. Correct Option: (a)
power on a High Court to issue writs for Explanation:
the enforcement of fundamental rights
as well as other rights. Hence, the writ  8 States have achieved 100% saturation in
jurisdictions of High Courts, under Article household electrification under Saubhagya
226, are broader than those of the namely Madhya Pradesh, Tripura, Bihar,
Supreme Court. J&K, Mizoram, Sikkim, Telangana, and
West Bengal.
21. Correct Option: (b)
25. Correct Option: (d)
Explanation:
Explanation:
 The Government has launched a unique
initiative called Bhasha Sangam to  Air forces of the United States and India
introduce school students to 22 Indian are scheduled to participate in a 12-day
languages. joint exercise ‘Cope India 2019’ at two air
 The initiative, under Ek Bharat Shreshtha force stations in West Bengal. The exercise
Bharat, was launched on the 22nd of this showcases the efforts and commitment of
month and will continue till the 21st of the two nations to a free and open Indo-
December. Pacific region.
22 TARGET PT 2020

TEST
DAY - 3

Time Allowed: 30 mins Maximum Marks: 50

1. Which of the following is/are not the 1. Promotion of Panchayati Raj System.
Fundamental Duties of a Citizen of
2. Adherence to Non-Alignment and
India under Article 51A of the Indian
Constitution? Panchsheel.
1. To vote in general elections. 3. The Maternity Benefit (Amendment)
2. To defend the country. Act of 2017.
3. To develop humanism and the spirit of 4. Formation of National Commission for
inquiry.
Backward Classes.
4. To strive towards excellence.
Select the correct answer using the code
Select the correct answer using the code
given below: given below:

(a) 1 only (a) 1 and 2 only


(b) 1 and 3 only (b) 3 and 4 only
(c) 2 and 4 only
(c) 2 and 3 only
(d) 2, 3 and 4 only
(d) 1, 2, 3 and 4
2. Which of the following Directive
Principles of State Policy is/are not 4. Which of the following pairs is correctly
based on Socialist principles? matched?
1. Equal pay for equal work.
1. 42nd Amendment: Opportunities for the
2. Free legal aid to the poor. healthy development of children.
3. Participation of workers in the
2. 44th Amendment: Promotion of
management of industries.
management of co-operative societies.
4. Protection and improvement of the
environment. 3. 97th Amendment: Minimize

Select the correct answer using the code inequalities in income, status, facility
given below: & opportunities.
(a) 1 and 3 only Select the correct answer using the code
(b) 4 only given below:
(c) 3 and 4 only
(a) 1 only
(d) 1, 2 and 3 only
(b) 2 and 3 only
3. Which of the following led to the
(c) 2 only
successful implementation of Directive
Principles of State Policy in India? (d) 1, 2, and 3
TARGET PT 2020 23

5. Consider the following statements 1. Morarji Desai government tried to


regarding Fundamental Duties: annul the Fundamental Duties in the
1. These were added by the 44th Lok Sabha but failed.
Constitutional Amendment Act. 2. There has not been any instance to
2. Some of the Fundamental Duties are amend the Fundamental Duties.
applicable to foreigners also.
3. They are not justiciable in nature. Select the correct option using the codes
given below:
4. They help the courts in examining and
determining the constitutional validity (a) 1 only
of a law.
(b) 2 only
Which of the above statements is/are (c) Both 1 and 2
correct?
(d) Neither 1 nor 2
(a) 1 and 2 only
(b) 1 and 4 only 9. Which of the following statements is/
are correct?
(c) 4 only
1. Rights and duties of the citizens are
(d) 3 and 4 only
correlative and inseparable.
6. Which of the following is/are not 2. Except for India, none of the
related to Fundamental duties? constitutions of any democratic country
1. Verma Committee contains a list of duties of citizens.
2. Swaran Singh Committee 3. The socialist countries, on the
3. H. R. Gokhale Committee contrary, gave equal importance to the
fundamental rights and duties of their
Select the correct option using the codes citizens.
given below:
(a) 2 only Select the correct option using the codes
given below:
(b) 1 and 2 only
(a) 2 only
(c) 3 only
(b) 2 and 3 only
(d) 1 and 3 only
(c) 1 and 3 only
7. Consider the following Acts enacted
by the Parliament? (d) 1, 2, and 3
1. The Wildlife (Protection) Act 10. Which of the following shall adopt the
2. The Unlawful Activities (Prevention) ideals of the Principles of Directive
Act Principles of State Policy while
3. The Protection of Civil Rights Act formulating policies and laws?
4. The Prevention of Insults to National 1. Union Legislature
Honour Act
2. State Government
Which of the above Acts had been enacted 3. Local authorities
before the enactment of Article 51A?
4. Public Sector Enterprises
(a) 1 only
Select the correct answer using the code
(b) 2, 3, and 4 only
given below:
(c) 1 and 4 only
(a) 1 only
(d) 1, 2, 3, and 4
(b) 1, 2 and 3 only
8. Which of the following statements
(c) 1, 3 and 4 only
regarding Fundamental Duties is
incorrect? (d) 1, 2, 3 and 4
24 TARGET PT 2020

11. Which of the following statements is/ Select the correct option using the codes
are correct? given below:
1. Indian Constitution strikes a balance
(a) 1 only
between socialism and liberalism.
2. Rawlsian theory of distributive justice (b) 2 only
can be observed in both Part III and IV
(c) Both 1 and 2
of the Indian Constitution.
(d) Neither 1 nor 2
Select the correct option using the codes
given below:
15. Consider the following statements
(a) 1 only regarding Directive Principles:
(b) 2 only 1. Fundamental rights are the rights to
(c) Both 1 and 2 citizens against the state.
(d) Neither 1 nor 2 2. Directive Principles are the right to
state against citizens.
12. Which of the following DPSPs is not
based on the Liberal–Intellectual 3. Fundamental rights promote the
Principles? welfare of the individual while DPSPs
(a) To secure for all citizens a uniform civil promote the welfare of the community.
code throughout the country
4. The courts cannot declare a law violative
(b) To secure the right to work to education of any of the Directive Principles as
and to public assistance in cases of unconstitutional and invalid.
unemployment, old age, sickness and
disablement Which of the above statements is/are
(c) To organize agriculture and animal incorrect?
husbandry on modern and scientific
lines (a) 2 only

(d) To promote international peace (b) 3 and 4 only


and security and maintain just and
(c) 1 and 2 only
honorable relations between nations.
(d) 1, 2, and 4
13. Consider the following list:
1. Part III 16. To implement which of the following
2. Part IV Articles of the Directive Principles,
3. Part IVA the Act of Parliament can contravene
the Fundamental Rights conferred by
Which of the above parts of the Constitution
Article 14 and Article 19?
was (were) amended by “The Constitution
(Eighty-sixth Amendment) Act, 2002”? 1. Article 39
(a) 1, 2, and 3 2. Article 39(a)
(b) 2 only 3. Article 39(b)
(c) 1 and 3 only 4. Article 39(c)
(d) 1 and 2 only
Select the correct option using the codes
14. Which of the following statements given below:
regarding DPSPs is correct?
(a) 3 and 4 only
1. The harmony and balance between Part
III and IV is an essential feature of the (b) 2, 3, and 4 only
basic structure of the Constitution.
(c) 1, 2 and 3 only
2. Fundamental Rights enjoy supremacy
over the Directive Principles. (d) 1, 2, 3, and 4
TARGET PT 2020 25

17. In which of the following cases 21. The Paramarsh Scheme is launched
the Supreme Court ruled that “the by:
Fundamental Rights cannot be (a) Ministry of Human Resources
amended for the implementation of Development
the Directive Principles”?
(b) Ministry of Corporate Affairs
(a) Champakam Dorairajan Case
(c) Ministry of Home Affairs
(b) Golaknath Case
(d) Ministry of Rural Development
(c) Waman Rao Case
(d) Minerva Mills Case
22. Which bank has conducted its first
18. The Legal Services Authorities Act e-auction for forest produce?
was established in 1987 to give effect (a) ICICI Bank
to which of the following Directive
Principles? (b) Yes Bank

(a) Article 39A (c) Canara Bank

(b) Article 39(a) (d) HDFC Bank

(c) Article 43A 23. Recently, the government has set up


(d) Article 43B Aruna Sundararajan Committee for
making recommendations on:
19. Which of the following is not related to (a) Indian Railways
the 97th Amendment Act?
(b) Telecom sector
(a) Article 19(1)
(c) Education sector
(b) Article 43B
(d) Coal auction
(c) Part IXA
24. The Indian government has recently
(d) None of the above extended the financial aid to which of
the following country for rebuilding
20. In which of the following parts of the
Earthquake-hit Houses?
Constitution has the provisions for
the protection of the environment and (a) Nepal
wildlife? (b) Philippines
1. Part III
(c) Indonesia
2. Part IV
(d) Turkey
3. Part IVA
25. Which bank will provide a line of
Select the correct answers using the options credit to Ghana and Mozambique for
given below: agricultural and railway projects?
(a) 2 and 3 only (a) Yes Bank
(b) 2 only (b) IDBI Bank
(c) 1, 2, and 3 (c) Exim Bank
(d) 3 only (d) None of the above

**********
26 TARGET PT 2020

ANSWER HINTS
DAY - 3

1. Correct Option: (a) child, or a ward between the ages of


6-14 years as the case may be.
Explanation:
Fundamental Duties 2. Correct Option: (b)

 According to Article 51A, it shall be the Explanation:


duty of every citizen of India:
DPSP based on Socialist Principle
! To abide by the Constitution and
respect its ideals and institutions,  To promote the welfare of the people by
the National Flag and the National securing a social order permeated by
Anthem. justice—social, economic and political—
and to minimize inequalities in income,
! To cherish and follow the noble ideals
status, facilities, and opportunities.
that inspired the national struggle for
freedom.  To secure
! To uphold and protect the sovereignty, ! the right to adequate means of
unity, and integrity of India. livelihood for all citizens;
! To defend the country and render ! the equitable distribution of material
national service when called upon to. resources of the community for the
! To promote harmony and the spirit of common good;
common brotherhood amongst all the ! prevention of concentration of wealth
people of India transcending religious,
and means of production;
linguistic and regional or sectional
diversities and to renounce practices ! equal pay for equal work for men
derogatory to the dignity of women. and women;
! To value and preserve the rich heritage ! preservation of the health and
of the country’s composite culture. strength of workers and children
! To protect and improve the natural against forcible abuse; and
environment including forests, lakes, ! opportunities for the healthy
rivers, and wildlife and to have development of children (Article 39).
compassion for living creatures.
 To promote equal justice and to provide
! To develop a scientific temper, free legal aid to the poor (Article 39A).
humanism and the spirit of
inquiry and reform.  To secure the right to work, to education
and to public assistance in cases of
! To safeguard public property and to
abjure violence. unemployment, old age, sickness and
disablement (Article 41).
! To strive towards excellence in all
spheres of individual and collective  To make provision for just and humane
activity so that the nation constantly conditions of work and maternity relief
rises to higher levels of endeavor and (Article 42).
achievement.  To secure a living wage, a decent standard of
! To provide opportunities for education living and social and cultural opportunities
by the parent the guardian, to his for all workers (Article 43).
TARGET PT 2020 27

 To take steps to secure the participation 5. Correct Option: (d)


of workers in the management of
industries (Article 43A). Explanation:

 To raise the level of nutrition and the Features of the Fundamental Duties
standard of living of people and to improve
 These were added by the 42nd
public health (Article 47).
Amendment Act, 1976.
3. Correct Option: (d)  These are the set of constitutional
duties for Indian citizens only.
Explanation:
 Some of them are moral duties while others
Implementation of DPSPs are civic duties. For instance, cherishing
 The 73rd Amendment Act (1992) has been noble ideals of freedom struggle is a moral
enacted to provide constitutional status precept and respecting the Constitution,
and protection to these Panchayati Raj National Flag and National Anthem is a
Institutions. It aims to achieve Gandhiji’s civic duty.
dream of every village being a republic and
 They refer to such values that have been
self-sufficient.
a part of the Indian tradition, mythology,
 Adherence to the policy of non-alignment religions, and practices. In other words,
and Panchsheel will promote international they essentially contain just a codification
peace and security. of tasks integral to the Indian way of life.
 The Maternity Benefit (Amendment) Act  Like the Directive Principles, the
2017 has been made to protect the interests fundamental duties are also non-
of women workers. justiciable. The Constitution does not
 National Commission for Backward provide for its direct enforcement by
Classes will promote and protect the social, the courts. Moreover, there is no legal
educational and economic interests of the sanction against their violation However,
weaker sections of the society. the Parliament is free to enforce them by
suitable legislation.
4. Correct Option: (b)  They are not enforceable by law but
Explanation: they help the courts in examining
and determining the constitutional
Amendments and DPSP added validity of a law.
 The 42nd Amendment Act of 1976 added
four new DPSP: 6. Correct Option: (c)
! Subclause in Article 39: To secure Explanation:
opportunities for the healthy
development of children. Committees for Fundamental duties

! Article 39 A: To promote equal justice  In 1976, the Congress Party set up the
and to provide free legal aid to the Sardar Swaran Singh Committee to make
poor. recommendations about fundamental
duties and amended the Constitution by
! Article 43 A: To take steps to secure
adding Article 51A.
the participation of workers in the
management of industries.  In 1999, the Verma Committee on
Fundamental Duties of the Citizens (1999)
! Article 48 A: To protect and improve
the environment and to safeguard identified the existence of legal provisions
forests and wildlife. for the implementation of some of the
Fundamental Duties
 The 44th Amendment Act of 1978 added
Article 38: It requires the State to minimize 7. Correct Option: (d)
inequalities in income, status, facilities,
and opportunities. Explanation:
 The 97th Amendment Act of 2011 added Acts related to the Fundamental Duties
Article 43B: It requires the state to
promote voluntary formation, autonomous  The Prevention of Insults to National
functioning, democratic control and Honour Act was enacted in 1971 to prevent
professional management of co-operative disrespect to the Constitution of India, the
societies. National Flag, and the National Anthem.
28 TARGET PT 2020

 The Protection of Civil Rights Act was  In India too, the original constitution
enacted in 1955 to provide punishments contained only the fundamental rights and
for offenses related to caste and religion. not the fundamental duties.
 The Unlawful Activities (Prevention) Act  These were added in the Constitution
was enacted in 1967 to provide for the during the Emergency by the 42nd
declaration of a communal organization as Amendment Act.
an unlawful association.
 The Fundamental Duties in the
 The Wildlife (Protection) Act of 1972 Indian Constitution are inspired by
prohibits trade in rare and endangered the Constitution of erstwhile USSR as
species. the socialist countries give equal
importance to the fundamental rights
8. Correct Option: (c) and duties of their citizens.
Explanation:
10. Correct Option: (d)
Significance of Fundamental Duties Explanation:
 In spite of criticisms and opposition,
the fundamental duties are considered Directive Principles of State Policy
significant as:  The phrase ‘Directive Principles of State
 they serve as a reminder to the citizens Policy’ denotes the ideals that the State
that while enjoying their rights, they should keep in mind while formulating
should also be conscious of the duties they policies and enacting laws.
owe to their country, their society and to  These are the constitutional instructions or
their fellow citizens; recommendations to the State in legislative,
 they serve as a warning against anti- executive and administrative matters.
national and anti-social activities like  According to Article 36, the term ‘State’
burning the national flag, destroying public in Part IV has the same meaning as
property and so on. in Article 12 Part III dealing with
 they serve as a source of inspiration for the Fundamental Rights. Therefore, it includes
citizens and promote a sense of discipline the legislative and executive organs of the
and commitment among them and central and state governments, all local
authorities and all other public authorities
 they help the courts in examining and in the country.
determining the constitutional validity of
a law.  Article12 defines the “state” which
includes:
 The Morarji Desai government sought
to undo many changes introduced in the ! Government and Parliament of India,
Constitution by the 42nd Amendment Act that is, executive and legislative
(1976) but did not annul Article 51A. organs of the Union government.
This shows that there was an eventual ! Government and legislature of
consensus on the necessity and desirability states, that is, executive and legislative
of including the Fundamental Duties in the
organs of state government.
Constitution.
! All local authorities, that is,
 In 2002, one more Fundamental Duty
municipalities, panchayats, district
was added by the 86th Amendment Act,
boards, improvement trusts, etc.
to provide opportunities for education to
his child or ward between the age of six and ! All other authorities, that is, statutory
fourteen years. or non-statutory authorities like LIC,
ONGC, SAIL, etc.
9. Correct Option: (c) ! According to the Supreme Court,
Explanation: even a private body or an agency
working as an instrument of the
Fundamental Duties State falls within the meaning of the
‘State’ under Article 12.
 Although the rights and duties of
the citizens are correlative and
inseparable, none of the Constitutions of
11. Correct Option: (c)
major democratic countries except India Explanation:
and Japan contain a list of duties of
citizens. Philosophy of Indian Constitution
TARGET PT 2020 29

 The guiding philosophy of the Indian 13. Correct Option: (a)


Constitution is enshrined in Part III
(Fundamental Rights) and Part IV Explanation:
(DPSPs). 86th Amendment Act, 2002
 Of the two main rights viz. political and  The Act enshrined right to education as a
socio-economic (enshrined in the Preamble fundamental right in Part-III of the Indian
too), Part III provides political rights Constitution by adding Article 21A. it
whereas, DPSPs provides Indian citizens states that “The State shall provide free and
the socio-economic rights and justice. compulsory education to all children of the
 Thus, Part III implies liberalism while Part age of six to fourteen years in such manner
as the State may, by law, determine”.
IV, the socialism. Since both are equally
important(As per the Supreme Court, in  Also, the Act substituted “age of fourteen
the Minerva Mills Case, 1980), the Indian years” with “age of six years” in the
Constitution strikes a perfect balance Article 45 and now its is read as “The State
between liberalism and socialism. shall endeavor to provide early childhood
care and education for all children until
 Articles 15(2), 16(2), 23, 24, etc of Part they complete the age of six years”
III and Articles 38, 39, 41, etc of Part IV
imply the Rawlsian theory of distributive  The clause (k) was added to the Article
justice. 51A stating that- “It shall be the duty
of every citizen of India who is a parent
12. Correct Option: (b) or guardian to provide opportunities
for education to his child or, as the case
Explanation: may be, ward between the age of six and
fourteen years”.
Liberal–Intellectual Principles
 A total of seven directive principles 14. Correct Option: (c)
are based on the Liberal–Intellectual Explanation:
Principles. These are-
Relationship between Fundamental Rights
! To secure for all citizens a uniform
and Directive Principles
civil code throughout the country
(Article 44).  The Supreme Court in the Minerva
Mills case (1980) held that the Indian
! To provide early childhood care and Constitution is founded on the bedrock
education for all children until they of the balance between the Fundamental
complete the age of six years9 (Article Rights and the Directive Principles.
45).
 To give absolute primacy to one over the
! To organize agriculture and other is to disturb the harmony of the
animal husbandry on modern and Constitution. This harmony and balance
scientific lines (Article 48). between the two is an essential feature of
! To protect and improve the the basic structure of the Constitution.
environment and to safeguard forests  Therefore, the present position is that the
and wildlife (Article 48 A). Fundamental Rights enjoy supremacy over
! To protect monuments, places, and the Directive Principles.
objects of artistic or historic interest  Yet, Directive Principles can be implemented
which are declared to be of national and the Parliament can amend the
importance (Article 49). Fundamental Rights for implementing
! To separate the judiciary from the the Directive Principles, so long as the
amendment does not damage or destroy
executive in the public services of the
the basic structure of the Constitution.
State (Article 50).
! To promote international peace 15. Correct Option: (a)
and security and maintain
just and honorable relations Explanation:
between nations; to foster respect
Distinction Between Fundamental
for international law and treaty
obligations, and to encourage
Rights and Directive Principles
settlement of international disputes  Fundamental Rights are negative as they
by arbitration (Article 51). prohibit the State from doing certain
30 TARGET PT 2020

things; DPSPs are positive as they require Principles even the Act contravenes the
the State to do certain things. Articles 14 and 19.
 Fundamental Rights are justiciable, that
17. Correct Option: (b)
is, they are legally enforceable by the
courts in case of their violation; DPSPs are Explanation:
non-justiciable.
Golaknath Case
 FRs have legal sanctions; DPSPs have
moral and political sanctions.  In the Champakam Dorairajan case (1951),
the Supreme Court ruled that in case of any
 FRs promote the welfare of the conflict between the Fundamental Rights
individual. Hence, they are personal and the Directive Principles, the former
and individualistic; DPSPs promote the would prevail.
welfare of the community. Hence, they are
societarian and socialistic.  But in the Golaknath case (1967), the
Supreme Court ruled that the Parliament
 FRs do not require any legislation for their cannot take away or abridge any of the
implementation as they are automatically Fundamental Rights.
enforced; DPSPs require legislation for
their implementation.  In other words, the Court held that the
Fundamental Rights cannot be amended
 The courts are bound to declare a law for the implementation of the Directive
violative of any of the Fundamental Rights Principles.
as unconstitutional and invalid; The courts
cannot declare a law violative of any of the 18. Correct Option: (a)
Directive Principles as unconstitutional
and invalid. However, they can uphold the Explanation:
validity of a law on the ground that it was
Article 39A
enacted to give effect to a directive.
 It directs the State “To take steps to
16. Correct Option: (a) secure the participation of workers in the
management of industries”.
Explanation:
 It was added to the Constitution by the 42nd
The 25th Amendment Act and Amendment Act 1976.
Kesavananda Bharati case
19. Correct Option: (c)
 The 25th Amendment Act inserted a new
Article 31C which contained the following Explanation:
two provisions:
97th Amendment Act, 2011
! No law which seeks to implement
the socialistic Directive Principles  It amended Article 19(1)(c) by adding the
specified in Article 39 (b) and (c) shall word “co-operatives”.
be void on the ground of contravention  It added a new Directive Principle (Article
of the Fundamental Rights conferred 43B). relating to co-operative societies. It
by Article 14, Article 19 or Article 31 directs the state to promote voluntary
(later repealed) formation, autonomous functioning,
! No law containing a declaration for democratic control and professional
giving effect to such policy shall be management of co-operative societies.
questioned in any court on the ground  It inserted Part IXB into the Constitution
that it does not give effect to such a explaining definitions, subjects of laws,
policy. terms, and conditions, etc of the co-
 But, in the Kesavananda Bharati case operative societies.
(1973), the Supreme Court held the
above second provision of Article 31C as 20. Correct Option: (c)
unconstitutional and invalid citing Judicial
Explanation:
Review as a “basic feature”.
 However, the above first provision of Environment Protection under
Article 31C was held to be constitutional Constitutional Framework of India
and valid.
 The specific provisions on environment
 Thus, the Parliament can implement protection can be found in the
Articles 39(b) and 39(c) of the Directive Constitution.
TARGET PT 2020 31

 Article 21 guarantees the fundamental a specific focus on quality as enumerated in


Right to Life and Personal L. The right to the UGC “Quality Mandate”.
clean environment, free from the danger of
diseases and infection is inherent in it. 22. Correct Option: (b)
 The right to a healthy environment is an Explanation:
important attribute of the right to live with
human dignity. E-auction for forest produce
 The right to live in a healthy environment
 YES Bank has conducted the first e-auction
as part of Article 21 of the Constitution was
of forest produce in Dhanora Mahasangh
recognized in the case of Rural Litigation
and Entitlement Kendra vs. State in in Maharashtra, something it will now aim
1988 (Popularly known as Dehradun to replicate in other states.
Quarrying Case).
23. Correct Option: (b)
 In M.C. Mehta vs. Union of India, 1987,
the Supreme Court treated the right to live Explanation:
in a pollution-free environment as a part of
the fundamental right to life under Article Aruna Sundararajan Committee:
21 of the Constitution.
 Union Telecom Minister has formed Aruna
 Article 48A of the constitution says that Sundararajan Committee to spell out
“the state shall endeavor to protect and revival plans for the Telecom sector
improve the environment and to safeguard
the forests and wildlife of the country”.
24. Correct Option: (a)
 Article 51-A (g), says that “It shall be the
duty of every citizen of India to protect and Explanation:
improve the natural environment including
forests, lakes, rivers, and wildlife and to Aid to Nepal
have compassion for living creatures.”  India extended a financial aid of 1.6 billion
Nepalese rupees to Nepal to help 50,000
21. Correct Option: (a) people in Nuwakot and Gorkha districts
Explanation: to rebuild their houses damaged in the
devastating earthquake in 2015.
Paramarsh Scheme:
 Recently, the Ministry for Human Resource 25. Correct Option: (c)
Development has launched ‘Paramarsh’-
a University Grants Commission (UGC) Explanation:
scheme for Mentoring Institutions
LoC to Ghana and Mozambique
aspiring to achieve National Accreditation
and Assessment Council (NAAC)  Export-Import Bank of India (Exim Bank)
Accreditation. will provide a line of credit worth USD
 The scheme will target 1000 Higher 245 million to Ghana and Mozambique for
Education Institutions for mentoring with agricultural and railway projects.

**********
32 TARGET PT 2020

TEST
DAY - 4

Time Allowed: 30 mins Maximum Marks: 50

1. Which of the following can sit in both 3. Project Monitoring Group which tracks
Houses of the Parliament? stalled investment projects is also
1. Comptroller and Auditor General of under its jurisdiction.
India
Which of the above statements is/are
2. Attorney General of India correct?
3. Prime Minister (a) 1 and 2 only
4. Cabinet Ministers
(b) 2 and 3 only
Select the correct option using the codes (c) 3 only
given below:
(d) 1, 2, and 3
(a) 2 and 3 only
(b) 3 only 4. Consider the following statements:
1. The minister is responsible and liable
(c) 2, 3, and 4 only
in the court for discharge of his/her
(d) 1, 2, and 4 only duties.

2. Which of the following do(es) not take 2. Deputy ministers are not members of
part in the presidential election? the cabinet and do not attend cabinet
meetings.
1. Nominated Anglo-Indian member from
Uttar Pradesh Assembly 3. Principal Secretary to the Prime
Minister is the ex-officio head of the
2. Elected members from Puducherry
Civil Services Board.
Assembly
3. Nominated members of Rajya Sabha 4. Only IAS can become the Principal
Secretary to the Prime Minister.
Select the correct option using the codes
given below: Which of the above statements is/are
correct?
(a) 2 only
(a) 2 only
(b) 1 and 2 only
(b) 1 and 3 only
(c) 1 and 3 only
(c) 1, 3, and 4 only
(d) None of the above
(d) 2, 3, and 4 only
3. Consider the following statements:
5. Given below are two statements, one is
1. Prime Minister’s Office(PMO) was
labeled as Assertion (A) and the other
established in 1977.
is as Reason(R).
2. Atomic Energy Regulatory Board
(AERB) comes under the jurisdiction of A. The president can dismiss the Prime
the PMO. Minister at any time.
TARGET PT 2020 33

R. The Prime minister holds office during (a) 1 only


the pleasure of the President.
(b) 1 and 2 only
Select the correct answer from the codes (c) 3 only
given below:
(d) 1, 2, and 3
(a) Both A and R are true and R is the
correct explanation of A. 9. The security money to be paid by the
(b) Both A and R are true but, R is not the Presidential and Vice-presidential
correct explanation of A. candidates are deposited in which of
the following banks?
(c) A is true, but R is false.
1. Reserve Bank of India
(d) A is false, but R is true.
2. State Bank of India
6. Which of the following statements is/ 3. Central Bank of India
are incorrect?
4. Government Treasury
1. The Constitution does not require that
a person must prove his majority in Select the correct option using the codes
the Lok Sabha before he is appointed given below:
as the Prime Minister.
(a) 1 only
2. The Prime Minister has to be a member
of any of the two Houses of parliament (b) 1 and 4 only
at the time of appointment. (c) 2 and 4 only
Select the correct option using the codes (d) 1, 2, 3 and 4
given below:
10. The word “cabinet ” is mentioned in
(a) 1 only which of the following Articles of the
(b) 2 only Indian Constitution?
(c) Both 1 and 2 (a) Article 75
(d) Neither 1 nor 2 (b) Article 78
(c) Article 352
7. Which of the following Amendment Acts
made it mandatary to the President to (d) Nowhere in the Constitution
be bound by the advice of the council
of ministers? 11. Given below are two statements, one is
labeled as Assertion (A) and the other
(a) 24th Amendment Act
is as Reason(R).
(b) 25th Amendment Act
A. In case of the vacancy in the president’s
(c) 42nd Amendment Act office, the Vice-President becomes the
(d) 44th Amendment Act President.

8. Which of the following statements is/ R. The office of Vice-President has been
are correct? created to maintain the political
continuity of the Indian State.
1. The state legislature cannot override
the veto power of the President. Select the correct answer from the codes
2. The 42nd Constitutional Amendment given below:
Act made it obligatory for the President (a) Both A and R are true and R is the
to give his assent to a constitutional
correct explanation of A.
amendment bill.
(b) Both A and R are true but, R is not the
3. Even the use of “Pocket Veto” is not the
correct explanation of A.
discretionary power of the President.
(c) A is true, but R is false.
Select the correct option using the codes
given below: (d) A is false, but R is true.
34 TARGET PT 2020

12. Given below are two statements, one is 3. The nominated members of either
labeled as Assertion (A) and the other House of Parliament can participate in
is as Reason(R). this process.
A. The President is the ex officio member of Which of the above statements is/are
the Parliament.
correct?
R. The President is a part of the (a) 1 only
Parliament.
(b) 3 only
Select the correct answer from the codes
given below: (c) 2 and 3 only

(a) Both A and R are true and R is the (d) 1, 2, and 3


correct explanation of A.
16. Which of the following groups
(b) Both A and R are true but, R is not the
participate(s) in the election of Vice-
correct explanation of A.
President?
(c) A is true, but R is false.
1. Elected members of Lok Sabha
(d) A is false, but R is true.
2. Nominated members of Lok Sabha
13. Consider the following statement: 3. Elected members of Legislative
1. The President has no constitutional Assemblies
discretion. 4. Nominated members of Rajya Sabha
2. He can appoint the Prime Minister
when no single party attains majority Select the correct answer using the code
after elections to the Lok Sabha. given below:

Which of the above statements is/are (a) 1 only


correct? (b) 1, 2, and 4 only
(a) 1 only
(c) 2, 3, 4 only
(b) 2 only
(d) 1, 2, 3 and 4
(c) Both 1 and 2
(d) Neither 1 nor 2 17. The Supreme Court has examined
the pardoning power of the President
14. With reference to the functions of the under different cases and laid down
Prime Minister, which of the following various principles. In this context,
statements is correct? which of the following statements are
(a) The salaries and allowances of correct?
Ministers are decided by him. 1. The petitioner for mercy has no right
(b) He advises the President to dismiss to an oral hearing by the President.
the minister in case of difference of
2. The power is to be exercised by the
opinion.
President on the advice of the Union
(c) He advises President for the Cabinet.
appointment of Speaker.
3. The pardoning power does not come
(d) He allocates funds to each Ministry. under judicial review.
15. Consider the following statements Select the correct answer using the code
regarding the removal of President:
given below:
1. The President can be removed from
office by a process of impeachment for (a) 1 and 2 only
‘proved misbehavior and incapacity’. (b) 2 and 3 only
2. The impeachment charges against
(c) 1 and 3 only
him can be initiated only by the Lower
House. (d) 1, 2 and 3
TARGET PT 2020 35

18. Consider the following statements: (c) 2 and 3 only


1. The Council of Ministers exists even (d) 1, 2 and 3
after the dissolution of Lok Sabha.
2. The advice tendered to the president 21. Which country recently became the
by the Council of Ministers does not Deputy Chair of the general body of the
come under judicial review. Parliamentary Union of Organisation
of Islamic Cooperation (OIC)?
Which of the above statements is/are
(a) Afghanistan
correct?
(b) Pakistan
(a) 1 only
(c) Kingdom of Saudi Arabia
(b) 2 only
(d) Qatar
(c) Both 1 and 2
(d) Neither 1 nor 2 22. A joint military exercise ‘Bold
Kurukshetra–2019’ is conducted
19. Which of the following statements are between:
incorrect?
(a) India and Indonesia
1. The Union Ministers are responsible to
the President only. (b) India and Nepal

2. The Prime Minister is appointed by the (c) India and Singapore


President but other ministers are by (d) India and Bhutan
the Prime Minister himself.
3. Parliamentary secretaries are the part 23. Door-step banking facility named
of the Council ministries. “Bank on Wheels” has been introduced
by which of the following bank?
Select the correct option using the codes
(a) India Bank
given below:
(b) Union Bank of India
(a) 1 only
(c) Indian Overseas Bank
(b) 2 only
(d) Punjab National Bank
(c) 1 and 2 only
(d) 1, 2, and 3 24. “Momo-3”, the first privately developed
rocket, belongs to which country?
20. Consider the following statements:
(a) China
1. The highest law officer of the
Government of India is appointed by (b) Japan
the Parliament. (c) Taiwan
2. He/she is responsible to the (d) South Korea
Parliament.
3. He/she must represent Government in 25. Amasebailu, recently in news is a:
Courts. (a) Festival of Karnataka
Which of the above statements is/are (b) Karnataka’s first solar-powered Gram
incorrect? Panchayat
(a) 1 only (c) Faction of Lingayat
(b) 1 and 2 only (d) Karnataka’s first Organic Block

**********
36 TARGET PT 2020

ANSWER HINTS
DAY - 4

1. Correct Option: (c)  The PMO provides secretarial assistance


to the Prime Minister. It is headed by the
Explanation: Principal Secretary to the Prime Minister.
 Council of Ministers including the  The office houses the Prime Minister
Prime Minister is responsible for and few selected officers of Indian Civil
Lok Sabha in particular & to the Service who work with him to manage and
Parliament in general. So they are coordinate government and his office. It is
entitled to be present in both Houses located at the South Block of the Secretariat
proceeding if he is deemed to do so. But Building.
she/he cannot vote in the House of which
 The PMO was originally called the
she/he is not the member.
Prime Minister’s Secretariat which
 Attorney-General too can sit in both was established in 1947
Houses but cannot vote in any of the  In 1977 it was renamed as PMO during
Houses. the Morarji Desai ministry.
 CAG does not sit in the Parliament but  AERB, BARC, UCIL, TIFR and its sister
his report is laid by the President on the institutes, Department of Space including
floor of the Houses. ISRO, etc are under its jurisdiction.
 Project Monitoring Group was created
2. Correct Option: (a)
to track stalled investment projects, both
Explanation: in the public and private sectors and to
remove the implementation bottlenecks. It
Presidential election was established in 2013 under Cabinet
Secretariat but it was removed into
 The President is elected not directly by
the PMO in 2014.
the people but by members of the electoral
college consisting of:
4. Correct Option: (c)
! the elected members of both the
Houses of Parliament;
Explanation:

! the elected members of the legislative Political and permanent executive


assemblies of the states; and  The Principal Secretary to the Prime
! the elected members of the Minister is generally a civil servant,
legislative assemblies of the commonly from the Indian Administrative
Union Territories of Delhi and Service and occasionally from the
Puducherry Indian Foreign Service (for example, P N
Haksar in the Indira Gandhi government.).
 The nominated members of the He/She is the senior-most bureaucrat in,
Parliament take part in the and the administrative head of, the Prime
impeachment process but not in the Minister’s Office.
election process.
 The head of the Civil Services Board is
the Cabinet Secretary who is also head
3. Correct Option: (b) of the Cabinet Secretariat, the Indian
Explanation: Administrative Service, and all civil
services under the rules of business of the
PMO government.
TARGET PT 2020 37

 Article 74 provides for a council of ministers a person must prove his majority in
with the Prime Minister at the head to aid the Lok Sabha before he is appointed
and advise the President in the exercise of as the Prime Minister. The President
his functions. may first appoint him the Prime Minister
and then ask him to prove his majority in
 In India, unlike in Britain, there is no
the Lok Sabha within a reasonable period.
provision in the Constitution for the
system of legal responsibility of a
7. Correct Option: (c)
minister.
Explanation:
 Moreover, the courts are barred from
inquiring into the nature of advice rendered Constitutional position of the President
by the ministers to the president.
 The Constitution of India has provided
 Deputy Ministers are not given independent for a parliamentary form of government.
charge of ministries/departments. They According to Dr. Ambedkar, the President
are attached to the cabinet ministers or is the head of the State but not of the
ministers of state and assist them in their Executive. He represents the nation but
does not rule the nation. He is the symbol
administrative, political, and parliamentary
of the nation. His place in administration
duties. They are not members of the is that of a ceremonial device or a seal
cabinet and do not attend cabinet by which the nation’s decisions are made
meetings. known.

5. Correct Option: (d)  There shall be a council of ministers with


the Prime Minister at the head to aid and
Explanation: advise the President who ‘shall’, in the
exercise of his functions, act in accordance
Office of Prime Minister with such advice. This mandatary provision
(‘shall’) was added to the Article 74 by 42nd
 The term of the Prime Minister is not
Amendment Act, 1976.
fixed and he holds office during the
pleasure of the president.  The President may return a matter once
for reconsideration of his ministers, but
 However, this does not mean that the the reconsidered advice shall be binding.
president can dismiss the Prime This provision was added by the 44th
Minister at any time. So long as the Amendment Act,1978.
Prime Minister enjoys the majority support
in the Lok Sabha, he cannot be dismissed 8. Correct Option: (a)
by the President. Explanation:
 However, if he loses the confidence of the
Powers of President
Lok Sabha, he must resign or the President
can dismiss him.  The President can neither ratify nor rejects
nor return the bill, but simply keeps the bill
6. Correct Option: (c) pending for an indefinite period. It is called
Pocket Veto and is the discretionary
Explanation: power of the President.

Office of Prime Minister  The President has no veto power in respect


of a constitutional amendment bill. The 24th
 Constitutionally, the Prime Minister may Constitutional Amendment Act of 1971
be a member of any of the two Houses of made it obligatory for the President to give
parliament. But in 1997, the Supreme his assent to a constitutional amendment
Court held that a person who is not a bill.
member of either House of Parliament  In case of the state bills, the President may
can be appointed as Prime Minister direct the governor to return the bill (if it
for six months, within which, he should is not a money bill) for the reconsideration
become a member of either House of of the state legislature. And if it is passed
Parliament; otherwise, he ceases to be the again, the President is not bound to give
Prime Minister. his assent to the bill. This means that
the state legislature, unlike Parliament,
 In 1980, the Delhi High Court held that cannot override the veto power of the
the Constitution does not require that President.
38 TARGET PT 2020

9. Correct Option: (b) 13. Correct Option: (c)


Explanation: Explanation:

Security deposit Situational discretionary powers of


president
 The Security Deposit for the election of
President and Vice-President of Rs.15000/-  As per Article 74 President is bound by the
advice of the council of ministers headed
is deposited either in cash with the
by the Prime Minister. Though he has
Returning Officer or a receipt showing
no constitutional discretion, he has some
that the amount has been deposited by the
situational discretion. In other words, the
candidate or on his behalf in the Reserve President can act on his discretion (that is,
Bank of India or in a Government without the advice of the ministers) under
Treasury. the following situations:
 The security deposit is liable to be forfeited ! Appointment of Prime Minister when
in case the candidate fails to secure one- no party has a clear majority in the
sixth of the votes polled. Lok Sabha or when the Prime Minister
in office dies suddenly and there is no
10. Correct Option: (c) obvious successor.

Explanation: ! Dismissal of the council of ministers


when it cannot prove the confidence in
Cabinet the Lok Sabha.

 Cabinet is the supreme policy-making, ! Dissolution of the Lok Sabha if the


council of ministers has lost its
highest decision making & supreme
majority.
executive body of the central government.
 Indian constitution didn’t mention it 14. Correct Option: (b)
originally but inserted in Article 352 by
Explanation:
the 44th Amendment Act, 1978.
Functions of the Prime Minister
11. Correct Option: (d)
 The Prime Minister enjoys the following
Explanation: powers as head of the Union Council of
Ministers:
Vice-President
! He recommends persons who can
 Unlike American Vice-President, the be appointed as ministers by the
Indian Vice-President does not assume President. The President can appoint
the office of the President when it falls only those persons as ministers
vacant for the unexpired term. He merely who are recommended by the Prime
Minister.
serves as an acting President until the
new President assumes charge. ! He allocates and reshuffles various
portfolios among the ministers.
 The Constitution has not assigned any
significant function to the Vice-President ! He presides over the meeting of the
in that capacity. Hence, some scholars call council of ministers and influences its
him ‘His Superfluous Highness’. decisions.

 This office was created with a view to


! He guides, directs, controls, and
coordinates the activities of all the
maintain the political continuity of the
ministers.
Indian State.
! He can bring about the collapse of the
 The Constitution (Article 64) has provided council of ministers by resigning from
the Vice-President as the ex officio office.
Chairman of Rajya Sabha.
 Article 75(6): The salaries and
allowances of Ministers shall be such
12. Correct Option: (d)
as Parliament may from time to time
Explanation: by law determine and, until Parliament
so determines, shall be as specified in the
President Second Schedule.
TARGET PT 2020 39

 The Speaker is elected by the Lok ! The petitioner for mercy has no right
Sabha from amongst its members; to an oral hearing by the President.
President has no role to play.
! The President can examine the
 Funds are granted by Parliament to evidence afresh and take a view
each ministry, Prime Minster does not different from the view taken by the
play an exclusive role in this context. court.

15. Correct Option: (b) ! The power is to be exercised by the


President on the advice of the union
Explanation: cabinet.

Removal of President ! The President is not bound to give


reasons for his order.
 The President may be removed from his
office before the expiry of his term, only for ! Where the earlier petition for mercy
the “violation of the Constitution” by has been rejected by the President,
the process of impeachment. stay cannot be obtained by filing
another petition.
 For impeachment, a charge against him
may be initiated by either house of  The exercise of power by the President
Parliament. The resolution can be moved is not immune to judicial review. The
only after giving at least 14 days written Supreme Court in the Epuru Sudhakar
notice signed by not less than 1/4th of the case has given a small window for judicial
total number of members of the House. review of the pardon powers of President
and governors for the purpose of ruling out
 The resolution must be passed by a
any arbitrariness.
majority of not less than 2/3rd of the total
membership of the House. The charge is
18. Correct Option: (c)
then investigated by the other house.
 The President has the right to appear and Explanation:
to be represented at such investigation. If
Article 74
the majority declaring that the charge is
proved, such resolution majority declaring  Article 74(1) states that there shall (must)
that the charge is proved, such resolution be a Council of Ministers with the Prime
shall have the effect of removing the Minister at the head to aid and advise the
President from his office from the date on President who shall, in the exercise of his
which the resolution is so passed. functions. This means that the Council of
Ministers exists even after the dissolution
 All the members of both Houses
participate in the impeachment process. of Lok Sabha.
 Article 74(2) says that the advice
16. Correct Option: (a) tendered to the president cannot be
inquired into any court, thus it does not
Explanation:
come under judicial review.
Elections of Vice-President
19. Correct Option: (c)
 Vice President is elected indirectly by the
Electoral College, consisting of all the Explanation:
Members of Parliament (both elected
& nominated). Council of Ministers

 State Legislature do not participate in  Article 75(1) states that the Prime Minister
Vice-President’s election. shall be appointed by the President and
the other Ministers shall be appointed
17. Correct Option: (a) by the President on the advice of the
Prime Minister.
Explanation:
 Article 75(2) states that the Ministers shall
Pardoning Power of the President hold office during the pleasure of the
President.
 Article 72 of the Indian Constitution
empowers the President to grant pardon.  Article 75(3) states that the Council of
Ministers shall be collectively responsible
 The Supreme Court examined the
to the House of the People.
pardoning power of the President under
different cases and laid down the following  Parliamentary secretaries the
principles: members of the last category of the
40 TARGET PT 2020

council of ministers. However, since 23. Correct Option: (c)


1967, no parliamentary secretaries have
been appointed except during the first Explanation:
phase of Rajiv Gandhi Government. Bank on Wheels

20. Correct Option: (d)  Indian Overseas Bank (IOB) announced


that it has launched ‘Bank on Wheels’
Explanation: facility in 14 of its lead districts in Tamil
Nadu and Kerala, besides Vijayawada.
Attorney General of India
 The bank said the mobile van facility will
 Article 76 says that the President shall enable the public, especially senior citizens,
appoint a person who is qualified to be conveniently get door-step banking facility
appointed a Judge of the Supreme Court to available at identified locations of the lead
be Attorney General for India. districts
 The Attorney General shall hold office  A dedicated banking correspondent will
during the pleasure of the President i.e. he/ accompany the vehicle with a micro ATM
she is responsible to the President. inside the van to take care of services
such as account opening, enrollment of
 Attorney General has the right of customers in Social Security Scheme,
audience in all courts in the territory of passbook printing, and other financial
India, but he/she can also send Solicitor inclusion activities.
General of India or Additional
Solicitor General to represent the 24. Correct Option: (b)
Government.
Explanation:
21. Correct Option: (b)
Momo-3
Explanation:  A Japanese aerospace startup
successfully launched a small rocket on
Parliamentary Union of Organisation of
Saturday, making it the country’s first
Islamic Cooperation (OIC)
privately developed model to reach outer
 Pakistan has been elected as the deputy space.
chairman of the general body of the
 Interstellar Technology Inc. said the
Parliamentary Union of Organisation of unmanned Momo-3 launched from its test
Islamic Cooperation (OIC), an influential site in Hokkaido and reached about 110
grouping of 57 countries, majority of which kilometers (68 miles) in altitude before
are Muslim-dominated. falling into the Pacific Ocean after 10
 According to an official announcement, minutes of flight time.
Pakistan was elected deputy
chairman of the general body of 25. Correct Option: (b)
the Parliamentary Union of the Explanation:
OIC member States (PUIC) in Rabat,
Morocco Amasebailu
 Amasebailu in Kundapura Taluk of Udupi
22. Correct Option: (c) district has become Karnataka’s first
Explanation: solar-powered Gram Panchayat.
 Over 1800 houses were provided solar
Bold Kurukshetra–2019 lamps at a total cost of 2.13 crore rupees.
 Singapore Armed Forces (SAF) and  The project was funded by Centre and
the Indian Army joined hands to conduct State Government in the ratio of 30:20 and
a three-day joint exercise in India between the rest was paid by the Panchayat and
April 8 and April 11. charitable organizations.
 Named as ‘Bold Kurukshetra 2019,’ the  Amasebailu Charitable Trust President A
joint exercise is being conducted to develop G Kodgi has said that the solar lamps have
military technology, enhance the maritime a guarantee of five years and any faulty
security and bolster the nations’ fight lamps will be replaced by the supplier
against terrorism. within the guarantee period.

**********
TARGET PT 2020 41

TEST
DAY - 5

Time Allowed: 30 mins Maximum Marks: 50

1. Consider the following statements 2. The Constitution provides that the


regarding the Question Hour in the maximum strength of the Lower House
Parliament: to be 550 elected members.
1. A starred question requires an oral 3. If a Cabinet Minister is from the Rajya
answer and supplementary questions Sabha, he shall be the Leader of the
cannot follow. House.
2. An unstarred question requires a 4. Prime Minister is always the leader of
written answer and supplementary the Lok Sabha.
questions cannot follow.
Select the correct option using codes given
Which of the above statements is/are below:
correct? (a) 1, 2 and 4 only
(a) 1 only (b) 1, 2, and 3 only
(b) 2 only (c) 3 and 4 only
(c) Both 1 and 2 (d) 1, 2, 3, and 4
(d) Neither 1 nor 2
4. Which of the following statements
regarding the prorogation of the house
2. Which of the following statements is/
is/are correct?
are incorrect?
1. Prorogation terminates both the sitting
1. The Constitution has fixed the term of
as well as the session of the House.
office of members of the Rajya Sabha
to six years. 2. Prorogation of the house is done by the
presiding officer.
2. The Rajya Sabha was first constituted
on 26 January 1950. Select the correct answer using the code
3. Allocation of seats in Rajya Sabha is given below:
similar to the Senate of the USA. (a) 1 only
Select the correct option using codes given (b) 2 only
below: (c) Both 1 and 2
(a) 3 only (d) Neither 1 nor 2
(b) 1 and 2 only
5. Consider the following statements
(c) 1 and 3 only regarding the Contingency Fund of
(d) 1, 2, and 3 India:
1. According to the Indian constitution, the
3. Which of the following statements are President can establish a Contingency
correct? Fund of India.
1. The origin of the Lok Sabha traces back 2. Contingency Fund of India is placed at
to the Charter Act of 1853. the disposal of the Parliament.
42 TARGET PT 2020

Which of the above statements is/are 9. Which of the following statements is/
incorrect? are correct?
(a) 1 only 1. The Chairman of Rajya Sabha presides
all the committees of the House.
(b) 2 only
2. Unlike the Speaker of Lok Sabha, the
(c) Both 1 and 2 Chairman of the Rajya Sabha does not
(d) Neither 1 nor 2 have a right to cast a vote.

6. Which of the following statements is/ Select the correct option using codes given
are correct? below:
1. There is no provision for a joint sitting (a) 1 only
in case of a deadlock over a Constitution (b) 2 only
Amendment Bill.
(c) Both 1 and 2
2. In the history of Parliament of India,
there have been only three occasions of (d) Neither 1 nor 2
the Joint Sitting of the Parliament.
10. Which of the following statements are
3. Rajya Sabha can neither introduce nor incorrect?
amend the Money Bills.
1. Deputy Speaker can decide whether a
Select the correct option using codes given bill is a money bill or not.
below: 2. Deputy Speaker alike Speaker can only
(a) 1 only exercise a casting vote in the event of a tie.
3. In the order of precedence, the Speaker
(b) 2 and 3 only
has a higher rank than all cabinet
(c) 1 and 3 only ministers, except the Prime Minister.
(d) 1, 2, and 3 4. Speaker can be removed by the Lok
Sabha by a resolution passed by an
7. Which of the following statements is/ absolute majority.
are correct?
Select the correct option using codes given
1. The office of the Speaker of Lok Sabha
below:
is responsible to the Parliament.
2. The Deputy Speaker is responsible to (a) 1 and 4 only
the Speaker. (b) 2 and 4 only
3. The term of Office of the Speaker ends (c) 1, 2 and 4 only
as soon as Lok Sabha gets dissolved.
(d) 1 and 3 only
4. Only Speaker or the Deputy Speaker
can preside over the Lok Sabha. 11. Which of the following statements
regarding parliamentary proceeding
Select the correct option using codes given is/are incorrect?
below:
1. The first hour of every parliamentary
(a) 1 only sitting is slotted for Zero Hour.
(b) 1, 2 and 4 only 2. Unlike the question hour, the zero
(c) 2 and 4 only hour is mentioned in the Rules of
Procedure.
(d) 1, 2, 3, and 4
3. A Bill becomes an Act as soon as passed
by both the Houses of Parliament.
8. Which of the following Presidents
of India was also a Speaker of Lok Select the correct option using codes given
Sabha? below:
(a) Neelam Sanjiva Reddy (a) 1 only
(b) Varahagiri Venkata Giri (b) 2 only
(c) Giani Zail Singh (c) 1 and 2 only
(d) None of the above (d) 1, 2 and 3
TARGET PT 2020 43

12. Which of the following statements Select the correct answer using the code
regarding the ‘Leader of the Opposition’ given below:
is/are correct? (a) 1 only
1. The Leader of the Opposition (LoP) is a (b) 1 and 2 only
statutory position in India.
(c) 1 and 3 only
2. The 1977 Act mandates that the Leader
of the Opposition to the Government (d) 2 and 3 only
must be from that party which have at
15. Which of the following statements
least one-tenth of the total number of regarding parliamentary proceeding
members of the House”. is/are correct?
3. The first LoP in Rajya Sabha was 1. Censure Motion can be moved against
formed in 1969. the entire council of ministers.
4. The Lok Sabha of India was without 2. If it is passed in the Lok Sabha, the
LoP during 1980-89. council of ministers must resign from
office.
Select the correct option using codes given
below: Select the correct option using codes given
below:
(a) 1, 2, and 3 only
(a) 1 only
(b) 1, 2, and 4 only
(b) 2 only
(c) 3 and 4 only
(c) Both 1 and 2
(d) 1,3, and 4 (d) Neither 1 nor 2
13. Consider the following statements: 16. Which of the following does not
1. Confidence motion can be moved in imply the Judicial Powers of the
both the Houses whereas, the no- Parliament?
confidence motion can only be moved (a) Impeachment of President
in Lok Sabha.
(b) Punishment for its contempt
2. Parliament cannot make laws on the
(c) Recommendation of the removal of
state list except in emergencies.
judges
3. Certain Financial Bills also cannot be
(d) Ratification of the pardoning by the
introduced in Rajya Sabha but it can
President
reject or amend the Bills.
17. With regard to enactment of Budget,
Which of the above statements is correct? consider the following statements:
(a) 3 only 1. The budget shall distinguish
expenditure on revenue account from
(b) 2 and 3 only
other expenditure.
(c) 1 and 2 only 2. Parliament can reduce or increase a
(d) 1 only tax but cannot abolish it.
3. No demand for a grant shall be made
14. Which of the following constitutes the except on the recommendation of the
special powers of the Rajya Sabha? Speaker.
1. It can authorize the Parliament to
Which of the above statements given above
create new All-India Services common
is/are correct?
to both the Centre and states.
(a) 1 and 2 only
2. It can authorize the Parliament to
make a law on a subject enumerated in (b) 1 only
the State List and Concurrent List. (c) 2 and 3 only
3. Removal of the Vice-President. (d) None of the above
44 TARGET PT 2020

18. Which of the following factors have 21. Gottiprolu, recently excavated by
led to the decline of the Indian the Archeological Survey of India, is
Parliament? situated on the bank of which river?
1. Frequent promulgation of ordinances. (a) Cauvery
2. Lack of strong and steady opposition in
(b) Narmada
the Parliament
3. Frequent amendment of the (c) Swarnamukhi
Constitution. (d) Krishna
4. Large size of the Parliament
22. Recently, the Shodh Shuddh initiative
Select the correct answer using the code is in news. It is____
given below: (a) Anti-corruption initiative
(a) 1, 2, and 3 only
(b) Program to give protection to
(b) 2, 3, and 4 only whistleblowers
(c) 1, 2, and 4 only (c) Plagiarism Detention Software
(d) 1, 2, 3, and 4 (d) Cleanliness program by Indian
Railways
19. When a Cut Motion states that the
amount of the demand be reduced by 23. Tatpar App is launched by___
Rs 1, it is called as___
(a) Central Information Commission
(a) Economy Cut Motion
(b) Delhi Police
(b) Token Cut Motion
(c) Central Bureau of Investigation
(c) Policy Cut Motion
(d) Department of Telecommunications
(d) None of the above
24. Which of the following countries is not
20. Which of the following statements
a member of the Shanghai Cooperation
regarding ‘delegated legislation’ is/are
incorrect? Organization?

1. It is in conflict with the ‘parliamentary (a) Uzbekistan


democracy’. (b) India
2. The rules made by this process need
(c) Turkmenistan
not to be laid in the Parliament.
3. It is not under the purview of judicial (d) Tajikistan
review.
25. Pegasus, which was recently in the
Select the correct option using codes given news, is a_____
below: (a) Extinct bird declared by IUCN
(a) 2 only (b) Spyware
(b) 3 only (c) A newly found disease in Central
(c) 1 and 3 only Africa
(d) 2 and 3 only (d) A protoplanet

**********
TARGET PT 2020 45

ANSWER HINTS
DAY - 5

1. Correct Option: (b)  The Constitution has not fixed the term
of office of members of the Rajya Sabha
Explanation: and left it to the Parliament. Subsequently,
Question Hour the Parliament in the Representation of
the People Act (1951) provided that
 The first hour of every parliamentary the term of office of a member of the
sitting is slotted for this. During this
Rajya Sabha shall be six years.
time, the members ask questions and the
ministers usually give answers.  The Fourth Schedule of the Constitution
 The questions are of three kinds, namely, deals with the allocation of seats in the
starred, unstarred and short notice. Rajya Sabha to the states and union
territories. The allocation is based on
! A starred question (distinguished the population and varies from 1 to 31
by an asterisk) requires an oral whereas in the USA it is equal (2) from
answer and hence supplementary
each state.
questions can follow.
! An unstarred question, on the 3. Correct Option: (b)
other hand, requires a written
answer and hence, supplementary Explanation:
questions cannot follow.
Parliament
! A short notice question is one that is
asked by giving a notice of less than  Parliament is the supreme legislative body
ten days. It is answered orally. of India.
 In addition to the ministers, the questions  The origin of Rajya Sabha can be traced
can also be asked to the private members. back to 1919 when in pursuance to the
Thus, a question may be addressed to a Government of India Act, 1919, a second
private member if the subject matter of the chamber known as the Council of States
question relates to some Bill, resolution or was created whereas, the origin of the
other matter connected with the business Lok Sabha traces back to the Charter
of the House for which that member is Act of 1853, when for the first time a
responsible. The procedure in regard
12 member Legislative Council was
to such a question is the same as that
established.
followed in the case of questions addressed
to a minister.  The Constitution (Article 81) provides
 The list of starred, unstarred, short notice that the maximum strength of the House
questions and questions to private members to be 552 (550 elected) members - 530
are printed in green, white, light pink and members to represent the States,
yellow colour, respectively, to distinguish 20 members to represent the Union
them from one another. Territories, and 2 members to be
nominated by the President from the
2. Correct Option: (d) Anglo-Indian Community. At present, the
Explanation: strength of the House is 545 members.
 The senior-most Minister, who is a
Rajya Sabha member of Rajya Sabha, is appointed
 The Rajya Sabha was first constituted in by the Prime Minister as Leader of the
April 1952 after the first general election. House.
46 TARGET PT 2020

 Prime Minister is the Leader of the  It is kept under the Finance Secretary on
House if he/she is a member of the behalf of the President.
House. For instance, if he/she is a member  Each state in India has its own consolidated
of the Rajya Sabha then he shall be the and contingency funds.
Leader of the Rajya Sabha but not Lok
Sabha. 6. Correct Option: (d)
4. Correct Option: (a) Explanation:

Explanation: Joint Sitting of the Houses

Prorogation  A joint sitting of the Houses is convened for


any deadlock between the two Houses over
 Prorogation means the termination a Bill.
of a session of the House by an order
 In the case of Money Bills, there is no
made by the President under article
question of a deadlock as the Rajya Sabha
85(2)(a) of the Constitution. has a limited say in such matters. Only
 Prorogation terminates both the Lok Sabha can introduce the Money
sitting and session of the House. Bills. Rajya Sabha has to return the
Money Bill to Lok Sabha within a period
 Usually, within a few days after the House of fourteen days from its receipt. Rajya
is adjourned sine die by the presiding Sabha cannot amend the Money
officer, the President issues a notification Bill directly; it can only recommend
for the prorogation of the session. However, amendments to the Bill and Lok Sabha
the President can also prorogue the House may either accept or reject all or any
while in session. of the recommendations made by the
Rajya Sabha.
 The period spanning between the
prorogation of a House and its reassembly  There is no provision for a joint
in a new session is called ‘recess’. sitting in case of a deadlock over a
Constitution Amendment Bill.
5. Correct Option: (c)  In the history of Parliament of India, there
Explanation: have been three occasions when both
Houses of Parliament held a joint sitting
Types of Funds of Indian Government to resolve deadlock on Bills between them,
i.e.,
 The Constitution of India provides for
the following three kinds of funds for the ! 6 and 9 May 1961 on the Dowry
Central government: Prohibition Bill, 1959;
! Consolidated Fund of India (Article ! 17 May 1978 on the Banking Service
266) Commission (Repeal) Bill, 1977; and
! Public Account of India (Article 266) ! 26 March 2002 on the Prevention of
Terrorism Bill, 2002.
! Contingency Fund of India (Article
267)
7. Correct Option: (a)
Contingency Fund of India Explanation:
 According to the Article 267(I) of the Indian
Presiding officers of Lok Sabha
constitution, Parliament may by law
establish a Contingency Fund in the  The Speaker and the Deputy Speaker are
nature of an imprest into which shall be the Presiding Officers of the Lok Sabha.
paid from time to time such sums as may But, When the Offices of both the Speaker
be determined by law. and the Deputy Speaker fall vacant, the
duties of the Office of the Speaker are
 Contingency Fund shall be placed at
performed by any member from the
the disposal of the President to enable
Panel of Chairpersons, as elected by the
advances to be made by him for the purposes
House.
of meeting unforeseen expenditure.
 President may appoint Speaker pro
 However, authorization of parliament
tem for this purpose.
is needed to recharge this fund from the
consolidated fund by law under Article 115  The Speaker continues in office till
or Article 116. immediately before the first meeting of Lok
TARGET PT 2020 47

Sabha after dissolution of the one to which total strength excluding vacancies) of the
he/she was elected, unless he/she ceases House.
to be a Member by any of the reasons  If the Speaker is absent from the
specified in articles 94, 101 and 102 of the House, the Deputy Speaker performs
Constitution. all the functions of the Speaker
 The Deputy Speaker presides over the Lok including decision on the Money Bills
Sabha when the Speaker is absent from or presiding the Joint Sitting of the
the sitting of the House. Houses.

 The Speaker and the Deputy Speaker are  Deputy Speaker unlike Speaker can
vote in the House similar to other
the Presiding Officers of the Lok Sabha.
members of the House if the Speaker
Both of them are selected by the Lok
is presiding the House. However, he
Sabha hence they are responsible to can exercise only a casting vote, in the
the House directly. event of a tie, if he performs the duty
of the Presiding Officer.
8. Correct Option: (a)
Explanation: 11. Correct Option: (d)
Explanation:
President and Speaker both
 Neelam Sanjiva Reddy was the First Hour and Zero Hour
Speaker of the Lok Sabha in 1967-69  The first hour of every parliamentary
and in 1977 before elected as the sixth sitting is slotted for ‘First Hour’. During
President of India. this time, the members ask questions and
the ministers usually give answers.
 PA Sangma was also the Speaker of Lok
Sabha and he was a candidate for the  Zero Hour is an informal device available
presidential election but couldn’t get to the members of the Parliament to raise
elected. matters without any prior notice.
 It starts immediately after the question
9. Correct Option: (d) hour and lasts until the agenda for the day
is taken up.
Explanation:
 Unlike the question hour, the zero
Chairman of Rajya Sabha hour is not mentioned in the Rules of
 The Chairman of Rajya Sabha (or, the Procedure of the Parliament.
Vice-President of India) is elected by both  A Bill becomes an Act when it is passed
the House. by both the Houses of Parliament and
gets assent from the President.
 S/he is the Chairman of the Business
Advisory Committee, General Purposes
Committee and Committee on Rules. The
12. Correct Option: (d)
Deputy Chairman is the Chairman of Explanation:
the Committee of Privileges.
Leader of the Opposition
 The Chairman has a casting vote in
the case of an equality of votes.  In each House of Parliament, there is the
‘Leader of the Opposition’.
10. Correct Option: (b)  The leader of the largest Opposition party
Explanation: having not less than one-tenth seats of the
total strength of the House is recognized as
Lok Sabha presiding officers the leader of the Opposition in that House.
This is why the post of LoP was vacant
 The Speaker has been given a very high during 1980-89 and at present, since
position in the order of precedence. 2014.
S/he is placed at seventh rank, along
with the Chief Justice of India. This  The 1977 Act defines LoP as that
means s/he has a higher rank than all member of the House who is the
cabinet ministers, except the Prime “Leader in that House of the party in
Minister or Deputy Prime Minister. opposition to the Government having
the greatest numerical strength and
 Speaker can be removed by the Lok recognized as such by the Chairman
Sabha by a resolution passed by an of the Council of States or the Speaker
effective majority ( more than 50% of of the House of the People.
48 TARGET PT 2020

 The Speaker’s decisions in this ! It is clear that the position of the Rajya
regard have so far been determined Sabha in our constitutional system is
by Direction 121(c) which laid down not as weak as that of the House of
one of the conditions for recognition Lords in the British constitutional
of party or group as having “at least system or as strong as that of the
a strength equal to the quorum fixed Senate in the American constitutional
to constitute a sitting of the House, system.
that is one-tenth of the total number
of members of the House”. ! Except in financial matters and
control over the council of ministers,
 Although they were accorded statutory the powers and status of the Rajya
recognition in 1977, it was in 1969 that Sabha in all other spheres are broadly
an official leader of the opposition was equal and coordinate with that of the
recognized for the first time in Rajya Lok Sabha.
Sabha.
 Lok Sabha and Rajya Sabha share
 His/her main functions are to provide a equal power related to the election
constructive criticism of the policies of the and removal of the Vice-President.
government. However, Rajya Sabha alone can initiate
 LoP plays a crucial role in bringing the removal of the vice-president.
bipartisanship and neutrality to
the appointments in institutions of 15. Correct Option: (a)
accountability and transparency viz. CVC,
Explanation:
CBI, CIC, Lokpal etc.
Censure Motion vs. No-Confidence Motion
13. Correct Option: (a)
 Censure Motion can be moved against
Explanation: an individual minister or a group
of ministers or the entire council
Relationship of the two Houses
of ministers whereas No-Confidence
 As the Council of Ministers is directly Motion can be moved against the
responsible to the Lower House, both entire council of ministers only.
Confidence and No-Confidence motions
 For Censure Motion, the reasons for its
can be moved in the Lok Sabha only.
adoption should be mentioned whereas no
 Apart from a Money Bill, certain other such thing is necessary in adopting the No-
introduced in Rajya Sabha but there Confidence Motion.
is no other limitation on the powers of
Rajya Sabha with regard to such Bills  If the No-Confidence Motion is
and Rajya Sabha has powers to reject passed in the Lok Sabha, the council
or amend such Financial Bills like any of ministers must resign from office
other Bill. whereas the council of ministers
need not resign from the office if the
 Besides emergency situations, Parliament Censure Motion is passed.
can make laws on the subjects under the
state list under Articles 249, 252, 253, 16. Correct Option: (d)
etc.
Explanation:
14. Correct Option: (a)
Judicial Powers of the Parliament
Explanation:
 The judicial powers and functions of the
Special Powers of Rajya Sabha Parliament include the following:

 The Rajya Sabha has been given two ! It can impeach the President for the
exclusive or special powers that are not violation of the Constitution.
enjoyed by the Lok Sabha: ! It can remove the Vice-President from
! It can authorize the Parliament to his office.
make a law on a subject enumerated ! It can recommend the removal of
in the State List (Article 249).
judges (including chief justice) of
! It can authorize the Parliament to the Supreme Court and the high
create new All-India Services common courts, chief election commissioner,
to both the Centre and states (Article comptroller and auditor general to the
312). president.
TARGET PT 2020 49

! It can punish its members or outsiders  The frequent promulgation of


for the breach of its privileges or its ordinances by the president dilutes
contempt. the Parliament’s power of legislation.
 Parliament has no role to play in the  The Parliament’s control is sporadic,
ratification of the pardoning of the general and mostly political in nature.
President. It is in the domain of the  Lack of strong and steady opposition
Cabinet. in the Parliament, and a setback in the
parliamentary behavior and ethics, have
17. Correct Option: (b) also contributed to the ineffectiveness of
legislative control over administration in
Explanation India. Etc.
Budget
19. Correct Option: (c)
 The Constitution of India contains the
following provisions with regard to the Explanation:
enactment of budget: Cut Motions
! Article 112(2) provides that  The Parliament while discussing the
the budget shall distinguish Demand for Grants can move motions to
expenditure on revenue account reduce any demand for the grant. Such
from other expenditure. motions are called ‘cut motion’, which
! Parliament can reduce or abolish are of three kinds viz. Policy Cut Motion,
a tax but cannot increase it. Economy Cut Motion, and Token Cut
Motion.
! Article 113(3) states that no
 Policy Cut Motion represents the
demand for a grant shall be made
disapproval of the policy underlying
except on the recommendation of
the demand. It states that the amount
the President. of the demand be reduced to Re 1.

18. Correct Option: (c)  Economy Cut Motion represents the


economy that can be affected in the
Explanation: proposed expenditure. It states that the
amount of the demand be reduced by a
Ineffectiveness of Parliamentary control specified amount.
 The parliamentary control over government  Token Cut Motion ventilates a specific
and administration in India is more grievance that is within the sphere of
theoretical than practical. In reality, the responsibility of the Government of India.
control is not as effective as it ought to be. It states that the amount of the demand be
The following factors are responsible for reduced by Rs 100.
this-
20. Correct Option: (d)
 Parliament’s financial control is hindered
by the technical nature of the demands for Explanation:
grants.
Delegated legislation
 The legislative leadership lies with the
Executive and it plays a significant role in  In most cases, the legislature enacts a law
formulating policies. covering the general principles and policies
and leaves detailed rule-making to the
 The very size of the Parliament is government to allow for expediency and
too large and unmanageable to be flexibility. The government is required
effective. to frame the rules in accordance with
the policy laid down by the legislature.
 The financial committees like the Public
Such rules are called subordinate
Accounts Committee examine the public legislation and may be referred to as-
expenditure after it has been incurred by rules, regulations, bye-laws, orders,
the Executive. Thus, they do post mortem and notification.
work.
 However, certain functions and powers
 The growth of ‘delegated legislation’ has cannot be delegated to the government.
reduced the role of Parliament in making These include framing the legislative policy,
detailed laws and has increased the powers exceeding the scope of the delegating Act,
of bureaucracy. retrospective effect of rules, etc.
50 TARGET PT 2020

 According to the Manual of Parliamentary the originality of ideas and publication of


Procedure, the rules have to be laid in the research scholars.
Parliament within 15 days from (a) the
date of their publication in the Official 23. Correct Option: (b)
Gazette if Parliament is in session; or (b) the
date of the commencement of Parliament, Explanation:
if it is not in session
Tatpar App
 Parliament has the power to oversee rules
 Delhi Police has launched Tatpar app
at various levels: (a) during the debate on
to provide safety and a convenient 24×7
the Act; (b) statutory motion to discuss
online mode to its citizens.
rules; (c) Question Hour; and (d) Committee
on Subordinate Legislation (Each House  The app brings together all the websites
has this Committee). and mobile applications of the Delhi Police
and over 50 services.
 Since delegated legislation forms a part
of law, courts have the jurisdiction to
examine them. The courts have held that 24. Correct Option: (c)
framing of legislative policy is an essential Explanation:
function and cannot be delegated to an
administrative body. However, they have Shanghai Cooperation Organization
generally leaned towards upholding the (SCO)
validity of rules.
 The Shanghai Cooperation Organization
 The growth of the delegated/ is a permanent intergovernmental
subordinate legislation has led to the international organization, the creation
ineffectiveness of the parliamentary of which was announced on 15 June
process and parliamentary 2001 in Shanghai by Kazakhstan, China,
democracy. Kyrgyzstan, the Russian Federation,
Tajikistan, and Uzbekistan.
21. Correct Option: (c)
 On 8-9 June 2017 in Astana, India
Explanation: and Pakistan were granted the full
membership.
Gottiprolu
 It is in Andhra Pradesh’s SPS Nellore 25. Correct Option: (b)
district and on the right bank of Explanation:
Swarnamukhiriver.
 The excavation has indicated that the area Pegasus
may have been a huge settlement and  It is a spyware that works by sending an
flourishing trade center, over 2,000 years exploit link, and if the target user clicks
ago. on the link, the malware or the code that
 Several bricks were unearthed which allows the surveillance is installed on the
were comparable to the Satavahana and user’s phone.
Ikshvaku dynasty periods  It is installed without the user’s knowledge
or permission and when installed, the
22. Correct Option: (c) attacker has complete access to the target
Explanation: user’s phone.
 It delivers a chain of zero-day exploits to
Shodh Shuddh initiative
penetrate security features on the phone
 It is a Plagiarism Detention Software and installs Pegasus without the user’s
(PDS) launched by the Ministry of Human knowledge or permission.
Resources Development.
 Pegasus has been attributed to the NSO
 It will significantly help to improve the Group, an Israeli company that uses it for
quality of research outcome by ensuring commercial purpose.

**********
TARGET PT 2020 51

TEST
DAY - 6

Time Allowed: 30 mins Maximum Marks: 50

1. Which of the following Constitutional 2. Recommendation for the imposition of


Amendment Acts facilitated the the President’s Rule in the state is the
appointment of the same person as a discretionary power of the Governor.
Governor for two or more states?
Which of the following statements is/are
(a) 1st Amendment Act correct?
(b) 7th Amendment Act (a) 1 only
(c) 24th Amendment Act (b) 2 only
(d) 42 Amendment Act
nd
(c) Both 1 and 2
2. The Draft Constitution provided for (d) Neither 1 nor 2
the direct election of the governor on
the basis of universal adult suffrage. 4. In which of the following cases, the
But, the Constituent Assembly opted Governor though has to consult the
for the present system of appointment council of ministers, acts finally on
due to which of the following reasons? her/his discretion?
1. The direct election of the governor is 1. With respect to the administration of
incompatible with the parliamentary tribal areas of Assam.
system established in the states. 2. With respect to the administration of
2. The governor being only a nominal plain areas of Manipur.
head, there is no point in making 3. With respect to law and order in
elaborate arrangements for his election Hyderabad.
and spending huge amount of money. 4. Establishment of separate development
3. The office of governor of a state is boards for Vidarbha and Marathwada.
an employment under the Central
Select the correct option using the codes
government.
given below:
Which of the following statements is/are (a) 1 and 4 only
correct?
(b) 2 and 3 only
(a) 1 only
(c) 1 and 3 only
(b) 2 only
(d) 1, 3, and 4
(c) 1 and 2 only
5. Consider the following statements:
(d) 1, 2, and 3
1. The 42nd Constitutional Amendment
3. Consider the following statements (1976) mandates both the President
regarding the discretionary power of and Governor to be bound by the
the Governor ministerial advice.
1. Unlike President, the Governor has 2. The ministers in the state is individually
constitutional discretion also. responsible to the Governor.
52 TARGET PT 2020

Which of the following statements is/are 9. Consider the following statements


correct? regarding the legislative powers of a
(a) 1 only Governor:
1. S/he nominates one-third of the
(b) 2 only
members of the state legislative
(c) Both 1 and 2 council.
(d) Neither 1 nor 2 2. S/he can reserve the bill of the state
legislature only if it is against the
6. Consider the following statements provisions of the Constitution.
regarding the pardoning power of
Governor: 3. Unlike President, s/he cannot
promulgate ordinances.
1. If a state law prescribes for death
sentence, then only the Governor can
10. Which of the following statements is/
pardon it.
are incorrect?
2. The Governor cannot pardon, suspend,
remit or commute a death sentence. (a) 1 only
(b) 1 and 3 only
Which of the following statements is/are
incorrect? (c) 2 and 3 only
(a) 1 only (d) 1, 2, and 3
(b) 2 only
11. Which of the following officials are
(c) Both 1 and 2 appointed as well as removed by the
(d) Neither 1 nor 2 Governor?
1. Advocate General of the state.
7. In which of the following cases the
2. State Election Commissioner.
reservation of a Bill by a Governor
for the consideration of President is 3. Members of the State Public Service
mandatary? Commission.
(a) If the Bill is related to the salary and 4. Members of the State Human Rights
allowances of the officers of All India Commission.
Services.
Select the correct answer using the code
(b) If the Bill is related to the transfer of given below:
High Court Judges.
(a) 1 only
(c) If the Bill endangers the position of the
state High Court. (b) 1, 2 and 4 only
(d) There is no such case. (c) 1, 3 and 4 only

8. Consider the following statements (d) 1, 2, 3 and 4


regarding the financial powers of the
12. Consider the following Cases:
Governor:
1. The Governor of a state cannot reserve 1. D.C. Wadhwa & Ors vs State Of Bihar
a Money Bill for the consideration of & Ors
the President of India. 2. Krishna Kumar Singh & Anr vs State
2. The Governor cannot return a Money of Bihar & Ors
Bill for the reconsideration of the state 3. S. R. Bommai v. Union of India
legislature.
Which of the above cases is/are related to the
Which of the following statements is/are
ordinance making power of a Governor?
correct?
(a) 1 only (a) 1 only

(b) 2 only (b) 2 only


(c) Both 1 and 2 (c) 1 and 2 only
(d) Neither 1 nor 2 (d) 1 and 3 only
TARGET PT 2020 53

13. In which of the following case(s) the (a) 1 only


Governor needs the instructions from
(b) 2 only
President for the promulgation of
ordinances? (c) Both 1 and 2
1. If a bill containing the same provisions (d) Neither 1 nor 2
would have required the previous
sanction of the President for its 16. Which of the following statements
introduction into the state legislature. regarding the post of Chief Secretary
of a state is/are correct?
2. If s/he would have deemed it necessary
to reserve a bill containing the same 1. The chief secretary acts as a secretary
provisions for the consideration of the to the state cabinet.
President. 2. The state consults the Union
3. If an act of the state legislature government regarding the appointment
containing the same provisions would and this consultation is binding.
have been invalid without receiving 3. S/he holds the office for 5 years.
the President’s assent.
Select the correct option using the codes
Select the correct option using the codes given below:
given below:
(a) 1 only
(a) 1 only
(b) 1 and 2 only
(b) 2 and 3 only
(c) 2 and 3 only
(c) 3 only
(d) 1, 2, and 3
(d) 1, 2, and 3
17. In case of a hung assembly following
14. Which of the following statements is/ are some of the recommendations of
are correct? the Sarkaria Commission. Arrange
1. The salaries of the Advocate-General the following recommendations in the
of a stare is determined by State order of preference for the Governor:
Legislature. A. Single largest party staking a claim to
2. S/he holds the position during the form the government with the support
pleasure of the Governor. of others, including independents

Select the correct option using the codes B. Post-electoral coalition of parties, with
given below: all the partners in the coalition joining
(a) 1 only the government

(b) 2 only C. Alliance of parties that was formed prior


to the elections
(c) Both 1 and 2
(d) Neither 1 nor 2 D. Post-electoral alliance of parties, with
some of the parties in the alliance forming
15. Consider the following statements a government and the remaining parties,
regarding Council of Ministers at including independents, supporting the
state: government from outside.
1. Unlike at Centre, there may not be a Select the correct option using the codes
Council of Ministers in state always. given below:
2. The salary and allowances of the
(a) A-C-B-D
Chief Minister are determined by the
Parliament. (b) C-A-B-D
(c) C-B-D-A
Which of the above statements is/are
correct? (d) B-A-D-C
54 TARGET PT 2020

18. Which of the above statements is/are Select the correct option using the codes
incorrect? given below:
1. The Constitution does not contain any (a) 1 only
specific procedure for the selection and
appointment of the Chief Minister. (b) 2 only
2. Upon death of incumbent Chief (c) Both 1 and 2
Minister, if the ruling party appoints
(d) Neither 1 nor 2
a new leader, then the Governor has
to appoint the new leader as Chief 22. Regarding Corporate Social
Minister. Responsibility, which of the following
Select the correct option using the codes below provisions are not included in
given below: the recently introduced bill amending
Companies Act 2013?
(a) 1 only
(a) The unspent amount of CSR fund in a
(b) 2 only
given year has to be spent in next 30
(c) Both 1 and 2 days.
(d) Neither 1 nor 2 (b) Companies violating CSR norms will
attract fines ranging from ` 50,000 to
19. Which of the following Commissions ` 5 lakh.
had suggested that the phrase “during
the pleasure of the President” should (c) Listed firms need to disclose their CSR
be deleted from the Constitution? activities.
(a) Venkatachaliah Commission (d) Companies need to have a CSR team
that provides a regular progress report
(b) Sarkaria Commission
to the CSR committee of the board.
(c) Punchhi Commission
(d) None of the above 23. Antimicrobial resistance (AMR)is an
important environment as well as a
20. Which of the following statements public health issue. State governments
regarding is/are incorrect? are coming up with action plans to
1. The Constitution does not lay down any manage this. Consider the following
grounds of the Governor’s removal. states with respect to this:
2. The ‘pleasure’ of the President is not 1. Madhya Pradesh
subject to the judicial review. 2. West Bengal
3. A Governor can hold office beyond her/ 3. Kerala
his term of five years
4. Maharashtra
Select the correct option using the codes
Which of the above states have developed
given below:
action plans for managing AMR?
(a) 1 only
(a) 2 only
(b) 2 only
(b) 1 and 3 only
(c) 1 and 2 only
(c) 3 only
(d) 1, 2, and 3
(d) 1, 2 and 3 only
21. Which of the following statements is/
are correct? 24. ZConsider the following statements
about Indus Valley inscriptions:
1. During the term of office, a Governor is
immune from any criminal proceedings, 1. These were discovered from 4,000
except in respect of his personal acts. ancient inscribed objects.
2. The oath of office to the governor 2. Majority of the Indus Valley inscriptions
is administered by the President of were written logographically (by using
India. word signs).
TARGET PT 2020 55

Which of the above statements is/are (c) Assam


correct?
(d) Chhattisgarh
(a) 1 only
(b) 2 only 26. Gulf of Hormuz, recently in news for
seizure of British ships by Iran, is
(c) Both 1 and 2
located between which two countries?
(d) Neither 1 nor 2
(a) Oman and Iran
25. Operation Bedlam is related to which
(b) Iran and UAE
of the following states?
(a) Nagaland (c) UAE and Qatar

(b) Jammu and Kashmir (d) Iran and Qatar

**********
56 TARGET PT 2020

ANSWER HINTS
DAY - 6

1. Correct Option: (b) ! The election of a governor would be


entirely on personal issues. Hence,
Explanation: it is not in the national interest to
Governor in a state involve a large number of voters in
such an election.
 Usually, there is a governor for each state,
but the 7th Constitutional Amendment ! An elected governor would naturally
Act of 1956 facilitated the appointment of belong to a party and would not be
the same person as a governor for two or a neutral person and an impartial
more states. head.

 By this Act, a new Article 258A was ! The election of governor would create
inserted to provide that the Governor separatist tendencies and thus affect
of a State may, with the consent of the the political stability and unity of the
Government of India, entrust any State country.
functions to the central Government or its
! The system of presidential nomination
officers.
enables the Centre to maintain its
control over the states.
2. Correct Option: (c)
! The direct election of the governor
Explanation:
creates a serious problem of leadership
Appointment of Governor at the time of a general election in the
state.
 A Governor is appointed by the president by
warrant under his hand and seal. In a way, ! The chief minister would like his
he is a nominee of the Central government nominee to contest for governorship
but not an employment under the etc.
Central government.
3. Correct Option: (c)
 The Draft Constitution provided for the
direct election of the governor on the Explanation:
basis of universal adult suffrage. But, the
Constituent Assembly opted for the present Discretionary powers of the Governor
system of appointment due to following
 The Governor has both Constitutional and
reasons:
Situational discretionary powers.
! The direct election of the
governor is incompatible with the  As far as situational discretionary power is
parliamentary system established concerned, it is similar to the President.
in the states.  During exercising situational
! The mode of direct election is more discretionary powers (in relation to
likely to create conflicts between the the State Legislature), s/he acts as
governor and the chief minister. a Constitutional head of the state.
Otherwise, s/he acts as a representative
! The governor being only a of the Centre.
constitutional (nominal) head,
there is no point in making  The Constitution makes it clear that if
elaborate arrangements for any question arises whether a matter falls
his election and spending huge within the governor’s discretion or not, the
amount of money. decision of the governor is final.
TARGET PT 2020 57

 The Governor has constitutional discretion 5. Correct Option: (b)


in the following cases:
Explanation:
! Reservation of a bill for the
consideration of the President. Constitutional position of the Governor
! Recommendation for the  The executive power of the state shall
imposition of the President’s Rule be vested in the governor and shall be
in the state. exercised by him either directly or through
officers subordinate to him in accordance
! While exercising his functions as
with this Constitution.
the administrator of an adjoining
Union territory (in case of additional  There shall be a council of ministers with
charge). the chief minister as the head to aid and
! Determining the amount payable advise the governor in the exercise of his
by the Government of Assam, functions, except in so far as he is required
Meghalaya, Tripura and Mizoram to to exercise his functions in his discretion.
an autonomous Tribal District Council  The Council of Ministers in a state
as royalty accruing from licenses for is individually responsible to the
mineral exploration7. Governor, and collectively responsible to
! Seeking information from the the State Legislature.
chief minister with regard to the  After the 42nd Constitutional Amendment
administrative and legislative matters (1976), ministerial advice has been made
of the state. binding on the President, but no such
provision has been made with respect
4. Correct Option: (a) to the Governor.
Explanation:
6. Correct Option: (c)
Special powers of the Governor
Explanation:
 The Governor has certain special
responsibilities to discharge according to Pardoning power of the Governor
the directions issued by the President. In  The Governor can pardon, reprieve, respite,
this regard, the Governor though has to remit, suspend or commute the punishment
consult the council of ministers led by the or sentence of any person convicted of any
chief minister, acts finally on his discretion. offense against a state law but cannot
They are as follows: pardon a death sentence.
 Maharashtra: Establishment of separate  But, the governor can suspend, remit
development boards for Vidarbha and or commute a death sentence.
Marathwada.
 Even if a state law prescribes for death
 Gujarat: Establishment of separate sentence, the power to grant pardon
development boards for Saurashtra and lies with the President and not the
Kutch. governor.
 Nagaland: With respect to law and
order in the state for so long as the 7. Correct Option: (c)
internal disturbance in the Naga
Hills–Tuensang Area continues.
Explanation:

 Assam: With respect to the Reservation of a Bill by Governor


administration of tribal areas.  When a bill is sent to the governor after it
 Manipur: Regarding the administration is passed by state legislature, he can
of the hill areas in the state. ! Give his assent to the bill, or
 Sikkim: For peace and for ensuring social ! Withhold his assent to the bill, or
and economic advancement of the different
sections of the population. ! Return the bill (if it is not a money
bill) for reconsideration of the state
 Arunachal Pradesh: With respect to
legislature. However, if the bill is
law and order in the state.
passed again by the state legislature
 Karnataka: Establishment of a separate with or without amendments, the
development board for Hyderabad- governor has to give his assent to the
Karnataka region. bill, or
58 TARGET PT 2020

! Reserve the bill for the consideration 10. Correct Option: (d)
of the president. In one case such
reservation is obligatory, that is, Explanation:
where the bill passed by the state Legislative Powers of Governor
legislature endangers the position
of the state High Court.  A governor is an integral part of the state
legislature.
 In addition, the governor can also reserve
the bill if it is of the following nature:  He nominates one-sixth of the members
of the state legislative council from
 Ultra-vires, that is, against the provisions amongst persons having special knowledge
of the Constitution. or practical experience in literature,
 Opposed to the Directive Principles of State science, art, cooperative movement, and
social service.
Policy.
 When a bill is sent to the governor after it
 Against the larger interest of the country.
is passed by state legislature, he can
 Of grave national importance.
! Give his assent to the bill, or
 Dealing with compulsory acquisition
! Withhold his assent to the bill, or
of property under Article 31A of the
Constitution. ! Return the bill (if it is not a money
bill) for reconsideration of the state
8. Correct Option: (b) legislature. However, if the bill is
passed again by the state legislature
Explanation: with or without amendments, the
governor has to give his assent to the
Role of Governor with regard to Money bill, or
Bills
! Reserve the bill for the
 Every money bill, after it is passed by consideration of the president.
the state legislature is presented to the
governor for his assent. He has three ! He can promulgate ordinances
alternatives: when the state legislature is not in
session.
! He may give his assent to the bill, the
bill then becomes an act. 11. Correct Option: (a)
! He may withhold his assent to the bill, Explanation:
the bill then ends and does not become
an act. Executive powers of Governor
! He may reserve the bill for the  The Advocate General is appointed by
consideration of the president. the Governor. The Constitution does not
contain the procedure and grounds for his
 Thus, the governor cannot return a removal. He/she holds office during the
money bill for the reconsideration pleasure of the Governor. This means
of the state legislature. Normally, the that he may be removed by the governor
governor gives his assent to a money bill as at any time.
it is introduced in the state legislature with
his previous permission.  Although the members of the State
Election Commission, members of
 When the governor reserves the money the SPSC, and the chairperson and
bill for the consideration of the President, members of a State Human Rights
he will not have any further role in the Commission are appointed by the
enactment of the bill. governor, they can be removed only
by the President.
 The President may give/withhold his assent
to the bill.
12. Correct Option: (c)
 If the President gives his assent to the
Explanation:
bill, it becomes an Act. This means that
the assent of the governor is no longer Cases related to Governor
required.
 All of the above cases are related to
9. Correct Option: (d) Governor’s role and powers. S. R. Bommai
v. Union of India, 1994 case is related
Explanation: to the misuse of Article 356 whereas
TARGET PT 2020 59

D.C. Wadhwa case of 1986 and Krishna Governor may determine.


Kumar Singh case of 1994 are related to
the misuse of ordinance making power of a 15. Correct Option: (a)
Governor (Article 213).
Explanation:
 In the Krishna Kumar Singh case,
the seven-judge constitution bench of Council of Ministers at state
the Supreme Court had held that re-
promulgation of ordinance is a fraud  Article 163 states that there shall be a
on the Constitution. The Court also Council of Ministers with the Chief Minister
held that the satisfaction of the President as the head to aid and advise the Governor
of India under Article 123 and of the in the exercise of his functions, except in
Governor under Article 213 while issuing so far as he is required to exercise his
an Ordinance is not immune from judicial functions in his discretion. Thus, there
review. may not be a Council of Ministers in
the state always.
 In D.C. Wadhwa case, the Supreme Court
held that it is unconstitutional to re-  In 1971, the Supreme Court ruled that a
promulgate ordinances, unless in council of ministers must always exist
exceptional circumstances. to advise the governor, even after the
dissolution of the state legislative assembly
13. Correct Option: (d) or resignation of a council of ministers.
Explanation: Hence, the existing ministry may continue
in the office until its successor assumes
Ordinance making power of Governor charge.
 The Governor cannot make an ordinance  The salary and allowances of the Chief
without the instructions from the President Minister are determined by the State
in three cases: Legislature.
! If a bill containing the same
provisions would have required 16. Correct Option: (a)
the previous sanction of the Explanation:
President for its introduction into
the state legislature.  The chief secretary acts as a
secretary to the state cabinet. S/he is
! If he would have deemed it
the administrative head of the cabinet
necessary to reserve a bill
secretariat and attends the meeting of the
containing the same provisions
for the consideration of the cabinet and its sub-committees.
President.  S/he acts as the principal advisor to the
! If an act of the state legislature chief minister on all matters of state
containing the same provisions administration.
would have been invalid without  The chief secretary is picked up by the
receiving the President’s assent. chief minister of the state who consults
the Union government regarding the
14. Correct Option: () appointment but this consultation is
Explanation: not binding.
Advocate general  The office of chief secretary has been
 Article 165 has provided for the office of the excluded from the operation of the tenure
advocate general for the states. system. There is no fixed tenure for
this post.
 The advocate general is appointed by
the governor.  As per the ARC’s view, the chief secretary
 Like of Attorney General, the term of office should be the senior-most person, due
of the advocate general is not fixed by the regard being paid to merit. S/he should
Constitution. have a minimum tenure of 3 to 4 years. S/
he should be relieved of routine and non-
 He/She holds office during the pleasure essential work and, where necessary, given
of the governor i.e. he/she may be proportionate staff assistance to enable him
removed by the governor at any time. to ensure quick implementation of cabinet
 The remuneration of the advocate decisions and effective co-ordination in
general is not fixed by the Constitution. the policies and programs of the state
He receives such remuneration as the government.
60 TARGET PT 2020

17. Correct Option: (b) removed before completion of their five-


year tenure, except in rare and compelling
Explanation: circumstances. This was meant to provide
 Sarkaria Commission was set up in June Governors with a measure of security of
1983 to examine the relationship and tenure, so that they could carry out their
balance of power between state and central duties without fear or favour. If such rare
governments and suggest changes within and compelling circumstances did exist,
the framework of the Constitution. the Commission said that the procedure
of removal must allow the Governors an
 The order of preference for the Governor in opportunity to explain their conduct, and
such case is as follows: the central government must give fair
consideration to such explanation. It was
A. An alliance of parties that was further recommended that Governors
formed prior to the elections, should be informed of the grounds of their
removal.
B. The single largest party staking a
claim to form the government with  The Venkatachaliah Commission (2002)
the support of others, including similarly recommended that ordinarily
Governors should be allowed to complete
independents,
their five-year term. If they have to be
C. A post-electoral coalition of parties, removed before completion of their term,
the central government should do so only
with all the partners in the coalition
after consultation with the Chief Minister.
joining the government,
 The Punchhi Commission (2010)
D. A post-electoral alliance of parties, suggested that the phrase “during the
with some of the parties in the pleasure of the President” should be
alliance forming a government and deleted from the Constitution, because
the remaining parties, including a Governor should not be removed at the
will of the central government; instead
independents, supporting the
he or she should be removed only by a
government from outside. resolution of the state legislature.
18. Correct Option: (d)  The above recommendations, however,
were never made into law by Parliament.
Explanation Therefore, they are not binding on the
central government.
Chief Minister
 The Constitution does not contain any 20. Correct Option: (b)
specific procedure for the selection and
Explanation:
appointment of the Chief Minister.
 However, Article 164 only says that the The term of Governor’s office
Chief Minister shall be appointed by the  The Constitution does not lay down
governor. However, this does not imply any grounds upon which a governor
that the governor is free to appoint anyone may be removed by the President.
as the Chief Minister.
 S/he holds the position during the pleasure
 In accordance with the convections of the of the President.
parliamentary system of government,
the Governor has to appoint the leader of  “Pleasure of the President” merely refers to
the majority party in the state legislative this will and wish of the central government.
assembly as the Chief Minister. The Supreme Court’s interpretation and in
2010, a constitutional bench of the Supreme
 On the Death of a Chief Minister, Court interpreted these provisions and laid
the ruling Party usually elects a new down some binding principles (B.P. Singhal
leader and the governor has no choice v. Union of India). These are:
but to appoint him as Chief Minister.
! The President, in effect the central
government, has the power to remove
19. Correct Option: (c)
a Governor at any time without giving
Explanation: him or her any reason, and without
granting an opportunity to be heard.
Important Commissions on the removal of
Governor
! However, this power cannot be
exercised in an arbitrary, capricious
 The Sarkaria Commission (1988) or unreasonable manner. The power
recommended that Governors must not be of removing Governors should only
TARGET PT 2020 61

be exercised in rare and exceptional `25 lakh, with the officers concerned liable
circumstances for valid and compelling for imprisonment of up to three years.
reasons.
Supplementary Notes:
! The mere reason that a Governor
is at variance with the policies and  The government has planned to include a
ideologies of the central government, specific penal provision in the Companies
or that the central government has Act in case of non-compliance with CSR.
lost confidence in him or her, is not Listed firms need to disclose their CSR
sufficient to remove a Governor. Thus, activities, amount spent and framework
a change in central government cannot created to ensure adherence to norms.
be a ground for removal of Governors,
 Under the new laws, any unspent amount
or to appoint more favourable persons
of CSR fund will have to be deposited into
to this post.
an escrow account within 30 days of the
! A decision to remove a Governor end of that fiscal.
can be challenged in a court of
 Companies need to have a CSR team that
law. In such cases, first the petitioner
provides a regular progress report and
will have to make a prima facie case updates to the CSR committee of the board.
of arbitrariness or bad faith on part The committee is given a report of the
of the central government. If a prima activities undertaken each quarter, along
facie case is established, the court can with targets and reasons for variance, if
require the central government to any. All this is needed to be submitted to
produce the materials on the basis of the corporate affairs ministry.
which the decision was made in order
to verify the presence of compelling  In news: Recently, Parliament has passed
reasons. amendments to the Companies Act, 2013 to
strengthen laws governing corporate social
 A Governor can hold office beyond his responsibility (CSR).
term of five years until his successor
assumes charge. The underlying idea is 23. Correct option: (b)
that there must be a governor in the state
and there cannot be an interregnum. Explanation:
 Option (b) is correct: Kerala and
21. Correct Option: (d) Madhya Pradesh are two states who
Explanation: have developed action plans to manage
Antimicrobial resistance (AMR).
Conditions of Governor’s office
Supplementary Notes:
 Like the President, the governor is also
entitled to a number of privileges and  AMR is “the ability of a microorganism (like
immunities. He enjoys personal immunity bacteria, viruses, and some parasites) to
from legal liability for his official acts. stop an antimicrobial (such as antibiotics,
antivirals and antimalarial) from working
 During his term of office, he is immune against it. As a result, standard treatments
from any criminal proceedings, even become ineffective, infections persist and
in respect of his personal acts. He may spread to others.”
cannot be arrested or imprisoned.
 Misuse of antibiotics in humans, animals,
 However, after giving two months’ notice, aquaculture, hospital effluents and
civil proceedings can be instituted against antibiotic use in livestock and poultry
him during his term of office in respect of contributes to AMR.
his personal acts.
 The vast scale and diversity of the country
 The oath of office to the governor is in terms of population, food animal sectors
administered by the Chief Justice of (for instance, commercial and backyard
the concerned state high court and in farming), extent of antibiotic use adds to
his absence, the senior-most judge of the challenges of addressing this multi-
that court available. sectoral issue.

22. Correct option: (b)  With rising AMR, antibiotics are


increasingly becoming ineffective for
Explanation: treating diseases in humans.
 Companies violating CSR norms will  If there is no timely containment, AMR
attract fines ranging from `50,000 to is likely to cause nearly 10 million deaths
62 TARGET PT 2020

by 2050 and result in significant global co-occurrence of restriction patterns


economic losses. demonstrated by certain classes of Indus
signs, it was inferred that such patterns
 Methods of controlling the spread of AMR:
can never be phonological co-occurrence
! Tracking antibiotic use in humans, restrictions (two or more sound units that
animals, fisheries, crops. cannot be pronounced together).
! Ensuring effective infection prevention
25. Correct Option: (b)
and control in human and animal
health, community and environment. Explanation:
! Optimised use of antimicrobial  Option (b) is correct: ‘Operation Bedlam’
agents in the health, animal and food for the tourism sector in Kashmir
sectors.
Supplementary Notes:
! Promoting investments for AMR
activities, research and innovations  Tourism has suffered a major jolt in
for AMR containment. Kashmir
 In news: Madhya Pradesh recently  Against an average footfall of 3,000 tourists
became the second state of the country per day, a trickle of 350 arrived in the
after Kerala to bring action plan to manage Valley
Antimicrobial resistance.  Most hotels in tourist hotspots including
Gulmarg and Pahalgam had been emptied
24. Correct option: (c)
 Kashmir Chamber of Commerce and
Explanation: Industry general secretary Farooq Amin
said the sudden advisory had proved
Option (a) is correct: “Operation Bedlam” for people and the
 Statement 1 is correct: Indus Valley State economy.
inscriptions are discovered from 4,000  Tourism, horticulture and other sectors of
ancient inscribed objects, including seals, our economy suffered tremendous losses
tablets, ivory rods, pottery shards. due to the decision of the government to
 Statement 2 is correct: Majority of the restrict movement of civilians on the only
Indus Valley inscriptions were written road link of Srinagar-Jammu.
logographically (by using word signs) and  In News - Operation Bedlam’ for the
not by using phonograms (speech sounds tourism sector in Kashmir
units).
Correct Option: (a)
Supplementary Notes:
Explanation:
 The Indus inscriptions are one of the
 Option (a) is correct: Gulf of Hormuz is
most enigmatic legacies of the Indus
between Oman and Iran
Valley civilization but these have not been
deciphered due to the absence of bilingual Supplementary Notes:
texts, extreme brevity of the inscriptions,
and ignorance about the language(s)
encoded by Indus script.
 In news: A research paper published
recently in Palgrave Communications,
a Nature group journal focuses on
understanding how Indus inscriptions
conveyed meanings, rather than on
deciphering what they conveyed.
 Another paper, titled “Interrogating Indus
inscription to unravel their mechanism of
meaning conveyance”, published recently
points out that the inscriptions can be
compared to the structured messages found
on stamps, coupons, tokens and currency
coins of modern times.
 Analysing the brevity of the inscriptions,
the rigid positional preferences maintained
by the signs of the inscriptions, and the
TARGET PT 2020 63

TEST
DAY - 7

Time Allowed: 30 mins Maximum Marks: 50

1. If a Money Bill is reserved by 2. In the case of Goa, it is fixed at 30.


the Governor of a State for the
consideration of the President. The Which of the above statements is/are
President may_ correct?
1. Give his assent to the Bill (a) 1 only
2. Withhold his assent to the Bill (b) 2 only
3. Direct the Governor of the State to return (c) Both 1 and 2
it to the House for reconsideration
(d) Neither 1 nor 2
Which of the above statements is/are
correct? 4. Arrange the following lists in the
increasing order of the composition of
(a) 1 only a Legislative Council:
(b) 1 and 2 only 1. Members of local bodies in the state like
(c) 1 and 3 only municipalities, district boards, etc.
(d) 1, 2, and 3 2. Graduates of three years standing and
residing within the state,
2. Consider the following statements 3. Members nominated by the governor,
regarding a state legislature:
1. Governor is the ex officio chairman of Select the correct option using the codes
the legislative council in a state. given below:
2. The State Assembly is authorized to (a) 1-2-3
modify the composition of a legislative (b) 3-2-1
council.
(c) 2-3-1
3. The term of the State Assembly can be
extended during the period of national (d) 2-1-3
emergency.
5. Which of the following is not a
Which of the above statements is/are power/duties of a Speaker in a State
incorrect? Assembly?
(a) 1 only (a) He is the final interpreter of the
(b) 2 and 3 only provisions of the Constitution of India
within the Assembly.
(c) 1 and 2 only
(b) He is the leader of the House.
(d) 1, 2 and 3
(c) He appoints the chairmen of all the
3. Consider the following statements: committees of the assembly and
supervises their functioning.
1. As per the Constitution, the maximum
strength of a Legislative Assembly is (d) He decides whether a bill is a Money
fixed at 500 and minimum strength at Bill or not and his decision on this
60. question is final.
64 TARGET PT 2020

6. Consider the following statements: 1. The maximum strength of a Legislative


1. The Deputy Speaker of Assembly is Council must be the half of Legislative
elected by the Speaker. Assembly.
2. S/he is removed by the Lower House 2. There is no lower limit in this regard.
only. Which of the above statements is/are
Which of the above statements is/are incorrect?
incorrect? (a) 1 only
(a) 1 only (b) 2 only
(b) 2 only (c) Both 1 and 2
(c) Both 1 and 2 (d) Neither 1 nor 2
(d) Neither 1 nor 2 10. Consider the following statements:
7. Which of the following statements is/ 1. Like the Rajya Sabha, the Legislative
are correct? Council is a continuing chamber.
1. The Rajya Sabha reflects the federal 2. Unlike the members of Rajya Sabha,
element of Indian polity whereas no the retiring members are not eligible
such issues in the case of a Legislative for re-election and re-nomination.
Council.
Which of the above statements is/are
2. The Rajya Sabha consists of mainly correct?
elected members whereas the Council,
of mainly nominated members. (a) 1 only
(b) 2 only
Select the correct option using the codes
given below: (c) Both 1 and 2
(a) 1 only (d) Neither 1 nor 2

(b) 2 only 11. Disqualification of Member of State


(c) Both 1 and 2 Legislative Assembly for matters other
than those in the Tenth schedule is
(d) Neither 1 nor 2 decided by__

8. Consider the following bills in State (a) Speaker of Legislative Assembly


Legislature: (b) Governor
1. A bill passed by the assembly but (c) High Court
pending in the council.
(d) State Election Commission
2. A bill passed by the council but pending
in the assembly. 12. Consider the following statements
3. A bill passed by the state legislature but regarding the legislative process in
pending the assent of the governor. State Legislature:
4. A bill passed by the state legislature 1. There is no provision of a joint sitting
but returned by the president for over ordinary bills in state legislature.
reconsideration. 2. Only the Legislative Assembly can has
the power to introduce any of the bills.
Which of the above bills lapse when the
State legislative assembly is dissolved? 3. The Legislative Council can delay a
maximum of four months in case of
(a) 1 only ordinary bills.
(b) 1 and 2 only Which of the above options is/are correct?
(c) 2 and 3 only (a) 1 only
(d) 3 and 4 only (b) 1 and 3 only
9. Consider the following statements (c) 2 and 3 only
regarding Legislative Council: (d) 1, 2 and 3
TARGET PT 2020 65

13. When it comes to the passage of Money (a) One-tenth of the total membership of
Bills, how is the situation in the Union the House
Parliament different from that in the
(b) Ten members or one-tenth of total
State Legislature?
membership whichever is more
1. As in the Parliament, it can be
introduced only in the lower house of (c) Ten members or one-tenth of total
state legislature. membership whichever is less
2. Unlike in Parliament, it is not sent to the (d) One-half of the total membership of
upper house of the state legislature. the House

Select the correct option using the codes 17. Which one of the following states does
given below: not have Vidhan Parishad?
(a) 1 only 1. Bihar
(b) 2 only 2. Maharashtra
(c) Both 1 and 2 3. Jammu and Kashmir

(d) Neither 1 nor 2 4. Tamil Nadu

Select the correct option using the codes


14. Which of the following statements
given below:
regarding State Legislature are
incorrect? (a) 1 and 2 only
1. Unlike Parliament, a Constitutional (b) 2 and 3 only
Amendment Bill can only be introduced (c) 3 and 4 only
in the Lower House.
(d) 2, 3 and 4 only
2. Constitution Amendment bills which
require ratification from states need 18. In the case of Puducherry, the
approval from both the Assembly and President of India can legislate by
the Council. making regulations only__
3. Resolution to abolish the council in a (a) When the Parliament passes a
state legislature has to be passed by resolution to that effect
both the Assembly and the Council.
(b) When the Assembly passes a resolution
Select the correct option using the codes to that effect
given below: (c) When the Assembly is suspended or
(a) 1 and 3 only dissolved

(b) 2 and 3 only (d) When the Lt. Governor requests him to
do so
(c) 1 and 2 only
(d) 1, 2, and 3 19. Which of the following can put a union
territory under the jurisdiction of the
15. What is the time limit within which high court of adjacent state?
an Ordinary Bill has to be sent to the (a) President of India
State Legislature by the Governor for
(b) Chief Justice of India
reconsideration?
(c) Parliament of India
(a) 4 months
(d) Administrator of the concerned union
(b) 3 month
territory
(c) 1 months
20. Consider the following statements:
(d) No time limit specified
1. Presence of states and Union Territories
16. The quorum to hold a meeting of a is a mark of federalism in India.
House of the State Legislature shall 2. Delhi is the only union territory that
be___ has a high court of its own.
66 TARGET PT 2020

Which of the above statements is/are 1. It was announced in 2015 to


correct? issue soil cards to carry crop wise
(a) 1 only recommendations of nutrients and
fertilisers.
(b) 2 only
2. It indicates soil health by testing it for
(c) Both 1 and 2 only primary nutrients.
(d) Neither 1 nor 2 3. It was launched by Ministry of
Agriculture and Farmers’ Welfare.
21. Consider the following statements with
reference to Protection of Children Which of the above statements is/are
from Sexual Offences (Amendment) correct?
Bill, 2019:
(a) 1 only
1. It proposes death penalty for penetrative
sexual assault on children. (b) 2 and 3 only
2. It redefines child pornography as (c) 3 only
“visual depiction of sexually explicit (d) 1 and 3 only
conduct involving a child”.
3. It considers offences only against 24. With reference to ‘One Nation One
female children as penalizing. Ration Card’ scheme, consider the
following statements:
4. It penalizes administrating of drugs to
make children sexually active. 1. Its objective is to introduce nation-
wide portability of ration card holders
Which one of the following provision is/are under National Food Security Act,
true? 2013 (NFSA).
(a) (a) 1, 2 and 3 only 2. It has been made available across the
country from July 1, 2020.
(b) (b) 2, 3 and 4 only
3. It has been launched by Ministry of
(c) (c) 1, 2 and 4 only Agriculture and Farmers’ Welfare.
(d) (d) 1, 3 and 4 only
Which of the above statements is/are
22. With reference to Sample Registration correct?
System, consider the following (a) 1 only
statements:
(b) 1 and 2 only
1. The sex ratio at birth (SRB) has been
dropping continuously since Census (c) 1 and 3 only
2011. (d) 1, 2 and 3
2. Only two states, Kerala and
Chhattisgarh had an SRB of above 950 25. Consider the following statements
girls per 1000 boys. regarding Economic Census of India:
1. It is carried out by Ministry of Home.
Which of the above statements is/are
incorrect? 2. It gives the count of all establishments
engaged in only non-agricultural
(a) 1 only economic activities located India.
(b) 2 only
Which of the above statements is/are
(c) Both 1 and 2 correct?
(d) Neither 1 nor 2 (a) 1 only
(b) 2 only
23. With reference to Soil Health Card
Scheme, consider the following (c) Both 1 and 2
statements:
(d) Neither 1 nor 2

**********
TARGET PT 2020 67

ANSWER HINTS
DAY - 7

1. Correct Option: (b) 3. Correct Option: (c)


Explanation: Explanation:
Composition of Assembly
State bills and the President
 The legislative assembly consists of
 When a money bill is reserved for representatives directly elected by the
consideration of the President: people on the basis of universal adult
franchise.
! The President may r give his
assent to the bill,  Its maximum strength is fixed at 500 and
minimum strength at 60. It means that its
! Withhold his assent to the bill strength varies from 60 to 500 depending
on the population size of the state.
! But, cannot return the bill for
reconsideration of the state  However, in the case of Arunachal
Pradesh, Sikkim and Goa, the
legislature. minimum number is fixed at 30.

2. Correct Option: (c)  In the case of Mizoram and Nagaland, it is


40 and 46 respectively.
Explanation:
4. Correct Option: (c)
State Assembly Explanation:
 The Chairman of the Legislative  Of the total number of members of a
Council is elected by the council legislative council:
itself from amongst its members. As a ! 1/3 are elected by the members
presiding officer, the powers and functions of local bodies in the state like
of the Chairman in the council are similar municipalities, district boards,
to those of the Speaker in the assembly. etc.,
! 1/12 are elected by graduates of
 The Parliament is authorized to three years standing and residing
modify the composition of a legislative within the state,
council.
! 1/12 are elected by teachers of three
 Like the Lok Sabha, the legislative assembly years standing in the state, not lower
in standard than secondary school,
is not a continuing chamber. Its normal
term is five years from the date of its first ! 1/3 are elected by the members of the
meeting after the general elections. legislative assembly of the state from
amongst persons who are not members
 However, the governor is authorized to of the assembly, and
dissolve the assembly at any time. ! the rest (1/6) are nominated by the
governor from amongst persons who
 Further, the term of the assembly can be
have special knowledge or practical
extended during the period of national experience of literature, science, art,
emergency by a law of Parliament for cooperative movement, and social
one year at a time. service.
68 TARGET PT 2020

5. Correct Option: (b) of the assembly. Such a resolution can be


moved only after giving 14 days’ advance
Explanation: notice.
The Speaker in a State Assembly
7. Correct Option: (a)
 The Speaker has the following powers and
duties: Explanation:
 He maintains order and decorum in the Rajya Sabha and the Legislative Council
assembly for conducting its business and
regulating its proceedings. This is his  Even though both the council and the
primary responsibility and he has final Rajya Sabha are second chambers, the
power in this regard. Constitution has given the council much
lesser importance than the Rajya Sabha
 He is the final interpreter of the due to the following reasons:
provisions of (a) the Constitution
of India, (b) the rules of procedure and  The Rajya Sabha consists of the
conduct of business of the assembly, and representatives of the states and
(c) the legislative precedents, within the thus reflect the federal element of the
assembly. polity. Therefore, it has to be an effective
revising body and not just an advisory body
 He adjourns the assembly or suspends the or dilatory body like that of the council. On
meeting in the absence of a quorum. the other hand, the issue of federal
 He does not vote in the first instance. But, significance does not arise in the case
he can exercise a casting vote in the case of a council.
of a tie.  The council is heterogeneously constituted.
 He can allow a ‘secret’ sitting of the It represents different interests and some
House at the request of the leader of nominated members (1/6 of the total). Its
the House. very composition makes its position weak
and reduces its utility as an effective
 He decides whether a bill is a Money
revising body. On the other hand, the Rajya
Bill or not and his decision on this
Sabha is homogeneously constituted.
question is final.
It represents only the states and
 He decides the questions of disqualification consists of mainly elected members
of a member of the assembly, arising on the (only 12 out of 250 are nominated).
ground of defection under the provisions of
 The council should yield to the assembly,
the Tenth Schedule.
which is a popular house.
 He appoints the chairmen of all the
committees of the assembly and  Thus, the Rajya Sabha is a second chamber
supervises their functioning. He while the Council is a secondary chamber.
himself is the chairman of the Business
Advisory Committee, the Rules Committee 8. Correct option: (b)
and the General Purpose Committee. Explanation:
6. Correct Option: (a) Bills in State Legislative Assembly
Explanation:  The position with respect to lapsing of
bills on the dissolution of the assembly is
Deputy Speaker of Assembly mentioned below:
 Like the Speaker, the Deputy Speaker  A Bill pending in the assembly lapses
is also elected by the assembly itself (whether originating in the assembly or
from amongst its members. He is elected transmitted to it by the council)
after the election of the Speaker has taken
place.  A Bill passed by the assembly but pending
in the council lapses.
 However, he also vacates his office earlier
in any of the following three cases:  A Bill pending in the council but not passed
by the assembly does not lapse.
 if he ceases to be a member of the
assembly;  A Bill passed by the assembly (in a
unicameral state) or passed by both the
 if he resigns by writing to the speaker; and
houses (in a bicameral state) but pending
 if he is removed by a resolution passed assent of the governor or the President
by a majority of all the then members does not lapse.
TARGET PT 2020 69

 Bill passed by the assembly (in a unicameral  When a bill is passed by the legislative
state) or passed by both the Houses (in assembly and transmitted to the legislative
a bicameral state) but returned by the council, the latter has four alternatives
president for reconsideration of Houses before it:
does not lapse.”
! it may pass the bill as sent by the
9. Correct Option: (c) assembly (i.e., without amendments);

Explanation: ! it may pass the bill with amendments


and return it to the assembly for
Structure of Legislative Council reconsideration;
 The maximum Strength of the ! it may reject the bill altogether; and
Legislative Council is fixed at one-
! it may not take any action and thus
third of the total strength of the
keep the bill pending.
Assembly.
 If the council passes the bill without
 The minimum strength is fixed at 40.
amendments or the assembly accepts the
 Though the Constitution has fixed the amendments suggested by the council, the
maximum and the minimum limits, the bill is deemed to have been passed by both
actual strength of a Council is fixed by the Houses and the same is sent to the
Parliament. governor for his assent.

10. Correct Option: (a)  On the other hand, if the assembly rejects
the amendments suggested by the council
Explanation: or the council rejects the bill altogether
or the council does not take any action for
Rajya Sabha and the Legislative Council
three months, then the assembly may
 Like Rajya Sabha, the legislative pass the bill again and transmit the same
council is a continuing chamber, to the council.
that is, it is a permanent body and is not
 If the council rejects the bill again or passes
subject to dissolution. But, one-third of its
the bill with amendments not acceptable
members retire on the expiration of every
second year. So, a member continues as to the assembly or does not pass the bill
such for six years. within one month, then the bill is deemed
to have been passed by both the Houses
 The retiring members are also eligible in the form in which it was passed by the
for re-election and re-nomination any assembly for the second time.
number of times.
 When a bill, which has originated in the
11. Correct Option: (b) council and was sent to the assembly, is
rejected by the assembly, the bill ends and
Explanation: becomes dead.
Disqualification of MLAs  Therefore, the ultimate power of passing
an ordinary bill is vested in the assembly.
 All the questions regarding disqualification
of Members of state legislature are decided At the most, the council can detain or delay
by the Governor in consultation with the the bill for a period of four months—three
Election Commission of India. months in the first instance and one month
in the second instance.
 Speaker has the power to decide questions
of disqualification only under the Tenth  Unlike parliament, there is not
schedule i.e. Anti- defection law. any provision for the joint sitting
over ordinary bills in the State
12. Correct Option: (b) Legislature.

Explanation 13. Correct Option: (a)


Comparison between State Legislative Explanation:
Assembly and Legislative Council
regarding ordinary bill Comparison between Parliament and
State Legislature
 Ordinary Bill can be introduced in
any house of both the Union and State  At Union level, a money bill can be
Legislature. introduced only in the Lok Sabha
70 TARGET PT 2020

and at the state level a money Bill  Quorum is the minimum number of
can be introduced only in the state members required to be present in the
Legislative assembly. House before it can transact any business.
 As in the Parliament, it is sent to the  It is ten members or one-tenth of the
upper house of the State Legislature total number of members of the House
and the time limit within which a money (including the presiding officer),
bill should be returned with or without whichever is greater.
recommendation by Upper Houses both in
case of Parliament and state Legislature is 17. Correct Option: (c)
14 days.
Explanation:
 There is no provision of Joint sitting in case
 Only 6 states viz. Andhra Pradesh,
of disagreement over money bill both in
Telangana, Uttar Pradesh, Bihar,
case of Parliament and state Legislature.
Maharashtra, Karnataka have the
Legislative Councils.
14. Correct Option: (d)
 J&K legislative council, with a strength of
Explanation: 36 members, was created in 1957, is now
abolished.
Comparison between State Legislative
Council and Legislative Assembly  The legislative council of Tamil
Nadu had been abolished in 1986. In
 State Legislature do not enjoy the 2010, the Legislative Assembly of Tamil
privilege to introduce a Constitutional Nadu passed a resolution for the revival
Amendment Bill. of the Legislative Council in the state.
 As per Article 368, a Constitutional Accordingly, the Parliament enacted
Amendment bill which requires the Tamil Nadu Legislative Council Act,
ratification from the states needs 2010 which provided for the creation of
approval only from Legislative Legislative Council in the state. However,
Assembly and that too by a simple before this Act was enforced, the Legislative
majority. Assembly of Tamil Nadu passed another
resolution in 2011 seeking the abolition of
 There is no power to the Legislative Council the proposed Legislative Council.
in case of abolishing the Council itself. The
resolution to abolish the Council has to 18. Correct Option: (c)
be passed by the legislative Assembly
only with a special majority. Explanation:
 The Parliament can make laws on any
15. Correct Option: (d) subject of the three lists (including the
Explanation: State List) for the union territories. This
power of Parliament also extends to
Legislation in a State Legislative Puducherry and Delhi, which have their
Assembly own local legislatures.
 After passing, the ordinary bill goes to  The President can make regulations for
Governor for assent. Here 4 courses of the peace, progress and good government
action arise for the Bill: of the Andaman and Nicobar Islands,
Lakshadweep, Dadra and Nagar Haveli,
! The Governor gives assent to bill and and Daman and Diu. In the case of
it becomes an act Puducherry also, the President can
! Governor withholds the assent legislate by making regulations but
only when the assembly is suspended
! Governor returns the bill, but there or dissolved.
is no time limit specified in the
Constitution for this. 19. Correct Option: (c)
! Reserve the bill for consideration of
Explanation:
the President.
 The Parliament can establish a high court
16. Correct Option: (b) for a union territory.

Explanation:  Also, the Parliament can put it under


the jurisdiction of the high court of
Quorum in a State Assembly the adjacent state.
TARGET PT 2020 71

20. Correct Option: (d) legal protection to 39 per cent of population


or 43 crore children, irrespective of whether
Explanation: they are a girl or boy.
Union Territories  There are cases wherein children were
administered drugs and hormones to make
 The states are the members of the
them sexually active. This bill is aimed at
federal system in India and share a
providing stringent punishment to such
distribution of power with the Centre.
offenders.
 The union territories, on the other  In news: Recently, the landmark legislation
hand, are those areas which are under for child rights, the Protection of Children
the direct control and administration from Sexual Offences (Amendment) Bill,
of the Central government. In this way, 2019, was passed by both Lok Sabha and
the existence of these territories constitutes Rajya Sabha and now requires President’s
a conspicuous departure from federalism assent to become the law.
in India.
 The Parliament can establish a high court 22. Correct option: (d)
for a union territory or put it under the Explanation:
jurisdiction of the high court of adjacent
state. Delhi has a high court of its own Supplementary notes:
since 1966. But after the enactment of
the Jammu and Kashmir Reorganisation Sample Registration System (SRS) data
Act, 2019, both the Union territories findings on sex ratio at birth (SRB)
Jammu & Kashmir and Ladakh have  The sex ratio at birth (SRB) has been
a high court. dropping continuously since Census 2011,
coming down from 909 girls per thousand
21. Correct option: (c) boys in 2011-2013 to 896 girls in 2015-
2017.
Explanation:
 Of the 21 large States, only two — Kerala
 The Protection of Children from Sexual and Chhattisgarh — had an SRB of above
Offences (Amendment) Bill, 2019 aims at 950 girls per 1000 boys
making offences against children gender-
neutral. It provides added legal protection  At present, about 5% of girls are ‘eliminated’
to 39 per cent of population or 43 crore before they are born, despite the promises
children, irrespective of whether they are of the Beti Bachao Beti Padhao scheme.
a girl or boy.
About Sample Registration System
Supplementary Notes;  Office of the Registrar General, India,
under the Home Ministry initiated Sample
 The amendment bill has a number of
Registration System (SRS) in 1964-65 on a
provisions to safeguard children from
pilot basis and on full scale from 1969-70.
offences of sexual assault and sexual
harassment:  The SRS since then has been providing
data on regular basis. It provides accurate,
 For aggravated penetrative sexual assault up-to-date fertility and mortality data via
on children, the bill proposes death penalty registration of births and deaths.
with a minimum of 20 years of rigorous
imprisonment.  It thus helps in an adequate evaluation of
a number of programs in the health sector,
 The bill significantly defines child including family planning, maternal and
pornography as “visual depiction of reproductive health, and immunization
sexually explicit conduct involving a child” programs.
including photograph, video, digital or
computer generated image. 23. Correct Option: (d)
 The punishment of using a child for Explanation:
pornographic purposes is a minimum of five
years of jail term and on a repeat offence  Uniform approach in soil testing is
for seven years along with a provision for adopted for 12 parameters viz. primary
nutrients (NPK); secondary nutrient
fine.
(S); micronutrients (B, Zn, Mn, Fe
 The bill aims at making offences against & Cu); and other (pH, EC & OC) for
children gender-neutral. It provides added comprehensiveness.
72 TARGET PT 2020

 It is promoted by the Department of  It would integrate the existing PDS


Agriculture & Co-operation under the systems/portals of States/UTs with the
Ministry of Agriculture and Farmers’ Central systems/portals.
Welfare.
25. Correct Option: (b)
Soil Health Card Scheme
Explanation:
 Soil Health Card Scheme was launched in
2015 to issue ‘Soil card’ to farmers which  Statement 1 is incorrect: Ministry of
will carry crop-wise recommendations of Statistics and Programme Implementation
nutrients and fertilizers required for the carries out economic census every five
individual farms. years.
 This is aimed to help farmers to improve  Statement 2 is correct: Economic census is
productivity through judicious use of the complete count of all establishments
inputs. engaged in only non-agricultural economic
 It is promoted by the Department of activities located within the geographical
Agriculture & Co-operation under the boundary of India.
Ministry of Agriculture and Farmers’
Welfare. Supplementary Notes:
 It is being implemented through the  Economic census is the complete count
Department of Agriculture of all the State of all establishments engaged in non-
and Union Territory Governments agricultural economic activities located
within the geographical boundary of India.
 It is made available once in a cycle of
3 years, which indicates the status of  The economic census provides disaggregated
soil health of a farmer’s holding for that information on various operational and
particular period. structural variables of all establishments
of the country.
24. Correct Option: (b)
 One of the main aims of the Economic
Explanation: Census is preparation of a National
Business Register which can be linked with
Statement 2 is incorrect: It will be available existing databases at the central and state
across the country from July 1, 2020. government levels.
Supplementary notes:  The seventh census would witness the
use of Information Technology for data
‘One Nation One Ration Card’ scheme
collection.
 Launched by Ministry of Consumer Affairs,
 Ministry of Statistics and Programme
Food and Public Distribution.
Implementation carries out economic
 It will be available across the country from census every five years. The ministry
July 1, 2020. has partnered with CSC e-Governance
 The main objective is to introduce nation- Services India Limited, a Special Purpose
wide portability of ration card holders Vehicle under the Ministry of Electronics
under National Food Security Act, 2013 and Information.
(NFSA)
Significance
 It will allow portability of food security
benefits i.e. to lift their entitlement food  It is a unique initiative in the sense that it
grains from any Fair Price Shop in the will set a precedent for the entire country
country for carrying out a large-scale survey work
like this.
 This means poor migrant workers will be
able to buy subsidised rice and wheat from  Also, planning of schemes would become
any ration shop in the country, so long as more effective with availability of accurate
their ration cards are linked to Aadhaar. data.

**********
TARGET PT 2020 73

TEST
DAY - 8

1. Which of the following Committees had (c) Both 1 and 2


been constituted after the enactment
(d) Neither 1 nor 2
of the 73rd Constitution Amendment
Act, 1992?
4. Which of the following is not a
1. Gadgil Committee compulsory provision of Part IX of the
2. Committee on Minor Forest Produce Constitution?
3. Thungon Committee 1. Reservation of one-third seats (both
4. B. D. Sharma Committee members and chairpersons) for women
in panchayats at all the three levels.
Select the correct option using the codes given
below: 2. Constitution of a State Finance
Commission every five years.
(a) 1 and 2 only
3. Providing reservation of seats (both
(b) 2 and 4 only
members and chairpersons) for
(c) 1 and 3 only backward classes in panchayats at any
(d) 1, 2, 3, and 4 level.
4. Authorizing panchayats them to levy,
2. Consider the following states: collect and appropriate taxes, duties,
1. Goa tolls and fees.
2. Sikkim Select the correct option using the codes given
3. Manipur below:

In which of the above states there is two-tier local (a) 3 only


bodies?
(b) 3 and 4 only
(a) 1 and 2 only
(c) 2 and 4 only
(b) 2 and 3 only
(d) 1, 2, 3, and 4
(c) 1 only
(d) 1, 2, and 3 5. The Panchayati Raj is included in
the___
3. Who of the following statements (a) Union List
regarding 73rd Amendment Act is/are
correct? (b) State List
1. The Act bars the interference by courts (c) Concurrent List
in the electoral matters of panchayats.
(d) Residuary List
2. No election to any panchayat is to
be questioned except by an election 6. Which one of the following is their
petition in such manner as provided by correct chronological order?
the State Election Commission.
(a) 3-4-2-1
Select the correct option using the codes given
below: (b) 2-1-3-4
(a) 1 only (c) 3-1-2-4
(b) 2 only (d) 2-4-3-1
74 TARGET PT 2020

7. Consider the following statements (a) 1 only


regarding State Election
(b) 1 and 2 only
Commissioner:
(c) 2 and 3 only
1. S/he is appointed by the Governor
2. S/he cannot be removed by the (d) 1, 2, and 3
Governor.
10. Which of the following can make the
Which of the above statements is/are correct? provisions to the maintenance of
(a) 1 only accounts by Panchayats?

(b) 2 only (a) State Finance Commission

(c) Both 1 and 2 (b) State Legislature

(d) Neither 1 nor 2 (c) Parliament


(d) District Collector
8. Which of the following is/are not
the objectives of the Panchayats 11. Consider the following statements:
(Extension to Scheduled Areas) Act?
1. A state can prescribe minimum
1. To extend the provisions of Part IX of educational qualifications to contest
the Constitution to the Schedule-VI local body elections.
areas with certain modifications.
2. A state cannot provide more than
2. To mainstream tribal communities. 33% reservation of seats for women in
3. To provide self-rule for tribal Panchayati Raj Institutions (PRIs).
population.
Which of the statements given above are
Select the correct option using the codes given
correct?
below:
(a) 1 only
(a) 2 only
(b) 2 only
(b) 1 and 2 only
(c) Both 1 and 2
(c) 1 only
(d) Neither 1 nor 2
(d) 1, 2, and 3
12. Which one of the following functions
9. Which of the following is/are probable
is not the concern of the Local
cause(s) of the ineffectiveness of the
Government in India?
PRIs in India?
1. The transfer of various governance (a) Public health
functions-like the provision of (b) Sanitation
education, health, sanitation, and
(c) Public utility services
water was not mandated by the 73rd
Amendment Act. (d) Maintenance of public order
2. Lack of finances for PRIs. 13. What should be the minimum
3. Despite there is the mandatory population to set up a metropolitan
provisions for the reservations for committee?
women and SC/STs in the PRIs, there
(a) 1 lakh
is hardly any representation of these
vulnerable groups. (b) 1 million
(c) 10 million
Select the correct option using the codes
given below: (d) None of the above
TARGET PT 2020 75

14. Which one of the following statements (d) 1, 3 and 4


with regard to the State Finance
Commission is not correct? 17. Which of the following statements
1. The provisions of State Finance regarding the Seventy-Fourth
Commission were added by the 73rd Amendment to the Constitution of
Amendment Act. India is/are correct?
2. It recommends principles and 1. It provides for the insertion of Schedule-
methodology as regards the devolution XI to the Constitution.
of funds to the rural local bodies only. 2. It provides for the reservation of seats
for women and Scheduled Castes in the
Select the correct option using the codes municipalities.
given below:
3. These reservations will apply for direct
(a) 1 only elections only.
(b) 2 only 4. Some seats may be filled by nomination
also.
(c) Both 1 and 2
(d) Neither 1 nor 2 Select the correct answer using the codes
given below:
15. Which of the following is/are not (a) 3 only
feature(s) of the Panchayats (Extension
to Scheduled Areas) Act? (b) 1, 2 and 3 only
1. The reservation for the Scheduled (c) 3 and 4 only
Tribes shall be less than one-half (d) 2, 3 and 4 only
but more than one-third of the total
number of seats. 18. Consider the following areas/states:
2. The Gram Sabha or the Panchayats 1. Darjeeling Gorkha Hill Council
shall be consulted before making the
2. Schedule-VI states
acquisition of land.
3. Schedule-V areas
3. Planning and management of minor
water bodies shall be entrusted to 4. Schedule-V states
Panchayats.
In which of the above areas/states, the 74th
4. All seats of Chairpersons of Panchayats Amendment Act is not applied?
at all levels shall be reserved for the
Scheduled Tribes only. (a) 1 only
(b) 1 and 2 only
Select the correct answer using the codes
given below: (c) 1, 2 and 3 only
(a) 1 only (d) 1, 2, 3 and 4
(b) 1 and 4 only 19. Consider the following statements:
(c) 2, 3, and 4 only 1. The Chairperson of every District
Planning Committee shall forward the
(d) 1, 2, and 3 only
development plan to the Governor of
16. Which of the following is/are the source the State.
of revenue for the urban local bodies 2. Not less than two-thirds of the total
in India? number of members of the District
1. Advertisement Taxes Planning Committee shall be elected,
from amongst, the elected members of
2. Life Insurance Corporation of India the Panchayat at the district level and
3. Grants the Municipalities in the district.
4. Entertainment Tax Which of the statements given above is/are
correct?
Select the correct answer from the codes
given below: (a) 1 only
(a) 1 and 2 (b) 2 only
(b) 1 and 3 (c) Both 1 and 2
(c) 1, 2, 3 and 4 (d) Neither 1 nor 2
76 TARGET PT 2020

20. Consider the following statements (a) 1, 2 and 3 only


regarding Metropolitan Planning
(b) 2, 3 and 4 only
Committee:
1. It prepares a draft development plan (c) 1, 2 and 4 only
for the metropolitan area. (d) 1, 3 and 4 only
2. Two-thirds of the members of a
metropolitan planning committee 23. Which one of the following international
should be elected by the elected conventions is not ratified by India?
members of the municipalities and (a) UN Convention on International
chairpersons of the panchayats in Settlement Agreements
the metropolitan area from amongst
(b) UN Convention against Corruption
themselves.
3. The chairpersons of such committees (c) UN Convention against Transnational
shall forward the development plan Organised Crime
to the municipal commissioner of the (d) ILO convention on Abolition of Forced
area. Labour
Which of the statements given above is/are 24. Khanij Bidesh India Ltd. is a joint
incorrect? venture of which of the following
(a) 3 only central public sector enterprises?
1. Hindustan Copper Ltd.(HCL)
(b) 2 only
2. Coal India Ltd.
(c) 1 only
3. National Aluminium Company Ltd
(d) 1 and 3 only
4. Mineral Exploration Company Ltd
21. Consider the following statements
Select the correct answer using the code
regarding the recently introduced
given below:
Jallianwala Bagh National Memorial
(Amendment) Bill, 2019: (a) 1 and 2 only
1. The position of President of the Indian (b) 1, 2 and 4 only
National Congress has changed from
chairperson to permanent member (c) 1, 3 and 4 only
of the Jallianwala Bagh National (d) 2, 3 and 4 only
Memorial.
2. It allows the central government to 25. Which one of the following is not
terminate the term of a nominated the main objective of Jai Bhim
trustee. Mukhyamantri Pratibha Vikas Yojana
launched by the Delhi Government?
Which of the above statements is/are 1. To provide quality coaching for
correct? economically disadvantaged Scheduled
(a) 1 only Castes candidates.

(b) 2 only 2. To provide farm loan waivers to the


small and medium farmers
(c) Both 1 and 2
3. To give free smart phones to the
(d) Neither 1 nor 2 anganwadi workers
4. To establish special government schools
22. Which of the following type of diabetes
for the differently-abled children.
is not an autoimmune disease?
1. Type 1 diabetes Select the correct answer from the codes
2. Type 2 diabetes given below:

3. Pre-diabetes (a) 2 and 3 only


4. Gestational diabetes (b) 2, 3 and 4 only
(c) 1 and 4 only
Select the correct answer from the codes
given below: (d) 1, 3 and 4 only
TARGET PT 2020 77

ANSWER HINTS
DAY - 8

1. Correct Option: (b) intermediate level Panchayat and Zilla


Parishad or district level Panchayat).
Explanation:
 However, a state having a population
Evolution of PRIs in India not exceeding 20 lakh may not
 In 1988, a sub-committee of constitute panchayats at the
the Consultative Committee of intermediate level.
Parliament was constituted under  Consequently, Dadra & Nagar Haveli,
the chairmanship of P.K. Thungon to Daman & Diu, Goa, Lakshadweep,
examine the political and administrative Manipur, Puducherry, and Sikkim have
structure in the district for the purpose the two-tier Panchayati Raj institutions in
of district planning. The Committee India.
recommended for the three-tier Panchayat
system and to provide the constitutional 3. Correct Option: (a)
status.
Explanation:
 The Committee on Policy and Programmes
was constituted in 1988 under the Bar to interference by Courts in Electoral
chairmanship of V.N. Gadgil. This Matters
committee was asked to consider the
question of “how best Panchayati Raj  The Act bars the interference by
institutions could be made effective”. This courts in the electoral matters of
committee too recommended for the three- panchayats. It declares that the validity
tier Panchayat system and to provide the of any law relating to the delimitation of
constitutional status. The committee also constituencies or the allotment of seats to
recommended for the reservation for SC/ such constituencies cannot be questioned
ST and women. in any court.

 Ministry of Panchayati Raj constituted  It further lays down that no election


three sub-committees in 2010, namely the to any panchayat is to be questioned
B. D. Sharma sub-committee on ‘Model except by an election petition
Guidelines to vest Gram Sabhas with presented to such authority and in
Powers as envisaged in PESA; the such manner as provided by the State
Raghav Chandra sub-committee on ‘Land Legislature.
Alienation, Displacement, Rehabilitation
& Resettlement’ and Shri A.K. Sharma 4. Correct Option: (b)
on Minor Forest Produce’. Report and
Explanation:
recommendations of all the sub-committees
have been forwarded to the PESA States. Compulsory provisions of the 73rd
Amendment Act.
2. Correct Option: (d)
 The mandatory provisions in the Act are as
Explanation: follows:
Structure of the PRIs in India ! Organization of Gram Sabha in a
village or group of villages.
 The Constitution (73rd Amendment)
Act, 1992 mandates provisions for the ! Establishment of panchayats at the
establishment of a three-tier structure village, intermediate and district
(Village Panchayat, Panchayat Samiti or levels.
78 TARGET PT 2020

! Direct elections to all seats in performance of PRIs in India. The


panchayats at the village, intermediate Committee recommended a 2-tier system
and district levels. viz. Parishad at the district level and
Mandal Panchayat.
! Indirect elections to the post of
chairperson of panchayats at the  The G.V.K. Rao committee was
intermediate and district levels. appointed in 1985 to recommend an
! 21 years to be the minimum age for integrated concept for growth and poverty
contesting elections to panchayats. alleviation and the performance of the
PRIs.
! Reservation of seats (both members
and chairpersons) for SCs and STs in 7. Correct Option: (c)
panchayats at all the three levels.
Explanation:
! Reservation of one-third seats
(both members and chairpersons)  As per the 73rd Amendment Act, the State
for women in panchayats at all election commission to supervise the
the three levels. election in local bodies.

! Fixing tenure of five years for  It consists of a state election commissioner


panchayats at all levels and holding to be appointed by the governor.
fresh elections within six months
 Though appointed by the Governor,
in the event of supersession of any
S/he cannot be removed by the
panchayat.
Governor.
! Establishment of a State Election
 As per the Act, S/he shall not be removed
Commission for conducting elections
from the office except in the manner
to the panchayats.
and on the grounds prescribed for the
! Constitution of a State Finance removal of a judge of the state high
Commission. court.

5. Correct Option: (b) 8. Correct Option: (b)


Explanation: Explanation:
Panchayati Raj Objectives of the PESA Act
 The term Panchayati Raj in India signifies  The objectives of the PESA Act are as
the system of rural local selfgovernment. follows:
 It has been established in all the states of ! To extend the provisions of Part
India by the Acts of the state legislatures IX of the Constitution relating to
to build democracy at the grass root level. the panchayats to the scheduled
 It is entrusted with rural development. areas (Schedule-V) with certain
modifications.
 It was constitutionalised through the 73rd
Constitutional Amendment Act of 1992. ! To provide self-rule for the bulk
of the tribal population.
 It is in the State List.
! To have village governance with
6. Correct Option: (c) participatory democracy and to make
the gram sabha a nucleus of all
Explanation:
activities.
Evolution of PRIs in India ! To evolve a suitable administrative
 The Balvant Rai Mehta Committee was framework consistent with traditional
constituted in 1957 that recommended a practices.
3-tier Panchayati Raj System in India. ! To safeguard and to preserve the
 Santhanam committee was formed in traditions and customs of tribal
1963 to study the financial matters of the communities.
PRIs.
! To empower panchayats at the
 The Ashok Mehta Committee was appropriate levels with specific powers
constituted in 1977 to study the conducive to tribal requirements.
TARGET PT 2020 79

! To prevent panchayats at the higher 10. Correct Option: (b)


level from assuming the powers and
authority of panchayats at the lower Explanation:
level of the gram sabha. Audit of accounts of Panchayats
9. Correct Option: (b)  As per the Article 243J, the State
Legislature may, by law, make provisions
Explanation: with respect to the maintenance of accounts
by the Panchayats and auditing of such
Causes of ineffectiveness of the PRIs in accounts.
India
 Generally, CAG audits Panchayat Raj
 Except representation of women and Institutions as they are principally funded
other vulnerable groups, the PRIs in India by grants from the State Governments for
have largely been ineffective. Some of the which CAG is the sole auditor.
reasons for this are as follows:
! Lack of adequate devolution: The 11. Correct Option: (a)
voluntary provisions of the devolution
of power to levt tax and duties is the
Explanation:
main reason for the failure of the PRIs
Panchayati Raj Institutions in India
in India.
 The Supreme Court in Rajbala &
! Low level of tax collections:
Ors vs State Of Haryana & Ors,
According to the Economic survey of
2018, Panchayats received 95 per cent 2015 case upheld the Haryana
of their revenues from the devolved Panchayati Raj (Amendment)
funds from the centre/state, while Act, 2015 which introduced the
generating only 5 per cent from own educational qualification as the basis
resources. of disqualification to contest local
body elections.
! Excessive control by bureaucracy:
In some States, the Gram Panchayats  The educational qualification as per
have been placed in a position of the Amendment Act, is specified as
subordination. Hence, the Gram passing the “matriculation examination
Panchayat Sarpanches have to or its equivalent examination from any
spend extraordinary amount of time
recognised institution/board: Provided
visiting Block Offices for funds and/or
that in case of a woman candidate or a
technical approval. These interactions
candidate belonging to Scheduled Caste,
with the Block staff office distort
the role of Sarpanches as elected the minimum qualification shall be middle
representatives. pass: Provided further that in case of a
woman candidate belonging to Scheduled
! Overwhelming dependency Caste contesting election for the post of
on government funding: When
Panch, the minimum qualification shall be
Panchayats do not raise resources
5th pass.
and instead receive funds from
outside, people are less likely  Among the States that already
to request a social audit, which follow 50% reservation for women in
results in an inadequate social Panchayati Raj Institutions (PRIs) are
audit of the PRIs. Bihar(first state to do this), Uttarakhand,
! Poor Infrastructure: A large Madhya Pradesh and Himachal Pradesh.
number of Gram Panchayats in the
country do not have even full time 12. Correct Option: (d)
Secretary. Around 25 percent of the
Gram Panchayats do not have basic Explanation:
office buildings. The database for
PRIs in India
planning, monitoring etc., are lacking
in most of the cases. Also, a large  Schedule XI or XII do nt contain the public
number of elected representatives of order because the maintanance of the
PRIs are a large number of elected law and order is not the concern of the
representatives of PRIs are semi- local bodies in India.
literate or literate and know little
about their roles & responsibilities,  It is under purview of the corresponding
programmes, procedures, systems. state.
80 TARGET PT 2020

13. Correct Option: (c) ! Legislation on Panchayats shall be in


conformity with the customary law,
Explanation: social and religious practices and
traditional management practices of
Metropolitan Committee
community resources;
 The Constitution of India makes it
! Every Gram Sabha to safeguard
mandatory for the States to set up and preserve the traditions and
Metropolitan Planning Committees (MPCs) customs of people, their cultural
in the metropolitan areas of the country. identity, community resources and
 A metropolitan area is defined as an the customary mode of dispute
area having a population of 1 million resolution.
or above. ! The Gram Sabhas have roles and
responsibilities in approving all
14. Correct Option: (b) development works in the village,
Explanation: identify beneficiaries, issue certificates
of utilization of funds; powers to
State Finance Commission control institutions and functionaries
in all social sectors and local plans.
 The provisions for the State Finance
Commission were added by the ! Gram Sabhas or Panchayats at
73rd Amendment Act. They have been appropriate level shall also have
enshrined in article 243-I and 243-Y in powers to manage minor water
Part IX-A and IX-B. bodies; power of mandatory
consultation in matters of land
 They are constituted by Governor every
acquisition; resettlement and
five years.
rehabilitation and prospecting licenses/
 They recommend principles and mining leases for minor minerals;
methodology as regards the devolution power to prevent alienation of
of funds to both rural and urban local land and restore alienated land;
bodies. regulate and restrict sale/consumption
 The recommendations of the state finance of liquor; manage village markets,
commissions include: control money lending to STs; and
ownership of minor forest produce.
! The distribution between the State
and the local bodies of the net proceeds ! Provided that the reservation for
of the taxes, duties, tolls and fees the Scheduled Tribes shall not
leviable by the State. be less than one-half of the total
number of seats;
! Determination of the taxes, duties,
tolls and fees levied or appropriated ! Provided further that all seats of
by the local bodies. Chairpersons of Panchayats at
all levels shall be reserved for the
! The grants-in-aid to the local bodies Scheduled Tribes;
from the Consolidated Fund of the
State. 16. Correct Option: (c)
! Measures needed to improve the Explanation:
financial position of the local bodies,
etc. The sources of revenue of urban local
bodies in India
15. Correct Option: (a)
 The major sources of receipts/income for
Explanation: the Urban Local Bodies are:
Features of the PESA Act ! Own Tax Revenue: Property tax,
OCTROI, Vacant land tax, Tax on
 The provisions of Panchayats with certain
animals, advertisement tax, etc.
modification and exceptions have been
extended to the Schedule V areas viz. the ! Own non-tax revenue: Municipal
ten States where the Panchayats exists in fees, user charges, etc.
the country.
! Borrowing: mainly from LIC, Central
 The provisions of the PESA Act are as governments, banks, and municipal
follows: bonds, etc.
TARGET PT 2020 81

! Grants-in-Aid from Governments.  As per the Article 243-ZC, the provision


! Assigned (Shared) Revenue: of the Act is not to apply to certain areas
Profession Tax, Surcharge on Stamp viz. Scheduled Areas and the Tribal Areas
Duty, Entertainment Tax of Article 244 i.e. Schedule-V areas and
Schedule-VI states.
! Etc.
 It is also not applied to the Darjeeling
17. Correct Option: (d) Gorkha Hill Council of West Bengal.
Explanation:
19. Correct Option: (d)
74 Amendment Act
th
Explanation:
 The Constitution (Seventy Fourth
Amendment) Act, 1992 has introduced a District Planning Committee
new Part IXA and a Schedule-XII in the
 As per the 74th Amendment Act, Every
Constitution. It provides for the constitution
state shall constitute at the district level, a
of three types of Municipalities: (i) Nagar
Panchayats for areas in transition from district planning committee to consolidate
a rural area to urban area; (ii) Municipal the plans prepared by panchayats and
Councils for smaller urban areas; and (iii) municipalities in the district, and to
Municipal Corporations for larger urban prepare a draft development plan for the
areas district as a whole.
 The Schedule XII contains 18 items on which  The state legislature may make provisions
the Urban Local Bodies can make laws. with respect to the following:
 It has given constitutional status to the ! The composition of such committees;
municipalities and brought them under the
justifiable part of the constitution. States ! The manner of election of members of
were put under constitutional obligation such committees;
to adopt municipalities as per system
enshrined in the Constitution. ! The functions of such committees in
relation to district planning; and
 As per the Act, besides the seats filled
by direct elections, some seats may be ! The manner of the election of the
filled by nomination of persons having chairpersons of such committees.
special knowledge and experience in
municipal administration. Persons so  The chairperson of such committee shall
nominated shall not have the right to vote forward the development plan to the state
in the meetings of the municipality. government (not the Governor).
 The proportion of seats to be reserved  Constitution provides that four-fifths of
for SC/ST to the total number of seats the members of a district planning
has to be same as the proportion of the committee should be elected by
population of SC/ST in the municipal
the elected members of the District
area.
Panchayat and municipalities in the
 The reservation has to be made for only district from amongst themselves.
those seats that are to be filled by the direct
elections.  The representation of these members in
the committee should be in proportion
 This Act also provides that not less
than one-third of the total number to the ratio between the rural and urban
of seats reserved for SC/ST shall be populations in the district.
reserved for women belonging to SC/
ST. 20. Correct Option: (a)
 In respect of women, the seats shall be Explanation:
reserved to the extent of not less than
one-third of the total number of seats. Metropolitan Planning Committee
This includes seats reserved for women
belonging to SC/ST. These reservations  Every metropolitan area shall have a
will apply for direct elections only. metropolitan planning committee to
prepare a draft development plan.
18. Correct Option: (c)
 The state legislature may make provisions
Explanation: with respect to the following:

Inapplicability of the 74th Amendment Act  The composition of such committees;


82 TARGET PT 2020

! The manner of election of members of of Opposition, then the Leader of the single
such committees; largest Opposition Party in that House”
will be on the trust.
! The representation in such committee
of the Central government, state  The bill allows the central government to
government and other organizations; terminate the term of a nominated trustee
before the expiry of the period of his term.
! The functions of such committees in
relation to planning and coordination  The bill is introduced so that such
for the metropolitan area; and organizations or trusts could not be
politicized and should instead be
! The manner of election of chairpersons nationalized.
of such committees.
 In news: The Lok Sabha recently passed
 The Constitution lays down that two- Jallianwala Bagh National Memorial
thirds of the members of a metropolitan (Amendment) Bill, 2019 by a voice vote.
planning committee should be
elected by the elected members of the 22. Correct Option: (b)
municipalities and chairpersons of Explanation:
the panchayats in the metropolitan
area from amongst themselves.  Type 1 diabetes is an autoimmune
disease. The immune system attacks
 The chairpersons of such committees and destroys cells in the pancreas, where
shall forward the development plan to insulin is made.
the state government.
Supplementary Notes:
21. Correct option: (b)  The South Asian population, including
Explanation: Indians, have been showing symptoms of
diabetes for at least 11,000 years now.
Option (b) is correct: Besides, the stature of this population
began to reduce and since 7,000 years, has
 The position of chairperson has
fallen by 8.5 cm among males and by 7.7
remained same with the Prime
cm among females, a new study has found.
Minister as in the Act of 1951 whereas,
the position of President of the Indian About Diabetes
National Congress as permanent member
is removed.  Diabetes is a metabolic disease that
causes high blood sugar. The hormone
 The bill allows the central government insulin moves sugar from the blood to cells
to terminate the term of a nominated to be stored or used for energy.
trustee before the expiry of the period
of his term.  With diabetes, body either doesn’t make
enough insulin or can’t effectively use
Supplementary Notes: the insulin it does make.
 The Jallianwala Bagh National Memorial  Untreated high blood sugar from diabetes
(Amendment) Bill, 2019 amends the can damage nerves, eyes, kidneys, and
Jallianwala Bagh National Memorial Act, other organs.
1951 which was passed to erect a National
Memorial in memory of those killed and Different types of diabetes:
wounded in the Jallianwala Bagh massacre  Type 1 diabetes: It is an autoimmune
of over 1,000 people on April 12, 1919. disease. The immune system attacks
 Under the provisions of the Act, the trustees and destroys cells in the pancreas, where
of the Memorial include the Prime Minister insulin is made. It’s unclear what causes
as Chairperson, the Congress president, the this attack. About 10 percent of people
Minister-in-charge of Culture, the Leader with diabetes have this type.
of Opposition in Lok Sabha, the Governor  Type 2 diabetes: It occurs when body
and Chief Minister of Punjab, and three becomes resistant to insulin, and sugar
eminent persons nominated by the Centre builds up in your blood. It stems from
as its members. The bill seeks to remove a combination of genetics and lifestyle
the Congress president as a permanent factors.
member of this trust.
 Pre-diabetes: It occurs when blood
 The bill says that “the Leader of Opposition sugar is higher than normal, but it’s
recognised as such in the House of the not high enough for a diagnosis of type
People, or where there is no such Leader 2 diabetes.
TARGET PT 2020 83

 Gestational diabetes: It is high blood ! Mineral Exploration Company Ltd.


sugar during pregnancy. Insulin- (MECL)
blocking hormones produced by the
 It would carry out identification, acquisition,
placenta cause this type of diabetes.
exploration, development, mining and
processing of strategic minerals overseas
23. Correct Option: (a) for commercial use and meeting country’s
Explanation: requirement of these minerals.

 The Union Cabinet had recently approved  The equity participation between NALCO,
the signing (not ratified) of the UN HCL and MECL is in the ratio of 40:30:30
Convention on International Settlement
Agreements. Significance
 It is a move to ensure a consistent supply
Supplementary Notes: of critical and strategic minerals to Indian
domestic market.
UN Convention on International
Settlement Agreements  In other words, it will ensure mineral
security of the Nation.
 It is also known as the “Singapore
Convention on Mediation”.It provides  It will help in building partnerships with
an efficient and harmonized framework other mineral rich countries like Australia
for cross-border enforcement of settlement and those in Africa
agreements  It would also help in realizing the overall
 It ensures that a settlement reached by objective of import substitution.
parties becomes binding and enforceable  In News - Ministry of Mines is setting
in accordance with a simplified and up a joint venture company namely Khanij
streamlined procedure. Bidesh India Ltd. (KABIL)
 It is an essential instrument in the
facilitation of international trade and 25. Correct Option: (b)
in the promotion of mediation as an Explanation:
alternative and effective method of
resolving trade disputes  It was launched by the Delhi Government
in 2018 to provide quality coaching
India’s Other Initiatives to promote for economically disadvantaged
Mediation Scheduled Castes (SCs) candidates to
enable them to compete in Competitive
 The Government is in the process of examination and succeed in obtaining an
establishing the New Delhi International appropriate job in Public/ Private sector.
Arbitration Centre (NDIAC) as a
statutory body Supplementary Notes:
 Commercial Courts Act, 2015 has been  The scheme will be implemented through the
amended and amendment to the Arbitration reputed institutions/centers run by the
and Conciliation Act, 1996, is currently registered private institutions /NGOS.
underway.
 Only students belonging to SCs residing
 In News - The Union Cabinet had in Delhi & who have passed 10th and
approved the signing of the UN Convention 12th class ( or studying in class XII) from
on International Settlement Agreements the schools in Delhi , having total family
resulting from mediation by India. income of not more than Rs.6.00 lakh per
annum will be eligible under the scheme.
24. Correct Option: (c)
 The ratio of students who have completed
Explanation: their studies from Govt. and Private
Schools to be coached under the Scheme
Supplementary notes: will be 75:25.

Khanij Bidesh India Ltd  Under the scheme, the government


currently pays a maximum of `40,000
 It will be set up with the participation to selected eight institutes for the
of three Central Public Sector coaching of each student. Apart from it, a
Enterprises namely, monthly stipend of `2,500 is given to
! National Aluminium Company Ltd. each student. Now, the amount will be
(NALCO) increased from `40,000 to `1,50,000 and
the duration of the coaching will also be
! Hindustan Copper Ltd.(HCL) extended.
84 TARGET PT 2020

TEST
DAY - 9

Time Allowed: 30 mins Maximum Marks: 50

1. Consider the following statements Supreme Court of India is different from


regarding ‘Advisory Jurisdiction’ of its normal Appellate Jurisdiction?
the Supreme Court of India: 1. It is a discretionary power of the
1. Supreme Court on any question of law Court.
or fact of public importance may refuse
to tender its opinion to the President. 2. It can be filed against the order of a high
court refusing to grant the certificate
2. The opinion expressed by the Supreme of fitness for appeal to the Supreme
Court is a judicial pronouncement, Court.
hence, it is binding on the President.
3. It can be granted in any judgment
3. it facilitates the government to have an
whether final or interlocutory.
authoritative legal opinion on a matter
to be decided by it. Select the correct option using the codes
Which of the above statements is/are given below:
correct? (a) 2 only
(a) 1 and 3 only (b) 1 and 3 only
(b) 1 only (c) 1, 2, and 3
(c) 2 and 3 only (d) 1 only
(d) 1, 2, and 3
4. Which of the following statements is/
2. Which of the following statements is/ are incorrect?
are correct? 1. Only the Supreme Court has the
1. Once a court is madea ‘Court of Record’ originaljurisdiction for any dispute
by statute, the power to punish for between two or more states.
contempt necessarily follows from that
position. 2. Any suit brought by a private citizen
against theCentre or a state is not
2. It also means that the acts and judicial entertained by the Supreme Court.
proceedings are enrolled for a perpetual
memorial and testimony. Select the correct option using the codes
3. Not all High Courts are the ‘Court of given below:
Record’.
(a) 1 only
Select the correct option using the codes (b) 2 only
given below:
(c) Both 1 and 2
(a) 1 only
(d) Neither 1 nor 2
(b) 1 and 2 only
(c) 3 only 5. Consider the following statements
regarding jurisdictions of the Supreme
(d) 1, 2, and 3
Court and High Courts:
3. In which of the following ways,the 1. Judicial review is the exclusive power
‘Appeal by Special Leave’ by the of the Supreme Court.
TARGET PT 2020 85

2. Only the Supreme Court can decide (c) Mahabharata


thedisputes regarding the election of (d) Mundaka Upanishad
the President and the Vice-President.
3. Only the Supreme Court can have the 9. Consider the following statements
‘Special Leave Petition’ against the regarding Lokpal in India:
decision by the Military Court and 1. It has been given constitutional
Tribunals. status.

Which of the above statements is/are 2. It has a wide jurisdiction including


Prime Minister of the country.
correct?1 only
3. The Lokpal has been the authority to
(a) 2 only take Suomoto cognizance of the cases
(b) 2 and 3 only of corruption.
(c) 1 and 3 only Which of the above statements is/are
(d) None of the above incorrect?
(a) 1 only
6. Which of the following statements is/
(b) 1 and 3 only
are correct?
1. The Supreme Court does not have (c) 2 and 3 only
jurisdiction toadjudicate water (d) 1 and 2 only
disputes.
10. Consider the following statements:
2. The Writ Jurisdiction is both original
as well as appellate. 1. The concept of curative petition was
first evolved by the Supreme Court of
Select the correct option using the codes India in the Menaka Gandhi Case.
given below: 2. It is the last judicial resort available
(a) 1 only for redressal of grievances in Court and
enjoyed after the ‘Review Petition’.
(b) 2 only
3. The court could impose “exemplary
(c) Both 1 and 2 costs” to the petitioner if his plea lacks
(d) Neither 1 nor 2 merit.

Which of the above statements is/are


7. In which of the following cases High correct?
Court has the original jurisdiction?
(a) 1 only
1. Disputes relating to the election of
members of Parliament and State (b) 2 and 3 only
legislatures. (c) 1 and 3 only
2. Enforcement of fundamental rights of (d) 1, 2, and 3
citizens.
3. Any Dispute between the Centre and 11. The jurisdiction of which of the
any state. following states is under the purview
of Gauhati High Court?
Select the correct answer using the code 1. Arunachal Pradesh
given below.
2. Mizoram
(a) 1 and 2 only
3. Manipur
(b) 1 only 4. Nagaland
(c) 2 and 3 only
Select the correct option using the codes
(d) 1, 2 and 3 given below:
(a) 2 and 3 only
8. The logo of the Supreme Court, “Yato
Dharma Tato Jaya” is taken from_____ (b) 1 and 4 only
(a) Rigveda (c) 1, 2, and 4 only
(b) Katha Upanishad (d) 2, 3, and 4 only
86 TARGET PT 2020

12. Consider the following statements 3. The Supreme Court itself never has
regarding appointment of judges: recommended the establishment of
1. The President can return the name for this.
reconsideration of the Supreme Court. Select the correct option using the codes
2. The President can consult the Judges given below:
of the High Courts of States to appoint (a) 1 only
a Supreme Court Judge.
(b) 1 and 3 only
Which of the above statements is/are
(c) 2 and 3 only
incorrect?
(d) 1, 2, and 3
(a) 1 only
(b) 2 only 16. Which of the following statements
regarding LokAdalats is incorrect?
(c) Both 1 and 2
(a) It is composed only of judicial officers.
(d) Neither 1 nor 2
(b) An award made by it is final and
13. Which of the following statements binding on the parties to the dispute.
regarding the removal of judges is (c) It is deemed to be a civil court under
correct? the Civil Procedure Court.
1. A removal motion has to be signed by at (d) All the above
least 100 members in the RajyaSabha
or the LokSabha. 17. Which of the following statements
2. The Speaker/Chairman has toadmit regarding Tribunal is/are correct?
the motion for the removal of a Judge 1. It is a quasi-judicial authority.
of the Supreme Court if it has been 2. It follows the procedures as laid down
passed by the House. under the Civil Procedure Code.
3. The grounds of removal is the violation 3. It was added to the Constitution based
of the Constitution. on the recommendation of Swaran
4. If the charge is proved by the Inquiry Singh Committee.
Committee, the Parliament is not
Select the correct option using the codes
bound to remove the Judge.
given below:
Select the correct option using the codes (a) 2 and 3 only
given below:
(b) 1 and 3 only
(a) 4 only
(c) 1 only
(b) 2 and 4 only
(d) 1, 2, and 3
(c) 2 and 3 only
18. Consider the following statements:
(d) 1, 2, and 4 only
1. The appointment ofdistrict judges in
14. Fourth Judges Case is related to ____ a state are made bythe President in
consultation with the state high court.
(a) Removal of Judges
2. Other than District Judge, all the other
(b) Appointment of Judges officers in the judicial service of the
(c) Transfer of Judges state are appointed by the Governor in
consultation of the State Public Service
(d) Public Interest Litigation Commission.

15. Which of the following statements Which of the above statements is/
regarding the ‘National Court of areincorrect?
Appeal’ is/are correct? (a) 1 only
1. The Supreme Court always sits in
(b) 2 only
Delhi as per the Constitution.
(c) Both 1 and 2
2. To establish it, there must be a
Constitutional Amendment for this. (d) Neither 1 nor 2
TARGET PT 2020 87

19. Consider the following statements: Which of the above statements is/are
1. Presently, the total number of judges correct?
in the Supreme Court is 34. (a) 1 only
2. Only the President has the power
(b) 2 only
to increase the number of Judges in
India. (c) Both 1 and 2
Which of the above statements is/are (d) Neither 1 nor 2
correct?
23. Consider the following statements
(a) 1 only
regarding Monetary Policy Committee
(b) 2 only (MPC)
(c) Both 1 and 2 1. It constitutes members from Reserve
(d) Neither 1 nor 2 Bank of India only.
2. It is required to meet at least four times
20. Which of the following statements is/ in a year.
are correct?
3. It decides the key policy interest rate
1. The Supreme Court does not enjoy the
i.e Bank Rate.
Right to Privacy.
2. The office of Chief Justice of India is Which of the above statements is/are
“public authority” and falls under the correct?
purview of the Right to Information
Act. (a) 3 only
(b) 2 only
Select the correct option using the codes
given below: (c) 1 and 3 only
(a) 1 only (d) 1 and 2 only
(b) 2 only
24. Consider the following statements
(c) Both 1 and 2 about Civilian Awards in India:
(d) Neither 1 nor 2 1. Bharat Ratna can be given
posthumously but Padma Awards are
21. Which one of the following program normally not conferred posthumously.
launched by Atal Innovation Mission
(AIM) fostering innovation involves 2. Government servants are not eligible
participation of Panchayati Raj for Padma Awards.
Institutions?
Which of the above statements is/are
(a) Atal Tinkering Labs correct?
(b) Atal Incubators
(a) 1 only
(c) Atal New India Challenges and Atal
(b) 2 only
Grand Challenges
(d) Atal Community Innovation Centre (c) Both 1 and 2
(d) Neither 1 nor 2
22. Consider the following statements
regarding Anti-Defection Law in 25. Who is empowered to decide what
India provisions of Indian Constitution
1. Both, the Speaker and the Chairman apply to J&K?
of the House have the power to decide
the disqualification of a member of the (a) President after parliament approves
respective house. (b) Cabinet alone
2. Resignation from the political party is (c) State legislature of J&K alone
the only criteria to ascertain whether
the member has voluntarily given up (d) President along with State legislature
the membership of his party or not. of J&K
88 TARGET PT 2020

ANSWER HINTS
DAY - 9

1. Correct Option: (a) to punish for contempt necessarily


follows from that position”.
Explanation:
Advisory Jurisdiction of the Supreme Court
3. Correct Option: (c)

 Article 143 authorizes the president to Explanation:


seek the opinion of the Supreme Court in Appeal by Special Leave
the two categories of matters:
 Special leave petition is a power of the
! On any question of law or fact of public Supreme Court of India granted by
importance which has arisen or which Constitution Article 136.
is likely to arise.
 It can be filed against any judgment of a
! On any dispute arising out of any high court within 90 days from the date of
pre-constitution treaty, agreement, judgment, or It can be filed within 60 days
covenant, engagement, etc. against the order of a high court refusing
 In the first case, the Supreme Court to grant the certificate of fitness for
may tender or may refuse to tender its appeal to the Supreme Court, whereas,
opinion to the President. But, in the the other Appellate Jurisdiction is if the
second case, the Supreme Court ‘must’ high court certifies that the case needs to
tender its opinion to the president. be decided by the Supreme Court.

 In both cases, the opinion expressed by  It can be granted in any judgment


the Supreme Court is only advisory and whether final or interlocutory whereas,
not a judicial pronouncement. Hence, the normal Appellate Jurisdiction is
it is not binding on the President. enjoyed only after the final judgment
by the high court.
 However, it facilitates the government
 It is a discretionary power of the
to have an authoritative legal opinion
Court and hence, cannot be claimed
on a matter to be decided by it.
as a matter of right,unlike the normal
Appellate Jurisdiction.
2. Correct Option: (b)
Explanation: 4. Correct Option: (d)
Explanation:
Court of Record
 A court whereof the acts and judicial Original Jurisdiction of the Supreme
proceedings are enrolled for a Court
perpetual memorial and testimony, and  As a federal court, the Supreme Court
which has power to fine and imprison decides the disputes between different
for contempt of its authority. units of the Indian Federation. The dispute
can be between:
 Article 129 provides that the Supreme
Court shall be a court of record and Article ! the Centre and one or more states; or
215 contains a similar provision in respect ! the Centre and any state or states on
of the High Court thus, both the Supreme one side and one or more states on the
Court as well as the High Courts are other; or
courts of record.
! between two or more states.
 Dr. Ambedkar in Constituent Assembly
said that “once you make a court a  In the above federal disputes, the
court of record by statute, the power Supreme Court has exclusive original
jurisdiction.
TARGET PT 2020 89

 But, the questions of political nature are ! Cases ordered to be transferred from
excluded from it and any suit brought a subordinate court involving the
before the Supreme Court by a private interpretation of the Constitution to
citizen against the Centre or a state its own file.
cannot be entertained under this.
 The four high courts (i.e., Calcutta,
Bombay, Madras, and Delhi High Courts)
5. Correct Option: (a)
have original civil jurisdiction in cases of
Explanation: higher value.

Comparison between the powers of the  Supreme Court holds the original
Supreme Court and high courts jurisdiction in case of any dispute between
the Centre and any state.
 Both the Supreme Court and High
Courts have the power of Judicial 8. Correct Option: (c)
Review. But the judgments of high courts
can be rejected or modified or upheld by the Explanation:
Supreme Court.
YatoDharmastato Jaya
 The Supreme Court decides the
disputes regarding the election of  It has been taken from Bhagavad Gita, a
the President and the Vice-President. part of Mahabharata.
In this regard, it has the original,  It means “Where there is Dharma
exclusive and final authority. (Righteousness), there will be Victory”.
 Supreme Court too, does not have the  The logo of the Supreme Court of India is the
power of the ‘Special Leave Petition’ Asoka Chakra but unlike the national logo
against the decision by the Military which has “SatyamevaJayate” inscribed on
Court and Tribunals. it, the Supreme Court logo inscribes “Yato
Dharma Tato Jaya”.
6. Correct Option: (b)
Explanation: 9. Correct Option: (b)

Powers of the Supreme Court


Explanation:

 The Supreme Court does not have Lokpal in India


the original jurisdiction over Inter-  Lokpal will consist of a chairperson and a
state water disputes but the aggrieved maximum of eight members, of which 50%
party(s) can approach the Court via
shall be judicial members and 50% shall be
appeal.
from SC/ST/OBCs, minorities and women.
 Writ Jurisdiction is both original ( if
 It is just a statutory position after the
any individual approach the Supreme
enactment of the Lokpal and Lokayuktas
Court directly) as well as appellate.
(Amendment) Bill, 2016.
7. Correct Option: (a)  The Lokpal will cover all categories of
public servants, including the Prime
Explanation: Minister. But the armed forces do not
Original Jurisdiction of High Court come under the ambit of Lokpal.

 Original Jurisdiction means the power of  The Lokpal has been deprived of
a high court to hear disputes in the first the authority of taking suomoto
instance, not by way of appeal. It extends cognizance of the cases of corruption
to the following: and maladministration.

! Matters of admiralty, will, marriage, 10. Correct Option: (b)


divorce, company laws and contempt
of court. Explanation:
! Disputes relating to the election Curative petition
of members of Parliament and
 The curative petition is fairly a new concept
state legislatures.
in the Indian legal system.
! Regarding revenue matter or an act  The concept of curative petition was first
ordered or done in revenue collection.
evolved by the Supreme Court of India
! Enforcement of fundamental in the matter of Rupa Ashok Hurra vs.
rights of citizens. Ashok Hurra and Anr. (2002).
90 TARGET PT 2020

 The Supreme Court in the above case held 13. Correct Option: (a)
that in order to prevent abuse of its process
and to cure gross miscarriage of justice, it Explanation:
may reconsider its judgments in exercise of
Removal of judges
its inherent powers. For this purpose, the
Court has devised what has been termed as  A judge of the Supreme Court can be
a “curative” petition. removed from his office by an order of the
 It is the last judicial resort available President.
for redressal of grievances in court,  The grounds of removal are two viz. proved
enjoyed after the review petition. misbehavior or incapacity.
 It is normally decided by judges in-chamber.  The procedure for this has been elaborated
 No time limit is given for filing Curative in the Judges Enquiry Act of 1968 which
petition. are as follows:
 If the majority of the judges on the above ! A removal motion signed by 100
bench agree that the matter needs hearing, members (in the case of LokSabha)
then it would be sent to the same bench or 50 members (in the case of
(as far as possible) and the court could RajyaSabha) is to be given to the
impose “exemplary costs” to the Speaker/Chairman.
petitioner if his plea lacks merit.
! The Speaker/Chairman may admit
11. Correct Option: (c) the motion or refuse to admit it.

Explanation: ! If it is admitted, then the Speaker/


Chairman is to constitute a three-
Gauhati High Court member committee to investigate the
charges.
 The Gauhati High Court has the
jurisdiction for Arunachal Pradesh, ! The committee should consist of (a)
Assam, Mizoram, and Nagaland. the chief justice or a judge of the
Supreme Court, (b) a chief justice of
 Its principal seat is at Guwahati in
a high court, and (c) a distinguished
Assam.
jurist.
 The Court has 3 outlying benches at
Kohima, Aizawl, and Itanagar. ! If the committee finds the judge
to be guilty of misbehavior or
12. Correct Option: (d) suffering from an incapacity, the
House can or cannot take up the
Explanation: consideration of the motion.

Appointment of judges of the Supreme ! After the motion is passed by each


Court House of Parliament by a special
majority, an address is presented
 The judges of the Supreme Court are
to the President for removal of the
appointed by the President.
judge.
 The chief justice is appointed by the
! Finally, the President passes an order
President after consultation with such
removing the judge.
judges of the Supreme Court and high
courts as s/he deems necessary.
14. Correct Option: (b)
 The other judges are appointed by President
after consultation with the collegium. Explanation:
 The president can consult the judged of Fourth Judges Case
the high courts ifdeemed necessary.
 The Supreme Court Advocates-on-
 In this matter, the President can Record-Association and another Vs.
return the name for reconsideration Union of India is also popularly known
by the collegium, but in the second as the Fourth judges’ case,
instance, s/he has to to appoint.
 It is related to the independence of the
 The Third Judges Case, 1998 ruled that the judiciary in the appointment of judges.
advice tendered by the collegium (plurality
of judges) is binding on the President in the  By this, the Supreme Court declared the
matters of appointment of the judges of the 99th Amendment Act and NJAC Act, 2014
Supreme Court. to be “unconstitutional and void”.
TARGET PT 2020 91

15. Correct Option: (a) Article 323 B dealing with tribunals for
other matters. It was added Constitution
Explanation: based on the recommendation of
Swaran Singh Committee.
National Court of Appeal
 It is a quasi-judicial body.
 The Supreme Court itself, as early as in 1986,
had recommended the establishment  They do not have to follow any uniform
of an NCA with regional Benches at procedure as laid down under the
Chennai, Mumbai and Kolkata to ease Civil Procedure Code and the Indian
the burden of the Supreme Court and Evidence Act but they have to follow
avoid hardship to litigants who have to the principles of Natural Justice.
come all the way to Delhi to fight their
cases. 18. Correct Option: (a)
 As per Article 130 of the Constitution, the Explanation:
Supreme Court always sits in Delhi, but
it may sit at the place(s) other than Delhi Subordinate courts
also.  The appointment, posting andpromotion of
district judges in a state are made by the
 The 229th report of the Law Commission too,
governor of the state inconsultation with
suggested retaining the New Delhi bench
the high court.
of the Supreme Court as a Constitutional
court and establishing Cassation Benches of  Appointment of persons (other thandistrict
the Supreme Court at New Delhi, Chennai/ judges) to the judicial service of a state are
Hyderabad, Kolkata and Mumbai. made by the governor ofthe state after
consultation with the State Public
 The report viewed that no Service Commission and thehigh court.
constitutional amendment is required
since the Article 130 of the Constitution  The subordinate courts are under
provides that the Supreme Court can administrative control of the High Court
be located in Delhi or at any other of the State to which the district concerned
place as the Chief Justice of India belongs. The decisions of District court are
subject to the appellate jurisdiction of the
with the approval of the President
High court.
may decide from time to time.
19. Correct Option: (a)
16. Correct Option: (a)
Explanation:
Explanation:
Supreme Court
LokAdalat
 As per the Article 124(1), only Parliament,
 Every LokAdalat consists of such number by law, has the power to increase the
of serving or retired judicial officers number of Supreme Court Judges.
and other persons of the area as may
be specified by the agency organizing such  Recently, it has been increased to 34
LokAdalat. Generally, a LokAdalat consists including the CJI.
of a judicial officer as the chairman and a
lawyer and a social worker as members. 20. Correct Option: (b)
 Every award made by a LokAdalat Explanation:
shall be final and binding on all parties
Supreme Court
to the dispute. No appeal shall lie in any
court against the award of LokAdalat.  A five-member bench led by CJI
RanjanGogoi upheld the judgment of the
 The LokAdalat has the same powers Delhi High Court in 2010, that the CJI’s
as are vested in a civil court under the office was under the ambit of the RTI.
CPC(1908).
 The Court also held that Judicial
17. Correct Option: (b) independence and Right to privacy has
to be kept in mind while dealing with
Explanation: transparency.

Tribunals  Although the Constitution does not


explicitly include the right to privacy,
 The 42nd Amendment Act of 1976 added a the Supreme Court has found that
new Part XIV-A, entitled as ‘Tribunals’ and the Constitution implicitly grants a
consists of only two Articles—Article 323 A right to privacy against governmental
dealing with administrative tribunals and intrusion.
92 TARGET PT 2020

21. Correct option: (d)  Statement 2 is incorrect: Voluntarily


giving up the membership is not the same
Explanation: as resigning from a party. An inference
 Option (d) is correct: Atal Community can also be drawn from the conduct of
Innovation Centre (ACIC)connects every the member that he has voluntarily given
Panchayati Raj Institution to help up the membership of his party.
innovators at grassroots level.
Supplementary Notes:
Supplementary Notes:
Anti-Defection provisions under the
 Atal Community Innovation Centre Tenth Schedule
(ACIC): This program is launched to foster
community innovation in underserved and Disqualification
un-served areas of the country.  If a member of a house belonging to a
 It can be connected to every Panchayati political party:
Raj Institutions to help innovators at ! Voluntarily gives up the membership
grassroots level become part of the policy of his political party, or
framework and leverage their creativity to
translate their products and services into ! Votes, or does not vote in the
innovation led commercial utilization for legislature, contrary to the directions
society. of his political party.

 The significance of innovation can be ! However, if the member has taken


understood by the fact that through prior permission, or is condoned by the
innovation India can become a USD 5 party within 15 days from such voting
trillion economy by 2024-25. or abstention, the member shall not
be disqualified.
 ACIC can serve as the bridge between the
knowledge base existing in communities  If an independent candidate joins a political
and the advanced technical ecosystem party after the election.
prevalent in the market base, addressing  If a nominated member joins a party six
the needs of society. months after he becomes a member of the
legislature.
 Atal Tinkering Labs –this programme
promotes creative, innovative mind set
in schools across all districts across the
23. Correct Option: (b)
country. Explanation:
 Atal Incubators – this programme  Statement 1 is incorrect: It is a six-
promotes entrepreneurship in universities member committee constituted by the
and industry. This has potential to create Central Government. Of these, three
thousands of job creators. Women led members are from RBI while three
incubators and entrepreneurial startups are other members are nominated by the
strongly encouraged by the programme. Central Government.
 Atal New India Challenges and Atal  Statement 3 is incorrect: It is mandate
Grand Challenges – this programme to decide the key policy interest rate, i.e
promotes technology driven innovations Repo Rate, required achieving the inflation
and product creation for social and target.
commercial impact. The challenges will be
launched in specific areas and sectors of Supplementary Notes:
national importance - such as Renewable
Monetary Policy Committee (MPC)
Energies, Energy Storage, Climate-smart
precision agriculture, Universal drinking  The MPC is required to meet at least four
water, Swaach Bharat, Transportation, times in a year.
Education, Healthcare using Robotic, IOT  The quorum for the meeting of the MPC is
technologies, Artificial Intelligence, Block- four members.
chain, Augmented and Virtual reality,
Battery Technologies etc.  Each member of the MPC has one vote,
and in the event of an equality of votes, the
 In news: Recently, the Atal Community Governor has a second or casting vote.
Innovation Centre (ACIC) program
was launched by Atal Innovation Mission  Repo Rate -It is the interest rate at which
(AIM). the Reserve Bank provides overnight
liquidity to banks against the collateral of
22. Correct Option: (a) government and other approved securities
under the liquidity adjustment facility
Explanation: (LAF).
TARGET PT 2020 93

 Reverse Repo Rate - The interest rate at deserving cases, the Government could
which the Reserve Bank absorbs liquidity, consider giving an award posthumously.
on an overnight basis, from banks against
 The total number of awards to be given in a
the collateral of eligible government
year (excluding posthumous awards and to
securities under the LAF.
NRI/foreigners/OCIs) should not be more
 Liquidity Adjustment Facility (LAF) than 120.
- The LAF consists of overnight as well
 The award does not amount to a title and
as term repo auctions. Progressively,
cannot be used as a suffix or prefix to
the Reserve Bank has increased the
the awardees’ name.
proportion of liquidity injected under
fine-tuning variable rate repo auctions of  All nominations received for Padma Awards
a range of tenors. The aim of term repo are placed before the Padma Awards
is to help develop the inter-bank term Committee, which is constituted by the
money market, which in turn can set Prime Minister every year.
market-based benchmarks for pricing of
 The Padma Awards Committee is
loans and deposits, and hence improve
headed by the Cabinet Secretary and
the transmission of monetary policy.
includes Home Secretary, Secretary to the
The Reserve Bank also conducts variable
President and four to six eminent persons
interest rate reverse repo auctions, as
as members.
necessitated under the market conditions.
 The recommendations of the committee are
24. Correct Option: (c) submitted to the Prime Minister and
the President of India for approval.
Explanation:
25. Correct Option: (d)
Supplementary Notes:
Explanation:
Bharat Ratna Awards
 Option (d) is correct: Article 370 of the
 Bharat Ratna is the highest civilian award Constitution of India describes it as a
of the country. temporary provision and that it will cease to
 It is awarded in recognition of exceptional be operative if the President issues a public
service/performance of the highest order in notification to that effect. However, prior to
any field of human endeavour. that, a recommendation is necessary from
the Constituent Assembly of Jammu and
 It is treated on a different footing from Kashmir.
Padma Award.
Supplementary Notes:
 The recommendations for Bharat Ratna
are made by the Prime Minister to the  Article 370 of the Constitution of India
President of India describes it as a temporary provision
and that it will cease to be operative if
 The number of Bharat Ratna Awards is
the President issues a public notification
restricted to a maximum of three in a
to that effect. However, prior to that, a
particular year.
recommendation is necessary from the
 It can be given posthumously. Constituent Assembly of Jammu and
Kashmir.
PADMA AWARDS
 As a result of Article 370, Jammu and
 The Padma Awards are one of the highest Kashmir had its own Constitution, and
civilian honours of India announced all laws passed by Parliament will not be
annually on the eve of Republic Day. applicable to the State, unless the State
 The Awards are given in three categories: government gives its concurrence.

! Padma Vibhushan - for exceptional  The President is empowered to decide


and distinguished service what provisions of the Constitution of
India would be applicable to the State and
! Padma Bhushan – for distinguished what are the exceptions, but with the State
service of higher order government’s concurrence.
! Padma Shri – for distinguished  The Constitution (Application to Jammu
service. and Kashmir) Order, 1954, lists the Articles
and provisions that apply to J&K. Further,
 Government servants including those working
the President also listed a set of exceptions
with PSUs, except doctors and scientists, are
under Article 35A of the Constitution (this
not eligible for these Awards.
Article does not figure in the text of the
 The award is normally not conferred Constitution of India, but figures only in
posthumously. However, in highly the J&K’s Constitution).
94 TARGET PT 2020

TEST
DAY - 10

Time Allowed: 30 mins Maximum Marks: 50

1. Which of the following statements is/ 1. He is barred from holding any


are incorrect regarding the office of government office after completion of
Comptroller & Auditor General (CAG) his tenure.
of India? 2. His salary except that of the staff
1. It has more freedom with regard to the is charged on the Public Account of
audit of expenditure than the audit of India.
receipts, stores and stock.
Select the correct answer using the code
2. It cannot conduct the propriety audit. given below:
3. It has no control over the issue of money (a) 1 only
from the Consolidated Fund of India.
(b) 2 only
Select the correct answer using the code
(c) Both 1 and 2
given below:
(d) Neither 1 nor 2
(a) 1 only
(b) 2 and 3 only 4. Which of the following are correct
regarding the duties and functions of
(c) 1 and 3 only CAG as laid down by the Parliament
(d) 2 only and the Constitution of India?
1. He audits balance sheets and other
2. Consider the following statements subsidiary accounts kept by any
regarding the removal procedure of department of central and state
Comptroller and Auditor General government.
of India:
2. He can audit the accounts of Panchayati
1. He can be removed on the grounds of Raj Institutions and Urban Local
proved misbehaviour or incapacity. Bodies on request of the President or
2. He is removed by the President of the Governor.
India on the basis of resolution passed 3. He ascertains and certifies the net
with special majority by both Houses of proceeds of any tax or duty.
Parliament.
Select the correct answer using the code
Which of the above statements is/are given below:
correct? (a) 1, 2 and 3 only
(a) 1 only (b) 2 and 3 only
(b) 2 only (c) 1 and 2 only
(c) Both 1 and 2 (d) None of the above
(d) Neither 1 nor 2
5. Consider the following statements
3. Under the Indian Constitutional regarding Comptroller & Auditor
Scheme how the independence of General (CAG) of India:
Comptroller and Auditor General 1. President of India prescribes the duties,
(CAG) is secured? exercise and power of CAG.
TARGET PT 2020 95

2. CAG maintains the accounts of the (c) 2 only


Central Government.
(d) 1, 2 and 3
3. Except Contingency Fund of India, CAG
audits and reports on all expenditure 8. Consider the following statements
from the accounts including Public regarding the features of Cabinet
accounts of the Union and of the states. Committees:
Which of the above statements is/are 1. They are constitutional in nature.
incorrect? 2. They can include Cabinet as well as
(a) 1 and 2 only Non-Cabinet Ministers.

(b) 2 only 3. They not only sort out issues


and formulate proposals for the
(c) 2 and 3 only consideration of the Cabinet, but also
(d) 1, 2 and 3 take decisions.

6. Which of the following statements are Which of the above statements are correct?
correct pertaining to Consultative (a) 1 and 2 only
Committee?
(b) 2 and 3 only
1. It provides a forum for informal
discussion between the ministers and (c) 1 and 3 only
the Members of the Parliament on
(d) 1, 2 and 3
policies and programs of the government
and manner of their implementation. 9. Consider the following statements
2. These committees are constituted by regarding the Public Account
the Ministry of Parliamentary Affairs. Committee in Indian Parliamentary
3. These committees shall stand dissolved System:
upon dissolution of every Lok Sabha 1. This committee was set up first in 1941
and shall be reconstituted upon under the provisions of the Government
constitution of each Lok Sabha. of India Act of 1935.
Select the correct answer using the code 2. At present, it consists of 22 members.
given below:
Which of the above statements is/are
(a) 1 and 3 only correct?
(b) 2 and 3 only (a) 1 only
(c) 1 and 2 only (b) 2 only
(d) 1, 2 and 3 (c) Both 1 and 2
7. Consider the following statements (d) Neither 1 nor 2
with regard to the Parliamentary
committees: 10. Consider the following statements
1. Committee on Public Undertaking regarding the Estimates Committee:
consists of 22 members where the term 1. Estimates committee has
of office does not exceed two years. representation only from Lok Sabha.
2. Committee on Estimates consists of 30 2. It examines the budget estimates
members where the term of office does before they have been voted by the
not exceed one year. Parliament.
3. Committee on Public Accounts
comprises of 25 members where the Which of the above statements is/are
term of office remains one year. correct?
(a) 1 only
Which of the above statements is/are
correct? (b) 2 only
(a) 1 and 3 only (c) Both 1 and 2
(b) 2 and 3 only (d) Neither 1 nor 2
96 TARGET PT 2020

11. Consider the following statements 3. Ethics Committee


regarding cabinet committees: 4. Rules Committee
1. The non-cabinet ministers can also be 5. General Purpose Committee
a member of Cabinet Committees.
2. All the Cabinet committees are headed Select the correct answer using the codes
by the Prime Minister. given below:
(a) 1 and 3 only
Which of the above statements is/are
correct? (b) 2, 4 and 5 only
(a) 1 only (c) 1, 4 and 5 only
(b) 2 only (d) 2 and 3 only
(c) Both 1 and 2 15. Which of the committees is are
(d) Neither 1 nor 2 joint committee of both Houses of
Parliament?
12. Consider the following statements 1. Estimates Committee
regarding cabinet committees:
2. Public Accounts Committee
1. Currently, there are eight Cabinet
Committees. 3. Committee on Public Undertakings
2. Only two members are in the Cabinet 4. Departmentally Related Standing
Committee on Appointment. Committees
3. Cabinet Committee on Security is also Select the correctoption using the codes
called as the ‘Super Cabinet’. given below:
Which of the above statements is/are (a) 1 and 4 only
correct? (b) 2 and 3 only
(a) 1 only (c) 2, 3, and 4 only
(b) 1 and 2 only (d) 3, and 4
(c) 3 only
16. Committee on Members of Parliament
(d) 1, 2, and 3 Local Area Development Scheme
(MPLADS) is a___
13. Which of the following statement(s) is/
are correct about the Public Accounts (a) Financial Committee
Committee? (b) Ad-hoc committee
1. It has representation from both the (c) Standing Committee on
Houses of the Parliament.
(d) None of the above
2. Its chairman is always selected from
the ruling party. 17. Which of the following statements
3. It consists of at least 30 members. is/are correct with regard to the
Committee on Public Undertakings?
Select the correct answer using the code
1. The members are elected every
given below:
year from both the houses of the
(a) 1 only parliament.
(b) 2 only 2. The chairman of this Committee can
be only from Lok Sabha.
(c) 1 and 3 only
(d) 2 and 3 only Select the correct answer using the code
given below:
14. Which of the following Parliamentary (a) 1 only
committees are chaired by the
Speaker? (b) 2 only
1. Public Accounts Committee (c) Both 1 and 2
2. Business Advisory Committee (d) Neither 1 nor 2
TARGET PT 2020 97

18. Consider the following statements (b) 2 and 3 only


regarding the Estimates Committee:
(c) 1 and 2 only
1. Estimates committee has
representation only from Lok Sabha. (d) 1, 2 and 3
2. It examines the budget estimates 21. The project titled “Zero Pendency
before they have been voted by the Courts” has been launched by____
Parliament.
(a) Supreme Court of India
Which of the above statements is/are (b) Delhi High Court
correct?
(c) Ministry of Law and Justice
(a) 1 only
(d) Law Commission of India
(b) 2 only
(c) Both 1 and 2 22. Which of the following statements
is incorrect regarding the
(d) Neither 1 nor 2 ‘HoneyMission?
19. Consider the following statements (a) It is to fulfill the dream of ‘Sweet
regarding the Parliamentary Revolution’
Committees: (b) Ministry of Agriculture is implementing
1. Standing Committees supervise the agency.
work of various departments, their (c) It was launched in August 2017.
budget, expenditure and bills that
come up in the house relating to the (d) None of the above
department.
23. ‘Tree Ambulance’ program has been
2. Joint Parliamentary Committees launched in _____
(JPCs) can be set up for the purpose of
investigating financial irregularities. (a) Maharashtra
3. Members of JPCs are selected from (b) Karnataka
both the houses of the Parliament. (c) Tamil Nadu
Which of the above statements are correct? (d) Uttar Pradesh
(a) 1 and 2 only
24. Consider the following statements:
(b) 2 and 3 only 1. Nabventures is public-private
(c) 1 and 3 only partnership venture.
(d) 1, 2 and 3 2. Its objective is to provide funds for
investments in agriculture and rural-
20. Which of the following statements focused startups.
are correct regarding Consultative
Committee? Which of the above statements is/are
correct?
1. It provides a forum for informal
discussion between the ministers and (a) 1 only
the members of the parliament on (b) 2 only
policies and programs of the government
and manner of their implementation. (c) Both 1 and 2
2. These committees are constituted by (d) Neither 1 nor 2
the Ministry of Parliamentary Affairs.
25. Pinaki Chandra Ghose, the first Lokpal,
3. These committees shall stand dissolved has been appointed by____
upon dissolution of every Lok Sabha
and shall be reconstituted upon (a) Cabinet Committee on Appointment
constitution of each Lok Sabha. (b) Lokpal Selection Committee, headed
by Prime Minister
Select the correct answer using the code
given below: (c) President
(a) 1 and 3 only (d) Chief Justice of India
98 TARGET PT 2020

ANSWER HINTS
DAY - 10

1. Correct Option: (d)  The Constitution of India visualizes the


CAG to be Comptroller as well as Auditor
Explanation: General. However, in practice, the CAG
 Statement 2 is incorrect: In addition to is fulfilling the role of an Auditor-General
the legal and regulatory audit, the CAG only.
can also conduct the propriety audit.
 In other words, the CAG has no control over
Supplementary Notes: the issue of money from the Consolidated
Fund and many departments are authorized
Comptroller & Auditor General (CAG) of to draw money by issuing cheques without
India specific authority from the CAG.
 The CAG is an agent of the Parliament and
conducts audit of expenditure on behalf of 2. Correct Option: (c)
the Parliament. Therefore, he is responsible
Explanation:
only to the Parliament.
 Statement 2 is incorrect:The CAG can
 The role of CAG is to uphold the Constitution
be removed by the President on the basis
of India and the laws of Parliament in
the field of financial administration. of a resolution passed to that effect by the
The accountability of the Executive (i.e., both Houses of the Parliament by special
Council of Ministers) to the Parliament in majority.
the sphere of financial administration is
Supplementary Notes:
secured through audit reports of the CAG.
 The CAG has more freedom with regard Removal of Comptroller and Auditor
to audit of expenditure than with regard General (CAG)
to audit of receipts, stores and stock. In
 Comptroller and Auditor General of India
relation to expenditure, he decides the
can be removed from office in like manner
scope of audit and frames his own audit
codes and manuals whereas he has to and on the like grounds as a Judge of the
proceed with the approval of the Executive Supreme Court.
government in relation to rules for the  In others words, he can be removed by
conduct of the other audits. the President on the basis of a resolution
 In addition to the legal and regulatory passed to that effect by both the Houses of
audit, the CAG can also conduct the the Parliament with special majority, either
propriety audit, that is, he can look into on the ground of proved misbehaviour or
the ‘wisdom, faithfulness and economy’ of incapacity.
government expenditure and comment on
the wastefulness and extravagance of such 3. Correct Option: (a)
expenditure.
Explanation:
 The CAG has to ascertain whether money
shown in the accounts as having been  Statement 2 is incorrect: The
disbursed was legally available for and administrative expenses of the office of the
applicable to the service or the purpose to CAG, including all salaries, allowances and
which they have been applied or charged pensions of persons serving in that office
and whether the expenditure conforms to are charged upon the Consolidated Fund
the authority that governs it. of India.
TARGET PT 2020 99

Supplementary Notes:  He audits all trading, manufacturing,


profit and loss accounts, balance
Comptroller and Auditor General of India sheets and other subsidiary accounts
(CAG) kept by any department of the Central
 The Constitution has made the following Government and state governments.
provisions to safeguard and ensure the
 He audits the receipts and expenditure of
independence of CAG:
the Centre and each state to satisfy himself
 He is not eligible for further office, that the rules and procedures in that behalf
either under the Government of India or of are designed to secure an effective check
any state, after he ceases to hold his office. on the assessment, collection and proper
 His salary and other service conditions are allocation of revenue.
determined by the Parliament. His salary  He audits the receipts and expenditure
is equal to that of a judge of the Supreme of the following:
Court.
! All bodies and authorities substantially
 The administrative expenses of the
financed from the Central or state
office of the CAG, including all salaries,
revenues.
allowances and pensions of persons
serving in that office are charged upon ! Government companies.
the Consolidated Fund of India.
! Other corporations and bodies, when
 Other Provisions: so required by related laws.
 He is provided with the security of tenure. ! He audits all transactions of the
He can be removed by the President only in Central and state governments related
accordance with the procedure mentioned to debt, sinking funds, deposits,
in the Constitution. Thus, he does not hold advances, suspense accounts and
his office till the pleasure of the President,
remittance business.
though he is appointed by him.
! He also audits receipts, stock accounts
 Neither his salary nor his rights in
and others, with approval of the
respect of leave of absence, pension or
age of retirement can be altered to his President, or when required by the
disadvantage after his appointment. President.

 The conditions of service of persons ! He audits the accounts of any


serving in the Indian Audit and Accounts other authority when requested
Department and the administrative powers by the President or Governor.
of the CAG are prescribed by the President For example, the audit of local
after consultation with the CAG. bodies.
 No minister can represent the CAG in ! He advises the President with regard
Parliament (both Houses) and no minister to prescription of the form in which
can be called upon to take any responsibility the accounts of the Centre and the
for any actions done by him. states shall be kept (Article 150).

4. Correct Option: (a) ! He submits his audit reports relating


to the accounts of the Centre to
Explanation: President, who shall, in turn, place
them before both the Houses of
All statements are correct
Parliament (Article 151).
Supplementary notes: ! He submits his audit reports relating
to the accounts of a state to governor,
The duties and functions of the CAG
who shall, in turn, place them before
 He audits the accounts related to all the state legislature (Article 151).
expenditure from the Consolidated Fund of
India, consolidated fund of each state and ! He ascertains and certifies the net
consolidated fund of each union territory proceeds of any tax or duty (Article
having a Legislative Assembly. 279). His certificate is final. The ‘net
proceeds’ means the proceeds of a tax
 He audits all expenditure from the or a duty minus the cost of collection.
Contingency Fund of India and the Public
Account of India as well as the contingency ! He acts as a guide, friend and
fund of each state and the public account of philosopher of the Public Accounts
each state. Committee of the Parliament.
100 TARGET PT 2020

! He compiles and maintains the Public Account of India as well as the


accounts of state governments. contingency fund of each state and the
In 1976, he was relieved of his public account of each state.
responsibilities with regard to the
compilation and maintenance of 6. Correct Option: (d)
accounts of the Central Government
due to the separation of accounts from Explanation:
audit, that is, departmentalization of
accounts. All statements are correct

Supplementary notes:
5. Correct Option: (d)
Explanation: Consultative Committees

 Statement 1 is incorrect: The  These committees are attached to various


Constitution (Article 149) authorises the ministries/departments of the Central
Parliament to prescribe the duties and Government.
powers of the CAG.
 They consist of members of both the Houses
 Statement 2 is incorrect: He compiles of Parliament. The Minister/ Minister of
and maintains the accounts of state State in charge of the Ministry concerned
governments. acts as the chairman of the consultative
 Statement 3 is incorrect: He audits all committee of that ministry.
expenditure from the Contingency Fund  These committees provide a forum
of India and the Public Account of for informal discussions between
India as well as the contingency fund of
the ministers and the members of
each state and the public account of each
Parliament on policies and programmes
state.
of the government and the manner of their
Supplementary Notes: implementation.

 Comptroller and Auditor General of  These committees are constituted by


India (CAG) theMinistry of Parliamentary Affairs.
 The Constitution of India (Article 148)  The guidelines regarding the composition,
provides for an independent office of the functions and procedures of these
Comptroller and Auditor General of India committees are formulated by this
(CAG). Ministry.
 He is the head of the Indian Audit and  The Ministry also makes arrangements
Accounts Department. He is the guardian for holding their meetings both during
of the public purse and controls the the session and the inter-session period of
entire financial system of the country Parliament.
at both the level of the Centre and the
state.  The membership of these committees is
voluntary and is left to the choice of the
 The Constitution (Article 149)
members and the leaders of their parties.
authorises the Parliament to prescribe
the duties and powers of the CAG in The maximum membership of a committee
relation to the accounts of the Union and is 30 and the minimum is 10.
of the states and of any other authority or  These committees are normally
body. Accordingly, the Parliament enacted constituted after the new Lok Sabha
the CAG’s (Duties, Powers and Conditions is constituted, i.e. after General
of Service) act, 1971. This Act was amended Elections for the Lok Sabha. In other
in 1976 to separate accounts from audit in
words, these committees shallstand
the Central government.
dissolved upon dissolution of every
 He compiles and maintains the Lok Sabha and shall be reconstituted
accounts of state governments. In 1976, upon constitution of each Lok Sabha.
he was relieved of his responsibilities
with regard to the compilation and 7. Correct Option: (c)
maintenance of accounts of the Central
Government due to the separation  Statement 1 is incorrect: The term of
of accounts from audit, that is, committee on Public Undertaking does not
departmentalisation of accounts. exceed one year.
 He audits all expenditure from the  Statement 3 is incorrect: Committee on
Contingency Fund of India and the Public Account comprises of 22 members.
TARGET PT 2020 101

Supplementary notes: 8. Correct Option: (b)


Parliamentary committees Explanation:
 Committee on Public undertaking  Statement 1 is incorrect: Cabinet
consists of total number of 22 members Committees are extra-constitutional in
where the term of office does not to exceed nature.
one year.
Supplementary Notes:
Functions:
Features of Cabinet Committees
 To examines the reports and accounts of
 They are extra-constitutional in
the Public Undertakings and Reports of
emergence. In other words, they are not
CAG of India.
mentioned in the Constitution. However,
 To examines whether the affairs of Public the Rules of Business provide for their
Undertakings are being managed in establishment.
accordance with sound business principles  They are of two types:
and prudent commercial practices.
 Standing: permanent in nature
 To exercise such other functions wasted
in the public accounts committee and the  Ad hoc: temporary in nature
estimate committee in relation to public  They are set up by the Prime Minister
undertakings which are allotted to it by according to the exigencies of the time and
the Speaker. requirements of the situation. Hence, their
 Committee on Estimates consists of number, nomenclature, and composition
total number of 30 members and term varies from time to time.
of office not to exceed one year.  Their membership varies from three
to eight. They usually include only
Functions: Cabinet Ministers. However, the non-
 To report on what economics, improvements cabinet Ministers are not debarred
in organisation, efficiency or administrative from their membership.
reforms, consistent with the policy  They not only include the Ministers in
underlying the estimates, may be effected. charge of subjects covered by them but also
 To suggest alternative policies in order include other senior Ministers.
to bring about efficiency and economy in  They are mostly headed by the Prime
administration. Minister. Sometimes other Cabinet
 To examine whether the money is well laid Ministers, particularly the Home Minister
out within the limits of policy implied in or the Finance Minister, also acts as their
the estimates. Chairman. But, in case the Prime Minister
is a member of a committee, he invariably
 To suggest the form in which estimates presides over it.
shall be presented to Parliament
 They not only sort out issues
 Committee on Public Accounts and formulate proposals for the
comprises of 22 members and term of consideration of the Cabinet, but also
office remains one year. take decisions. However, the Cabinet can
review their decisions.
Functions:
 They are an organizational device to reduce
 To examine the Appropriation Accounts the enormous workload of the Cabinet.
and Annual Finance Accounts of Union They also facilitate in-depth examination
Government. of policy issues and effective coordination.
They are based on the principles of division
 To examine the Report of CAG of India on
of labour and effective delegation.
these accounts and other matters.
 To examine the expenditure by various 9. Correct Option: (b)
departments and accounts of autonomous
bodies. Explanation:
 Statement 1 is incorrect:This committee
 To examine various aspects of tax
was set up first in 1921 under the
administration.
provisions of the Government of India
 To ascertain that Government spent money Act of 1919 and has since been in
within the scope of the demand. existence.
102 TARGET PT 2020

Supplementary Notes: Supplementary Notes:

Public Accounts committee Estimates Committee


 At present, it consists of 22 members  Estimates Committee is a Parliamentary
(15 from the Lok Sabha and 7 from the Committee in India consisting of 30 Lok
Rajya Sabha). Sabha members, to examine the budget
estimates of the Union Government.
 The members are elected by the Parliament
every year from amongst its members  All the thirty members are from Lok
according to the principle of proportional Sabha only. The Rajya Sabha has no
representation by means of the single representation in this committee.
transferable vote. Thus, all parties get due
 These members are elected by the Lok
representation in it.
Sabha every year from amongst its own
 The term of office of the members is one members, according to the principles of
year. A minister cannot be elected as a proportional representation by means
member of the committee. of a single transferable vote. Thus, all
parties get due representation in it.
 The chairman of the committee is
appointed from amongst its members by  The term of office is one year.
the Speaker.
 A minister cannot be elected as a
 Since 1967 a convention has developed member of the committee.
whereby the chairman of the committee is
 The chairman of the committee is appointed
selected invariably from the Opposition.
by the Speaker from amongst its members
Role of the committee: and he is invariably from the ruling party.

 The function of the committee is to examine  Functions of the Estimates


the annual audit reports of the Comptroller Committee:
and Auditor General of India (CAG), which  To report what economies, improvements in
are laid before the Parliament by the organisation, efficiency and administrative
President. reform consistent with the policy underlying
 The committee examines public expenditure the estimates, can be affected.
not only from legal and formal point of  To suggest alternative policies in order
view to discover technical irregularities to bring about efficiency and economy in
but also from the point of view of economy, administration.
prudence, wisdom and propriety to bring
out the cases of waste, loss, corruption,  To examine whether the money is well laid
extravagance, inefficiency and nugatory out within the limits of the policy implied
expenses. in the estimates.

 It examine the accounts of autonomous  To suggest the form in which the estimates
and semi-autonomous bodies, the audit of are to be presented to Parliament.
which is conducted by the CAG.  Role of estimate committee:
 It consider the report of the CAG relating  The effectiveness of the role of the
to the audit of any receipt or to examine committee is limited by the following:
the accounts of stores and stocks.
 It examines the budget estimates only
 It examine the money spent on any service after they have been voted by the
during a financial year in excess of the Parliament, and not before that.
amount granted by the Lok Sabha for that
purpose.  It cannot question the policies laid down by
the Parliament.
 In the fulfilment of the above functions,
the committee is assisted by the CAG. In  Its recommendations are advisory and
fact, the CAG acts as a guide, friend and not binding on the ministries.
philosopher of the committee.  It examines every year only certain
selected ministries and departments.
10. Correct Option: (a) Thus, by rotation, it would cover all of them
over a number of years.
Explanation:
 It lacks the expert assistance of the CAG
 Statement 2 is incorrect: It examines
which is available to the Public Accounts
the budget estimates only after they
Committee.
have been voted by the Parliament,
and not before that.  Its work is in the nature of a post-mortem.
TARGET PT 2020 103

11. Correct Option: (a) resignation from RajyaSabha on 05 August


2019.
Explanation:
 This committee was set up first in 1921
Cabinet committees under the provisions of the Government
 They are extra-constitutional, powerful, of India Act of 1919 and has since been in
decision-making body, constituted by the existence. Since 1967 a convention has
Prime Minister as standing (permanent)or developed whereby the chairman of
ad hoc (temporary) from time to time. the committee is selected invariably
from the Opposition. Presently,
 They are an organizational device to reduce AdhirRanjanChowdhury is the
the enormous workload of theCabinet. chairman of the Committee.
 Generally, they include onlyCabinet
 The members are elected by the
Ministers. But, the non-cabinet
Parliament (both House) every year
Ministers are not debarredfrom their
from amongst its members according
membership.
to the principle of proportional
 They are generally headed by the Prime representation by means of the single
Minister but other CabinetMinisters, transferable vote. The term of office
particularly the Home Minister or the of the members is one year. A minister
Finance Minister, alsoacts as their cannot be elected as a member of the
Chairman. But, in case the Prime committee. The chairman of the committee
Minister is a member of acommittee, is appointed from amongst its members by
he invariably presides over it. the Speaker.

12. Correct Option: (b) 14. Correct Option: (b)


Explanation: Explanation:
 Under the Transaction of Business Rules,
the Government has reconstituted Cabinet Committees chaired by Speaker
Committees in June 2019.  The Speaker appoints the chairman
 Currently (as on 12.11.2019), the total of all the parliamentary committees
number of the Cabinet Committees is of the LokSabha and supervises their
eight. functioning. He himself is the chairman
of the Business Advisory Committee,
 These are (i) Appointments Committee of
the Rules Committee and the General
the Cabinet, (ii) Cabinet Committee on
Accommodation, (iii) Cabinet Committee Purpose Committee.
on Economic Affairs, (iv) Cabinet  Business Advisory Committee: It regulates
Committee on Parliamentary Affairs, (v) the program and time table of the House.
Cabinet Committee on Political Affairs, It allocates time for the transaction of
(vi) Cabinet Committee on Security, (vii) legislative and other business brought
Cabinet Committee on Investment and before the House by the government.
Growth, and (viii) Cabinet Committee on The LokSabha committee consists of 15
Employment & Skill Development. members including the Speaker as its
 In the Appointments Committee of the chairman. In the RajyaSabha, it has 11
Cabinet, only the Prime Minister and members including the Chairman as its ex-
the Minister of Home Affairs are the officio chairman.
members.
 Rules Committee: It considers the matters
 Cabinet Committee on Political of procedure and conduct of business in
Affairs, headed by the Prime Minister, the House and recommends necessary
is also called the ‘Super Cabinet’. amendments or additions to the Rules of the
House. The LokSabha committee consists
13. Correct Option: (a) of 15 members including the Speaker as
Explanation its ex-officio chairman. In RajyaSabha,
it consists of 16 members including the
Public Accounts Committee Chairman as its ex-officio chairman.
 At present, the Public Accounts Committee  General Purposes Committee: It considers
consists of 21 members (15 from the and advises on matters concerning affairs
LokSabha and 6 from the RajyaSabha). of the House, which do not fall within the
Shri Bhubaneswar Kalita ceased to be a jurisdiction of any other parliamentary
Member of Committee consequent upon his committee.
104 TARGET PT 2020

15. Correct Option: (c)  The Committee on MPLADS consists of 10


members and is headed by Hon’ble Deputy
Explanation: Chairman, RajyaSabha. The Committee
was first constituted on 5th September
Joint Committees of both Houses
1998 to look into various aspects of the
 Presently, there are 14 joint committees of MPLAD schemes.
Parliament viz.
 The Committee performs its functions in
! Committee on Empowerment of close coordination with the Ministry of
Women Statistics &Programme Implementation
and helps the individual M.P. relating to
! Committee on Food Management in the problems being faced by him in his
Parliament House Complex
constituency/state in the implementation
! Committee on Installation of of the MPLAD Schemes.
Portraits/Statues of National Leaders
and Parliamentarians in Parliament 17. Correct Option: (c)
House Complex
Explanation:
! Joint parliamentary committee
! Joint Parliamentary Committee on Committee on Public Undertakings
Maintenance of Heritage Character  As the members are elected every year for
and Development of Parliament House a term of 1 year.
Complex
 Originally, it had 15 members (10 from the
! Library Committee Lok Sabha and 5 from the Rajya Sabha).
! Committee on Members Salary and However, in 1974, its membership was
Allowances raised to 22 (15 from the Lok Sabha and
7 from the Rajya Sabha). The elections are
! Committee on Offices of Profit done through proportional representation
! Committee on Public Accounts using a single transferable vote.

! Committee on Public  The chairman of the committee is appointed


Undertakings by the Speaker from amongst its members
who are drawn from the Lok Sabha only. A
! Committee on Railway Convention minister cannot be elected as a member of
Committee the committee.
! Committee on Security in Parliament
House Complex 18. Correct Option: (a)
! Committee on Welfare of Other Explanation:
Backward Classes
Estimates Committee
! Committee on Welfare of Scheduled
Castes and Scheduled Tribes  Estimates Committee is a Parliamentary
Committee in India consisting of 30 Lok
 Additionally, Departmentally Related Sabha members, to examine the budget
Standing Committees (DRSCs) are the estimates of the Union Government.
joint committee of both the Houses.
 All the thirty members are from Lok
 Estimates Committee is the exclusive Sabha only. The Rajya Sabha has no
financial committee of the LokSabha. representation in this committee.
16. Correct Option: ()  These members are elected by the Lok
Sabha every year from amongst its own
Explanation: members, according to the principles of
 Both the Houses have separate proportional representation by means
Committee on Members of Parliament of a single transferable vote. Thus, all
Local Area Development Scheme as parties get due representation in it.
the ad-hoc committee.  The term of office is one year.
 The Committee on MPLADS (LokSabha),  A minister cannot be elected as a
an ad hoc Committee was constituted for member of the committee.
the first time on 22 February1999 by the
Speaker as per provisions of Rule 254(1)  The chairman of the committee is appointed
of the Rules of Procedureand Conduct by the Speaker from amongst its members
of Business in LokSabha. Initially the and he is invariably from the ruling party.
Committee consisted of 20Members.
Later, the membership was raised to Functions of the Estimates Committee:
24. The Chairperson is appointedby the
 To report what economies, improvements in
Speaker from amongst the Members of the
organisation, efficiency and administrative
Committee.
TARGET PT 2020 105

reform consistent with the policy underlying cease to exist on completion of the task
the estimates, can be affected. assigned to them.
 To suggest alternative policies in order Standing Committees
to bring about efficiency and economy in
administration.  Standing Committees supervise the work
of various departments, their budget, their
 To examine whether the money is well laid expenditure and bills that come up in the
out within the limits of the policy implied house relating to the department.
in the estimates.
 On the basis of the nature of functions
 To suggest the form in which the estimates performed by them, standing committees
are to be presented to Parliament. can be classified into the following six
categories:
Role of estimate committee
 Financial Committees
 The effectiveness of the role of the
committee is limited by the following: ! Public Accounts Committee
! It examines the budget estimates ! Estimates Committee
only after they have been voted
! Committee on Public Undertakings
by the Parliament, and not before
that.  Departmental Standing Committees (24)
! It cannot question the policies laid  Committees to Inquire
down by the Parliament.
 Committees to Scrutinise and Control
! Its recommendations are advisory  Committees Relating to the Day-to-Day
and not binding on the ministries.
Business of the House
! It examines every year only  House-Keeping Committees or Service
certain selected ministries and
Committees
departments. Thus, by rotation, it
would cover all of them over a number Ad Hoc Committees
of years.
 Ad hoc committees can be divided into two
! It lacks the expert assistance of the categories, that is, Inquiry Committees
CAG which is available to the Public and Advisory Committees.
Accounts Committee.
 Inquiry Committees are constituted from
! Its work is in the nature of a post- time to time, either by the two Houses on
mortem. a motion adopted in that behalf, or by the
Speaker / Chairman, to inquire into and
19. Correct Option: (d) report on specific subjects. For example:
Explanation: ! Committee on the Conduct of Certain
Members during President’Address
Parliament
! Committee on Draft Five-Year Plan
 The Parliament is the legislative organ of
the Union government. ! Railway Convention Committee etc
 Articles 79 to 122 in Part V of the Advisory Committees
Constitution deal with the organisation,
composition, duration, officers, procedures,  It includes select or joint committees on
bills, which are appointed to consider and
privileges, and powers of the Parliament.
report on particular bills.
Parliamentary committees  These committees are distinguishable from
 A significant feature of the legislative the other ad hoc committees in as much
process is the appointment of committees as they are concerned with bills and the
for various legislative purposes. procedure to be followed by them is laid
down in the Rules of Procedure and the
 These committees play a vital role not Directions by the Speaker / Chairman.
merely in law making, but also in the day-
to-day business of the House. Joint Parliamentary Committees
 Broadly, parliamentary committees are  Joint Parliamentary Committees (JPCs)
of two kinds—Standing Committees can be set up for the purpose of
and Ad Hoc Committees. The former discussing a particular bill, like the
are permanent (constituted every year or joint committee to discuss bill, or for
periodically) and work on a continuous the purpose of investigating financial
basis, while the latter are temporary and irregularities.
106 TARGET PT 2020

 Members of the committee are selected Systems Committee in January 2017, in


from both Houses. collaboration with DAKSH.
 It was started to study the pending cases
20. Correct option: (d)
as well as the requirement of the number
Explanation: of judges in lower courts.

Consultative Committees  Its report was published in May 2019.

 Consultative committees are attached 22. Correct Option: (b)


to various ministries / departments of
the Central Government. They consist of Explanation:
members of both the Houses of Parliament.  The mission was launched in 2017 to fulfill
The Minister / Minister of State in charge of the vision of Sweet Revolution.
the Ministry concerned acts as the chairman of
the consultative committee of that ministry.  Khadi& Village Industries Commission
(KVIC) is the implementing agency which
 These committees provide a forum
is under the Ministry of Micro, Small and
for informal discussions between
Medium Enterprises.
the ministers and the members of
Parliament on policies and programmes  The KVIC for this distributes bee-boxes
of the government and the manner of their among farmers and unemployed youths
implementation. across India.
 These committees are constituted by the
Ministry of Parliamentary Affairs. 23. Correct Option: (c)

 The guidelines regarding the composition, Explanation:


functions and procedures of these  It is a reforestation program launched in
committees are formulated by this Ministry. Chennai.
The Ministry also makes arrangements for
holding their meetings both during the  an attempt to revive trees after lakhs of
session and the inter- session period of trees in Tamil Nadu were uprooted by
Parliament. cyclones like Vardah (2016) and Gaja
(2018).
 The membership of these committees is
voluntary and is left to the choice of the  It has been launched in Delhi already, in 2010.
members and the leaders of their parties.
The maximum membership of a committee 24. Correct Option: (b)
is 30 and the minimum is 10.
Explanation:
 These committees are normally constituted
after the new Lok Sabha is constituted, after  It is a 100% subsidiary of NABARD i.e. a
General Elections for the Lok Sabha. In government agency.
other words, these committees shall stand  Recently, it has announced aRs 700-crore
dissolved upon dissolution of every venture capital fund for equity investments
Lok Sabhaand shall be reconstituted in agriculture and rural-focused startups.
upon constitution of each Lok Sabha.
 The fund is registered with SEBI as a
 Separate Informal Consultative Committees Category II Alternative Investment Fund.
of the members of Parliament are also
constituted for all the Railway Zones. 25. Correct Option: (c)
Members of Parliament belonging to the
area falling under a particular Railway Zone Explanation:
are nominated on the Informal Consultative
 The Lokpal is appointed by the President
Committee of that Railway Zone.
on the recommendation of the Lokpal
 Unlike the Consultative Committees Selection Committee.
attached to various ministries / departments,
 The Committee includes Prime Minister,
the meetings of the Informal Consultative
Chief Justice of India or any Supreme Court
Committees are to be arranged during the
Judge as nominated by the Chief Justice,
session periods only.
LokSabha Speaker, Leader of Opposition
(or, the Largest opposition party) in
21. Correct Option: (b)
LokSabha, and an eminent jurist.
Explanation:  The President appointed Pinaki Chandra
 The project was kickstarted by the Delhi Ghose, former Supreme Court judge as the
High Court’s State Court Management first Lokpal.
TARGET PT 2020 107

TEST
DAY - 11

Time Allowed: 30 mins Maximum Marks: 50

1. ‘Suspension of Fundamental Rights (d) 1, 2, and 3


during emergencies’ has been
borrowed from which of the following 4. Which of the following statements
sources? regarding the State emergency is/are
correct?
(a) Constitution of the Fifth Republic
1. The President can make the
(b) Weimar Republic’s Constitution proclamation of State Emergency only
(c) Government of India Act 1935 on the report of the Governor.
(d) Government of India Act 1919 2. It must be approved by the Parliament
within 2 months.
2. With reference to the President’s rule 3. It can last for a maximum of 3 years
consider the following statements: after once the Parliament approves the
1. It must have been supported by the proclamation.
Parliament using a special majority.
Select the correct option using the codes
2. During its operation, both the state given below:
executive state legislature are
dismissed. (a) 2 only
(b) 3 only
Which of the above statements is/are
correct? (c) 1 and 2 only
(a) 1 only (d) 1, 2, and 3
(b) 2 only 5. Consider the following statements:
(c) Both 1 and 2 1. The Constitution has imposed the
(d) Neither 1 nor 2 duty on the Centre to ensure that the
government of every state is carried on
3. Consider the following statements in accordance with the provisions of
regarding the National emergency: the Constitution.
1. It can be extended for up to an indefinite 2. The President’s Rule can be proclaimed
period using subsequent proclamation. for the violation of Article 365.
2. It needs to be approved by both the 3. A law made by the Parliament or
Houses of Parliament within 14 days President or any other specified
by special majority. authority ceases to be operative after
the President’s Rule.
3. It has been proclaimed only once in
India. Which of the above statements is/are
correct?
Which of the above statements is/are
incorrect? (a) 1 only
(a) 1 only (b) 2 and 3 only
(b) 2 and 3 only (c) 1 and 2 only
(c) 1 and 3 only (d) 1, 2, and 3
108 TARGET PT 2020

6. Which of the following Supreme Court to bypass the Cabinet’s concurrence to


cases held that the proclamation of advise the President for revocation of
emergency issued by the President any of the emergencies.
under Article 352(1) is not immune to
the judicial review? Which of the above statements is/are
correct?
(a) Minerva Mills vs Union of India
(a) 1 only
(b) I.R. Coelho v. State of Tamil Nadu
(b) 2 only
(c) Waman Rao Vs Union of India
(c) Both 1 and 2
(d) S. R. Bommai v. Union of India
(d) Neither 1 nor 2
7. Which of the following statements is/
are correct? 10. If any of the lists in the Seventh
1. During a national emergency, the Schedule is to be amended, it can be
legislative power of a state legislature is amended by which of the following
not suspended, but the Parliament can king of majority?
make laws on any subject mentioned in (a) Amendment by simple majority of the
the State List. Parliament.
2. Centre becomes entitled to give (b) Amendment by special majority of the
executive directions to a state on any Parliament.
matter.
(c) Amendment by special majority of the
3. The constitutional distribution of
Parliament and the ratification of half
revenues between the center and the
of the state legislatures.
states cannot be violated during the
National emergency. (d) Absolute majority of the Parliament.

Select the correct option using the codes 11. The secular word was added to the
given below: constitution by which of the following
(a) 1 and 2 only Constitutional Amendment Acts?

(b) 3 only (a) 44th Constitutional Amendment Act

(c) 1 and 3 only (b) 42nd Constitutional Amendment Act

(d) 2 and 3 only (c) 38th Constitutional Amendment Act


(d) 25th Constitutional Amendment Act
8. Consider the following statements:
1. The financial emergency has been 12. Which of the following statements is/are
proclaimed only once in India and that correct regarding the Constitutional
too, during the Balance of Payment Amendment under Art. 368?
crisis of the early 1990s. 1. Articles 368 provides for two types
2. Once approved it continues indefinitely of Amendments, that is, by a special
till it is revoked. majority of Parliament and also
through the ratification of half of the
Which of the above statements is/are states by a simple majority.
correct?
2. In case of disagreement between two
(a) 1 only Houses over passing an Amendment
(b) 2 only Bill under 368, there is a provision for
joint sitting for both Houses
(c) Both 1 and 2
Select the correct answer using the code
(d) Neither 1 nor 2
given below:
9. Consider the following statements: (a) 1 only
1. Only Article 19 can be suspended (b) 2 only
during the external emergency.
(c) Both 1 and 2
2. ‘Rule 12’, recently in the news, confers
special powers to the Prime Minister (d) Neither 1 nor 2
TARGET PT 2020 109

13. Consider the following statements (a) 1 and 2 only


regarding the theory of the basic
(b) 2 only
structure of the Constitution:
1. The basic structure doctrine has (c) 2 and 3 only
consolidated the balance between (d) None
rigidity and flexibility.
2. It places the Judiciary as the final 16. Consider the following statements
authority in deciding what constitutes regarding the President’s Rule:
the basic structure. 1. 44th Amendment Act of 1978 introduced
restraint on the power of the parliament
Which of the above statements is/are to extend the President’s Rule beyond
correct? one year.
(a) 1 only 2. President rule can be extended beyond
(b) 2 only one year when the National Emergency
is in operation in the whole of India or
(c) Both 1 and 2 in the whole or any part of the state.
(d) Neither 1 nor 2 3. For the extension of the President’s rule
beyond one year, the Supreme Court
14. Which of the following statements of India must certify that the general
is/are correct regarding the 98th election to the Legislative Assembly
Constitutional Amendment Act? of concerned state cannot be held on
1. 98th constitutional Amendment Act account of difficulties.
inserted Article371-J, which provided
for special provision for the Hyderabad Which of the above statements is/are correct?
Karnataka region. (a) 1 only
2. The special provisions aim to establish (b) 1 and 3 only
an institutional mechanism for an
equitable allocation of funds to meet (c) 1 and 2 only
the development needs over the region (d) 1, 2 and 3
3. Article 371-J provided for reservation
of seats in educational and vocational 17. Which of the following cases
training institutions in the region for contributed towards the emergence of
students who belong to the region Basic Structure of the Constitution?
1. Waman Rao case
Select the correct answer using the code
given below: 2. Indira Sawhney case

(a) 1 and 2 only 3. Golak Nath case

(b) 2 and 3 only Select the correct option using the codes
given below:
(c) 1, 2 and 3
(a) 2 only
(d) 1 and 3 only
(b) 1 and 2 only
15. Consider the following statements with
(c) 1 and 3 only
reference to the Emergency Powers of
the President: (d) 1 ,2 and 3
1. He cannot declare a National
Emergency before the onset of war. 18. Which of the following elements are
included in the ‘basic structure’ of the
2. He must be communicated in writing Constitution?
for declaration of National Emergency
by the Union Cabinet. 1. Federal character of the Constitution

3. He can suspend Fundamental Rights 2. Parliamentary system


during the National Emergency. 3. Effective access to justice

Which of the above statements is/are Select the correct option using the codes
correct? given below:
110 TARGET PT 2020

(a) 1 only 23. Which of the following central taxes


(b) 1 and 2 only have not been subsumed under Goods
and Services Tax?
(c) 2 and 3 only
1. Basic Customs Duty
(d) 1, 2 and 3 2. Octroi and Entry tax
19. The amendment of the constitution can 3. Luxury tax
be initiated by which of the following? 4. Countervailing Duty
(a) Parliament
Select the correct answer using the code
(b) Lok Sabha only given below:
(c) President (a) 1 only
(d) State legislature (b) 1 and 3 only
20. Which of the following provisions is/ (c) 2, 3 and 4 only
are amended by a simple majority of
the two Houses of Parliament? (d) 1, 2 and 3 only
1. Salaries and allowances of the members 24. The Directorate of Enforcement is
of Parliament responsible for the enforcement of
2. Use of official language which of the following Acts?
3. Citizenship-acquisition and 1. Foreign Exchange Management Act,
termination 1999

Select the correct option using the codes 2. Prevention of Money Laundering Act,
given below: 2002

(a) 3 only 3. Fugitive Economic Offenders Act,


2018
(b) 1 and 2 only
4. The Special Court (trial of Offences
(c) 2 and 3 only relating to Transactions in Securities)
(d) 1, 2 and 3 Act, 1992

21. Oxytocine harmone is secreted by Select the correct answer using the codes
which gland? given below:
(a) Pineal Gland (a) 1 and 4 only
(b) Sebaceous Gland (b) 1 and 2 only
(c) Adrenal Gland (c) 1, 2 and 3 only
(d) None of these (d) 2, 3 and 4 only

22. Consider the following statements: 25. Consider the following statements
1. Foreign Direct Investment (FDI) is a regarding Press Council of India:
long term investment whereas Foreign 1. It is a statutory body.
Portfolio Investment (FPI) is a short
2. It has the power to review the
term investment.
functioning of the electronic media like
2. Ministry of Commerce and Industry radio and television etc.
and Ministry of Finance is authorized
to issue directions for enhancing the 3. Most of the members in PCI are from
flow of FPIs in India. the press.

Which of the above statements is/are Which of the statements given above is/are
correct? correct?
(a) 1 only (a) 3 only
(b) 2 only (b) 1 and 3 only
(c) Both 1 and 2 (c) 1 and 2 only
(d) Neither 1 nor 2 (d) 1, 2 and 3 only
TARGET PT 2020 111

ANSWER HINTS
DAY - 11

1. Correct Option: (b) when this Emergency was in operation, a


third proclamation of National Emergency
Explanation: was made in June 1975. Both the second
 Both Emergency powers to be enjoyed and third proclamations were revoked in
by the Union and the Suspension of March 1977.
Fundamental Rights during emergency  The first two proclamations (1962 and
have been borrowed from the Weimar 1971) were made on the ground of ‘external
Republic’s Constitution (Germany) aggression’, while the third proclamation
 However, some provisions are also from (1975) was made on the ground of ‘internal
the Government of India Act 1935. disturbance’.

2. Correct Option: (d) 4. Correct Option: (a)

Explanation: Explanation:
 During its operation, the state  Article 356 empowers the President to
executive is dismissed and the state issue a proclamation if s/he is satisfied
legislature is either suspended or that a situation has arisen in which the
dissolved. The President administers government of a state cannot be carried
the state through the governor and the on in accordance with the provisions of the
Parliament makes laws for the state. Constitution. Notably, the president can
act either on a report of the governor of the
 Every resolution of Parliament approving state or otherwise too (ie, even without
its proclamation or its continuance can be the governor’s report). The Sarkaria
passed only by a simple majority. Commission had recommended that the
proclamation of the President’s Rule should
3. Correct Option: (b) be only on the governor’s report.
Explanation:  A proclamation imposing President’s
Rule must be approved by both the
National emergency Houses of Parliament within two
 It can be proclaimed only when the security months from the date of its issue.
of India or a part of it is threatened by war,  However, if the proclamation of President’s
external aggression or armed rebellion. Rule is issued at a time when the Lok
 Once proclaimed, there is no maximum Sabha has been dissolved or the dissolution
period prescribed for its operation. of the Lok Sabha takes place during the
It can be continued indefinitely with period of two months without approving
the approval of Parliament every six the proclamation, then the proclamation
months. survives until 30 days from the first sitting
of the Lok Sabha after its reconstitution,
 It has been proclaimed three times provided the Rajya Sabha approves it in
so far-in 1962, 1971 and 1975. The first the meantime.
proclamation of National Emergency was
 Once proclaimed, the President’s Rule
issued in October 1962 on account of Chinese
continues for six months.
aggression and was in force till January
1968. The second proclamation of national  It can be extended for a maximum
emergency was made in December 1971 period of three years with the approval
in the wake of attack by Pakistan. Even of the Parliament, every six months.
112 TARGET PT 2020

 However, if the dissolution of the Lok 7. Correct Option: (a)


Sabha takes place during the period of
six months without approving the further Explanation:
continuation of the President’s Rule, then
Effect of National emergency on the
the proclamation survives until 30 days
Centre-State relations
from the first sitting of the Lok Sabha
after its reconstitution, provided the Rajya  While a proclamation of Emergency is in
Sabha has in the meantime approved its force, the normal fabric of the Center-State
continuance. relations undergoes a basic change.
 During a national emergency, the
5. Correct Option: (c) Parliament becomes empowered to make
Explanation: laws on any subject mentioned in the
State List. Although the legislative power
President’s rule of a state legislature is not suspended, it
becomes subject to the overriding power of
 Article 355 imposes a duty on the
the Parliament.
Centre to ensure that the government of
every state is carried on in accordance with
8. Correct Option: (b)
the provisions of the Constitution.
Explanation:
 Hence, It is this duty in the performance
of which the Centre takes over the Financial emergency
government of a state under Article 356 in
case of failure of constitutional machinery  The financial emergency has not been
in the state. proclaimed in India so far.

 Article 365 says that whenever a state  Once approved by both the Houses of
fails to comply with or to give effect to any Parliament, the Financial Emergency
direction from the Centre, it will be lawful continues indefinitely till it is
for the president to hold that a situation has revoked.
arisen in which the government of the state
cannot be carried on in accordance with the 9. Correct Option: (b)
provisions of the Constitution. Hence, the Explanation:
proclamation of the President’s rule
can be due to the violation of Article Effect of Emergency on the Fundamental
365. Rights
 A law made by the Parliament or  According to Article 358, when a
president or any other specified proclamation of national emergency is
authority continues to be operative made, the six Fundamental Rights under
even after the President’s Rule. But it Article 19 are automatically suspended.
can be repealed or altered or re-enacted by When the National Emergency ceases to
the state legislature. operate, Article 19 automatically revives
and comes into force.
6. Correct Option: (a)
 Under Article 359, the Fundamental
Explanation: Rights as such are not suspended, but only
their enforcement.
Judicial review of the proclamation of
emergency  Thus, only Article 19(1) is suspended
during the External emergency
 The 38th Amendment Act of 1975 made because none of the Fundamental
the declaration of a National Emergency Rights can be suspended either
immune from the judicial review. during an Internal emergency or State
emergency or Financial emergency.
 But, this provision was subsequently
deleted by the 44th Amendment Act of  Further, Article 20 and 21 cannot be
1978. suspended during any of the emergencies.
 Further, in the Minerva Mills case,  Rule 12 of the Government of India
(1980), the Supreme Court held that the (Transaction of Business) Rules, 1961,
proclamation of a national emergency can allows the Prime Minister to depart
be challenged in a court on the ground of from laid down norms at his discretion.
malafide or that the declaration was based The Cabinet can subsequently give
on wholly extraneous and irrelevant facts post-facto approval for any decision
or is absurd or perverse. taken under Rule 12.
TARGET PT 2020 113

 Thus, by using this rule, the Prime 12. Correct option: (a)
Minister gave the advice to the
President to revocate the President’s Explanation:
rule in Maharashtra. Constitutional Amendment Under 368
10. Correct Option: (c)  An Amendment of the Constitution can be
initiated only by the introduction of a bill for
Explanation: the purpose in either House of Parliament
and not in the state legislature. The bill
Amended to Any of the lists in the Seventh can be introduced either by a minister or
Schedule by a private member and does not require
 The Constitution can be amended in three prior permission of the President.
ways:  Article 368 provides for the two types of
! Amendment by simple majority of the Amendments, that is, by a special majority
Parliament, of Parliament and also through the
ratification of half the states by a simple
! Amendment by special majority of the majority.
Parliament, and
 The Constitutional Amendment Bill must
! Amendment by special majority of the be passed in each house by a special
Parliament and the ratification of half majority, that is, a majority of the total
of the state legislatures. membership of the House and a majority
 The following provisions can be amended of two- thirds of the members of the
By Special Majority of Parliament and House present and voting. Each house
must pass the bill separately. In case of
Consent of States:
a disagreement between the two Houses,
! Election of the President and its there is no provision for holding a joint
manner. sitting of two Houses for the purpose of
! Extent of the executive power of the deliberation and passage of the bill.
Union and the states.
13. Correct Option: (c)
! Supreme Court and high courts.
Explanation:
! Distribution of legislative powers
between the Union and the states. Basic structure and evolution of the
Constitution
! Any of the lists in the Seventh
Schedule.  The theory of the basic structure has had
a long-lasting effect on the evolution of the
! Representation of states in Parliament. Indian Constitution.
! Power of Parliament to amend the  The Judiciary advanced this theory
Constitution and its procedure (Article in the famous case of Kesavananda
368 itself). Bharati (1973). This ruling has contributed
to the evolution of the Constitution in the
11. Correct option: (b) following ways:
Explanation: ! It has set specific limits to the
Parliament’s power to amend
Secular the Constitution. It says that no
 The secular word was added to Preamble amendment can violate the basic
to Indian Constitution by the 42nd structure of the Constitution.
Constitutional Amendment Act of 1976. ! It allows the Parliament to amend
 However, as the Supreme Court said in any and all parts of the Constitution
1974, although the words ‘Secular state’ (within this limitation).
was not expressedly mentioned in the ! It places the Judiciary as the
Constitution, there can be no doubt that final authority in deciding if
Constitution- makers wanted to establish an amendment violates basic
such a state and accordingly Article 25 to structure and what constitutes
28 have been included in the constitution. the basic structure.
 The Indian Constitution embodies the  In fact, the theory of basic structure is
positive concept of secularism that is, all itself an example of a living constitution
religions in our country shall have the as there is no mention of this theory in the
same status and support from the state. Constitution.
114 TARGET PT 2020

 It has emerged from judicial interpretation.  The President can also issue different
The Judiciary and its interpretation have proclamations on grounds of war, external
practically amended the Constitution aggression, armed rebellion, or imminent
without a formal amendment. All living danger thereof, whether or not there is
documents evolve in this manner through a proclamation already issued by him
debates, arguments, competition, and and such proclamation is in operation.
practical politics. This provision was added by the 38th
 In a sense, the basic structure doctrine Amendment Act of 1975.
has further consolidated the balance  The President can proclaim a national
between rigidity and flexibility: emergency only after receiving a
by saying that certain parts cannot be written recommendation from the
amended, it has underlined the rigid cabinet. This means that the emergency
nature while allowing amendments to all can be declared only on the concurrence of
others it has underlined the flexible nature the cabinet and not merely on the advice of
of the amending process. the prime minister.

14. Correct option: (c)  The President can suspend Fundamental


Rights during the National Emergency.
Explanation: Article 358 deals with the suspension of
the Fundamental Rights guaranteed by
98th Constitutional Amendment Act Article 19, while Article 359 deals with
 Article 371-J was inserted by the 98th the suspension of other Fundamental
Constitutional Amendment Act which Rights (except those guaranteed by
provided for special provisions for Articles 20 and 21).
Hyderabad Karnataka Region.
16. Correct Option: (c)
 The special Provisions aims to establish
an institutional mechanism for equitable Explanation:
distribution of funds to meet the
development needs of the region, as well as Extension of the President’s Rule beyond
to enhance human resources and promote one year
employment from the region by providing  The 44th Amendment Act of 1978
for local cadres in services and reservation introduced a new provision to put
in educational and vocational training a restraint on the power of the
institution.
parliament to extend a proclamation
 Article 371-J provides for reservation of of President’s Rule beyond one year.
seats in educational and vocational training
 The 44 amendment Act provided that
institutions in the region for students
beyond one year the President’s Rule can
belonging to the region
be extended by six months at a time
 Article 371-J provides that Governor only when two conditions are fulfilled:
of Karnataka would have special
! A Proclamation of National emergency
responsibility for the establishment of
should be in operation in the whole of
a separate development board for the
India, or in the whole or any part of
Hyderabad Karnataka Region.
the state;
15. Correct option: (c) ! The Election Commission must
certify that the general elections
Explanation:
to the legislative assembly of the
Emergency powers of President concerned state cannot be held on
account of difficulties.
 Under Article 352 President can declare
a national emergency when the security 17. Correct Answer: (c)
of India threatened by war or external
aggression. He can declare a national Explanation:
emergency even before the actual
occurrence of war. Basic structure of the constitution

 The president can declare a national The cases related to the Basic structure
emergency even before the actual are mentioned below:
occurrence of war or external aggression or
 Shankari Prasad v. Union of India, (1951)
armed rebellion if he is satisfied that there
is an imminent danger.  Golak Nath v. State of Punjab, (1967)
TARGET PT 2020 115

 Kesavananda Bharati v. State of Kerala, ! An amendment of the Constitution can


(1973) be initiated only by the introduction of
a bill for the purpose in either House
 Indira Nehru Gandhi v. Raj Narain (1975)
of Parliament and not in the state
 Minerva Mills v. Union of India, (1980) legislatures.
 Waman Rao v. Union of India, (1981) ! The bill can be introduced either by a
minister or by a private member and
18. Correct Option: (d) does not require prior permission of
Explantion: the president.
! The bill must be passed in each House
Elements included in the Basic Structure by a special majority, that is, a majority
of the Constitution
(that is, more than 50 per cent) of the
The following have emerged as ‘basic total membership of the House and a
features’ of the Constitution or elements majority of two-thirds of the members
/ components / ingredients of the ‘basic of the House present and voting.
structure’ of the constitution: ! Each House must pass the bill
 Supremacy of the Constitution separately. In case of a disagreement
between the two Houses, there is no
 Sovereign, democratic and republican provision for holding a joint sitting
nature of the Indian polity
of the two Houses for the purpose of
 Secular character of the Constitution deliberation and passage of the bill.
 Separation of powers between the ! If the bill seeks to amend the federal
legislature, the executive and the judiciary provisions of the Constitution, it must
also be ratified by the legislatures of
 Federal character of the Constitution
half of the states by a simple majority,
 Unity and integrity of the nation that is, a majority of the members of
 Welfare state (socio-economic justice) the House present and voting.

 Judicial review ! After duly passed by both the Houses


of Parliament and ratified by the state
 Freedom and dignity of the individual legislatures, where necessary, the
 Parliamentary system bill is presented to the president for
assent.
 Rule of law
! The president must give his assent to
 Harmony and balance between Fundamental
the bill. He can neither withhold his
Rights and Directive Principles
assent to the bill nor return the bill for
 Principle of equality reconsideration of the Parliament.
 Free and fair elections ! After the president’s assent, the bill
 Independence of Judiciary becomes an Act (i.e., a constitutional
amendment act) and the Constitution
 Limited power of Parliament to amend the stands amended in accordance with
Constitution the terms of the Act.
 Effective access to justice
20. Correct Option: (d)
 Principles (or essence) underlying
fundamental rights. Explanation:
 Powers of the Supreme Court under Articles Amendment by Simple Majority of
32, 136, 141 and 142 Parliament
 Powers of the High Court’s under Articles
226 and 227 A number of provisions in the Constitution
can be amended by a simple majority of
19. Correct Option: (a) the two Houses of Parliament outside
the scope of Article 368. These provisions
Explanation: include:
Procedure For Amendment  Admission or establishment of new states.
 The procedure for the amendment of the  Formation of new states and alteration
Constitution as laid down in Article 368 is of areas, boundaries or names of existing
as follows: states.
116 TARGET PT 2020

 Abolition or creation of legislative councils 22. Correct Option: (a)


in states.
Explanation
 Second Schedule—emoluments, allowances,
privileges and so on of the president, the Foreign Direct Investment (FDI) and
governors, the Speakers, judges, etc. Foreign Portfolio Investment (FPI)
 Quorum in Parliament.  Statement 1 is correct: Foreign Direct
 Salaries and allowances of the members of Investment (FDI) is a long term
Parliament. investment whereas Foreign Portfolio
Investment (FPI) is a short term
 Rules of procedure in Parliament.
investment.
 Privileges of the Parliament, its members
 Statement 2 is incorrect: Recently,
and its committees.
Securities and Exchange Board of
 Use of English language in Parliament. India (SEBI) has liberalised norms for
 Number of puisne judges in the Supreme Foreign Portfolio Investors.
Court.
Supplementary Notes:
 Conferment of more jurisdiction on the
Supreme Court. Difference between Foreign Portfolio
Investment (FPI) and Foreign Direct
 Use of official language.
Investment (FDI)
 Citizenship—acquisition and termination.
 FPI consist of securities and other financial
 Elections to Parliament and state like Bonds, Mutual Funds held by an
legislatures. investor in another country. It does not
 Delimitation of constituencies. provide direct ownership of the asset and
is relatively liquid. They are basically short
 Union territories.
term investors.
 Fifth Schedule—administration of
 FDI lets an investor purchase a direct
scheduled areas and scheduled tribes.
business interest in a foreign country. They
 Sixth Schedule—administration of tribal are long term investments.
areas.
Securities and Exchange Board of India
21. Correct Option: (d)
 Securities and Exchange Board of India is a
Explanation: government established in 1988 authority
which controls the securities market in
Oxytocine Hormone India. Indian Parliament passed SEBI
 It has also been dubbed the hug hormone, Act 1992 in 1992 India which made SEBI
cuddle chemical, moral molecule, and a statutory body. SEBI functions to fulfill
the bliss hormone due to its effects on the requirements of the following three
behavior, including its role in love and in categories.
female reproductive biological functions in
reproduction. ! Issuers –It provides a marketplace in
which the issuers can increase finance
 It is secreted by Pituitary Gland. properly.
 It is a hormone that is made in the brain, ! Investors –It ensure safety and supply
in the hypothalamus. It is transported of precise and accurate information
to, and secreted by, the pituitary gland,
which is located at the base of the brain. ! Intermediaries –It enables a
competitive professional market for
 It acts both as a hormone and as a brain
intermediaries.
neurotransmitter.
 The release of oxytocin by the pituitary  The headquarters of SEBI is situated in
gland acts to regulate two female Mumbai. The regional offices of SEBI are
reproductive functions: Childbirth and located in Ahmadabad, Kolkata, Chennai
Breast-feeding. and Delhi.

 It induces contractions of the uterine  In News - Securities and Exchange Board


muscles and initiates labour for the child of India has liberalised norms for Foreign
birth. Portfolio Investors.
TARGET PT 2020 117

23. Correct Option: (a)  Its prime objective is enforcement of two


key Acts of Government of India namely:
Explanation
! Foreign Exchange Management Act
Goods and Services Tax 1999 (FEMA) and
 Statement 1 is correct: Basic Customs ! Prevention of Money Laundering Act
Duty is not subsumed under GST. 2002 (PMLA)

Supplementary Notes:  To check money laundering by curb black


money and hawala trade cases.
Taxes subsumed into GST  It is composed of officers from the Indian
At the Central level, the following taxes Revenue Service, Indian Police Service and
are being subsumed: the Indian Administrative Service.

 Central Excise Duty,  It was established in 1956 as ‘Enforcement


Unit’ in Department of Economic Affairs.
 Additional Excise Duty,
 It is headquartered in New Delhi.
 Service Tax,
 In News - The Enforcement Directorate is
 Additional Customs Duty commonly known investigating the money laundering angle
as Countervailing Duty, and in the INX MEDIA case.
 Special Additional Duty of Customs.
25. Correct Option: (b)
At the State level, the following taxes are
Explanation:
being subsumed:
 Subsuming of State Value Added Tax/Sales Press Council of India
Tax,  Statement 2 is incorrect: It does not
 Entertainment Tax (other than the tax have the power to review the functioning
levied by the local bodies), Central Sales of the electronic media like radio and
Tax (levied by the Centre and collected by television etc.
the States),
Supplementary Notes:
 Octroi and Entry tax,
Press Council of India
 Purchase Tax,
 It was first set up in the year 1966 by the
 Luxury tax, and Parliament on the recommendations of the
 Taxes on lottery, betting and gambling. First Press Commission with the object of
preserving the freedom of the press and of
 In News – Developments in GST is maintaining and improving the standards
frequently in news. of press in India.

24. Correct Option: (b)  The present Council functions under the
Press Council Act, 1978.
Explanation
 It is a statutory, quasi judicial authority
Directorate of Enforcement functioning as a watchdog of the press,
for the press and by the press.
 Statement 2 and 3 are incorrect:
Enforcement Directorate is responsible  It adjudicates the complaints against
for the administration of Prevention and by the press for violation of ethics
of Money Laundering Act, 2002 and and for violation of the freedom of the press
Foreign Exchange Management Act, respectively.
1999 only.
Composition
Supplementary Notes:  The Press Council is headed by a Chairman,
Enforcement Directorate who has by convention, been a retired
judge of the Supreme Court of India.
 It is economic intelligence and law
enforcement agency responsible for  The Council consists of 28 other members
enforcing economic laws and fighting of whom 20 represent the press and are
economic crime in India. nominated by the press organisations/news
agencies recognised and notified by the
 It functions under aegis of Department of Council as all India bodies of categories such
Revenue, Union Ministry of Finance. as editors, working journalists and owners
118 TARGET PT 2020

and managers of newspaper and news Functions


agencies, five members are nominated
from the two Houses of Parliament and The important functions of the Council
three represent cultural, literary and are:
legal fields as nominees of the Sahitya  to help newspapers and news agencies to
Academy, University Grants Commission maintain their independence,
and the Bar Council of India.  to build up a code of conduct for newspapers,
 The members serve on the Council for a news agencies and journalists,
term of three years.  to keep under review any development
 A retiring member shall be eligible for likely to restrict supply and dissemination
of news of public interest and importance.
renomination for not more than one
term. (max 2 consecutive terms)  To concern itself with the developments
such as concentration of or other aspects of
FUNDING ownership of newspapers and news agencies
which may affect the independence of the
 The Council is funded by the revenue
press.
collected by it as fee levied on the
registered newspapers in the country  In News - The Press Council of India had
on the basis of their circulation. described the ban on communication and
free movement in Jammu and Kashmir as
 No fee is levied on newspapers with being in the interest of the integrity and
circulation less than 5000 copies. The sovereignty of the nation. While on the
deficit is made good by way of grant by the other hand, the ban has severely affected
Central Government. the functioning of the press in J&K.

**********
TARGET PT 2020 119

TEST
DAY - 12

Time Allowed: 30 mins Maximum Marks: 50

1. Consider the following statements report to the Parliament upon those


regarding Comptroller and Auditor matters.
General of India (CAG):
1. He holds office for a period of six years 3. Consider the following statements:
or upto the age of 65 years, whichever 1. The National Commission for SCs and
is earlier STs came into being consequent upon
2. He can also be removed by the passing of the 65th Constitutional
president on same grounds and in Amendment Act of 1990.
the same manner as a Chief Election
2. The new Ministry of Tribal Affairs was
Commissioner.
created before the creation of National
3. His salary and other service conditions Commission for SCs and STs.
are determined by the President and
his salary is equal to that of a Supreme 3. National Commission for SCs and
Court Judge. STs were bifurcated by the 93rd
Constitutional Amendment Act.
Which of the above statements is/are
correct? Which of the above statements is/are
(a) 1 and 3 only incorrect?

(b) 2 and 3 only (a) 1 and 2 only


(c) 1 and 2 only (b) 1 only
(d) 1 only (c) 2 and 3 only

2. Which of the following statement is (d) 1, 2 and 3


correct regarding the Special Officer
for Linguistic Minorities? 4. Which of the following statement
is incorrect regarding the Attorney
(a) It has been inserted in the Constitution
General of India?
by the Seventh Constitutional
Amendment Act of 1956. (a) He enjoys all the privileges and
(b) The Constitution specifies the immunities that are available to a
qualifications, tenure, salaries and member of Parliament.
allowances, service conditions and (b) He is removed from the office in the same
procedure for removal of the Special manner as a judge of Supreme Court.
Officer for Linguistic Minorities.
(c) He may continue his private legal
(c) The Commissioner has his head
quarters at Delhi. practice when in office.

(d) It is the duty of the special officer to (d) He has the right to speak and to take part
investigate all matters relating to in the proceedings of both the Houses of
the safeguards provided for linguistic Parliament or their joint sitting and any
minorities under this constitution and committee of the Parliament
120 TARGET PT 2020

5. Which of the following statement is/ (a) 1, 2 and 3 only


are correct regarding the Union Public
(b) 2 and 4 only
Service Commission (UPSC)?
1. Any irregularity in consultation (c) 2, 3 and 4 only
with the UPSC or acting without (d) 1 and 3 only
consultation does not invalidate the
decision of the government. 8. Which of the following statements is
incorrect regarding the Joint State
2. A selection by the UPSC confers the
Public Service Commission (JSPSC)?
right to the post upon the candidate.
(a) The Constitution does not makes a
3. The UPSC can advice the President to
provision for the establishment of a
create new All India Services.
Joint State Public Service Commission
Select the correct answer from the codes (JSPSC) for two or more states.
given below: (b) JSPSC is neither a statutory body nor
(a) 1 only a constitutional body.

(b) 1 and 2 only (c) The members hold office for a term of
six years or until they attain the age of
(c) 2 and 3 only 65 years, whichever is earlier.
(d) 1, 2 and 3 (d) The number of members of a JSPSC
and their conditions of service are
6. Which of the following is incorrect determined by the President.
regarding the role and function of
UPSC? 9. Regarding the Election Commission,
(a) UPSC is only a central recruiting consider the following statements:
agency while the Department of 1. The security of tenure is provided only to
Personnel and Training is the central the Chief Election Commissioner
personnel agency in India. 2. According to the Constitution, the retiring
Election Commissioners are not barred from
(b) Recommendations made by UPSC are reappointment.
only of advisory nature and hence, not
3. The Election Commission determine the
binding on the government.
territorial areas of the electoral constituencies
(c) It is mandatory to consult UPSC while throughout the country.
making reservations of appointments
or posts in favour of any backward Which of the following statements is/are
class of citizens. correct?

(d) The president can exclude posts, (a) 3 only


services and matters from the purview (b) 1 and 2 only
of the UPSC.
(c) 1, 2 and 3
7. With reference to Finance Commission (d) None of the above
(FC), consider the following
statement: 10. Consider the following statements
1. It is constituted by the Parliament regarding the Advocate General of the
every fifth year or at such earlier time State:
as considered necessary. 1. He is appointed by the Governor but
can only be removed by the President.
2. The members are eligible for
reappointment. 2. He must be a citizen of India and must
have held a judicial office for seven
3. The members of FC hold office for a
years or been an advocate of a high
period of 5 years.
court for ten years.
4. Among the four members of the FC,
one of them with special knowledge of Which of the following statements is/are
economics must be a member. incorrect?
(a) 1 only
Which of the following statement is/are
correct? (b) 2 only
TARGET PT 2020 121

(c) Both 1 and 2 3. Unlike other Constitutional bodies, its


(d) None of the above recommendations are binding on the
Government.
11. Consider the following statements
regarding the Protection of Human Which of the above statements is/are
Rights (Amendment) Bill, 2019: correct?
1. The Bill mandates that there should (a) 2 only
be a separate State Human Rights
Commission for Delhi. (b) 1 and 2 only
2. The Bill provides that a person who (c) 2 and 3 only
has been a Judge of the Supreme Court
will be the chairperson of the National (d) 1, 2, and 3
Human Rights Commission.
14. Which of the following statements
3. The Bill provides NHRC and SHRCs
regarding the constitutional bodies, at
the constitutional status.
the Centre, in India is/are correct?
Which of the above statements is/are 1. All the Constitutional bodies of India
correct? are quasi-judicial in nature.
(a) 1 and 3 only
2. All the Constitutional bodies’
(b) 3 only recommendations are only advisory in
(c) 2 only nature.

(d) 1, 2, and 3 3. All the Constitutional bodies are


appointed by the President, and hence
12. Consider the following statements: the Council of Ministers.
1. NHRC and SHRCs are the quasi-
judicial authorities. Select the correct option using the codes
2. They cannot inquire the matter suo given below:
motu. (a) 2 only
3. The Commissions are empowered
(b) 1 and 2 only
to inquire into any matter after the
expiry of one year from the date on (c) 1 and 3 only
which the act constituting violation of
human rights is alleged to have been (d) 2 and 3 only
committed.
15. Which of the following is/are statutory
Which of the above statements is/are bodies?
incorrect? 1. Unique Identification Authority of
(a) 1 only India
(b) 2 only 2. NITI Ayog
(c) 3 only 3. Central Bureau of investigation
(d) 1, 2, and 3 4. Central Vigilance Commission

13. Consider the following statements Select the correct option using the codes
regarding the finance commission: given below:
1. The Finance Commission of India
cannot give recommendations for more (a) 1 only
than 5 years. (b) 1 and 4 only
2. To be appointed as a member of the
(c) 1, 3, and 4 only
commission, the person should be
qualified as a judge of high court. (d) 2, 3, and 4 only
122 TARGET PT 2020

16. Consider the following statements 3. They hold office for a term of five years
regarding the Central Information or until they attain the age of sixty five
Commission: years, whichever is earlier.
1. It was constituted under the provisions
Which of the following statement is/are
of the Right to Information Act (2005).
incorrect?
2. It works under the Ministry of
Information and Broadcasting. (a) 1 only

3. The CIC is appointed by the Prime (b) 2 and 3 only


Minister on the recommendation of the (c) 1 and 3 only
Cabinet.
(d) 1 and 2 only
Which of the following statement is/are
incorrect? 19. Which of the following statement is
correct regarding the Central Bureau
(a) 1 only
of Investigation (CBI)?
(b) 1 and 2 only
(a) It is a statutory body.
(c) 2 and 3 only
(b) Home minister is one of the members
(d) 1 and 3 only in the committee setup to recommends
the Director of CBI.
17. Consider the following statements
regarding the State Information (c) It is established under the Delhi Special
Commission: Police Establishment Act, 1946.
1. While the State Chief Information (d) It works under The Ministry of
Commissioner is appointed by the Personnel, Public Grievances and
Governor, it can only be removed by Pensions.
the President.
2. The State Chief Information 20. Consider the following statements
Commissioner can be removed from regarding the Central Bureau of
the office if he has been convicted of Investigation (CBI):
an offence which, in the opinion of the 1. The CBI is required to obtain the prior
Election Commissioner, involves a approval of the Central Government
moral turpitude. before conducting any inquiry or
investigation into an offence committed
Which of the following statement is/are
by officers of the rank of joint secretary
correct?
and above in the Central Government
(a) 1 only and its authorities.
(b) 2 only 2. The CBI acts as the “National Central
(c) Both 1 and 2 Bureau” of Interpol in India.

(d) Neither 1 nor 2 3. The Director of Prosecution, appointed


for the prosecution of cases under the
18. Regarding the Central Vigilance Lokpal and Lokayuktas Act, 2013,
Commission (CVC), consider the shall be an officer not below the rank of
following statements: Director General of Police of a State.
1. Originally the CVC was neither a
Which of the following statement is/are
constitutional body nor a statutory
correct?
body.
2. The CVC is a multi-member body (a) 2 only
consisting of a Central Vigilance (b) 2 and 3 only
Commissioner (chairperson) and
(c) 1, 2 and 3
not more than four vigilance
commissioners. (d) 1 and 3 only
TARGET PT 2020 123

21. At which of the following place the 23. Which of the following CSIR institutes
International Conference on Women’s has developed OneerTM?
Health, Wellness and Empowerment
2018 was organized? (a) National Chemical Laboratory, Pune
(a) Lucknow (b) Central Drug Research Institute,
(b) Kanpur Lucknow
(c) Varanasi (c) National Botanical Research Institute,
(d) Ahemdabad Lucknow

Correct Option: (b) (d) Indian Institute of Toxicology Research,


Explanation: Lucknow

International Conference on Women’s 24. ‘Artificial Moon’ is an initiative of


Health, Wellness and Empowerment
which of the following coumtries?
 The event was organized by the Federation
of Obstetric and Gynaecological Societies (a) USA
of India and the Kanpur Obstetric and
Gynaecological Society in Kanpur.
(b) China

 Held on 6 October, the theme of this year (c) France


edition was “Give her wings and let her
soar”. (d) Japan

22. Which company’s was the first in 25. Astana Declaration is related to which
India, to move its container on inland of the following goals?
waterways?
(a) Healthcare
(a) Tata Motors
(b) Anti-Terrorism
(b) Coca-Cola
(c) Pepsico (c) Food Security

(d) All of the above (d) None of the above

**********
124 TARGET PT 2020

ANSWER HINTS
DAY - 12

1. Correct Option: (c) of India. Thus, they are not subject to


the vote of Parliament.
Explanation:
 He holds office for a period of six years or 2. Correct Option: (a)
upto the age of 65 years, whichever is
Explanation:
earlier. He can resign any time from his
office by addressing the resignation letter  Originally, the Constitution of India did not
to the president. He can also be removed make any provision with respect to the Special
by the president on same grounds and Officer for Linguistic Minorities. Later, the
in the same manner as a judge of the States Reorganisation Commission (1953-
Supreme Court (which is also the same 55) made a recommendation in this regard.
for Chief Election Commissioner). Accordingly, the Seventh Constitutional
 The Constitution has made the following Amendment Act of 1956 inserted a
provisions to safeguard and ensure the new Article 350-B in Part XVII of the
independence of CAG: Constitution.

! He is provided with the security of  It would be the duty of the Special Officer
tenure. He can be removed by the to investigate all matters relating to the
president only in accordance with safeguards provided for linguistic minorities
the procedure mentioned in the under the Constitution. He would report
Constitution. Thus, he does not hold to the President upon those matters
his office till the pleasure of the at such intervals as the President may
president, though he is appointed by direct. The President should place all such
him. reports before each House of Parliament
and send to the governments of the states
! He is not eligible for further office, concerned.
either under the Government of India
or of any state, after he ceases to hold  The Commissioner takes up all the
his office. matters pertaining to the grievances
arising out of the non-implementation of
! His salary and other service the Constitutional and Nationally Agreed
conditions are determined by the Scheme of Safeguards provided to linguistic
Parliament. His salary is equal to minorities that come to its notice or are
that of a judge of the Supreme Court. brought to its knowledge by the linguistic
! Neither his salary nor his rights in minority individuals, groups, associations
respect of leave of absence, pension or or organisations at the highest political
age of retirement can be altered to his and administrative levels of the state
disadvantage after his appointment. governments and UT administrations and
recommends remedial actions to be
! The conditions of service of persons taken.
serving in the Indian Audit and
Accounts Department and the  It must be noted here that the
administrative powers of the CAG Constitution does not specify the
are prescribed by the president after qualifications, tenure, salaries and
consultation with the CAG. allowances, service conditions and
procedure for removal of the Special
! The administrative expenses of
Officer for Linguistic Minorities.
the office of the CAG, including all
salaries, allowances and pensions  The Commissioner has his
of persons serving in that office are headquarters at Allahabad (Uttar
charged upon the Consolidated Fund Pradesh). He has three regional offices
TARGET PT 2020 125

at Belgaum (Karnataka), Chennai (Tamil fall in the category of government servants.


Nadu) and Kolkata (West Bengal). Each is Further, he is not debarred from private
headed by an Assistant Commissioner legal practice.

3. Correct Option: (c) 5. Correct Option: (a)


Explanation: Explanation:
 The National Commission for SCs and STs The UPSC performs the following
came into being consequent upon passing functions:
of the 65th Constitutional Amendment
Act of 1990.  It conducts examinations for appointments
to the all-India services, Central services
 The Commission was established under and public services of the centrally
Article 338 of the Constitution with the administered territories.
objective of monitoring all the safeguards
provided for the SCs and STs under the  It assists the states (if requested by two
Constitution or other laws. Geographically or more states to do so) in framing and
and culturally, the STs are different operating schemes of joint recruitment for
from the SCs and their problems are also any services for which candidates possessing
different from those of SCs. In 1999, a new special qualifications are required.
Ministry of Tribal Affairs was created  It serves all or any of the needs of a state on
to provide a sharp focus to the welfare and the request of the state governor and with
development of the STs.
the approval of the president of India.
 In order to safeguard the interests of the
 It is consulted on the following matters
STs more effectively, it was proposed to
related to personnel management:
set up a separate National Commission for
STs by bifurcating the existing combined ! All matters relating to methods of
National Commission for SCs and STs. recruitment to civil services and for
civil posts.
 This was done by passing the 89th
Constitutional Amendment Act of ! The principles to be followed in making
2003. This Act further amended Article appointments to civil services and
338 and inserted a new Article 338-A in the posts and in making promotions and
Constitution. transfers from one service to another.
! The suitability of candidates for
4. Correct Option: (b) appointments to civil services and
Explanation: posts; for promotions and transfers
from one service to another; and
 The term of office of the AG is not fixed by the appointments by transfer or deputation.
Constitution. Further, the Constitution The concerned departments make
does not contain the procedure and recommendations for promotions and
grounds for his removal. He holds office request the UPSC to ratify them.
during the pleasure of the president. This
means that he may be removed by the ! All disciplinary matters affecting a
president at any time. person serving under the Government
of India in a civil capacity including
 Conventionally, he resigns when the memorials or petitions relating to
government (council of ministers) resigns such matters. These include:
or is replaced, as he is appointed on its
advice. » Censure (Severe disapproval)
 In the performance of his official duties, » Withholding of increments
the Attorney General has the right of » Withholding of promotions
audience in all courts in the territory of
India. Further, he has the right to speak » Recovery of pecuniary loss
and to take part in the proceedings
» Reduction to lower service or rank
of both the Houses of Parliament or
(Demotion)
their joint sitting and any committee
of the Parliament of which he may be » Compulsory retirement
named a member, but without a right to
» Removal from service
vote. He enjoys all the privileges and
immunities that are available to a » Dismissal from service
member of Parliament.
! Any claim for reimbursement of legal
 The Attorney General is not a full-time expenses incurred by a civil servant in
counsel for the Government. He does not defending legal proceedings instituted
126 TARGET PT 2020

against him in respect of acts done in services and Central services and posts may
the execution of his official duties. make regulations specifying the matters in
which, it shall not be necessary for UPSC
! Any claim for the award of a pension
to be consulted.
in respect of injuries sustained by
a person while serving under the  The role of UPSC is not only limited,
Government of India and any question but also recommendations made by it
as to the amount of any such award. are only of advisory nature and hence,
! Matters of temporary appointments not binding on the government. It
for period exceeding one year and on is upto the Union government to accept
regularisation of appointments. or reject that advise. The only safeguard
is the answerability of the government
! Matters related to grant of extension to the Parliament for departing from
of service and re-employment of the recommendation of the Commission.
certain retired civil servants. Further, the government can also make
! Any other matter related to personnel rules which regulate the scope of the
management. advisory functions of UPSC.

 The Supreme Court has held that if the  The Constitution visualises the UPSC to
government fails to consult UPSC in the be the ‘watch-dog of merit system’ in India.
matters (mentioned above), the aggrieved It is concerned with the recruitment to the
public servant has no remedy in a court. all-India services and Central services—
In other words, the court held that any group A and group B and advises the
irregularity in consultation with the government, when consulted, on promotion
UPSC or acting without consultation and disciplinary matters.
does not invalidate the decision of
 It is not concerned with the classification of
the government. Thus, the provision is
services, pay and service conditions, cadre
directory and not mandatory. Similarly,
management, training, and so on.
the court held that a selection by the
UPSC does not confer any right to the  These matters are handled by the
post upon the candidate. However, the Department of Personnel and Training—
government is to act fairly and without one of the three departments of the
arbitrariness or malafides. Ministry of Personnel, Public Grievances
and Pensions. Therefore, UPSC is only
6. Correct Option: (c) a central recruiting agency while the
Department of Personnel and Training
Explanation:
is the central personnel agency in
 The following matters are kept outside India.
the functional jurisdiction of the UPSC. In
other words, the UPSC is not consulted 7. Correct Option: (b)
on the following matters:
Explanation:
! While making reservations of
appointments or posts in favour Finance Commission
of any backward class of citizens.
 Article 280 of the Constitution of India
! While taking into consideration provides for a Finance Commission as a
the claims of scheduled castes quasi judicial body. It is constituted by
and scheduled tribes in making the President of India every fifth year
appointments to services and posts. or at such earlier time as he considers
! With regard to the selections for necessary.
chairmanship or membership of  The Finance Commission consists of a
commissions or tribunals, posts of the chairman and four other members to be
highest diplomatic nature and a bulk appointed by the president. They hold
of group C and group D services. office for such period as specified by
! With regard to the selection for the president in his order.
temporary or officiating appointment  They are eligible for reappointment.
to a post if the person appointed is not
likely to hold the post for more than  The chairman should be a person having
a year. experience in public affairs and the four
other members should be selected from
 The president can exclude posts,
amongst the following:
services and matters from the purview
of the UPSC. The Constitution states that ! A judge of high court or one qualified
the president, in respect to the all India to be appointed as one.
TARGET PT 2020 127

! A person who has specialised India provides for a separate State Election
knowledge of finance and accounts of Commission.
the government.
 Article 324 of the Constitution has made
! A person who has wide experience the following provisions to safeguard and
in financial matters and in ensure the independent and impartial
administration. functioning of the Election Commission:
! A person who has special ! The chief election commissioner
knowledge of economics. is provided with the security of
tenure. He cannot be removed from
8. Correct Option: (d) his office except in same manner and
Explanation: on the same grounds as a judge of the
Supreme Court. In other words, he
 The Constitution makes a provision can be removed by the president on
for the establishment of a Joint State the basis of a resolution passed to that
Public Service Commission (JSPSC) for effect by both the Houses of Parliament
two or more states. with special majority, either on the
 While the UPSC and the SPSC are created ground of proved misbehaviour or
directly by the Constitution, a JSPSC can incapacity. Thus, he does not hold his
be created by an act of Parliament on the office till the pleasure of the president,
request of the state legislatures concerned. though he is appointed by him.
Thus, a JSPSC is a statutory and not a
constitutional body. ! The service conditions of the chief
election commissioner cannot be
 The chairman and members of a JSPSC varied to his disadvantage after his
are appointed by the president. They appointment.
hold office for a term of six years or
until they attain the age of 62 years, ! Any other election commissioner or
whichever is earlier. a regional commissioner cannot be
removed from office except on the
 They can be suspended or removed by the recommendation of the chief election
president. They can also resign from their commissioner.
offices at any time by submitting their
resignation letters to the president.  Though the constitution has sought to
safeguard and ensure the independence and
 The number of members of a JSPSC
impartiality of the Election Commission,
and their conditions of service are
some flaws can be noted, viz.,
determined by the President.
 A JSPSC presents its annual performance ! The Constitution has not prescribed
report to each of the concerned state the qualifications (legal, educational,
governors. Each governor places the report administrative or judicial) of the
before the state legislature. members of the Election Commission.
! The Constitution has not specified the
9. Correct Option: (c) term of the members of the Election
Explanation: Commission.

 The Election Commission is a permanent ! The Constitution has not


and an independent body established by debarred the retiring election
the Constitution of India directly to ensure commissioners from any further
free and fair elections in the country. appointment by the government.
 Article 324 of the Constitution provides that  The Election Commission determine
the power of superintendence, direction the territorial areas of the electoral
and control of elections to parliament, state constituencies throughout the country
legislatures, the office of president of India on the basis of the Delimitation
and the office of vice president of India Commission Act of Parliament.
shall be vested in the election commission.
Thus, the Election Commission is an all- 10. Correct Option: (c)
India body in the sense that it is common to
both the Central government and the state Explanation:
governments.  The Constitution (Article 165) has provided
 It must be noted here that the election for the office of the advocate general for the
commission is not concerned with the states. He is the highest law officer in the
elections to panchayats and muncipalities state. Thus he corresponds to the Attorney
in the states. For this, the Constitution of General of India.
128 TARGET PT 2020

 The advocate general is appointed by reappointment of members of the NHRC


the governor. He must be a person who and SHRCs for a period of five years.
is qualified to be appointed a judge of a The Bill removes the five-year limit for
high court. In other words, he must be a reappointment.
citizen of India and must have held a
judicial office for ten years or been an  The Bill provides for including the
advocate of a high court for ten years. chairpersons of the National Commission
for Backward Classes, the National
 The term of office of the advocate general is Commission for the Protection of Child
not fixed by the Constitution. Rights, and the Chief Commissioner for
 Further, the Constitution does not contain Persons with Disabilities as members of
the procedure and grounds for his removal. the NHRC.
He holds office during the pleasure
 The Bill does not change the statutory
of the governor. This means that he
status of the NHRC or SHRCs.
may be removed by the governor
at any time. He may also quit his office
by submitting his resignation to the 12. Correct Option: (d)
governor. Explanation:
 Conventionally, he resigns when the  The Protection of Human Rights Act, 1993
government (council of ministers) resigns provides for a National Human Rights
or is replaced, as he is appointed on its Commission (NHRC), State Human Rights
advice. The remuneration of the advocate
Commissions (SHRC), as well as Human
general is not fixed by the Constitution. He
Rights Courts (at the district level).
receives such remuneration as the governor
may determine.  They are the watchdog of human rights in
the country.
11. Correct Option: (c)
 The chairman and members are appointed
Explanation: by the president on the recommendations
of a six-member committee consisting of
The Protection of Human Rights the prime minister as its head, the Speaker
(Amendment) Bill, 2019 of the Lok Sabha, the Deputy Chairman of
 The Bill amends the Protection of Human the Rajya Sabha, leaders of the Opposition
Rights Act, 1993. in both the Houses of Parliament and the
Central home minister.
 Under the original Act, the chairperson
of the NHRC (SHRC) is a person who  The Commissions are not empowered
has been a Chief Justice of the Supreme to inquire into any matter after the
Court (High Court). The Bill amends expiry of one year from the date on
the provision that to provide that a which the act constituting violation
person who has been Chief Justice of of human rights is alleged to have
the Supreme Court, or a Judge of the been committed. In other words, it can
Supreme Court (High Court) will be look into a matter within one year of its
the chairperson of the NHRC (SHRC). occurrence.
 The 1993 Act provides for two persons  A. M. Ahmadi Committee set up by the
having knowledge of human rights to be Commission recommended that the
appointed as members of the NHRC. The Commission should be empowered to
Bill amends this to allow three members to inquire into any matter after the expiry of
be appointed, of which at least one will be
one year if there is sufficient reason for not
a woman.
filing the complaint within the said period.
 The Bill provides that the central
 The Commissions enquire into any
government may confer on an SHRC
human rights functions being discharged violation of human rights or negligence
by Union Territories. Functions relating in the prevention of such violation by
to human rights in the case of Delhi a public servant, either suo motu or
will be dealt with by the NHRC. on a petition presented to it or on an
order of a court.
 The original Act states that the chairperson
and members of the NHRC and SHRC will  The functions of the commission are
hold office for five years or till the age of mainly recommendatory in nature. It
seventy years, whichever is earlier. The has no power to punish the violators
Bill reduces the term of office to three years of human rights, nor to award any
or till the age of seventy years, whichever relief including monetary relief to the
is earlier. Further, the Act allows for the victim.
TARGET PT 2020 129

13. Correct Option: (a) ! The Union Public Commission


Explanation: ! Finance Commission

Finance Commission of India


! Special Officer for

 The Constitution of India envisages the


! Linguistic Minorities
Finance commission as the balancing wheel ! The office of the Comptroller of Auditor
of fiscal federalism in India. & Accounts General.
 It is constituted by the President of India  Not all are quasi-judicial in nature for
every fifth year. instance, CAG.
 The chairman should be a person having  All the recommendations of these
experience in public affairs and the four bodies are non-binding on the
other members should be selected from Government.
amongst the following:
 These are appointed by the President,
! A judge of high court or one qualified hence, the Council of Ministers.
to be appointed as one.
! A person who has specialised 15. Correct Option: (b)
knowledge of finance and accounts of Explanation:
the government.
! A person who has wide experience Statutory and Executive bodies in India
in financial matters and in  Statutory body is a non-constitutional
administration. body. This body is created by statute i.e. by
! A person who has special knowledge Parliament by law.
of economics.  Executive bodies are those bodies that are
 Recently, the Union Cabinet has created by an executive order - i.e. order of
extended the term of 15th Finance a ministry of union or state. Such bodies
Commission by one-year. The cabinet have no constitutional or law backing
decision means the Commission will them.
recommend its award to six fiscal  Originally an executive body, the Unique
years, instead of the usual five. Identification Authority of India (UIDAI)
 This means that while the Commission can is now a statutory authority established
give recommendations for six years through under the provisions of the Aadhaar
two reports (2020-21 to 2025-26). (Targeted Delivery of Financial and Other
Subsidies, Benefits and Services) Act, 2016
 When the Sixteenth Finance Commission
(“Aadhaar Act 2016”) on 12 July 2016 by the
will be set up, it will consider devolution
Government of India, under the Ministry
for 2025-26 to 2029-30, and not from 2026-
of Electronics and Information Technology
27. This will essentially keep the award
(MeitY).
period of the 15th Finance Commission at
five years,  NITI Ayog is an executive body, formed
via a resolution of the Union Cabinet on
 The commission submits its report to the
January 1, 2015.
President. He lays it before both the Houses
of Parliament.  CBI is not a statutory body as it is not
 The recommendations made by established by an Act of the Parliament. It
the Finance Commission are only is just an executive body.
of advisory nature and hence, not  Originally an executive body, the
binding on the government. It is up to CVC is got the statutory status via the
the Union government to implement its Central Vigilance Commission Act 2003.
recommendations on granting money to
the states. 16. Correct Option: (c)
14. Correct Option: (d) Explanation:
Explanation:  The Central Information Commission was
established by the Central Government
Constitutional bodies in 2005. It was constituted through an
Official Gazette Notification under the
 The following are the Constitutional bodies
provisions of the Right to Information
at the Centre, in India.
Act (2005). Hence, it is not a constitutional
! The Election Commission. body.
130 TARGET PT 2020

 The Central Information Commission is a corruption in the Central government. It


high-powered independent body which was established in 1964 by an executive
inter alia looks into the complaints made to resolution of the Central government. Its
it and decides the appeals. establishment was recommended by the
 The Commission consists of a Chief Santhanam Committee on Prevention of
Information Commissioner and not more Corruption (1962–64).
than ten Information Commissioners. They  Thus, originally the CVC was neither
are appointed by the President on the a constitutional body nor a statutory
recommendation of a committee consisting body. Later, in 2003, the Parliament
of the Prime Minister as Chairperson, the enacted a law conferring statutory status
Leader of Opposition in the Lok Sabha and on the CVC.
a Union Cabinet Minister nominated by
the Prime Minister.  The CVC is a multi-member body consisting
of a Central Vigilance Commissioner
 The Chief Information Commissioner and
(chairperson) and not more than two
an Information Commissioner hold office
vigilance commissioners.
for a term of 5 years or until they attain the
age of 65 years, whichever is earlier. They  They hold office for a term of four years
are not eligible for reappointment. or until they attain the age of sixty
five years, whichever is earlier. After their
17. Correct Option: (d) tenure, they are not eligible for further
employment under the Central or a state
Explanation:
government.
 State Chief Information Commissioner
are appointed by the Governor on the 19. Correct Option: (b)
recommendation of a committee consisting
of the Chief Minister as Chairperson, the Explanation:
Leader of Opposition in the Legislative  The Central Bureau of Investigation
Assembly and a State Cabinet Minister
(CBI) was set up in 1963 by a resolution
nominated by the Chief Minister.
of the Ministry of Home Affairs. Later,
 They should be persons of eminence in public it was transferred to the Ministry of
life with wide knowledge and experience in Personnel and now it enjoys the status
law, science and technology, social service, of an attached office. The Special Police
management, journalism, mass media or Establishment (which looked into vigilance
administration and governance. cases) setup in 1941 was also merged with
 They should not be a Member of Parliament the CBI.
or Member of the Legislature of any State  The establishment of the CBI was
or Union Territory. recommended by the Santhanam
 The Governor can remove the State Committee on Prevention of Corruption
Chief Information Commissioner or any (1962-1964). The CBI is not a statutory
State Information Commissioner from the body. It derives its powers from the
office under the following circumstances: Delhi Special Police Establishment
Act, 1946, however it is not established
! if he is adjudged an insolvent; or under it.
! if he has been convicted of an
 The Central Government shall appoint the
offence which (in the opinion of
the Governor) involves a moral Director of CBI on the recommendation
turpitude; or of a three-member committee consisting
of the Prime Minister as Chairperson,
! if he engages during his term of office the Leader of Opposition in the Lok
in any paid employment outside the Sabha and the Chief Justice of India or
duties of his office; or Judge of the Supreme Court nominated
! if he is (in the opinion of the Governor) by him.
unfit to continue in office due to
infirmity of mind or body; or 20. Correct Option: (b)
! if he has acquired such financial or Explanation:
other interest as is likely to affect
prejudicially his official functions.  The Lokpal and Lokayuktas Act (2013)
amended the Delhi Special Police
18. Correct Option: (b) Establishment Act (1946) and made the
following changes with respect to the
Explanation: composition of the CBI:
 The Central Vigilance Commission ! The Central Government shall
(CVC) is the main agency for preventing appoint the Director of CBI on the
TARGET PT 2020 131

recommendation of a three-member  Held on 6 October, the theme of this year


committee consisting of the Prime edition was “Give her wings and let her soar”.
Minister as Chairperson, the Leader
of Opposition in the Lok Sabha and 22. Correct Option: (c)
the Chief Justice of India or Judge
of the Supreme Court nominated by Explanation:
him.
India’s first container movement on inland
! There shall be a Directorate of waterways
prosecution headed by a Director for
 On the National Waterway-1, 16 containers
conducting the prosecution of cases
of Pepsico was carried from Kolkata to
under the Lokpal and Lokayuktas Act,
Varanasi.
2013. The Director of Prosecution
shall be an officer not below the  The vessel was named as MV RN Tagore
rank of Joint Secretary to the
 It was India’s first container movement on
Government of India. He shall Inland Vessel post-independence.
function under the overall supervision
and control of the Director of CBI.
23. Correct Option: (d)
He shall be appointed by the Central
Government on the recommendation Explanation:
of the Central Vigilance Commission.
He shall hold office for a period of two OneerTM
years.  OneerTM is Water Disinfection System in
! The Central Government shall appoint meeting the requirements of potable water
officers of the rank of SP and above in rural and urban areas.
in the CBI on the recommendation  It has been developed by Indian Institute
of a committee consisting of the of Toxicology Research (CSIR-IITR),
Central Vigilance Commissioner Lucknow.
as Chairperson, the Vigilance
Commissioners, the Secretary of the  It is useful for the treatment of water by
Home Ministry and the Secretary of eliminates all disease-causing pathogens
the Department of Personnel. such as virus, bacteria, fungi, protozoa and
cyst to provide safe.
 The CBI is required to obtain the prior
approval of the Central Government before 24. Correct Option: (b)
conducting any inquiry or investigation into
an offence committed by officers of the rank Explanation:
of joint secretary and above in the Central
Artificial Moon
Government and its authorities. However,
on May 6, 2014, the Supreme Court  China is planning to launch its own
held as invalid the legal provision that ‘artificial moon’ by 2020 to replace
makes prior sanction mandatory for streetlamps and lower electricity costs in
the Central Bureau of Investigation urban areas.
to conduct a probe against senior  Also called an illumination satellite, it will
bureaucrats in corruption cases under be developed by Xichang Satellite Launch
the Prevention of Corruption Act. Center in Sichuan.
 The CBI acts as the “National Central
Bureau” of Interpol in India. The 25. Correct Option: (a)
Interpol Wing of the CBI coordinates Explanation:
requests for investigation-related activities
originating from Indian law enforcement Astana Declaration
agencies and the member countries of the
Interpol.  The Global Conference on Primary
Health Care in Astana, Kazakhstan in
October 2018 endorsed a new declaration
21. Correct Option: (b)
emphasizing the critical role of primary
Explanation: health care around the world.

International Conference on Women’s  It was convened by UNICEF and the World


Health Organisation (WHO).
Health, Wellness and Empowerment
 India and all other members of the United
 The event was organized by the Federation
Nations signed the Declaration.
of Obstetric and Gynaecological Societies
of India and the Kanpur Obstetric and  The first such conference was held in
Gynaecological Society in Kanpur. Almaty, Kazakhstan in 1978.
132 TARGET PT 2020

TEST
DAY - 13

Time Allowed: 30 mins Maximum Marks: 50

1. Which of the following statements 2. The laws made by state legislatures


regarding Centre-State relations in cease to exist immediately after the
India is/are correct? Resolution passed by Rajya Sabha,
1. Like the USA, residuary powers in under Article 249, has ceased to be in
India are left to the Union. force.

2. The subject matters that fall under the Which of the above statements is/are
residuary power or not is to be decided incorrect?
by Rajya Sabha.
(a) 1 only
Select the correct option using the codes (b) 2 only
given below:
(c) Both 1 and 2
(a) 1 only
(d) Neither 1 nor 2
(b) 2 only
4. Consider the following statements with
(c) Both 1 and 2
reference to Centre-State relations in
(d) Neither 1 nor 2 India:
1. The Governor of a state may with the
2. If two or more states resolve that it is
consent of the central government,
lawful of Parliament to make laws with
entrust to that government any of the
respect to any matter enumerated in
executive functions of the state.
the State List and Parliament makes
the law under Article 252. 2. The President can entrust the executive
function of the center to a state with its
1. The Law automatically applies in all
consent.
the States/UTs in India.
2. Only Parliament can repeal/amend the Which of the above statements given above
Law. is/are correct?
3. Wild Life (Protection) Act, 1972 is an (a) 1 only
example of this situation. (b) 2 only
Which of the above statements is/are (c) Both 1 and 2
incorrect?
(d) Neither 1 nor 2
(a) 1 only
5. With reference to the Constitution of
(b) 1 and 2 only India which one of the following pairs
(c) 2 and 3 only is incorrectly matched?
(d) 1, 2 and 3 only (a) Stock Exchange: The State List
(b) Forest: The Concurrent List
3. Consider the following statements:
(c) Insurance: The Union List
1. A state legislature cannot make laws
on the state lists during a national (d) Marriage and divorce: The Concurrent
emergency. List
TARGET PT 2020 133

6. Consider the following statements 4. The seat of the North Eastern Council
regarding financial relations between is at Guwahati.
the Centre and states:
Which of the above statements given above
1. The limits on the loan borrowed
is/are correct?
by the states are fixed by the state
legislatures. (a) 1 only
2. State Governments cannot borrow (b) 2, 3, and 4 only
directly from foreign agencies.
(c) 1, 2, and 3 only
Which of the above statements given above
(d) 1, 2, 3, and 4
is/are correct?
(a) 1 only 10. Consider the following statements:
(b) 2 only 1. The Commission recommended the
Goods and Services Tax.
(c) Both 1 and 2
2. It recommended that the Governor
(d) Neither 1 nor 2 should be removed either by
impeachment or by resolution in the
7. Which of the following statements
regarding Inter-State Council is state assembly.
correct? 3. It recommended that the Centre should
1. It is a permanent Constitutional body. consult states before introducing bills
on items in the concurrent list through
2. It ha was established by the 7th
the inter-state council.
Constitutional Amendment Act.
3. The Prime minister is its ex officio Which of the following Commissions made
chairperson. the above recommendations?

Select the correct option using the codes (a) Rajamannar Committee
given below: (b) Sarkaria Commission
(a) 1 only (c) Punchhi Commission
(b) 1 and 3 only (d) 2nd Administrative Reforms
(c) 2 only Commission
(d) 2 and 3 only
11. With which of the following countries,
8. Article 301 of the Indian Constitution India has signed the ‘Red Fort
is related to___ Declaration of March 1997’ and the
‘Tshwane Declaration of October
(a) Right to Property
2006’?
(b) Inter-state water disputes
(a) Brazil
(c) Inter-state trade and commerce
(b) South Africa
(d) International Trade and Commerce
(c) Japan
9. Consider the following statements (d) South Korea
regarding the Zonal Councils in India:
1. The Zonal Councils are the statutory 12. ‘Future of Work’ is an initiative of___
bodies. (a) World Economic Forum
2. These have been set up by the (b) World Trade Organization
Governments on the recommendations
of the Sarkaria Commission. (c) International Labour Organization
3. The Prime Minister is the ex officio (d) United Nations Economic and Social
chairperson of these Councils. Commission for Asia and the Pacific
134 TARGET PT 2020

13. Which of the following pairs is (c) Both 1 and 2


incorrectly matched?
(d) Neither 1 nor 2
1. World Competitiveness Ranking:
International Institute for Management 17. With regard to Legislative Relations
and Development between Union and states, consider
2. Global talent competitiveness index: the following statements:
World Economic Forum
1. The State law, that has been reserved
3. The Democracy Index: Transparency for the assent of President and received
International
his assent, can prevail over the law
Select the correct option using the codes made by the Parliament for the whole
given below: country.
(a) 1 only 2. The Constitution expressly secures
(b) 1 and 3 only the predominance of Union List and
Concurrent List over the State List.
(c) 2 and 3 only
3. All the matters enumerated in the State
(d) 1, 2, and 3
List can be introduced in Legislature
14. ‘Room for the River’, the model that without prior sanction from Union or
inspired Kerala, is a flagship project of the President.
which of the following countries?
Which of the above statements is/are
(a) Switzerland
correct?
(b) France
(a) 1 only
(c) The Netherlands
(b) 2 and 3 only
(d) Belgium
(c) 2 only
15. Which of the following countries has
the largest Exclusive Economic Zones (d) 1, 2 and 3
in the Indo-Pacific region?
18. Under which of the following
(a) France
circumstances can the Union make
(b) Britain laws on items in the state list?
(c) The United States 1. When a proclamation of Financial
(d) Australia Emergency is in place.
2. To give effect to an international
16. Regarding the mutual delegation
agreement.
of function between the Centre and
the State, consider the following 3. When a State request the Parliament
statements: to legislate on a matter of national
1. A law made by the Parliament on a importance.
subject of the Union List can confer 4. If Rajya Sabha initiates a resolution
powers and impose duties on a state.
with a simple majority and declares
The same thing can be done by the
state legislature also after taking the that a subject is of national interest.
consent of the Centre.
Select the correct answer using the codes
2. The Centre cannot delegate its given here:
legislative powers to the states.
(a) 1, 2 and 3 only
Which of the above statements is/are
incorrect? (b) 2 only

(a) 1 only (c) 1, 3 and 4 only


(b) 2 only (d) 2 and 3 only
TARGET PT 2020 135

19. Consider the following statements Select the correct answer using the code
with reference to the Inter-State Water given below:
dispute:
(a) 1 only
1. It is in the Concurrent List.
(b) 2 only
2. Parliament can provide that not even
the Supreme Court shall exercise (c) Both 1 and 2
jurisdiction over any such dispute. (d) Neither 1 nor 2
3. The Constitution provides for the
formation of a separate tribunal to deal 22. Which of the following statements is/
with such cases. are correct as per the revised Economic
Capital Framework of Reserve Bank of
Which of the above statements is/are India (RBI)?
correct?
1. The objective of the Economic Capital
(a) 1 and 3 only Framework is to build harmony
(b) 2 only between the central bank’s need for
autonomy and the Government’s
(c) 1 and 2 only
objectives of the development.
(d) 2 and 3 only
2. Some of the recommendations of the
20. Article 301 declares that trade, Bimal Jalan panel have been accepted
commerce and intercourse throughout by the RBI.
the territory of India shall be free.
Select the correct answer using the code
Consider the following statements
given below:
regarding this:
1. The freedom under this provision is (a) 1 only
confined to inter-state trade, commerce (b) 2 only
and intercourse and does not extends
to intra-state trade, commerce and (c) Both 1 and 2
intercourse. (d) Neither 1 nor 2
2. The legislature of a state can impose
reasonable restrictions on the freedom 23. Consider the following statements
of trade, commerce and intercourse regarding Electronic Cigarettes:
with that state or within that state in 1. The Commerce Ministry has put a
public interest but only with the prior blanket ban on the import of Electronic
consent of the Governor. cigarettes.
Select the correct answer using the code 2. They contain vegetable glycerine,
given below: propylene glycol but does not contain
tobacco.
(a) 1 only
3. They fall within the ambit of the
(b) 2 only Cigarettes and Other Tobacco Products
(c) Both 1 and 2 (Prohibition of Advertisement and
(d) Neither 1 nor 2 Regulation of Trade and Commerce,
Production, Supply and Distribution)
21. Which of the following statements is/ Act, 2003.
are correct regarding the recently
launched, Fit India Movement? Which of the above statements is/are
incorrect?
1. Prime Minister of India has launched it
on the occasion of National Sports (a) 1 and 3 only
Day. (b) 1 and 2 only
2. National Sports Day is celebrated
(c) 2 and 3 only
on 29th August, on the birth anniversary
of hockey legend Udham Singh. (d) 1, 2 and 3
136 TARGET PT 2020

24. Consider the following statements 1. A gravitational lens is a distribution


regarding child well-being index: of matter such as a cluster of galaxies
1. This Report is released by UNICEF. between a distant light source and an
2. Kerala topped the chart in the child observer that is capable of bending
well-being index. the light from the source as the light
travels towards the observer.
3. The index captures the performance of
each state and union territories also. 2. Amount of bending is one of the
predictions of Newton’s Corpuscular
Which of the above statements is/are
theory of light.
correct?
(a) 1 and 2 only Which of the above statements is/are
correct?
(b) 2 and 3 only
(c) 1 and 3 only (a) 1 only

(d) 1,2 and 3 (b) 2 only


(c) Both of them
25. Consider the following Statements
regarding Gravitational Lensing (d) None of the above

**********
TARGET PT 2020 137

ANSWER HINTS
DAY - 13

1. Correct Option: (d)  Such a law can be amended or repealed


only by the Parliament and not by the
Explanation: legislatures of the concerned states.
Residuary powers  On the other hand, the state legislature
ceases to have the power to make a law
 Apart from the three legislative subject lists
with respect to that matter.
(Union, State, and Concurrent), there are
also some subject of the national interests  Some examples of laws passed under the
viz. residuary power. above provision are
 The residuary powers have been  Prize Competition Act, 1955; Wild Life
granted to the Union contrary to the (Protection) Act, 1972; Water (Prevention
convention in other federations of the and Control of Pollution) Act, 1974; Urban
world, where the residuary powers are Land (Ceiling and Regulation) Act, 1976;
given to the States. and Transplantation of Human Organs
Act, 1994.
 However, in case of any conflict,
whether a particular matter falls 3. Correct Option: (c)
under the residuary power or not is to
be decided by the court. Explanation:
 The Constitution follows the scheme of the Centre-State Relations
Government of India 1935 Act but with
 During a National Emergency, the
one difference, that is, under this act, the
Parliament acquires the power to legislate
residuary powers were given neither to the
with respect to matters in the State List,
federal legislature nor to the provincial
but the state legislature can also make
legislature but to the governor-general of
laws on the state lists. However, in case
India. of repugnancy between a state law and a
 In this respect, India follows the parliamentary law, the latter is to prevail.
Canadian precedent.  When Rajya Sabha Passes a Resolution
using Article 249, then the Parliament
2. Correct Option: (a) becomes competent to make laws on the
Explanation: state lists.
 The resolution remains in force for one year;
Article 252 it can be renewed any number of times but
 Under the Article, if the legislatures of two not exceeding one year at a time.
or more states pass resolutions requesting  The laws cease to have effect on the
the Parliament to enact laws on a matter expiration of six months after the
in the State List, then the Parliament can resolution has ceased to be in force.
make laws for regulating that matter.
 A law so enacted applies only to 4. Correct Option: (c)
those states which have passed the Explanation:
resolutions.
Centre-State Relations
 However, any other state may adopt it
afterward by passing a resolution to  The distribution of executive power,
that effect in its legislature. in general, follows the distribution of
138 TARGET PT 2020

legislative powers. But, such a rigid guarantee has been given by the Centre.
division in the executive sphere may lead
 The Union Cabinet in 2017 has
to occasional conflicts between the two.
approved the policy guidelines
 Hence, the Constitution provides for to allow financially sound State
inter-government delegation of executive Government entities to borrow
functions in order to mitigate rigidity and directly from bilateral ODA (Official
avoid a situation of deadlock. development Assistance) partners for
 Accordingly, the President may, with implementation of vital infrastructure
the consent of the state government, projects.
entrust to that government any of the  The guidelines will facilitate the State
executive functions of the Centre. Government entities to directly borrow
 Conversely, the Governor of a from the external bilateral funding agencies
State may, with the consent of the subject to fulfilment of certain conditions
Central government, entrust to that and all repayments of loans and interests
government any of the executive to the funding agencies will be directly
functions of the state. remitted by the concerned borrower.

 This mutual delegation of administrative  The concerned State Government will


functions may be conditional or furnish guarantee for the Loan. The
unconditional. Government of India will provide counter
guarantee for the loan.
5. Correct Option: (a)
7. Correct Option: (b)
Explanation:
Explanation:
Seventh Schedule of Indian Constitution
Inter-State Council
• Under the seventh schedule of the
Constitution of India Union list has the subject  Article 263 contemplates the establishment
‘Stock exchanges and futures markets’ under of an Inter-State Council to effect
entry 48. coordination between the states and
between Centre and states. But until
 Under the seventh schedule of the
1990, it had not been established by the
constitution of India Concurrent list has
Government of India.
the subject ‘Forests’ under the entry 17A.
 Under the seventh schedule of the  In pursuance of the recommendations
constitution of India Union list has the of the Sarkaria Commission, the
subject ‘Insurance’ under entry 47. Government headed by established
the Inter-State Council in 1990.
 Under the seventh schedule of constitution
of India Union list has the subject ‘Marriage  It is a permanent Constitutional body,
and Divorce; infants and minors; adoption; established by the President and
wills, intestacy and succession; joint family consists of:
and partition; all matters in respect of ! Prime minister as the
which parties in judicial proceedings were Chairperson
immediately before the commencement of
this Constitution subject to their personal ! Chief ministers of all the states
law’ under the entry 5. ! Chief ministers of union territories
having legislative assemblies
6. Correct Option: (a)
! Administrators of union territories
Explanation: not having legislative assemblies
Borrowing by the States ! Governors of States under President’s
rule
 State government can borrow within India
upon the security of the Consolidated Fund  Six Central cabinet ministers, including
of the State or can give guarantees, but the home minister, to be nominated by the
both within the limits fixed by the Prime Minister.
legislature of that state.
 Five Ministers of Cabinet rank / Minister
 A state cannot raise any loan without of State (independent charge) nominated
the consent of the Centre if there is still by the Chairman of the Council (i.e., Prime
outstanding any part of a loan made to the Minister) are permanent invitees to the
state by the Centre or in respect of which a Council.
TARGET PT 2020 139

 The council is a recommendatory body on  The Southern Zonal Council, comprising


issues relating to inter-state, Center-state the States of Andhra Pradesh, Karnataka,
and Centre–union territories relations. It Kerala, Tamil Nadu and the Union
aims at promoting coordination between Territory of Puducherry.
them by examining, discussing and
 The North Eastern States i.e. (i) Assam
deliberating on such issues.
(ii) Arunachal Pradesh (iii) Manipur (iv)
Tripura (v) Mizoram (vi) Meghalaya and
8. Correct Option: (c) (vii) Nagaland are not included in the
Explanation: Zonal Councils and their special problems
are looked after by the North Eastern
Inter-state trade and commerce Council, set up under the North Eastern
Council Act, 1972. The State of Sikkim has
 Articles 301 to 307 in Part XIII of the
also been included in the North Eastern
Constitution deal with the trade, commerce
Council vide North Eastern Council
and intercourse within the territory of
(Amendment) Act, 2002.
India.
 The seat of the North Eastern Council
 Article 301 declares that trade,
is at Shillong.
commerce and intercourse throughout
the territory of India shall be free.
10. Correct Option: (c)
 The object of this provision is to break
Explanation:
down the border barriers between the
states and to create one unit with a view to Punchhi Commission
encourage the free flow of trade, commerce
and intercourse in the country.  The Second Commission on Centre-State
Relations was set-up by the Government of
9. Correct Option: (a) India in April 2007 under the Chairmanship
of Madan Mohan Punchhi, former Chief
Explanation: Justice of India which submitted its reports
in 2010 and made 310 recommendations.
Zonal Councils
 Some of the recommendations are:
 These are the statutory bodies
established under the States ! The Commission recommended
Reorganisation Act 1956. that the Centre should consult
states before introducing bills
 Each zonal council consists of the on items in the concurrent list
following members: (a) home minister through the inter-state council.
of Central government as the ex
officio chairperson, (b) chief ministers ! On the question of dismissal of a
of all the States in the zone, (c) Two other Chief Minister, the Governor should
ministers from each state in the zone, (d) invariably insist on the Chief Minister
Administrator of each union territory in proving his majority on the floor of the
the zone. House for which he should prescribe a
time limit.
 The present composition of each of these
Zonal Councils is as under: ! In case of appointment of state
chief ministers, the Commission
 The Northern Zonal Council, comprising recommended that:
the States of Haryana, Himachal Pradesh,
Jammu & Kashmir, Punjab, Rajasthan, ! There should be clear guidelines on
National Capital Territory of Delhi and the chief ministers’ appointment so
Union Territory of Chandigarh; that the discretionary powers of the
governor are limited in this regard.
 The Central Zonal Council, comprising
the States of Chhattisgarh, Uttarakhand, ! A pre-poll alliance to be regarded as
one political party.
Uttar Pradesh and Madhya Pradesh;
 The Eastern Zonal Council, comprising
! Order of precedence to form the state
government is:
the States of Bihar, Jharkhand, Orissa,
Sikkim and West Bengal; » The group/alliance with the largest
pre-poll alliance with the highest
 The Western Zonal Council, comprising
number.
the States of Goa, Gujarat, Maharashtra
and the Union Territories of Daman & Diu » The single largest party with
and Dadra & Nagar Haveli; support from others.
140 TARGET PT 2020

» The post-poll alliance with a few  After South Africa achieved democracy in
parties joining the government. 1994, it was the Red Fort Declaration on
Strategic Partnership between India
» The post-poll alliance with a few
and South Africa, signed in March
parties joining the government and
1997.
remaining including independents
giving outside support.  This Strategic Partnership between the
two countries was again re-affirmed in the
 In case of the appointments and removal of Tshwane Declaration (October 2006).
the governors:
 Both these countries are also co-operating
! The Doctrine of Pleasure should be with each other in multilateral platforms
deleted from the Constitution. like IBSA, BRICS, G20, and the World
! Only a resolution by the state Trade Organisation.
legislature should remove the
governor. 12. Correct Option: (c)
! It supported the right of the governor Explanation:
to sanction the prosecution of ministers
against the state government’s Future of Work Initiative
advice.  It was launched by the International
! It recommended that there be Labour Organisation in 2015 in
provision for the impeachment order to understand and to respond
of the governor by the state effectively to the new challenges posed
legislature. by the changes that the world of work
is undergoing.
! The Commission also
recommended for the Goods  Recently, released by the Global
Commission on the Future of Work, set up
and Services Tax. It recommend
under the Future of Work Initiative.
the adoption of the dual GST to be
levied by the Centre and the States
concurrently on a common base
13. Correct Option: (a)
with fewer exemptions. Exemptions Explanation:
may be limited to unprocessed
food services rendered by the Indices and Ranking
governmental organizations and local  The global talent competitiveness
bodies. All area-based exemptions index is published by INSEAD in
should be replaced by cash subsidy partnership with the Adecco Group
linked to investment. The aggregate and Tata Communications. India was
GST base should be large enough to in 80th position (out of 125) in 2019.
permit lower rates. The EC may work
towards building up consensus in this  World Competitiveness Rankings is
direction published by International Institute
for Management and Development,
11. Correct Option: (b) Switzerland. India was at 43rd position in
2019 while Singapore is the on the top.
Explanation:
 The Democracy Index is an index
India- South Africa compiled by the Economist Intelligence
Unit (EIU), a UK-based company.
 India’s relationship with South Africa
is both fundamental and unique, dating  India (ranked 41 ) ranks below the US
back several centuries and is anchored in (ranked 25th) and other so-called ‘flawed
common ideals, ideas, interests, and icons – democracies’ like Italy, France, Botswana
like Mahatma Gandhi and Nelson Mandela. and South Africa in the Democracy Index
However, their bilateral relationship 2018.
remained strained for a long time due to
South Africa’s apartheid government. 14. Correct Option: (c)
 In May 1993, a Cultural Center was Explanation:
opened in Johannesburg. In November
Room for the River
1993, diplomatic and consular relations
were restored during the visit of the South  It is a flagship project of the Dutch
African Foreign Minister Pik Botha to government, centered on protecting areas
India. adjoining rivers from routine flooding and
TARGET PT 2020 141

improving water management systems in the executive functions of the Centre.


delta regions. Conversely, the Governor of a State may,
with the consent of the Central government,
 Kerala, which had witnessed the century’s
entrust to that government any of the
worst floods recently, is to incorporate the
executive functions of the state.
model for flood control in the Kuttanad
region.  This mutual delegation of administrative
functions may be conditional or
15. Correct Option: (a) unconditional.

Explanation:  The Constitution also makes a


provision for the entrustment of the
Largest EEZ in the Indo-Pacific region executive functions of the Centre to
a state without the consent of that
 France, due to its numerous overseas state. But, in this case, the delegation
departments and territories scattered is by the Parliament and not by the
all over the oceans, has the largest president.
EEZ in the Indo-Pacific region but also
in the world.  Thus, a law made by the Parliament on
a subject of the Union List can confer
 The total area of the French Republic’s EEZ powers and impose duties on a state, or
is 4,514,000 sq mi, which is about 8% of the authorize the conferring of powers and
world’s exclusive economic zones, while the imposition of duties by the Centre upon
country’s land area represents only 0.45% a state (irrespective of the consent of the
of the world’s land area. state concerned). Notably, the same
 The United States has the world’s thing cannot be done by the state
second-largest EEZ, which includes areas legislature.
in the Caribbean Sea, the Gulf of Mexico  From the above, it is clear that the mutual
and three oceans. The United States EEZ delegation of functions between the Centre
occupies an area of about 4,383,000sq mi and the state can take place either under
which is about 7.77% of the world’s total an agreement or by legislation. While the
EEZ area. Centre can use both methods, a state can
use only the first method.
 Australia has the third-largest EEZ,
which occupies an area of about 3,283,933
17. Correct Option: (c)
sq mi.
 Russia and the United Kingdom are at 4th
Explanation:
and 5th place correspondingly. Legislative Relations between Union and
States
16. Correct Option: (a)
 In case of a conflict between the Central law
Explanation: and the state law on a subject enumerated
in the Concurrent List, the Central law
Mutual Delegation of Functions prevails over state law. But, there is
 The distribution of legislative powers an exception. If the state law has been
between the Centre and the States is reserved for the consideration of the
rigid. Consequently, the Centre cannot president and has received his assent,
delegate its legislative powers to the then the state law prevails in that
states and a single state cannot request state. But, it would still be competent for
the Parliament to make a law on a state the Parliament to override such a law by
subject. subsequently making a law on the same
matter.
 The distribution of executive power,
in general, follows the distribution of  The Constitution expressly secures the
legislative powers. But, such a rigid predominance of the Union List over the
division in the executive sphere may State List and the Concurrent List and that
lead to occasional conflicts between the of the Concurrent List over the State List.
Thus, in case of overlapping between the
two. Hence, the Constitution provides for
Union List and the State List, the former
inter-government delegation of executive
should prevail. In case of overlapping
functions in order to mitigate rigidity and
between the Union List and the Concurrent
avoid a situation of deadlock.
List, it is again the former which should
 Accordingly, the President may, with prevail. Where there is a conflict between
the consent of the state government, the Concurrent List and the State List, it
entrust to that government any of is the former that should prevail.
142 TARGET PT 2020

 The bills on certain matters (and not legislatures of the concerned states. The
all matters) in the State List can be effect of passing a resolution under the
introduced in the State Legislature above provision is that the Parliament
only with the previous sanction becomes entitled to legislate with respect
of the President (For example bills to a matter for which it has no power to
imposing restriction on freedom of trade & make a law. On the other hand, the state
commerce. legislature ceases to have the power to
make a law with respect to that matter.
18. Correct Option: (b) The resolution operates as abdication
Explanation: or surrender of the power of the state
legislature with respect to that matter
Parliamentary Legislation in the State and it is placed entirely in the hands of
Field Parliament which alone can then legislate
 Constitution empowers the Parliament with respect to it.
to make laws on any matter enumerated  To Implement International
in the State List under the following five Agreements: The Parliament can make
extraordinary circumstances: laws on any matter in the State List for
 When Rajya Sabha Passes a Resolution: implementing the international treaties,
If the Rajya Sabha declares that it is agreements or conventions. This provision
necessary for the national interest that enables the Central government to
Parliament should make laws on a matter fulfill its international obligations and
in the State List, then the Parliament commitments. Some examples of laws
becomes competent to make laws on enacted under the above provision are the
that matter. Such a resolution must be United Nations (Privileges and Immunities)
supported by two-thirds of the members Act, 1947; Geneva Convention Act, 1960;
present and voting. The resolution remains Anti-Hijacking Act, 1982 and legislations
in force for one year; it can be renewed any relating to environment and TRIPS.
number of times but not exceeding one
year at a time. The laws cease to have an  During President’s Rule: When the
effect on the expiration of six months after President’s rule is imposed in a state, the
the resolution has ceased to be in force. Parliament becomes empowered to make
This provision does not restrict the power laws with respect to any matter in the
of a state legislature to make laws on the State List in relation to that state. A law
same matter. But, in case of inconsistency made so by the Parliament continues to be
between state law and a parliamentary operative even after the president’s rule.
law, the latter is to prevail. This means that the period for which such
 During a National Emergency: The a law remains in force is not co-terminus
Parliament acquires the power to legislate with the duration of the President’s rule.
with respect to matters in the State List, But, such a law can be repealed or altered
while a proclamation of national emergency or re-enacted by the state legislature.
is in operation. The laws become inoperative  If Rajya Sabha initiates a resolution with
on the expiration of six months after the
the special majority and declares that a
emergency has ceased to operate. Here also,
subject is of national interest. (Under Art
the power of a state legislature to make
249).
laws on the same matter is not restricted.
But, in case of repugnancy between state
law and parliamentary law, the latter is to 19. Correct Option: (b)
prevail. Explanation:
 When States Make a Request: When
the legislatures of two or more states pass Inter-State Water Dispute
resolutions requesting the Parliament to  Interstate water disputes are part of The
enact laws on a matter in the State List, Union List under the Seventh Schedule of
then the Parliament can make laws for the constitution.
regulating that matter. A law so enacted
applies only to those states which have  The Constitution does not provide
passed the resolutions. However, any other for the formation of special tribunals
state may adopt it afterward by passing a for the adjudication of inter-state
resolution to that effect in its legislature. water disputes. Rather Article 262 of the
Such a law can be amended or repealed Constitution provides for the adjudication
only by the Parliament and not by the of inter-state water disputes.
TARGET PT 2020 143

 In case of disputes, Article 262 provides: and intercourse. Thus, Article 301 will be
violated whether restrictions are imposed
! Parliament may by law provide
for the adjudication of any dispute at the frontier of any state or at any prior
or complaint with respect to the or subsequent stage.
use, distribution or control of the  The freedom guaranteed by Article 301
waters of, or in, any inter-State is a freedom from all restrictions, except
river or river valley.
those which are provided for in the other
! Under this provision, the Parliament provisions (Articles 302 to 305) of Part
has enacted two laws i.e. the River XIII of the Constitution itself. These are
Boards Act (1956) and the Inter-State explained below:
Water Disputes Act (1956).
! Parliament can impose restrictions
! The Inter-State Water Disputes Act on the freedom of trade, commerce
empowers the Central government and intercourse between the states
to set up an ad hoc tribunal for the
or within a state in public interest.
adjudication of a dispute between two
But, the Parliament cannot give
or more states in relation to the waters
preference to one state over another
of an inter-state river or river valley.
or discriminate between the states
! The decision of the tribunal would except in the case of scarcity of goods
be final and binding on the parties in any part of India.
to the dispute. Neither the Supreme
Court nor any other court is to have ! The legislature of a state can
jurisdiction in respect of any water impose reasonable restrictions on
dispute which may be referred to as the freedom of trade, commerce
such a tribunal under this Act. and intercourse with that state or
! Notwithstanding, anything in this within that state in public interest.
Constitution, Parliament may, by law But, a bill for this purpose can
provide that neither the Supreme be introduced in the legislature
Court nor any other court shall only with the previous sanction
exercise jurisdiction in respect of of the President. Further, the state
any such dispute or complaint as is legislature cannot give preference to
referred to in Clause (1). one state over another or discriminate
! Entry 56 of Union list enables the between the states.
Union to deal with interstate rivers if ! The legislature of a state can impose
Parliament legislates for this purpose. on goods imported from other states
It states that “Regulation and or the union territories any tax to
development of inter-State rivers and
which similar goods manufactured
river valleys to the extent to which
in that state are subject. This
such regulation and development
provision prohibits the imposition of
under the control of the Union is
declared by Parliament by law to be discriminatory taxes by the state.
expedient in the public interest.” ! The freedom (under Article 301) is
subject to the nationalisation laws (i.e.,
20. Correct Option: (d) laws providing for monopolies in favour
Explanation: of the Centre or the states). Thus, the
Parliament or the state legislature can
Inter-State Trade And Commerce make laws for the carrying on by the
 Article 301 declares that trade, commerce respective government of any trade,
and intercourse throughout the territory business, industry or service, whether
of India shall be free. The object of this to the exclusion, complete or partial,
provision is to break down the border of citizens or otherwise.
barriers between the states and to create
one unit with a view to encourage the free 21. Correct Option: (a)
flow of trade, commerce and intercourse in
the country. Explanation:
 The freedom under this provision  Statement 2 is incorrect: National
is not confined to inter-state trade, Sports Day is celebrated on the birth
commerce and intercourse but also anniversary of hockey legend Major
extends to intra-state trade, commerce Dhyan Chand.
144 TARGET PT 2020

Supplementary Notes: ! The report has also removed the


interim payout structure in general
Fit India Movement circumstances.
 Prime Minister has launched nation- ! All the recommendations of the
wide Fit India Movement on the panel have been accepted by the
occasion of National Sports Day. RBI.
 Fit India Movement aims to motivate every
Indian to incorporate simple, easy ways of 23. Correct Option: (a)
staying fit in their everyday life.
Explanation:
Na onal Sports Day  Statement 1 is incorrect: In the absence
 It is celebrated on 29 August, on of a domestic legislation, it would not
the birth anniversary of hockey be possible to put a blanket ban on its
legend Major Dhyan Chand. imports.

 On this Day, President confers the National  Statement 2 is correct: Electronic


Sports Awards, National Adventure cigarettes do not contain tobacco.
Awards, Arjuna Award, Khel Ratna,
Dronacharya Award and Dhyanchand Supplementary Notes:
Award to recognise the exceptional
Electronic Cigarette
achievements of Indian sportspersons.
 The Commerce Ministry has asked
22. Correct Option: (a) the Health Ministry to frame a law
banning manufacture and sale of
Explanation: e-cigarettes in the country as in the
Supplementary Notes: absence of a domestic legislation, it
would not be possible to put a blanket
RBI Economic Capital Framework ban on its imports. E-cigarettes are set
 Recently, the Reserve Bank of India to be banned entirely on grounds that they
(RBI) accepted the recommendations are “drugs”.
of Bimal Jalan panel; set up to review  An electronic cigarette (or e-cig) is a
the RBI’s Economic Capital Framework.
battery-powered vaporizer that mimics
 Following these recommendations, RBI tobacco smoking. It works by heating up
decided to transfer Rs 1.76 lakh crore in nicotine liquid, called “juice”.
dividend and surplus reserves to the
! Nicotine juice (or e-juice) comes in
government.
various flavours and nicotine levels.
 The objective of the economic capital
! E-liquid contains vegetable
framework is to build harmony
glycerine (a material used in all
between the central bank’s need for
types of food and personal care
autonomy and the Government’s
products, like toothpaste) Propylene
objectives of the development.
glycol (a solvent most commonly used
 The Bimal Jalan-led panel in fog machines). Propylene glycol is
recommended holistic risk capital the ingredient that produces thicker
frameworks to assess the adequacy of RBI clouds of vapour.
reserves. Some recommendations of the
committee are:  Proponents of e-cigs argue that the practice
is healthier than traditional cigarettes
! RBI to maintain the Contingency Risk because users are only inhaling water
Buffer (CRB), which is the country’s vapour and nicotine.
fund to handle financial stability
within the range of 5.5% to 6.5% of the  As e-cigarettes contain nicotine and
RBI’s balance sheet. not tobacco, they do not fall within
the ambit of the Cigarettes and Other
! It recommended a review of the Tobacco Products (Prohibition of
RBI’s Economic Capital Framework Advertisement and Regulation of
(ECF) every five years. Trade and Commerce, Production,
! The RBI’s accounting year of July- Supply and Distribution) Act, 2003
June can be brought in sync with (COTPA), which mandates stringent
the fiscal year of April-March from the health warnings on the packaging and
financial year 2020-21. advertisements of tobacco products.
TARGET PT 2020 145

24. Correct Option: (b)  The report also emphasised on triggering


policy level changes, seek better budgetary
Explanation: allocations and initiate discussions with all
 Statement 1 is incorrect: The Report stakeholders which can help in enhancing
is released by the non- government the quality of life of all children in the
organization World Vision India and country.
research institute IFMR LEAD.
25. Correct Option: (a)
Supplementary notes:
Explanation:
Child well-being index  Statement 2 is incorrect: Amount of
 The India child well-being index is a bending is one of the predictions of Albert
crucial report that can be mined both by Einstein’s general theory of relativity.
the Government and civil organisations to
Supplementary notes:
achieve the goal of child well-being and we
will use this report effectively. Gravitational Lensing
 This report provides insights on health,  A gravitational lens is a distribution of
nutrition, education, and sanitation and matter (such as a cluster of galaxies)
child protection. between a distant light source and an
 The dimensions of the index include observer that is capable of bending the
light from the source as the light travels
healthy individual development, positive
towards the observer. This effect is known
relationships and protective contexts.
as gravitational lensing.
 Focusing on the three key dimensions, 24
 Amount of bending is one of the predictions
indicators were selected to develop the
of Albert Einstein’s general theory of
computation of the child well-being index. relativity.
 The index captures the performance of each  The phenomenon occurs when a huge
state and union territory on a composite amount of matter, such as a massive galaxy
child well-being score. or cluster of galaxies, creates a gravitational
 Kerala, Tamil Nadu, Himachal Pradesh field that distorts and magnifies the light
and Puducherry topped the charts in the from objects behind it, but in the same line
child well-being index. of sight.

 Meghalaya, Jharkhand and Madhya  These large celestial objects will magnify
Pradesh featured at the bottom. the light from distant galaxies that are at
or near the peak of star formation.
 Among the union territories, Puducherry
 Gravitational lensing is useful to
led the way and Dadra and Nagar Haveli
cosmologists because it is directly sensitive
featured at the other end.
to the amount and distribution of dark
 The report has called for states to look at matter.
their respective scores on the dimensions of  Lensing can therefore help astronomers
child well-being and to prepare for priority work out exactly how much dark matter
areas of intervention with specific plans of there is in the Universe as a whole, and
action. also how it is distributed.

**********
146 TARGET PT 2020

TEST
DAY - 14

Time Allowed: 30 mins Maximum Marks: 50

1. Which of the following decides the (a) 4 only


question of disqualification of a
(b) 2 only
member of the Parliament, arising on
the ground of defection? (c) 3 and 4 only
(a) Election Commission (d) 1, 2, and 4 only
(b) Supreme Court 4. Consider the following statements
(c) President of India regarding the nature of the political
parties in India:
(d) Presiding Officer of the House 1. Except for the communist parties, all
other parties do not have a clear-cut
2. Consider the following statements
ideology.
regarding Association for Democratic
Reforms: 2. Every major party in India advocates
Gandhism.
1. It is a non-governmental organization,
founded in 1999. 3. Only a few parties have declared itself
as the ‘public authority’ under the
2. It was established by a group of
Right to Information Act, 2005.
professors from the Indian Institute of
Management Ahmedabad. Which of the above statements is/are correct?
3. The concept of electoral bonds was (a) 2 only
introduced by it.
(b) 2 and 3 only
Which of the above statements is/are correct? (c) 1 and 2 only
(a) 1 and 2 only (d) 1, 2, and 3
(b) 3 only
5. Consider the following statements
(c) 2 only regarding the election in India:
(d) 1 and 3 only 1. Article 324 provides equality in the
removal of the election commissioners.
3. Consider the following statements 2. In the T.N. Seshan ECI vs Union
regarding the use of the Electronic of India case, the Supreme Court
Voting Machines in India: ruled that the CEC is first among the
1. EVMs were first used in Assembly equals.
Constituency of Kerala in 1982. 3. The Model Code of Conduct was
2. EVMs do not run where there is no introduced for the first time in the 1991
electricity. General Election.
3. The EVMs have been devised and Which of the above statements are
designed by the Indian Public Sector incorrect?
undertakings.
(a) 2 only
4. VVPATs with EVMs were used for the
first time in Nagaland. (b) 1 and 3 only

Which of the above statements is/are (c) 1 and 3 only


incorrect? (d) 1, 2, and 3
TARGET PT 2020 147

6. Which of the following pressure/ 1. It is an executive body, set up by the


interest groups was (were) founded cabinet resolution.
before the Indian independence? 2. It defines the ‘child’ as a person in the
1. Jamaat-e-Islami 0 to 18 years age group.
2. All India Trade Union Congress 3. It monitors the implementation of
Protection of Children from Sexual
3. All India Students Federation
Offences (POCSO) Act.
4. ASSOCHAM
Select the correct option using the codes
Select the correct option using the codes given below:
given below:
(a) 2 only
(a) 1 only
(b) 3 only
(b) 2 and 4 only (c) 2 and 3 only
(c) 2, 3, and 4 (d) 1, 2, and 3
(d) 1, 2, 3, and 4
10. For which of the following purposes
7. Which of the following statements the Justice G. Rohini Commission has
regarding the Competition Commission been constituted?
of India is correct? (a) Sub-categorization of Other Backward
1. It is a statutory body responsible for Classes
enforcing the Competition Act, 2013. (b) Sub-categorization of Scheduled
2. Its aim is to prevent monopoly as well Castes
as cartelization of the Indian economy. (c) Reservations for Economic Weaker
Section
Select the correct option using the codes
given below: (d) Reservations for Marathas
(a) 1 only 11. Consider the following statements
(b) 2 only regarding the Geneva Convention:
(c) Both 1 and 2 1. The origin of the Geneva Conventions
dates back to the Second World War.
(d) Neither 1 nor 2
2. All the Conventions and Protocols were
signed in 1949.
8. Consider the following statements
regarding the National Financial 3. Any nation that has ratified the Geneva
Reporting Authority: Conventions but not the Protocols
are not bound by all provisions of the
1. It has been established under the
Conventions.
Companies Act, 2013.
2. It is the auditor of auditors. Which of the above statements is/are
incorrect?
3. It can impose sanctions as well as
penalties against defaulting auditors (a) 3 only
and audit firms (b) 1 and 2 only
4. Its powers extend to the listed (c) 2 and 3 only
companies only.
(d) 1, 2, and 3
Which of the above statements are correct?
12. Recently, the World Health
(a) 1 and 2 only Organization has designated the
(b) 1, and 3 only year 2020 as the “Year of Nurse and
Midwife”, in honor of which of the
(c) 1, 2, and 3 only following personalities?
(d) 2 and 4 only (a) Mother Teresa
9. Which of the following statements (b) Florence Nightingale
regarding the National Commission for (c) Harper Lee
Protection of Child Rights is correct?
(d) Dorothea Dix
148 TARGET PT 2020

13. Which of the following statements is 3. Liberal parties: Aim at reforming the
correct regarding the “Davos in the existing institutions
desert”? 4. Radical parties: Aim at establishing a
(a) It is a branch office of the World new order by overthrowing the existing
Economic Forum in Dubai. institutions
(b) It is used for the new International Select the correct option form the codes
football stadium in Doha for the next given below:
FIFA world cup.
(a) 1, 2, 3 and 4
(c) It is an initiative of Saudi Arabia to
(b) 2 and 3 only
diversify the country’s economy.
(c) 1 and 2 only
(d) All of the above.
(d) 3 and 4 only
14. Consider the following countries:
17. Regarding the Representation of the
1. India
People Act, 1950, which the following
2. Japan statement is incorrect?
3. Indonesia (a) It provides for the allocation of seats
4. Thailand in the House of the People and in the
Legislative Assemblies and Legislative
5. Singapore Councils of States.
Which of the above countries is/are a part of (b) It confers the powers on the Parliament
the Asian Tea Alliance? to delimit the various constituencies
after consultation with the Election
(a) 1 and 2 only
Commission.
(b) 1, 2, and 3 only
(c) It provides for the registration of electors
(c) 1, 2, 3, and 5 only for Parliamentary Constituencies
and for the Assembly and Council
(d) 2, 3, 4, and 5 only
Constituencies.
15. Which of the following statements (d) It also provides for the qualifications
regarding Greta Thunberg is/are and disqualifications for registration of
correct? electors.
1. She is a Norwegian environmental
18. Consider the following statements:
activist.
1. Notification of general elections
2. She is the youngest person to be named
as Time Person of the Year. 2. Registration of political parties
3. She has been awarded the “Alternative 3. Qualifications and disqualifications for
Nobel Prize”. membership of Parliament and State
Legislatures
Select the correct statements using the 4. Disputes regarding elections
codes given below:
Which of the following matters is/are
(a) 1 and 3 only
provided in the Representation of the
(b) 2 only People Act, 1951?
(c) 2 and 3 only (a) 1, 2 and 4 only
(d) 1, 2, and 3 (b) 1, 3 and 4 only
(c) 1, 2 and 3 only
16. hich of the following pairs is correctly
matched regarding the types of (d) 1, 2, 3 and 4
political parties:
19. Which of the following committee/s is/
1. Reactionary parties: Believe in the
are related to Electoral Reforms?
status-quo
1. J.S. Verma Committee
2. Conservative parties: Cling to the
old socio-economic and political 2. Prakash Singh Committee
institutions 3. Indrajit Gupta Committee
TARGET PT 2020 149

Select the correct answer form the codes (b) 2 only


given below:
(c) 3 only
(a) 1 only
(d) None of the above
(b) 1 and 2 only
(c) 1 and 3 only 23. Consider the following:

(d) 3 only List I List II


1. Jeeraphool Rice Chhattisgarh
20. Regionalism is a subsidiary process of
political integration in India. Which 2. Dindigul lock Tamil Nadu
of the following events manifests 3. Kandangi Saree Uttar Pradesh
regionalism in India?
1. Demand of the people of certain states Which of the above given pairs is/are
for secession from the Indian Union correctly matched?
2. Inter-state boundary disputes (a) 1 only
3. Violent disputes and agitations over (b) 1 and 2 only
the reservation policy
(c) 2 and 3 only
4. Emergence of pressure groups
(d) 2 only
5. Sons of the soil theory

Select the correct answer form the codes 24. Consider the following statements
given below: regarding the World Skills
Competition:
(a) 1, 2 and 5 only
1. The 45th World Skills Competition is
(b) 1, 2, 3, 4 and 5 being held at Beijing, China.
(c) 1, 2, 4 and 5 2. It is the biggest vocational education and
(d) 1, 2, 3 and 4 skills excellence event in the world.

21. Which one of the following is a Legacy Which of the above statements is/are
Dispute Resolution scheme which correct?
offers to those taxpayers who wish to (a) 1 only
disclose any previously undisclosed
tax liability without any penalty or (b) 2 only
prosecution? (c) Both 1 and 2
(a) Sabka Vishwas Scheme
(d) Neither 1 nor 2
(b) KALIA Scheme
25. Consider the following Statements
(c) PAHAL Scheme
regarding Open Acreage Licensing
(d) Pandit Deendayal Upadhyay Shramev Policy (OALP)
Jayate Yojana
1. Oil and gas acreages will be available
22. Consider the following statements round the year instead of cyclic bidding
regarding the Project SURE: rounds as in NELP.
1. It aims to set a sustainable pathway 2. Setting up of National Data Repository
for India’s food industry. is one of the milestones achieved for
2. It was launched by Minsitry of Open Acreage Licensing Policy.
Environment, Forest and Climate Which of the above statements is/are
Change.
correct?
3. It would help in reduction of carbon
emissions. (a) 1 only
(b) 2 only
Which of the above statements is/are
correct? (c) Both of them
(a) 1 and 2 only (d) None of the above
150 TARGET PT 2020

ANSWER HINTS
DAY - 14

1. Correct Option: (d)  It opposes the Electoral Bonds, even


moved Supreme Court for stay on the
Explanation: Electoral Bond scheme in 2018.
Anti-Defection Law
3. Correct Option: (b)
 The 52nd Amendment Act of 1985 provided
for the disqualification of the members of Explanation:
Parliament and the state legislatures on
Electronic Voting Machines in India
the ground of defection from one political
party to another.  Electronic Voting Machine (EVM) is an
electronic device for recording votes. An
 For this purpose, it added a new Schedule
Electronic Voting Machine consists of two
(the Tenth Schedule) to the Constitution.
Units – a Control Unit and a Balloting
 This act is often referred to as the Anti- Unit – joined by a five-meter cable.
Defection Law.
 EVMs were first used in 70-Parur
 The question of disqualification of a Assembly Constituency of Kerala in
member of the Parliament, arising on the year 1982.
the ground of defection, is decided
 EVMs do not require electricity. EVMs
by the Speaker/Chairperson of Lok
run on an ordinary battery assembled by
sabha/Rajya Sabha.
Bharat Electronics Limited/Electronics
Corporation of India Limited.
2. Correct Option: (a)
 The EVMs have been devised and designed
Explanation: by the Technical Experts Committee
(TEC) of the Election Commission in
Association for Democratic Reforms (ADR) collaboration with two Public Sector
 The Association for Democratic undertakings viz., Bharat Electronics Ltd.,
Reforms (ADR) was established Bangalore and Electronic Corporation
as an NGO in 1999 by a group of of India Ltd., Hyderabad. The EVMs
professors from the Indian Institute of are manufactured by the above two
Management (IIM) Ahmedabad. undertakings.
 In 1999, Public Interest Litigation (PIL)  Voter Verifiable Paper Audit Trail (VVPAT)
was filed by them with Delhi High Court is an independent system attached with
asking for the disclosure of the criminal, the Electronic Voting Machines that allows
financial and educational background of the voters to verify that their votes are cast
the candidates contesting elections. Based as intended.
on this, the Supreme Court in 2002, and  VVPAT too runs on a power pack Battery.
subsequently in 2003, made it mandatory
for all candidates contesting elections to  VVPATs with EVMs were used for the
disclose criminal, financial and educational first time in a bye-election from 51-
background prior to the polls by filing an Noksen (ST) Assembly Constituency
affidavit with the Election Commission. of Nagaland.
 It conducts the ‘election watch’ ( since
2002) for state assembly elections and the
4. Correct Option: (a)
‘National Election Watch’. Explanation:
 It conducts multiple projects aimed at
Nature of political parties in India
increasing transparency and accountability
in the political and electoral system of the  Except for the BJP and the two
country. communist parties (CPI and CPM),
TARGET PT 2020 151

all other parties do not have a clear-  The Act also mandates that the Election
cut ideology. They (i.e., all other parties) Commission may, by unanimous decision,
are ideologically closer to each other. They regulate the procedure for transaction of its
have a close resemblance in their policies business as also allocation of its business
and programs. amongst the Chief Election Commissioner
 Almost every party in India advocates and other Election Commissioner.
democracy, secularism, socialism, and  T. N. Seshan, arguably the most honest
Gandhism. and effective CEC in India, challenged
 Political parties across the ideological provisions of the Act but, the Supreme
spectrum are in consonance to not Court in the T.N. Seshan ECI vs Union of
complying with the RTI Act, 2005. India 1995, dismissed the petition.
 Despite a June 2013 ruling from the Central  The Apex Court of India ruled that the
Information Commission (CIC) that they business of the Commission may be carried
(parties) fall within the ambit of the out by unanimous decision as far a possible
transparency law, parties insist that they but in cases of there being no unanimity
cannot be considered public authorities the rule of majority shall prevail.
under the Act.
 The supreme guardian of the
 The public interest litigation filed by Constitution thus, held that the chief
Ashwini Kumar Upadhyay, a Bharatiya election commissioner was not superior
Janata Party (BJP) leader, wants political
to the election commissioners, rather
parties registered under section 29A of
of the same position. Thus, CEC is the
Representation of People Act, 1951 to be
declared as ‘public authority’ under the first among equals but not superior to
Right to Information Act, 2005, (RTI) is yet other ECs.
to be decided by the apex court of India.
6. Correct Option: (d)
5. Correct Option: (c) Explanation:
Explanation:
Pressure/interest groups
Election and Election Commission of India  A pressure group is a group of people/
 The MCC is a set of guidelines issued by the organizations who are organized activities
Election Commission to regulate political for promoting and defending their common
parties and candidates prior to elections, to interest. It attempts to bring a change in
ensure free and fair elections in consonance public policy by exerting pressure on the
with Article 324 of the Constitution. It was government. It acts as a liaison between
first introduced in the state assembly the government and its members.
elections in Kerala in 1960. But, the
Election Commission first effectively put to  The pressure groups are also called interest
use the Model Code of Conduct in 1991. groups or vested groups.
 As per Article 324, other election  It can include political parties (viz.
commissioners cannot be removed from Congress, BJP, CPI, etc.), trade unions
office except on the recommendation of the (AITUC, MKS, etc), student unions(ABVP,
chief election commissioner. Thus, Article NSUI, AISA. SFI, AISF, etc.), Business
324 does not provide parity in the Groups (CII, ASSOCHAM, etc.), religious
removal of the officers of the ECI. organizations(RSS, VHP, etc), etc.
 The 61st Amendment Act of 1988 has  Jamaat-e-Islami, an Islamist political and
reduced the voting age from 21 to 18 years right-wing Muslim nationalist movement,
and two more election commissioners. But, was founded in 1941 by the Islamist Abul
the two posts of election commissioners Ala Maududi.
were abolished in January 1990 and the
Election Commission was reverted to the  The All India Trade Union Congress is the
earlier position. oldest trade union federations in India and
was founded on 31 October 1920.
 Further, the Government of India enacted
EC (Conditions of Services of Election  The All India Students’ Federation is
Commissioners and Transactions of the oldest student organization in India
Business Act,1991) Act which mandates founded in 1936 influenced by left-wing
that the chief election commissioner and ideology.
the two other election commissioners
would have equal powers and receive equal  Both CII and ASSOCHAM were founded
salary, allowances and other perquisites, before 1947. ASSOCHAM was established in
which are similar to those of a judge of the 1920 by promoter Chambers, representing
Supreme Court. all regions of India.
152 TARGET PT 2020

7. Correct Option: (b)  The powers of the NFRA include the


governing of auditors of companies
Explanation: listed in any stock exchange, in India
or outside of India, unlisted public
Competition Commission of India companies above certain thresholds,
 The Competition Act, 2002 was passed and other companies.
by the Parliament and was subsequently  Companies are required to disclose
amended by the Competition (Amendment) information of their auditors to the NFRA
Act, 2007. through Form NFRA-1.
 In accordance with the provisions of
the Amendment Act, the Competition 9. Correct Option: (c)
Commission of India and the Explanation:
Competition Appellate Tribunal were
established in 2003. National Commission for Protection of
 Its aim is to promote and sustain an Child Rights
enabling robust competition culture  The National Commission for Protection
through engagement and enforcement of Child Rights (NCPCR) was set up in
that would inspire businesses to be fair, March 2007 under the Commissions for
competitive and innovative; enhance Protection of Child Rights (CPCR)
consumer welfare, and support economic Act, 2005. Thus, it is a statutory body
growth. under the administrative control
of the Ministry of Women & Child
 It prevents activities that have an
Development.
adverse effect on competition such as
monopolization and the cartelization  The Commission’s Mandate is to ensure
of the Indian economy and market. that all Laws, Policies, Programmes,
and Administrative Mechanisms are
8. Correct Option: (c) in consonance with the Child Rights
perspective as enshrined in the Constitution
Explanation: of India and also the UN Convention on the
Rights of the Child.
National Financial Reporting Authority
 The Child, as per the Act, is defined
 The need for establishing NFRA has arisen as a person in the 0 to 18 years age
on account of the need felt across various group.
jurisdictions in the world, in the wake of
accounting scams(for instance, Satyam  Besides other functions performed by
scam), to establish independent regulators, it, the Commission monitors the
independent from those it regulates, for implementation of the POCSO Act.
enforcement of auditing standards and
ensuring the quality of audits to strengthen 10. Correct Option: (a)
the independence of audit firms, quality of Explanation:
audits and, therefore, enhance investor and
public confidence in financial disclosures of Justice G. Rohini Commission
companies.
 Union Government with approval
 NFRA has been established as an of President had constituted a five-
independent regulator for the auditing member Commission under article 340 of
profession which is one of the key Constitution in October 2017.
changes brought in by the Companies
 It has been formed to take up the exercise
Act, 2013. of identifying castes, communities,
 The jurisdiction of NFRA for sub-castes, synonyms in the central
investigation of Chartered Accountants list of OBCs and classifying them into
and their firms under section 132 of sub-categories.
the Act.  Article 340 of the Constitution was also
 According to Section 132 of Companies used to establish landmark Mandal
Act 2013, “NFRA is responsible for Commission (set up in 1979) more than
recommending accounting and two decades ago which recommend 27%
auditing policies and standards in the reservation for socially and educationally
country, undertaking investigations backward classes in higher education
and imposing sanctions against and government jobs. Over the years, the
defaulting auditors and audit firms benefits of this reservation were mostly
cornered mostly by dominant OBC groups.
in the form of monetary penalties and
debarment from practice for up to 10  Even National Commission for Backward
years. Classes (NCBC) had in 2015 had noted
TARGET PT 2020 153

that unequal cannot be treated equally and 12. Correct Option: (b)
recommended that OBCs be categorized
into extremely backward classes, more Explanation:
backward classes, and backward classes.
At present, 11 states have sub-categorized Florence Nightingale
OBCs for their state services.  Florence Nightingale was a British social
reformer who founded modern nursing.
11. Correct Option: (d) Her biggest contribution to humanity
was when she volunteered to help in the
Explanation: Crimean war to tend to wounded soldiers.
Geneva Conventions  Later in life, she tried to bring about
 The Geneva Conventions is a body of reforms regarding the hygiene problems of
Public International Law, also known as the Army hospitals.
the Humanitarian Law of Armed Conflicts,  She wrote many books and letters to
whose purpose is to provide minimum improvise the treatments and care given to
protections, standards of humane wounded soldiers.
treatment, and fundamental guarantees of
respect to individuals who become victims  She laid the foundation for pursuing
of armed conflicts. nursing as a profession by elite British
women and others as well.
 Their origin dates back to 1864 when
the Swiss businessman and the first Nobel  She was called “The Lady with the Lamp”
peace laureate Henry Dunant led to the for making rounds on wounded soldiers.
establishment of the Red Cross society and  In the honor of 200th birth anniversary of
the 1864 Geneva Convention, the first Florence Nightingale, the World Health
codified international treaty that covered Organization designated the year 2020 as
the sick and wounded soldiers on the the “Year of Nurse and Midwife”.
battlefield.
 The conventions are as follows: 13. Correct Option: (c)
! The First Geneva Convention “for the Explanation:
Amelioration of the Condition of the
Wounded and Sick in Armed Forces Davos in the desert
in the Field” (first adopted in 1864,
 It is an informal name for the ‘Future
revised in 1906, 1929 and finally
1949); Investment Initiative’, an ambitious
project of Saudi Arabia.
! The Second Geneva Convention “for
the Amelioration of the Condition  It is an initiative by the Saudi Crown
of Wounded, Sick and Shipwrecked Prince Mohammad bin Salman to
Members of Armed Forces at Sea” diversify the kingdom’s economy and
(first adopted in 1949, successor of reduce its dependence on petroleum
the Hague Convention (X) 1907); products.

! The Third Geneva Convention  It is an annual investment forum held in


“relative to the Treatment of Prisoners Riyadh, Saudi Arabia, to discuss trends
of War” (first adopted in 1929, last in the world economy and investment
revision in 1949); environment.

! The Fourth Geneva Convention 14. Correct Option: (b)


“relative to the Protection of Civilian
Persons in Time of War” (first Explanation:
adopted in 1949, based on parts of
the Hague Convention (II) of 1899 and Asian Tea Alliance
Hague Convention (IV) 1907).  The Asian Tea Alliance (ATA) announced
 Two additional protocols were in April 2019 is a memorandum of
adopted in 1977 and the third protocol understanding between the main industry
was added in 2005. and policy organizations in five tea-growing
countries.
 In total, 196 countries have signed and
ratified the 1949 conventions over the years,  These are the Indian Tea Association,
including many that did not participate or China Tea Marketing Association,
sign until decades later. Indonesian Tea Marketing Association,
Sri Lanka Tea Board and Japan Tea
 Any nation that has ratified the Geneva Association.
Conventions but not the Protocols are
still bound by all provisions of the  The whole idea is to boost tea trade from
conventions. these countries.
154 TARGET PT 2020

15. Correct Option: (c) of seats in the House of the People


and in the Legislative Assemblies and
Explanation: Legislative Councils of States.
Greta Thunberg  In allotting seats in the House of the
 She is a Swedish environmental activist People to different States and in fixing the
on climate change whose campaigning has total number of seats in the Legislative
gained international recognition. Assemblies of different States, the
population of each State as on 1st March
 A 16-year-old from Stockholm she went 1950 was taken into account.
from a solitary protest on the cobblestones
outside her country’s Parliament to leading  The Act also sought to confer on the
a worldwide youth movement. President the powers to delimit, after
consultation with the Election Commission,
 She marshaled “Fridays for Future” the various constituencies for the purpose
protests throughout Europe by thundering, of elections to fill seats in the House of the
“How dare you!” at the world’s most
People and in the Legislative Assemblies
powerful leaders in her viral U.N. speech
and Legislative Councils of States.
which led some 7 million climate strikers
across the world in September and tens of  The Act further provided for the registration
thousands more in Madrid. of electors for Parliamentary
 Thunberg is the TIME’s Person of Constituencies and for the Assembly
2019, the youngest individual ever and Council Constituencies, and the
conferred with. qualifications and disqualifications
for such registration.
 She has been awarded the ‘Right
Livelihood Award’ is widely known  A special provision has been included for
as the ‘Alternative Nobel Prize’, relaxation of the residence qualification
distributed by Sweden. in the case of displaced persons who
migrated before the 25th day of July, 1949,
 Thunberg was also nominated for the 2019 to India from the territory of Pakistan.
Nobel Peace Prize.
The provisions have been made for the
preparation of electoral rolls, the period of
16. Correct Option: (d) currency of such rolls, and the revision and
Explanation: connection of such rolls during such period
in special cases.
Types of Political Parties
 There are four types of political parties in 18. Correct Option: (d)
the modern democratic states, viz., Explanation:
! reactionary parties which cling to
the old socio-economic and political Representation of the People Act, 1951
institutions;  The Representation of the People Act, 1950
! conservative parties which believe in did not contain all the provisions relating
the status-quo; to elections but merely provided for the
allocation of seats in and the delimitation of
! liberal parties which aim at reforming constituencies for the purpose of elections
the existing institutions; and to the House of People and Legislatures of
! radical parties which aim at States, the qualifications of voter at such
establishing a new order by election and the preparations of electoral
overthrowing the existing rolls.
institutions.
 Broadly speaking, this Act contains
 In their classification of political parties provisions relating to the following electoral
on the basis of ideologies, the political matters:
scientists have placed the radical parties
on the left and the liberal parties in the ! Qualifications and
centre and reactionary and conservative disqualifications for membership
parties on the right. of Parliament and State
Legislatures
17. Correct Option: (b) ! Notification of general elections
Explanation: ! Administrative machinery for the
conduct of elections
Representation of the People Act, 1950
! Registration of political parties
 The Representation of the People Act, 1950,
was enacted to provide for the allocation ! Conduct of elections
TARGET PT 2020 155

! Free supply of certain material to ! Law Commission of India 255th


candidates of recognised political Report on Electoral Reforms (2015).
parties
! Disputes regarding elections 20. Correct Option: (a)
! Corrupt practices and electoral Explanation:
offences
Regionalism
19. Correct Option: (c)
 Regionalism refers to sub-nationalism and
Explanation: sub-territorial loyalty. It implies the love
for a particular region or state in preference
Electoral Reforms
to the country as a whole.
 The various committees and commissions
which have examined our electoral  There is also sub-regionalism, that is, love
system, election machinery as well as for a particular region in preference to the
election process and suggested reforms are state of which the region forms a part.
mentioned here.
 Regionalism is “a subsidiary process
! Joint Parliamentary Committee on of political integration in India. It is a
Amendments to Election Laws (1971- manifestation of those residual elements
72). which do not find expression in the
! Tarkunde Committee was appointed national polity and national culture, and
in 1974 by Jaya Prakash Narayan being excluded from the centrality of the
(JP) during his “Total Revolution” new polity, express themselves in political
movement. This unofficial committee discontent and political exclusionism”.
submitted its report in 1975.
 Regionalism is a country-wide phenomenon
! Dinesh Goswami Committee on
which manifests itself in the following six
Electoral Reforms (1990)
forms:
! Vohra Committee on the Nexus
between Crime and Politics (1993) ! Demand of the people of certain
states for secession from the
! Election Commission of India
Indian Union (like Khalistan, Dravid
Recommendations on Electoral
Nad, Mizos, Nagas and so on).
Reforms (1998).
! Indrajit Gupta Committee on ! Demand of the people of certain areas
State Funding of Elections (1998) for separate statehood (like Telengana,
Bodoland, Uttarkhand, Vidharbha,
! Law Commission of India 170th
Gorkhaland and so on).
Report on Reform of the Electoral
Laws (1999) ! Demand of people of certain Union
! National Commission to Review Territories for full-fledged statehood
the Working of the Constitution (like Manipur, Tripura, Puducherry,
(2000-2002). It was headed by M.N. Delhi, Goa, Daman and Diu and so
Venkatachaliah. on).
! Election Commission of India Report ! Inter-state boundary disputes
on Proposed Electoral Reforms (like Chandigarh and Belgaum) and
(2004).
river water disputes (like Cauvery,
! Second Administrative Reforms Krishna, Ravi-Beas and so on).
Commission of India Report on Ethics
in Governance (2007). It was headed ! Formation of organisations with
by Veerappa Moily. regional motives which advocates
a militant approach in pursuing its
! Tankha Committee (Core Committee)
policies and goals (like Shiv Sena,
was appointed in 2010 to look into the
whole gamut of the election laws and Tamil Sena, Hindi Sena, Sardar Sena,
electoral reforms. Lachit Sena and so on).
! J.S. Verma Committee Report on ! ‘Sons of the soil theory’ which
Amendments to Criminal Law advocates preference to local people
(2013). in government jobs, private jobs,
! Law Commission of India 244th permits and so on. Their slogan will
Report on Electoral Disqualifications be Assam for Assamese, Maharashtra
(2014). for Maharashtrians and so on.
156 TARGET PT 2020

21. Correct Option: (a) Supplementary Notes:

Explanation Project SURE


 Option (a) is correct: Sabka Vishwas  The SURE project is a commitment
Scheme is a Legacy Dispute Resolution by India’s apparel industry to set a
scheme which offers to those taxpayers who sustainable pathway for the Indian
wish to disclose any previously undisclosed fashion industry.
tax liability without any penalty or
 SURE stands for ‘Sustainable Resolution’ –
prosecution.
a firm commitment from the industry to
Supplementary Notes: move towards fashion that contributes to a
clean environment.
Sabka Vishwas Scheme
 The project has been launched by
 The scheme targets those taxpayers who the Union Textiles Ministry, along
want to close their pending disputes with Clothing Manufacturers Association
related to Service Tax and Excise Tax of India (CMAI); United Nations in India;
(now subsumed under Goods and Services and IMG Reliance.
Tax). Hence, the term ‘legacy’.  It will be the first holistic effort by the
 The two main components of the apparel industry towards gradually
Scheme are dispute resolution and introducing a broader framework for
amnesty. establishing critical sustainability goals
for the industry.
! The dispute resolution component
is aimed at liquidating the legacy cases  This framework would help the
of Central Excise and Service Tax that industry reduce its carbon emissions,
are subsumed in GST and are pending increase resource efficiency, tackle waste
in litigation at various forums. and water management, and create
positive social impact to achieve long-term
! The amnesty component of the sustainability targets.
Scheme offers an opportunity to the
taxpayers to pay the outstanding tax 23. Correct Option: (b)
and be free of any other consequence
under the law. The most attractive Explanation:
aspect of the Scheme is that it provides  Option (b) is correct:
substantial relief in the tax dues for
all categories of cases as well as full List I List II
waiver of interest, fine, penalty, In all
1. Jeeraphool Rice Chhattisgarh
these cases, there would be no other
liability of interest, fine or penalty. 2. Dindigul lock Tamil Nadu
There is also a complete amnesty from 3. Kandangi Saree Tamil Nadu
prosecution.
! The Scheme is especially tailored Supplementary notes:
to free the large number of small
Geographical Indication (GI) tag
taxpayers of their pending disputes
with the tax administration.  Recently, two well-known products
from Tamil Nadu — Dindigul lock and
 In News - Sabka Vishwas Scheme was
Kandangi Saree — have been given the
announced in the Union Budget 2019.
Geographical Indication (GI) tag.
It has now been notified and will be
operationalized from 1st September 2019. Dindigul locks

22. Correct Option: (c)  The Dindigul locks are known throughout
the world for their superior quality and
Explanation: durability, so much so that even the city is
called Lock City.
 Statement 1 is incorrect: The SURE
project is a commitment by India’s  Government institutions such as prisons,
apparel industry to set a sustainable godowns, hospitals, and even temples use
pathway for the Indian fashion these locks instead of other machine-made
industry. ones.
 Statement 2 is incorrect: The project  There are over 50 varieties of locks made
has been launched by the Union Textiles by the artisans using raw materials such
Ministry. as MS flat plates and brass plates.
TARGET PT 2020 157

Kandangi Sarees Supplementary Notes:


 The Kandangi Sarees are manufactured Open Acreage Licensing Policy (OALP)
in the entire Karaikuditaluk in Sivaganga
district.  Open Acreage Licensing Policy (OALP)
gives an option to a company looking
 They are characterised by large contrast for exploring hydrocarbons to select the
borders and some are known to have exploration blocks on its own, without
borders covering as far as two-thirds of the
waiting for the formal bid round from the
saree which is usually around 5.10 m-5.60
Government.
m in length.
 Under Open Acreage Licensing Policy
 Worn in summer, these cotton sarees are
(OALP), a bidder intending to explore
usually bought by customers in bulk.
hydrocarbons like oil and gas, coal bed
Other products that got a GI tag this year methane, gas hydrate etc., may apply
(2019): to the Government seeking exploration
of any new block (not already covered by
 Odisha – Kandhamal Haldi exploration).
 Himachal Pradesh – Kala Zeera  The Government will examine the
 Chhattisgarh – Jeeraphool Rice Expression of Interest and justification. If
it is suitable for award, Govt. will call for
 Karnataka – Coorg Arabica Coffee competitive bids after obtaining necessary
 Andhra Pradesh – Araku Valley Arabica environmental and other clearances.
 Kerala – Wayanad Robusta Coffee  OALP was introduced vide a Cabinet
decision of the Government as part of the
 Karnataka- Sirsi Supari new fiscal regime in exploration sector
called HELP or Hydrocarbon Exploration
24. Correct Option: (b) and Licensing Policy, so as to enable a
Explanation: faster survey and coverage of the available
geographical area which has potential for
World Skills Competition oil and gas discovery.
 Statement 1 is incorrect: The 45th World  What distinguishes OALP from New
Skills Competition is being held at the Exploration and Licensing Policy (NELP)
KAZAN EXPO International Exhibition is that under OALP, oil and gas acreages
Centre in Kazan, Russia. will be available round the year instead
of cyclic bidding rounds as in NELP.
Supplementary Notes: Potential investors need not have to wait
 The 45th World Skills Competition is for the bidding rounds to claim acreages.
being held at the KAZAN EXPO  Setting up of National Data Repository
International Exhibition Centre is one of the milestones achieved for
in Kazan, Russia. Open Acreage Licensing Policy. To make
 It is a flagship event of the World Skills India a favorable destination globally for
International, organised every two years. Exploration of Crude Oil and Natural Gas,
the Government plans to move to the OALP
 It is the biggest vocational
regime soon.
education and skills excellence event
in the world that truly reflects global  It is well acknowledged that there is
industry. a need for a faster vehicle of awarding
blocks in order to bring more area under
 The aims of the competition
exploration.
include demonstrating the advantages of
learning a vocational skill, and encouraging  As India has vast unexplored sedimentary
‘parity of esteem’ between vocational and basins, a strategy which facilitates a time
academic qualifications. bound full coverage has become a necessity.
Moreover, even the OALP pre-supposes
25. Correct Option: (c) offering of data to the interested companies
for them to submit their bids/ interest.
Explanation:
Hence, availability of data is no longer an
 Both the statements are correct option, but a pre-condition.

**********
158 TARGET PT 2020

TEST
DAY - 15

Time Allowed: 30 mins Maximum Marks: 50

1. Consider the following statements: 1. A transgender person is one whose


1. A bribe-taking MP who votes gender does not match the gender
assigned at birth.
in the Parliament is entitled to
protection under Article 105(2) of the 2. They have been declared as the OBCs.
Constitution. 3. A transgender person has the right to
2. An MLA from a state does not enjoy self-perceived gender identity without
this privilege. undergoing sex reassignment surgery.

Which of the above statements is/are


Which of the above statements is/are
correct?
correct?
(a) 1 only
(a) 1 only
(b) 1 and 2 only
(b) 2 only
(c) 2 and 3 only
(c) Both 1 and 2
(d) 1, 2, and 3
(d) Neither 1 nor 2
4. Which the following offences have been
2. Consider the following statements added by the ‘National Investigation
regarding the Inter-State River Water Agency (Amendment) Bill, 2019 ’?
Disputes (Amendment) Bill, 2019? 1. Explosive substances
1. The Bill requires the central 2. Offences related to counterfeit currency
government to set up a Disputes or bank notes
Resolution Committee for adjudicating
3. Human trafficking
any inter-state water dispute.
4. Cyber-terrorism
2. The Bill also proposes to set up an
Inter-State River Water Disputes Which of the above statements is/are
Tribunal with multiple benches. correct?
(a) 1 and 4 only
Which of the above statements is/are
correct? (b) 2 and 3 only
(a) 1 only (c) 1, 2 and 3 only
(b) 2 only (d) 1, 2, 3, and 4

(c) Both 1 and 2 5. Consider the following statements:


(d) Neither 1 nor 2 1. Hindus are in minority in nine States/
UTs, hence they can establish the
3. Consider the following statements Hindu religious universities in these
regarding Transgender rights in India states.
with reference to the Transgender 2. The Allahabad High Court ruled in
Persons (Protection of Rights) Act, 2005 that Aligarh Muslim University
2019: is a minority institution.
TARGET PT 2020 159

Which of the above statements is/are Which of the above statements is/are
correct? correct?
(a) 1 only (a) 1 only
(b) 2 only (b) 1 and 2 only
(c) Both 1 and 2 (c) 3 and 4 only
(d) Neither 1 nor 2 (d) 1, 2, 3, and 4

6. What were the salient features of the 9. Which of the following provisions is/are
Government of India Act, 1935? not borrowed from the Government of
1. Dyarchy in the Provinces India Act, 1935?
2. Abolition of Council of India 1. Federal Scheme
3. Establishment of Reserve Bank of 2. Office of governor
India 3. Removal of Supreme Court and high
4. Introduction of bicameralism in India court judges
4. Emergency provisions
Select the correct answer using the codes
given below: Which of the above statements is/are
(a) 2 and 3 correct?

(b) 1, 2 and 3 (a) 3 only

(c) 1, 3 and 4 (b) 4 only

(d) 1, 2, 3 and 4 (c) 3 and 4 only


(d) 1, 2, and 3 only
7. Arrange the following events
chronologically in post-independent 10. What of the following is the key
India? takeaway from the judgment of the
1. Bank Nationalization Act Supreme Court in Bijoe Emmanuel v.
State of Kerala?
2. Establishment of Research and
Analysis Wing (a) Reasonable restrictions are equally
important as the Fundamental Rights.
3. Abolition of Privy purse
4. Formation of the Nuclear Suppliers (b) To sing the National Anthem is a
Group Fundamental Duty.
(c) The right of free speech and expression
Select the correct order using the codes also includes the right to remain
given below: silent.
(a) 1-2-4-3 (d) All of the above.
(b) 2-1-3-4
11. Consider the following statements
(c) 3-2-1-4 regarding People’s Union for Civil
(d) 2-3-1-4 Liberties:
1. It is a non-governmental human rights
8. Which of the following pairs is body.
incorrectly matched?
2. It was founded by Jayaprakash
1. Beohar Rammanohar Sinha: Narayan.
Calligrapher of the original Constitution
in Hindi Which of the above statements is/are
2. Vasant Krishan Vaidya: Calligrapher correct?
of the original Constitution (a) 1 only
3. S.N. Mukerjee: Chief draftsman of the (b) 2 only
Constitution
(c) Both 1 and 2
4. Elephant: Symbol of the Constituent
Assembly (d) Neither 1 nor 2
160 TARGET PT 2020

12. Which of the following statements is/ (c) 3 only


are correct?
(d) 1, 2, and 3
1. Only the state governments can decide
whether a community is a minority or 15. Consider the following statements:
not.
1. Article 334 of the original Constitution
2. The State cannot interfere in the liberty provided for the reservation of seats
of the minority institutions to appoint for SCs and STs in the Lok Sabha and
teachers. states assemblies for 70 years.
Which of the above statements is/are 2. Recently, it has been extended for
correct? the next 70 years by a Constitutional
Amendment Act.
(a) 1 only
3. But the further extension of the
(b) 2 only
reservation for the Anglo-Indian
(c) Both 1 and 2 community has not been granted.
(d) Neither 1 nor 2 Which of the above statement is/are
incorrect?
13. Consider the following statements:
1. The backwardness of SCs and STs (a) 1 only
is the sole criteria for providing the (b) 1 and 2 only
reservations in job to SCs and STs.
(c) 2 only
2. It is mandatory for the government
to give reservations in promotions in (d) 1, 2, and 3
public sector jobs.
16. Consider the following statements
3. The states need not collect quantifiable
regarding the Constitution (One
data on the backwardness of SCs and
Hundred and Twenty-fifth Amendment)
STs for giving quota in job promotion.
Bill, 2019?
4. The Creamy layer principle is also
1. The Bill aims to provide for Village
applied in the reservation of the SCs
and STs. and Municipal Councils in addition to
the District and Regional Councils in
Which of the above statements is/are Schedule-V states.
correct? 2. The Bill adds that the Governor may
(a) 3 and 4 only make rules for the disqualification
of such members on the grounds of
(b) 2 and 4 only
defection.
(c) 3 only
3. It provides for the appointment of a
(d) 1 and 3 only State Finance Commission for the
states under Schedule-VI.
14. Which of the following pairs is/are
incorrectly matched? Which of the above statements is/are
1. Constitution (100 Amendment) Act:
th correct?
Goods and Services Tax (a) 2 only
2. Constitution (103rd Amendment) Act: (b) 1 and 3 only
Constitutional status to National
Commission for Backward Classes (c) 2 and 3 only
3. Constitution (102nd Amendment) Act: (d) 1, 2, and 3
10% Reservation for Economically
Weaker Sections 17. Why it is necessary to have
Parliamentary Committees in a
Select the correct option using the democracy?
codes given below:
1. They are an instrument of the
(a) 1 and 3 only Parliament for its own effective
(b) 2 only functioning.
TARGET PT 2020 161

2. They have ample time to discuss over a (c) State public service commission
bill, even more than that on the floor of
(d) Solicitor General
the Houses.
3. Members of the Committees are not 22. Consider the following statements
bound by party whips. regarding Parliamentary System of
Government:
Which of the above statements are correct?
1. The parliamentary government is
(a) 1 and 3 only known as ‘responsible government’ as
the cabinet is accountable to the Lok
(b) 1 and 2 only
Sabha.
(c) 2 and 3 only 2. The parliamentary system is not
(d) 1, 2, and 3 conductive for the formulation and
implementation of long-term policies.
18. Consider the following statements 3. Under this system, the executive
regarding the Estimates Committee: authority is vested in a group of
1. It cannot question the policy laid down individuals and not in a single person.
by the Parliament.
Which of the above statements is/are
2. Its recommendations are binding on
correct?
the ministries.
3. The demands for grants cannot be (a) 1 and 2 only
voted before the Committee makes the (b) 1 and 3 only
report.
(c) 2 and 3 only
Which of the above statements is/are (d) 1,2 and 3
incorrect?
(a) 3 only 23. Which of the following is/are the
features of the federal government?
(b) 2 and 3 only 1. Appointment of Governor
(c) 1 and 3 only 2. All-India Services
(d) 1 and 2 only 3. Bicameralism
19. Gian Kaur v. State of Punjab and P. 4. Supremacy of the Constitution
Rathinam v. Union of India cases are Select the correct option from the codes
related to__ given below:
(a) Scrapping Section 66A (a) 1,2 and 3 only
(b) Hoisting Tricolour as Fundamental (b) 2 and 4 only
Right
(c) 1 and 3 only
(c) Right to Die
(d) 3 and 4 only
(d) Right to Privacy
24. Consider the following regarding
20. Which of the following is not an Federal Government:
element of the basic structure of Indian
Constitution ? 1. According to the Supreme Court,
Federalism is a part of basic structure.
(a) Harmony and balance between
Fundamental Rights and Directive 2. The process of constitutional
amendment is less rigid in case of
(b) Welfare state Federal Government.
(c) Freedom and dignity of the individual Which of the following statement is
(d) Unitary character of the Constitution correct?
(a) 1 only
21. Which of the following is not a
Constitutional Body? (b) 2 only
(a) Special officer for linguistic minorities (c) Both 1 and 2
(b) Advocate general of states (d) Neither 1 nor 2
162 TARGET PT 2020

25. Who is empowered to direct that an 1. It works on the principle of the rule of
act of Parliament does not apply to a ‘locus standi’.
scheduled area in the state or apply 2. Ordinarily both High Court and
with specified modifications and
Supreme Court can entertain a writ
exceptions?
petition by way of PIL questioning
(a) Governor constitutionality or validity of a statute
(b) State Legislature or a statutory rule.
3. In a PIL, unlike traditional dispute
(c) Parliament
resolution mechanism, there is no
(d) President determination on adjudication of
individual rights.
26. During which of the following
circumstances can Parliament frame Which of the following statements is/are
laws on the state subjects? incorrect?
1. When the legislatures of one or more (a) 1 and 2 only
states pass resolutions requesting the
Parliament to enact laws on a matter (b) 2 and 3 only
in the State List (c) 1 and 3 only
2. When a Rajya Sabha resolution
(d) 3 only
supported by one-third of the members
present and voting declares that it is 29. Which of the following can be a possible
necessary in the national interest that justification of Judicial Activism:
Parliament should make laws on a
matter in the State List 1. Judicial Enthusiasm
2. Legislative Vacuum
Select the correct option using the codes
given below: 3. Organic nature of the Constitution
(a) 1 only 4. Failure of legislature to discharge its
duties
(b) 2 only
(c) Both 1 and 2 Select the correct answer from the codes
given below.
(d) Neither 1 nor 2
(a) 3 and 4 only
27. Which of the following was/were the (b) 1 and 2 only
recommendations of the Sarkaria
Commission? (c) 1, 2, 3 and 4
1. A permanent Inter-State Council (d) 2, 3 and 4 only
should be set up under Article 263.
30. Which of the following statement is
2. The zonal councils should be constituted
incorrect regarding the Public Services
afresh and reactivated to promote the
spirit of federalism. in India?

3. The institution of All-India Services (a) All-India services are those services
should be further strengthened and which are common to both Central and
some more such services should be state governments
created. (b) The All-India Services Act of 1951
authorised the Central government to
Select the correct options from the codes
make rules in consultation with the
given below:
state governments for the regulation of
(a) 1 only recruitment and service conditions of
(b) 1 and 3 only the members of all-India services.

(c) 2 and 3 only (c) The all-India services are controlled


jointly by the Central and state
(d) 1, 2 and 3 governments.
28. Consider the following statements (d) The immediate control of the All-
regarding Public Interest Litigation India Services is vested in the Central
(PIL): governments.
TARGET PT 2020 163

31. Consider the following statements: financial institutions and services,


1. The President has the power to specify deposits and securities that were
as to what castes or tribes in each state previously approved by regulators such
and union territory are to be treated as as SEBI, RBI and IFSC.
the SCs and STs.
Which of the following statements is/are
2. Unlike in the case of SCs, STs and correct?
OBCs, the Constitution has defined
the persons who belong to the Anglo- (a) 1 only
Indian community. (b) 2 only
Which of the following statements is/are (c) Both 1 and 2
correct? (d) Neither 1 nor 2
(a) 1 only
35. Department of Pharmaceutical
(b) 2 only Industry works under which of the
(c) Both 1 and 2 following Ministry?
(d) Neither 1 nor 2 (a) Ministry of Science and Technology
(b) Ministry of Chemical and Fertilizer
32. Which of the following statement is
incorrect regarding the Assamese (c) Ministry of Commerce and Industry
Language?
(d) Ministry of Health and Family
(a) Assamese and Bodo are the official Welfare
languages in Assam.
36. Regarding Transgender Persons
(b) BTAD (Bodoland Territorial
(Protection of Rights) Bill 2019,
Administrative Districts), Barak
consider the following statements:
Valley and Hill districts of Assam are
excluded from recognizing Assamese 1. A National Council for Transgender
as a state language. Persons (NCTP) is being constituted
under it.
(c) Assamese is one of the languages
recognized and listed in the 8th 2. Some of the transgender person are
Schedule of the Constitution of India. provided with a Right of Residence
with parents and immediate family
(d) Assamese has been added to the 8th members and be included in his
Schedule by the 71st Constitutional household.
Amendment Act of 1992
Which of the following statements is/are
33. Which state has recently passed the correct?
‘Disha Act’ for the amendment of
criminal laws in the State? (a) 1 only
(a) Chhattisgarh (b) 2 only
(b) Karnataka (c) Both 1 and 2
(c) Andhra Pradesh (d) Neither 1 nor 2
(d) Madhya Pradesh 37. Consider the following aspects of
“India’s Nuclear Doctrine”.
34. Consider the following statements
regarding International Financial 1. Building and maintaining a credible
Services Centre Authority Bill: minimum deterrent.
1. Among the nine members in the 2. Non-use of nuclear weapons against
authority, two will be from Ministry of non-nuclear weapon states.
Commerce and Industry and one each 3. Nuclear retaliatory attacks to be
from SEBI, RBI, IRDAI and PRFDA. authorized by civilian political
2. The authority will regulate financial leadership through the Strategic
products like contracts of insurance, Forces Command.
164 TARGET PT 2020

Which of the statements given above is/are (c) SEBI


correct?
(d) RBI
(a) 1 and 3 only
41. Consider the following statements
(b) 1 and 2 only
about United Nations Security Council
(c) 2 and 3 only (UNSC):
(d) 1, 2 and 3 only 1. It consists of 15 members with
presidency rotating monthly among its
38. Consider the following statements members.
regarding World Trade Organization:
2. It meets only once in a year.
1. It came into existence as a result of
Paraguay rounds of Negotiations held 3. India currently is not a member of
under General Agreement on Trade UNSC.
and Tariffs.
Which of the above statements is/are
2. All decisions in WTO Ministerial are
incorrect?
made by the 5 permanent members of
UNSC. (a) 1 only
3. WTO provides detailed criteria for (b) 1 and 2 only
the classification of member countries
into Developing, Developed and Least (c) 2 only
Developed nations. (d) 2 and 3 only
Which of the statements given above is/are 42. The ‘ASEAN Way’ often seen in the
incorrect?
news is:
(a) 1 and 3 only
(a) A reflection of the culture of ASEAN
(b) 2 only members.
(c) 2 and 3 only (b) A form of interaction among ASEAN
(d) 1, 2 and 3 members that is informal and
cooperative.
39. Consider the following statements
(c) The Economic policy followed by the
regarding Gallantary Awards in
ASEAN members.
India:
1. Only Indian Navy, Army and Air Force (d) A Free Trade Agreement among the
personnel are eligible to receive these ASEAN Members.
awards.
43. Operation Sunrise, is a joint operation
2. Param Vir Chakra is the highest level
conducted between India and which of
of Gallantary award instituted by the
the following countries?
Government of India.
3. These awards announced once a year. (a) Veitnam
(b) Australia
Which of the statements given above is/are
correct? (c) Myanmar
(a) 1 only (d) Thailand
(b) 2 only
44. Marshall Plan is related to which of
(c) 1 and 3 only the following organisations?
(d) 1 and 2 only (a) Organisation for European Economic
Cooperation
40. Sandbox Model recently seen in news
is related to which of the following (b) International Monetary Fund (IMF)
organization? (c) World Trade Organisation (WTO)
(a) IRCTC
(d) Organisation for Economic Co-
(b) IMD operation and Development (OECD)
TARGET PT 2020 165

45. Recently, India-Japan has organized Which of the above statements is/are
its first Space Dialogue. In this context, correct?
consider the following statements:
(a) 1 only
1. It was held in New Delhi.
2. It aims to keep pace with the U.S., (b) 2 only
China, and Russia in the area of (c) Both 1 and 2
Space.
(d) Neither 1 nor 2
Which of the above statements is/are
correct? 48. The ‘goAML’ platform recently seen in
(a) 1 only the news. In this context, consider the
(b) 2 only following statements:

(c) Both 1 and 2 1. It is a platform to deal with financial


crimes.
(d) Neither 1 nor 2
2. UAE is the first country in the Gulf to
46. Which of the following are the founding launch this platform.
members of Shanghai Cooperation
Organisation (SCO)? Which of the above statements is/are
1. China correct?
2. Russia (a) 1 only
3. India
(b) 2 only
4. Kazakhstan
5. Pakistan (c) Both 1 and 2
(d) Neither 1 nor 2
Select the correct answer using the code
given below:
49. Afghanistan Peace Process is important
(a) 2 and 3 only for India because:
(b) 3 and 4 only 1. It will help India to overcome China’s
(c) 1, 2 and 4 only One-Road-One-Belt initiative.
(d) 1, 2, 3, 4 and 5 2. Afghanistan is a gateway for the North-
South Corridor for India.
47. In the context of International
Relations, consider the following 3. Afghanistan’s rich source of oil can
statements regarding the Triangular help India to fulfill its demand.
Cooperation for Global Development:
1. It provides a framework for promoting Which of the above statements is/are
cooperation between the two countries correct?
to meet the developmental aspirations
(a) 1 only
of partner countries.
2. India and Japan recently signed the (b) 2 only
First Amendment to the Statement (c) 1 and 2 only
of Guiding Principles on Triangular
Cooperation for Global Development. (d) 1, 2 and 3

**********
166 TARGET PT 2020

ANSWER HINTS
DAY - 15

1. Correct Option: (a)  It amends the Inter-State River Water


Disputes Act, 1956. The Act provides for the
Explanation: adjudication of disputes relating to waters
of inter-state rivers and river valleys.
Article 105 and the JMM bribery case
 The Bill seeks to streamline the adjudication
 Article 105(1) and 194(1) provide of inter-state river water disputes and
safeguards MPs’ and MLAs’ freedom make the present institutional architecture
of speech in Parliament and state robust.
legislatures respectively.
 Under the Bill, when a state puts in a
 The provisions of Article 105(2) of the request regarding any water dispute,
Constitution confer immunity on a Member the central government will set up a
of Parliament from criminal prosecution Disputes Resolution Committee (DRC),
only in respect of the “freedom of speech” to resolve the dispute amicably. The
and the “right to give vote” by him in DRC will comprise of a Chairperson, and
Parliament or any committee thereof. experts with at least 15 years of experience
 The immunity or protection is available in relevant sectors, to be nominated by the
only in regard to these parliamentary or central government. It will also comprise
official activities. Such immunity is not one member from each state (at Joint
available for any acts done in his private or Secretary level), who are party to the
personal capacity. dispute, to be nominated by the concerned
state government.
 Article 105(2) confers no immunity on a
Member of Parliament involved in a case  The DRC will seek to resolve the dispute
of bribery. through negotiations, within one year
(extendable by six months), and submit
 Some JMM members of Parliament were its report to the central government. If
alleged to have accepted money to vote a dispute cannot be settled by the DRC,
against a 1993 no-confidence motion facing the central government will refer it to the
the then Congress government headed by Inter-State River Water Disputes Tribunal.
PV Narasimha Rao. Such referral must be made within three
 But, the 5-judges bench of the Supreme months from the receipt of the report from
Court in the P.V. Narasimha Rao v. State the DRC.
1998 case or, the JMM bribery case, ruled  The central government will also set
that a lawmaker has protection under the up an Inter-State River Water Disputes
Constitution from facing a criminal case Tribunal, for the adjudication of water
despite accepting money to vote on the floor disputes. This Tribunal can have
of the House. multiple benches. All existing Tribunals
will be dissolved, and the water disputes
 The court said that the bribe-
pending adjudication before such existing
taking MPs who voted on the no-
Tribunals will be transferred to the new
confidence motion were entitled to
Tribunal.
protection under Article 105(2) of the
Constitution.  The Tribunal will consist of a Chairperson,
Vice-Chairperson, three judicial members,
2. Correct Option: (c) and three expert members. They will be
appointed by the central government on the
Explanation: recommendation of a Selection Committee.
Inter-State River Water Disputes  Each Tribunal Bench will consist of
(Amendment) Bill, 2019 a Chairperson or Vice-Chairperson, a
TARGET PT 2020 167

judicial member, and an expert member. Justice; (iv) one representative from
The central government may also appoint ministries including Health, Home Affairs,
two experts serving in the Central Water and Human Resources Development.
Engineering Service as assessors to advise Other members include representatives of
the Bench in its proceedings. the NITI Aayog and the National Human
Rights Commission. State governments
 Under the Bill, the proposed Tribunal must
will also be represented. The Council
give its decision on the dispute within two
will also consist of five members from the
years, which may be extended by another
year. transgender community and five experts
from non-governmental organizations.
 Under the 1956 Act, if the matter is again
referred to the Tribunal by a state for  However, it failed to incorporate yet other
further consideration, the Tribunal must principles in line with the same judgment,
submit its report to the central government such as the right of transgender persons
within a period of one year. This period can to self-perceived gender identity
be extended by the central government. without undergoing sex reassignment
The Bill amends this to specify that such surgery, treating the community as
extension may be up to a maximum of six socially and economically backward
months. classes and extending reservation in
educational institutions and for public
 The Bill provides that the central appointments.
government will appoint or authorize an
agency to maintain such data bank. 4. Correct Option: (d)
3. Correct Option: (b) Explanation:
Explanation: National Investigation Agency
(Amendment) Bill, 2019
The Transgender Persons (Protection of
Rights) Act, 2019  The Bill amends the National Investigation
Agency (NIA) Act, 2008 which provides for
 The Act defines a transgender person a national-level agency to investigate and
as one whose gender does not match the prosecute offences listed in a schedule
gender assigned at birth. It includes (scheduled offences), established in
trans-men and trans-women, persons with response to the Mumbai terror attack of
intersex variations, gender-queers, and 2008.
persons with socio-cultural identities, such
as kinnar and hijra.  The schedule to the Act specifies a list
of offences which are to be investigated
 It prohibits the discrimination against a and prosecuted by the NIA which include
transgender person, including denial of offences under Acts such as the Atomic
service or unfair treatment in relation to (i) Energy Act, 1962, and the Unlawful
education; (ii) employment; (iii) healthcare;
Activities Prevention Act, 1967. The Bill
(iv) access to, or enjoyment of goods,
seeks to allow the NIA to investigate
facilities, opportunities available to the
the following offences, in addition:
public; (v) right to movement; (vi) right to
(i) human trafficking, (ii) offences
reside, rent, or otherwise occupy property;
related to counterfeit currency or
(vii) opportunity to hold public or private
bank notes, (iii) manufacture or sale of
office; and (viii) access to a government
prohibited arms, (iv) cyber-terrorism,
or private establishment in whose care or
and (v) offences under the Explosive
custody a transgender person is.
Substances Act, 1908.
 No government or private entity can
 The Bill also states that now officers of
discriminate against a transgender
the NIA will have the power to investigate
person in employment matters, including
scheduled offences committed outside
recruitment, and promotion. Every
India, subject to international treaties and
establishment is required to designate a
domestic laws of other countries.
person to be a complaint officer to deal with
complaints in relation to the Act.
5. Correct Option: (d)
 The Act provides for the constitution of a
National Council for Transgender persons, Explanation:
that will consist of: (i) Union Minister for
Minority institutions
Social Justice (Chairperson); (ii) Minister of
State for Social Justice (Vice-Chairperson);  According to the 2011 census, Hindus
(iii) Secretary of the Ministry of Social were in minority in eight states/UTs
168 TARGET PT 2020

Lakshadweep (2.5 %), Mizoram (2.75  It abolished dyarchy in the provinces


%), Nagaland (8.75 percent), Meghalaya (introduced by the goI Act, 1919) and
(11.53 %), erstwhile J&K (28.44 %), introduced ‘provincial autonomy’ in its
Arunachal Pradesh (29 %), Manipur place.
(31.39 %) and Punjab (38.40 %). Thus,
today, they are in minority in 6 states  It provided for the adoption of dyarchy at
and 3 UTs. the Centre. However, this provision of the
Act also did not come into operation at all.
 The Supreme Court recently dismissed
the petition that requested the Court  It extended the bicameralism in India
to treat Hindus as a minority in these by introducing it in six out of eleven
states. provinces.
 The Court observed that languages are  It abolished the Council of India,
restricted to a state and in fact, states were established by the Government of India
reorganized on the basis of language, but Act of 1858.
it is not so about religion that is pan India.
 It extended franchise. About 10 percent of
Numbers are taken throughout the country
the total population got the voting right.
to determine who is a minority. Therefore,
religion must be considered pan-India  It provided for the establishment of
 The court also did not agree to plea for a Reserve Bank of India and a Federal
framing any guidelines for declaring a Court.
community as a minority, saying, “Who
does not know who is Hindu, Muslims and 7. Correct Option: (b)
Parsis. How can there be guidelines?”
Explanation:
 Article 30 gives Minorities, both linguistic
and religious, the fundamental right to Indira Gandhi
establish and administer educational  All the above four events took place during
institutions of their choice. However, in the reign of the former Prime Minister
Azzez Basha’s case, the Court denied Indira Gandhi.
minority status to AMU on the ground that
it was not created by a minority individual  In response to the loss in Indo-China war
or body, but rather by a Central Legislature due to the intelligence failure, the external
in 1920. intelligence wing was separated from
the Intelligence Bureau in 1968 and a
 To nullify the effect of Azzez Basha,
the Parliament passed the Aligarh full-fledged second security service was
(Amendment) Act, 1981 restoring the established in the name of the Research
Minority status of AMU. and Analysis Wing under Rameshwar
Nath Kao.
 This Amendment was struck down by
the Allahabad High Court in 2005 and  In 1969, fourteen major private
this led to a joint appeal by AMU and the commercial banks were nationalized
UPA Government in 2006, withdrawn by through the Banking Companies
the NDA government in 2016. (Acquisition and Transfer of Undertakings)
Ordinance, 1969, and later Bill.
 The Supreme Court on 12th February,
2019 referred the issue of AMU’s (Aligarh  The privy purses were abolished by
Muslim University) status as a Minority the 26th Amendment Act, 1971.
Institution to a 7 judge Constitution
 The Nuclear Suppliers Group was
Bench.
founded in response to the Indian
 Thus, presently, AMU is not a minority nuclear test in May 1974.
institution.
8. Correct Option: (b)
6. Correct Option: (a)
Explanation:
Explanation:
Important facts of the Constituent
Features of the Government of India Act, Assembly
1935
 Elephant was adopted as the symbol
 It provided for the establishment of an All- (seal) of the Constituent Assembly.
India Federation consisting of provinces
and princely states as units. However, the  Sir B.N. Rau was appointed as the
federation never came into being as the constitutional advisor (Legal advisor) to
princely states did not join it. the Constituent Assembly.
TARGET PT 2020 169

 H.V.R. Iyengar was the Secretary to the rights to freedom expression under Article
Constituent Assembly. 19 and freedom of religion under Article
25 of the Constitution of India. The High
 S.N. Mukerjee was the chief draftsman
Court dismissed the petition on the
of the constitution in the Constituent
ground that no word or thought in
Assembly.
the national anthem could offend any
 Prem Behari Narain Raizada was the religious beliefs.
calligrapher of the Indian Constitution.
 Subsequently, they appealed the decision
The original constitution was handwritten
to the Supreme Court of India.
by him in italic style.
 The Court found their expulsion in
 The original version was beautified and violation both Articles 19 and 25 of the
decorated by artists from Shantiniketan Constitution, holding that a reasonable
including Nand Lal Bose and Beohar limitation on the right to freedom of
Rammanohar Sinha. expression must be based on a “‘a law’
 Beohar Rammanohar Sinha illuminated, having statutory force and not a mere
beautified and ornamented the original executive or departmental instruction.
Preamble calligraphed by Prem Behari  The Court noted that the right of free
Narain Raizada. speech and expression also includes
 The calligraphy of the Hindi version the right to remain silent and that
of the original constitution was done standing for the national anthem
by Vasant Krishan Vaidya and elegantly showed proper respect.
decorated and illuminated by Nand Lal
Bose. 11. Correct Option: (c)
Explanation:
9. Correct Option: (a)
People’s Union for Civil Liberties
Explanation:
 PUCL is the largest human rights
Sources of the Constitution organization in the country, striving
 The features borrowed from the Act to defend civil liberties and human
are Federal Scheme, Office of rights of all members of society.
governor, Judiciary, Public Service  The Loknayak founded the People’s
Commissions, Emergency provisions Union for Civil Liberties and
and administrative details. Democratic Rights (PUCLDR), in 1976
 The provisions for the removal of during the emergency.
Supreme Court and high court judges  It was re-christened as the People’s
have been borrowed from the US Union for Civil Liberties (PUCL) in
Constitution. the 1980 conference.
 Suspension of Fundamental Rights during  The conference elected V. M. Tarkunde
Emergency has been borrowed from the as its President and Arun Shourie as the
Weimar Constitution of Germany. General Secretary.
 As of 2019, the President of the PUCL
10. Correct Option: (c)
is Ravi Kiran Jain and Dr. Binayak
Explanation: Sen is the national Vice-President.

Bijoe Emmanuel v. State of Kerala case 12. Correct Option: (a)


 Three children namely Bijoe, Binu and Explanation:
Bindu, belonging to a sect called Jehovah’s
Witnesses, studying in a school in Articles 29 and 30
Ettumanoor near Kottayam, were expelled
 In the TMA Foundation vs. State of
from school after they refused to sing the
Karnataka 2002 case, the Supreme Court
national anthem of India.
held that only the State government
 Through their representative, they filed a can determine the status of a religious
writ petition in the High Court of Kerala or linguistic minority.
State, seeking to restrain authorities from
 Even for a Central law, for the purpose
preventing their school attendance.
of determining the minority, the unit
 They alleged that their expulsion amounted will be the State and not the whole of
to an infringement of their fundamental India; thus, religious and linguistic
170 TARGET PT 2020

minorities, who have been put on a  While on the one hand, the Court struck
par in Article 30, have to be considered down the further backwardness criterion, it,
State-wise. on the other hand, introduced the principle
of creamy layer exclusion. It held that
 Recently, the Supreme Court upheld
creamy layer exclusion extends to SC/
a law framed by the West Bengal
STs and, hence the State cannot grant
government in 2008 viz. “West Bengal reservations in promotion to SC/ST
Madrasah Service Commission Act, individuals who belong to the creamy
2008” to constitute a Commission in layer of their community.
connection with the appointment of
teachers in the Islamic seminaries.  The Supreme Court held that there is
no need to collect the quantifiable data
 Holding that the good quality of education on the backwardness of SCs and STs
cannot be compromised in national interest for giving quota in job promotions.
irrespective of whether it is minority or
majority educational institutions, the apex 14. Correct Option: (d)
court ruled that the decision of West Bengal
government to put a regulatory mechanism Explanation:
for appointment of teachers did not amount
Recent Amendment Acts
to interfering with the administration of
minority institutions.  Constitution (100th Amendment)
Act 2015 ratified the land boundary
 Further, the Court observed that if merit
agreement between India and
is not the sole and governing criteria, the
Bangladesh whereas, the Constitution
minority institutions may lag behind the
(101st Amendment) Act introduces the
non-minority institutions rather than keep
GST in India.
in step with them.
 To safeguard the interests of the backward
13. Correct Option: (a) classes, the government amended the
Constitution via the Constitution (102nd
Explanation: Amendment) Act 2019, to give more
powers to the National Commission on
Reservation in promotions Backward Classes by conferring it the
 In Indra Sawhney vs Union of India, the constitutional status.
Supreme Court had held that reservations  Making a note of the Article 16 and 46
under Article 16(4) could only be provided of the Constitution, the government has
at the time of entry into government service amended the Constitution via Constitution
but not in matters of promotion. (102nd Amendment) Act 2019, to provide
 On June 17, 1995, Parliament, acting in its 10% reservation for the Economically
constituent capacity, adopted the seventy- Weaker Sections of the society.
seventh amendment by which clause (4A)
was inserted into Article 16 to enable 15. Correct Option: (b)
reservation to be made in promotion for Explanation:
SCs and STs.
Article 334 and Amendments
 This was challenged in the M. Nagraj
vs. Union of India 2006. In this case, the  Article 334 of the Constitution lays down
Supreme Court ruled that the State is not that the provisions of the Constitution
bound to make reservation for the SCs relating to the reservation of seats for the
and STs in promotions. But, if it seeks Scheduled Castes and the Scheduled Tribes
to do so, it must collect quantifiable data and the representation of the Anglo-Indian
on three facets-the backwardness community by nomination in the Lok Sabha
of the class; the inadequacy of the and in the Legislative Assemblies.
representation of that class in public  Article 334 of the Constitution had
employment; and the administrative originally required the reservation
efficiency of service. of seats to cease in 1960, but this
 In the Jarnail Singh versus Lachhmi was extended to 1970 by the 8th
Narain Gupta 2018, the apex court Amendment, and the 23rd Amendment
upheld its 2006 order that it is not extended this period to 1980.
mandatory for the government to give  The 45th Amendment extended the period
reservations in promotions in public of reservation to 1990. Further, the period
sector jobs. of reservation was extended to 2000, 2010
TARGET PT 2020 171

and 2020 by the 62nd, 79th and 95th rules for the disqualification of such
Amendments respectively. members on the grounds of defection.
 Now, the Constitution (104th
Amendment) Act 2019 extends the
17. Correct Option: (d)
the the reservation of seats for SCs Explanation:
and STs in the Lok Sabha and states
assemblies for the next 10 years until Parliamentary Committees in a
2030. democracy
 But it does not extend the same for the  In a parliamentary democracy, Parliament
Anglo-Indian community. has broadly two functions, which are
lawmaking and oversight of the executive
16. Correct Option: (c) branch of the government. Parliament is
the embodiment of the people’s will.
Explanation:
 Committees are an instrument of
The Constitution (One Hundred and Parliament for its own effective
Twenty-fifth Amendment) Bill, 2019 functioning.
 The Bill amends provisions related to  Given the volume of legislative business,
the Finance Commission and the Sixth discussing all Bills under the consideration
Schedule of the Constitution. The Sixth of Parliament in detail on the floor of the
Schedule relates to the administration House is impossible. Committees have
of tribal areas in the states of Assam, more time and expertise for this.
Meghalaya, Tripura and Mizoram.
 Committee meetings are ‘closed-door’
 The Bill provides the appointment of and members are not bound by party
a State Finance Commission for these whips, which allows them the latitude for
states, to review the financial position a more meaningful exchange of views.
of District, Village, and Municipal
Councils.  While lawmaking gets increasingly complex,
lawmakers cannot infinitely expand their
 The Finance Commission will make knowledge into ever-expanding areas of
recommendations on measures to augment human activities such as new technology,
the Consolidated Fund of a state to provide metadata, etc. It is through committees that
resources to District Councils, Village such expertise is drawn into lawmaking.
Councils, and Municipal Councils in tribal
areas in the four Sixth Schedule state  The first Parliamentary Committee was
constituted in 1571 in Britain. The Public
 The Commission will make Accounts Committee was established in
recommendations regarding (i) distribution 1861. In India, the first Public Accounts
of taxes between states and District Committee was constituted in April
Councils, (ii) grants-in-aid to District, 1950.
Village, and Municipal Councils from
the Consolidated Fund of the state, (iii) 18. Correct Option: (b)
and measures to improve the financial
position of District, Village, and Municipal Explanation:
Councils.
Estimates committee
 The Bill aims to provide for Village and
Municipal Councils in addition to the  The first Estimates Committee in the post-
District and Regional Councils. independence era was constituted in 1950.
 Further, the District Councils may make  All the thirty members of the Committee
laws on various issues, including: (i) are from Lok Sabha only. The Rajya Sabha
number of Village and Municipal Councils has no representation in this committee.
to be formed, and their composition, (ii) These members are elected by the Lok
delimitation of constituencies for election Sabha every year from amongst its own
to the Village and Municipal Councils, members, according to the principles of
(iii) powers and functions of Village and proportional representation by means of a
Municipal Councils. single transferable vote. Thus, all parties
get due representation in it.
 The Sixth Schedule provides that
the Governor may make rules for the  A minister cannot be elected as a member
constitution of District and Regional of the committee. The chairman of the
Councils, including qualifications for being committee is appointed by the Speaker from
elected as members of these councils. The amongst its members and he is invariably
Bill adds that the Governor may make from the ruling party.
172 TARGET PT 2020

 The functions of the committee are:  This judgment was in line with the Aruna
Ramachandra Shanbaug v Union of India
! To report what economies,
and Ors.
improvements in organization,
efficiency, and administrative reform
20. Correct Option: (d)
consistent with the policy.
! To suggest alternative policies in order Explanation:
to bring about efficiency and economy Elements of the basic structure
in administration.
 Parliament under Article 368 can amend
! To examine whether the money is well any part of the Constitution including the
laid out within the limits of the policy Fundamental Rights but without affecting
implied in the estimates. the ‘basic structure’ of the Constitution.
! To suggest the forth in which the  However, the Supreme Court is yet to
estimates are to be presented to define or clarify as to what constitutes
Parliament. the ‘basic structure’ of the Constitution.
 However, the effectiveness of the role of the From the various judgments, the following
committee is limited by the following: have emerged as ‘basic features’ of the
Constitution or elements/components/
! It examines the budget estimates ingredients of the ‘basic structure’ of the
only alter they have been voted constitution:
by the Parliament, and not before
that. ! Supremacy of the Constitution
! It cannot question the policy laid ! Sovereign, democratic and republican
down by the Parliament. nature of the Indian polity

! Its recommendations are ! Secular character of the Constitution


advisory and not binding on the ! Separation of powers between the
ministries. legislature, the executive and the
! It examines every year only certain judiciary
selected ministries and departments. ! Federal character of the
! It lacks the expert assistance of the Constitution
CAG which is available to the Public ! Unity and integrity of the nation
Accounts Committee.
! Welfare state (socio-economic
! Its work is in the nature of a post- justice)
mortem.
! Judicial review
19. Correct Option: (c) ! Freedom and dignity of the
individual
Explanation:
! Parliamentary system
Euthanasia
! Rule of law
 Both of the above cases are related to
passive euthanasia. ! Harmony and balance between
Fundamental Rights and Directive
 A five Judge Bench of Supreme Court in
Gian Kaur v. State of Punjab held that 21. Correct Option: (d)
both euthanasia and assisted suicide are
unlawful in India. Explanation:
 In P. Rathinam case, the Supreme The Constitutional Bodies in India
Court held that the “right to die” is a
 The Constitutional Bodies in India are as
right enshrined under Article 21 of the
follows:
Constitution.
 In the Common Cause (registered society)
! Election commission (Article 324)
v Union of India & Anr, the Supreme Court ! Union public service commission (
of India held that the right to die with Article 315 to 323)
dignity is a fundamental right and a part
! State public service commission
of Art.21. The Bench further held that
(Article 315 to 323)
passive euthanasia and a living will are
legally valid. ! Finance Commission (Article 280)
TARGET PT 2020 173

! National Commission for Scheduled The state governments, on the other hand,
Castes (Article 338) look after the matters of regional and local
importance like public order, agriculture,
! National Commission for Scheduled
health, local government and so on.
Tribes (Article 388-A)
 Written Constitution-The Constitution
! Special Officer for Linguistic
is not only a written document but also
Minorities (Article 350-B)
the lengthiest Constitution of the world.
! Comptroller and Auditor General of Originally, it contained a Preamble, 395
India (Article 148) Articles (divided into 22 Parts) and 8
Schedules.2 At present (2016), it consists
! Attorney General of India (Article 76)
of a Preamble, about 465 Articles (divided
! Advocate General of States (Article into 25 Parts) and 12 Schedules.3 It
165) specifies the structure, organization,
powers and functions of both the Central
22. Correct Option: (c) and state governments and prescribes the
limits within which they must operate.
Explanation: Thus, it avoids the misunderstandings and
Parliamentary System disagreements between the two.

 The Constitution of India provides for a  Division of Powers - The Constitution


parliamentary form of government, both at divided the powers between the Centre and
the Centre and in the states. the states in terms of the Union List, State
List and Concurrent List in the Seventh
 Articles 74 and 75 deal with the Schedule. The Union List consists of 100
parliamentary system at the Centre and subjects (originally 97), the State List 61
Articles 163 and 164 in the states. subjects (originally 66) and the Concurrent
 The parliamentary system of government is List 52 subjects (originally 47). Both the
the one in which the executive is responsible Centre and the states can make laws on
to the legislature for its policies and acts. the subjects of the concurrent list, but in
case of a conflict, the Central law prevails.
 Under this system, the executive The residuary subjects (ie, which are not
authority is vested in a group of mentioned in any of the three lists) are
individuals (council of ministers) and given to the Centre.
not in a single person.
 Supremacy of the Constitution -
 The parliamentary government is also The Constitution is the supreme (or the
known as ‘responsible government’ highest) law of the land. The laws enacted
as the cabinet (the real executive) is by the Centre and the states must confirm
accountable to the Parliament and to its provisions. Otherwise, they can be
stays in office so long as it enjoys the declared invalid by the Supreme Court
latter’s confidence. or the high courts through their power of
 But, the parliamentary system is not judicial review. Thus, the organs of the
conductive for the formulation and government (legislative, executive and
implementation of long-term policies. judicial) at both the levels must operate
within the jurisdiction prescribed by the
23. Correct Option: (d) Constitution.
Rigid Constitution- The division of
Explanation:

powers established by the Constitution as
FEDERAL FEATURES OF THE well as the supremacy of the Constitution
CONSTITUTION can be maintained only if the method of its
amendment is rigid. Hence, the Constitution
The federal features of the Constitution of is rigid to the extent that those provisions
India are explained below: which are concerned with the federal
structure (i.e., Centre–state relations and
 Dual Polity - The Constitution establishes
judicial organization) can be amended only
a dual polity consisting the Union at the
by the joint action of the Central and state
Centre and the states at the periphery.
governments. Such provisions require for
Each is endowed with sovereign powers
their amendment a special majority4 of the
to be exercised in the field assigned to
Parliament and also an approval of half of
them respectively by the Constitution. The
the state legislatures.
Union government deals with the matters
of national importance like defence, foreign  Independent Judiciary -The
affairs, currency, communication and so on. Constitution establishes an independent
174 TARGET PT 2020

judiciary headed by the Supreme Court for ! The Parliament can make laws for the
two purposes: one, to protect the supremacy whole or any part of the territory of
of the Constitution by exercising the power India. The territory of India includes
of judicial review; and two, to settle the the states, the union territories, and
disputes between the Centre and the states any other area for the time being
or between the states. The Constitution included in the territory of India.
contains various measures like security of ! A state legislature can make laws for
tenure to judges, fixed service conditions the whole or any part of the state.
and so on to make the judiciary independent The laws made by a state legislature
of the government. are not applicable outside the state,
 Bicameralism -The Constitution provides except when there is a sufficient nexus
for a bicameral legislature consisting of an between the state and the object.
Upper House (Rajya Sabha) and a Lower ! The Parliament alone can make ‘extra-
House (Lok Sabha). The Rajya Sabha territorial legislation’. Thus, the laws
represents the states of Indian Federation, of the Parliament are also applicable to
while the Lok Sabha represents the people the Indian citizens and their property
of India as a whole. The Rajya Sabha (even in any part of the world.
though a less powerful chamber) is required
 However, the Constitution places certain
to maintain the federal equilibrium by
restrictions on the plenary territorial
protecting the interests of the states against
jurisdiction of the Parliament. In other
the undue interference of the Centre.
words, the laws of Parliament are not
applicable in the following areas:
24. Correct Option: (a)
! The President can make regulations
Explanation: for the peace, progress and good
government of the four Union
Federal Government
Territories—the Andaman and Nicobar
 A federal government, on the other hand, Islands, Lakshadweep, Dadra and
is one in which powers are divided between Nagar Haveli and Daman and Diu. A
the national government and the regional regulation so made has the same force
governments by the Constitution itself and and effect as an act of Parliament. It
both operate in their respective jurisdictions may also repeal or amend any act of
independently. Parliament in relation to these union
territories.
 The US, Switzerland, Australia, Canada,
Russia, Brazil, Argentina and so on have ! The governor is empowered to
the federal model of government. direct that an act of Parliament
does not apply to a scheduled area
 In a federal model, the national government in the state or apply with specified
is known as the Federal government or modifications and exceptions.
the Central government or the Union
government and the regional government ! The Governor of Assam may likewise
is known as the state government or the direct that an act of Parliament does
provincial government. not apply to a tribal area (autonomous
district) in the state or apply with
 In Bommai case (1994), the Supreme specified modifications and exceptions.
Court laid down that the Constitution The President enjoys the same
is federal and characterized federalism power with respect to tribal areas
as its ‘basic feature’. (autonomous districts) in Meghalaya,
Tripura and Mizoram.
 The process of constitutional
amendment is less rigid in case of
26. Correct Option: (d)
Unitary Government.
Explanation:
25. Correct Option: (a)
 Parliamentary Legislation in the
Explanation: State Field: The Constitution empowers
the Parliament to make laws on any
Centre State Relations matter enumerated in the State List
under the following five extraordinary
 The Constitution defines the territorial
circumstances:
limits of the legislative powers vested in
the Centre and the states in the following  When Rajya Sabha Passes a Resolution:
way: If the Rajya Sabha declares that it is
TARGET PT 2020 175

necessary in the national interest that  To Implement International


Parliament should make laws on a matter Agreements: The Parliament can make
in the State List, then the Parliament laws on any matter in the State List for
becomes competent to make laws on implementing the international treaties,
that matter. Such a resolution must be agreements or conventions. This provision
supported by two-thirds of the members enables the Central government to
present and voting. The resolution remains fulfil its international obligations and
in force for one year; it can be renewed commitments. Some examples of laws
any number of times but not exceeding enacted under the above provision are
one year at a time. The laws cease to have United Nations (Privileges and Immunities)
effect on the expiration of six months after Act, 1947; Geneva Convention Act, 1960;
the resolution has ceased to be in force. Anti-Hijacking Act, 1982 and legislations
This provision does not restrict the power relating to environment and TRIPS.
of a state legislature to make laws on the
 During President’s Rule: When the
same matter. But, in case of inconsistency
President’s rule is imposed in a state, the
between a state law and a parliamentary
Parliament becomes empowered to make
law, the latter is to prevail.
laws with respect to any matter in the
 During a National Emergency: The State List in relation to that state. A law
Parliament acquires the power to legislate made so by the Parliament continues to be
with respect to matters in the State List, operative even after the president’s rule.
while a proclamation of national emergency This means that the period for which such
is in operation. The laws become inoperative a law remains in force is not co-terminus
on the expiration of six months after the with the duration of the President’s rule.
emergency has ceased to operate. Here also, But, such a law can be repealed or altered
the power of a state legislature to make or re-enacted by the state legislature.
laws on the same matter is not restricted.
But, in case of repugnancy between a state 27. Correct Option: (d)
law and a parliamentary law, the latter is
to prevail. Explanation:
 When States Make a Request: When Sarkaria Commission
the legislatures of two or more states pass
 In 1983, the Central government appointed
resolutions requesting the Parliament to
a three-member Commission on Centre–
enact laws on a matter in the State List,
state relations under the chairmanship
then the Parliament can make laws for
of R S Sarkaria, a retired judge of the
regulating that matter. A law so enacted
Supreme Court. The Commission made 247
applies only to those states which have
recommendations to improve Centre–state
passed the resolutions. However, any other
relations. The important recommendations
state may adopt it afterwards by passing a
are mentioned below:
resolution to that effect in its legislature.
Such a law can be amended or repealed  The Commission made 247
only by the Parliament and not by the recommendations to improve Centre–state
legislatures of the concerned states. The relations. The important recommendations
effect of passing a resolution under the are mentioned below:
above provision is that the Parliament
! A permanent Inter-State Council
becomes entitled to legislate with respect to
called the Inter-Governmental Council
a matter for which it has no power to make a
should be set up under Article 263.
law. On the other hand, the state legislature
ceases to have the power to make a law ! Article 356 (President’s Rule) should
with respect to that matter. The resolution be used very sparingly, in extreme
operates as abdication or surrender of the cases as a last resort when all the
power of the state legislature with respect available alternatives fail.
to that matter and it is placed entirely in
! The institution of All-India Services
the hands of Parliament which alone can
should be further strengthened and
then legislate with respect to it. Some
some more such services should be
examples of laws passed under the above
created.
provision are Prize Competition Act, 1955;
Wild Life (Protection) Act, 1972; Water ! The residuary powers of taxation
(Prevention and Control of Pollution) Act, should continue to remain with the
1974; Urban Land (Ceiling and Regulation) Parliament, while the other residuary
Act, 1976; and Transplantation of Human powers should be placed in the
Organs Act, 1994. Concurrent List.
176 TARGET PT 2020

! When the president withholds his  The Central government has implemented
assent to the state bills, the reasons 180 (out of 247) recommendations of the
should be communicated to the state Sarkaria Commission. The most important
government. is the establishment of the Inter-State
Council in 1990
! The National Development Council
(NDC) should be renamed and
28. Correct Option: (a)
reconstituted as the National
Economic and Development Council Explanation:
(NEDC).
Public Interest Litigation (PIL)
! The zonal councils should be
constituted afresh and reactivated to  The Supreme Court evolved the following
promote the spirit of federalism. principles in regard to PIL:
! The Centre should have powers to ! The Court in exercise of powers under
deploy its armed forces, even without Articles 32 and 226 of the Constitution
the consent of states. However, it is can entertain a petition filed by any
desirable that the states should be interested person in the welfare of the
consulted. people who are in a disadvantaged
position and thus not in a position
! The Centre should consult the states to knock the doors of the Court. The
before making a law on a subject of Court is constitutionally bound to
the Concurrent List. protect the Fundamental Rights of
! The procedure of consulting the chief such disadvantaged people and direct
minister in the appointment of the the State to fulfil its constitutional
state governor should be prescribed in promises.
the Constitution itself. ! When the issues of public importance,
! The net proceeds of the corporation tax enforcement of the fundamental rights
may be made permissibly shareable of large number of people vis-à-vis the
with the states. constitutional duties and functions of
the State are raised, the court treat a
! The governor cannot dismiss the council letter or a telegram as a PIL. In such
of ministers so long as it commands a cases, the court relaxes the procedural
majority in the assembly. laws and also the law relating to
! The governor’s term of five years in a pleadings.
state should not be disturbed except for ! Whenever injustice is meted out to
some extremely compelling reasons. a large number of people, the court
! No commission of enquiry should be will not hesitate to step in to invoke
set up against a state minister unless Articles 14 and 21 of the Constitution
a demand is made by the Parliament. of India as well as the International
Conventions on Human Rights which
! The surcharge on income tax should provide for a reasonable and fair
not be levied by the Centre except for trial.
a specific purpose and for a strictly
limited period. ! The common rule of locus standi
is relaxed so as to enable the
! The present division of functions court to look into the grievances
between the Finance Commission complained on behalf of the poor,
and the Planning Commission is deprived, illiterate and the disabled
reasonable and should continue. who cannot vindicate the legal wrong
or legal injury caused to them for
! Steps should be taken to uniformly
violation of any constitutional or legal
implement the three language formula
right.
in its true spirit.
! No autonomy for radio and television ! When the Court is prima facie satisfied
about violation of any constitutional
but decentralisation in their
right of a group of people belonging
operations.
to the disadvantaged category, it may
! No change in the role of Rajya Sabha not allow the State or the Government
and Centre’s power to reorganise the from raising the question as to the
states. maintainability of the petition.
! The commissioner for linguistic ! Although procedural laws apply on PIL
minorities should be activated. cases, the question as to whether the
TARGET PT 2020 177

principles of res judicata or principles Constitution and democracy amongst


analogous thereto would apply depend the citizens.
on the nature of the petition and also
! The citizens of the country look up to
facts and circumstances of the case.
the judiciary for the protection of their
! The dispute between two warring rights and freedoms. This leads to
groups purely in the realm of private tremendous pressure on judiciary to
law would not be allowed to be agitated step in aid for the suffering masses.
as a PIL.
! Judicial Enthusiasm, that is, the
! However, in an appropriate case, judges like to participate in the
although the petitioner might have social reforms that take place in
moved a Court in his private interest the changing times. It encourages
and for redressal of the personal the Public Interest Litigation and
grievances, the Court in furtherance liberalises the principle of ‘Locus
of the public interest may treat it Standi’.
necessary to enquire into the state of
affairs of the subject of litigation in ! Legislative Vacuum, that is, there
the interest of justice. may be certain areas, which have
not been legislated upon. It is
! The Court in special situations may therefore, upon court to indulge
appoint Commission or other bodies in judicial legislation and to meet
for the purpose of investigating into the changing social needs.
the allegations and finding out facts.
It may also direct management of a ! The Constitution of India has itself
public institution taken over by such adopted certain provisions, which
Commission. gives judiciary enough scope to
legislate or to play an active role.
! The Court will not ordinarily
transgress into a policy. It shall also  Similarly, Subhash Kashyap observes that
take utmost care not to transgress certain eventualities may be conceived
its jurisdiction while purporting to when the judiciary may have to overstep
protect the rights of the people from its normal jurisdiction and intervene in
being violated. areas otherwise falling within the domain
of the legislature and the executive:
! The Court would ordinarily not step
out of the known areas of judicial ! When the legislature fails to
review. The High Court although discharge its responsibilities.
may pass an order for doing complete
! In case of a ‘hung’ legislature when
justice to the parties, it does not have
the government it provides is weak,
a power akin to Article 142 of the
insecure and busy only in the struggle
Constitution of India.
for survival and, therefore, unable to
! Ordinarily the High Court should take any decision which displeases
not entertain a writ petition by way any caste, community, or other group.
of PIL questioning constitutionality
or validity of a statute or a statutory ! Those in power may be afraid of taking
rule. honest and hard decisions for fear of
losing power and, for that reason, may
 In a PIL, unlike traditional dispute have public issues referred to courts
resolution mechanism, there is no as issues of law in order to mark time
determination on adjudication of individual and delay decisions or to pass on the
rights. odium of strong decision-making to
the courts.
29. Correct Option: (c)
! Where the legislature and the
Explanation: executive fail to protect the basic
rights of citizens, like the right to live
Judicial Activism a decent life, healthy surroundings,
 According to Dr. B.L. Wadehra, the reasons or to provide honest, efficient and just
for judicial activism are as follows: system of laws and administration.
! There is near collapse of the responsible ! Where the court of law is misused by
government, when the Legislature a strong authoritarian parliamentary
and Executive fail to discharge their party government for ulterior motives,
respective functions. This results as was sought to be done during the
in erosion of the confidence in the emergency aberration.
178 TARGET PT 2020

! Sometimes, the courts themselves by a subsequent Presidential notification.


knowingly or unknowingly become Presidents have issued several orders
victims of human, all too human, specifying the SCs and STs in different
weaknesses of craze for populism, states and union territories and these have
publicity, playing to the media and also been amended by the Parliament.
hogging the headlines.  Similarly, the Constitution has neither
specified the BCs nor used a single uniform
30. Correct Option: (d) expression to characterise the BCs. The
Explanation: expression ‘BCs’ means such backward
classes of citizens other than the SCs and
 All-India services are those services the STs as may be specified by the Central
which are common to both Central Government. Thus the expression ‘BCs’ in
and state governments. The members of this context means the ‘Other Backward
these services occupy top positions (or key Classes’ (OBCs) as the SCs and STs are
posts) under both the Centre and the states also backward classes of citizens.
and serve them by turns.
 Unlike in the case of SCs, STs and
 At present, there are three all-India OBCs, the Constitution has defined
services. They are: the persons who belong to the Anglo-
Indian community. Accordingly, ‘an
! Indian Administrative Service (IAS) Anglo-Indian means a person whose father
! Indian Police Service (IPS) or any of whose other male progenitors in
the male line is or was of European descent
! Indian Forest Service (IFS) but who is domiciled within the territory
 The All-India Services Act of 1951 of India and is or was born within such
authorised the Central government to territory of parents habitually resident
make rules in consultation with the therein and not established there for
state governments for the regulation temporary purposes only’.
of recruitment and service conditions
of the members of all-India services. 32. Correct Option: (d)
 The members of these services are recruited Explanation:
and trained by the Central government but
are assigned to different states for work. Assamese Language

 The all-India services are controlled  On December 23, 2019, the Assam Cabinet
jointly by the Central and state nodded to make Assamese language
governments. the state language of Assam. This will
exclude BTAD (Bodoland Territorial
 The ultimate control lies with the Central Administrative Districts), Barak
government while the immediate control Valley and Hill districts of Assam.
is vested in the state governments. Any
 The principal language spoken in Assam
disciplinary action (imposition of penalties)
is Assamese or Asamiya. Assamese is
against these officers can only be taken by
regarded as the lingua-franca of the whole
the Central government.
northeast India. Assamese is one of the
languages recognized and listed in
31. Correct Option: (c) the 8th Schedule of the Constitution
Explanation: of India. Majority of its speakers live in
the Brahmaputra valley of the Assam,
 The Constitution does not specify the castes which accounts for two third of total Assam
or tribes which are to be called the SCs or population (approximately 20 million as
the STs. It leaves to the President the per Census Report 2011).
power to specify as to what castes or
tribes in each state and union territory  Assamese and Bodo are the official
are to be treated as the SCs and STs. languages in Assam, whereas in some of
Thus, the lists of the SCs or STs vary from the districts in the Barak Valley, Bengali
enjoys the official status. It is the eastern-
state to state and union territory to union
most language of the Indo-Aryan European
territory.
family of languages. It gradually evolved
 In case of the states, the President from the Sanskrit language about the
issues the notification after consulting 7th century AD. However, its vocabulary,
the governor of the state concerned. But, phonology and grammar have substantially
any inclusion or exclusion of any caste or been influenced by the original inhabitants
tribe from Presidential notification can of Assam, such as the Bodos and the
be done only by the Parliament and not Kacharis.
TARGET PT 2020 179

 Sindhi was added in 1967 by 21st ! one member each to be nominated


constitutional amendment act; Konkani, from the Reserve Bank of India
Manipuri and Nepali were added in (RBI), the Securities and Exchange
1992 by 71st Constitutional Amendment Board of India (SEBI), the Insurance
Act; and Bodo, Dogri, Maithili and Santali Regulatory and Development
were added in 2003 by 92nd Constitutional Authority of India (IRDAI), and
Amendment Act. the Pension Fund Regulatory and
Development Authority (PFRDA),
33. Correct Option: (c) ! two members from among officials
Explanation: of the Ministry of Finance, and
! two members to be appointed on
Disha Act the recommendation of a Search
 The Andhra Pradesh Legislative Committee. Members will have
Assembly, on December 13, 2019, a term of three years, subject to
passed the Andhra Pradesh Disha Bill, reappointment.
2019 (Andhra Pradesh Criminal Law  Functions of the Authority: The
(Amendment) Act 2019). The bill provides Authority will regulate financial
for awarding death sentence for offences of products (such as securities, deposits
rape and gangrape and expediting trials of or contracts of insurance), financial
such cases to within 21 days. services, and financial institutions
 The Act envisages the completion of which have been previously approved
investigation in seven days and trial in by any appropriate regulator (such as
14 working days, where there is adequate RBI or SEBI), in an IFSC. It will follow
conclusive evidence, and reducing the all processes which are applicable to such
total judgment time to 21 days from the financial products, financial services, and
existing four months. The AP Disha Act financial institutions under their respective
also prescribes life imprisonment for laws. The appropriate regulators are listed
other sexual offences against children and in a Schedule to the Bill, and includes
includes Section 354 F and 354 G in IPC. the RBI, SEBI, IRDAI, and PFRDA. The
central government may amend this
 Disha is the name given to a veterinarian schedule through a notification.
who was raped and murdered in Hyderabad
 Other functions of the Authority include:
on November 27.
 The government of India has launched a
! regulating any other financial
products, financial services, or
National Registry of Sexual offenders
financial institutions in an IFSC,
but the database is not digitized and is
which may be notified by the central
not accessible to the public. In the Andhra
government, and
Pradesh Disha Act, 2019, the Andhra
Pradesh government will establish, operate ! recommending any other financial
and maintain a register in electronic form, products, financial services, or
to be called the ‘Women & Children financial institutions to the central
Offenders Registry’. government, which may be permitted
in an IFSC.
 This registry will be made public and will
be available to law enforcement agencies.  International Financial Services
Centres Authority Fund: The Bill sets
34. Correct Option: (b) up the International Financial Services
Centres Authority Fund. The following
Explanation: items will be credited to the Fund:
 The Bill will apply to all International ! all grants, fees and charges received
Financial Services Centres (IFSCs) set by the Authority, and
up under the Special Economic Zones Act,
2005. ! all sums received by the Authority
from various sources, as decided by
 International Financial Services the central government.
Centres Authority: The Bill sets up the
 The Fund will be used for:
International Financial Services Centres
Authority. It will consist of nine members, ! salaries, allowances and other
appointed by the central government. remuneration of members and
Members of the Authority will include: employees of the Authority, and
! the Chairperson, ! expenses incurred by the Authority.
180 TARGET PT 2020

 Further, the central government may 36. Correct Option: (a)


provide grants to the Authority for the
Explanation:
regulation of IFSCs.
 The proposed Bill has following provisions:
 Performance Review Committee: The
Authority will constitute a Performance ! Non discrimination against a
Review Committee to review its functioning. Transgender Person in educational
The Review Committee will consist of at institutions, employment, healthcare
least two members of the Authority. It services etc.
will review whether: (i) the Authority has ! Recognition of identity of Transgender
adhered to the provisions of the applicable Persons and to confer upon them right
laws while exercising powers or performing to self perceived gender identity.
functions, (ii) the regulations made by ! Provision of right of Residence
the Authority promote transparency and with parents and immediate
best practices of governance, and (iii) family members.
the Authority is managing risks to its
! Provision for formulation of welfare
functioning in a reasonable manner.
schemes and programmes for
 Transaction in foreign currency: As education, social security and health
per the Bill, all transactions of financial of Transgender Persons.
services in IFSCs will be in such foreign ! Provision for National Council for
currency as specified by the Authority, in Transgender Persons to advice,
consultation with the central government. monitor and evaluate measures for
the protection of their rights.
35. Correct Option: (b)  The Bill will make all the stakeholders
Explanation: responsive and accountable for upholding
the principles underlying the Bill. It
 The Department of Pharmaceuticals was will bring greater accountability on the
created on the 1st of July in the year 2008 part of the Central Government and
in the Ministry of Chemicals & Fertilizers State Governments/Union Territories
with the objective to give greater focus Administrations for issues concerning
and thrust on the development of Transgender persons.
pharmaceutical sector in the country and  The Bill will benefit a large number of
to regulate various complex issues related transgender persons, mitigate the stigma,
to pricing and availability of medicines at discrimination and abuse against this
affordable prices, research & development, marginalized section and bring them into
protection of intellectual property rights the mainstream of society. It will lead to
and international commitments related greater inclusiveness and will make the
to pharmaceutical sector which required transgender persons productive members
integration of work with other Ministries. of the society.
 It provides every transgender person
 VISION:
a Right of Residence with parents and
! To make India the Largest Global immediate family members and be included
Provider of Quality Medicines at in his household.
Reasonable Prices.
37. Correct Option: (b)
 MISSION:
Explanation:
! Ensure availability of quality drugs at
reasonable prices as per the Pharma Supplementary notes:
Policy.
Main Features of India’s Nuclear Doctrine
! Promote Pharma Brand India.
 Building and maintaining a credible
! Encourage environmentally minimum deterrent.
sustainable development of
 A “No First Use” posture; nuclear weapons
Pharmaceutical Industry to be used only “in retaliation against a
! To establish NIPERs as nationally nuclear attack on Indian territory or on
and internationally recognized brand Indian forces anywhere”.
in the field of education and research  Nuclear retaliation to a first strike will
of pharmaceutical sciences for the be “massive” and designed to inflict
benefit of human kind. “unacceptable damage”.
TARGET PT 2020 181

 Nuclear retaliatory attacks to be authorized  Ministerial Conference is WTO’s top


only by civilian political leadership level decision making body. It meets once
through the Nuclear Command in two years.
Authority.
 WTO Agreements:
 Non use of nuclear weapons against non
nuclear weapon states. ! For Goods – Marrakesh Agreement
(1995) and Trade Facilitation
 Continuance of strict controls on export Agreement ( 2017)
of nuclear and missile related materials
and technologies, participation in FMCT ! For Services – General Agreement
negotiations, continued moratorium on on Trade in Services
testing. ! For Intellectual Property - The
 India to retain option of retaliating with Agreement on Trade-Related Aspects
nuclear weapons in the event of a major of Intellectual Property Rights
attack against it with biological or chemical (TRIPS)
weapons.
Functions
 Continued commitment to goal of nuclear
weapon free world, through global, verifiable ! Administering WTO Trade
and non discriminatory disarmament. Agreement
! Act as forum for trade negotiation
38. Correct Option: (d)
! Handling trade disputes
Explanation:
! Monitoring trade policies
 Statement 1 is incorrect: WTO came into
existence in Uruguay Rounds of 1986- ! Cooperation with other international
1994 negotiations being held under the organization
General Agreement on Trade and Tariffs  In News - US President Donald Trump
(GATT). has questioned ‘Developing Tag’ for India
 Statement 2 is incorrect: All decisions in and China.
WTO Ministerial are made by consensus.
39. Correct Option: (b)
 Statement 3 is incorrect: There are
no WTO definitions of “developed” and Explanation:
“developing” countries. Members announce
for themselves whether they are “developed”  Statement 1 is incorrect: Along with
or “developing” countries. Air Force, Navy and Army, even para-
military, state police and civilians
Supplementary Notes: are also eligible to receive the Gallantary
awards for their act of bravery.
World Trade Organization (WTO)
 Statement 3 is incorrect: These awards
 It is a successor to the General Agreement announced twice in a year.
on Tariffs and Trade (GATT) established
in the wake of the Second World War. Supplementary Notes:
 Several rounds of negotiations were held
Gallantary Awards
under GATT.
 Post-independence, first three gallantry
 WTO came into existence in Uruguay
awards namely the Param Vir Chakra,
Rounds of 1986-1994 through Marrakesh
Agreement in 1995. the Maha Vir Chakra and the Vir
Chakra were instituted by the Government
 Members – 164 and together, they of India on 26th January, 1950 which
constitute 98% of the global trade were deemed to have effect from the 15th
 In 2000, 4th WTO Ministerial led to a new August, 1947.
round of negotiation, Doha Development  The Ashoka Chakra Class-I, the Ashoka
Agenda, which is still in progress. Chakra Class-II and the Ashoka Chakra
 Mandate – to open trade for the benefit Class-III were instituted by the Government
for all of India on 4th January, 1952, which
were deemed to have effect from the 15th
 All decisions are made by consensus. August, 1947. These awards were renamed
 WTO Headquarters is in Geneva, as the Ashoka Chakra, the Kirti Chakra
Switzerland. It has no other branch and the Shaurya Chakra respectively in
office. January, 1967.
182 TARGET PT 2020

 These gallantry awards are announced a controlled/test regulatory environment


twice in a year - first on the occasion of the for which regulators may permit certain
Republic Day and then on the occasion regulatory relaxations for the limited
of the Independence Day. purpose of the testing.
 Order of precedence of these awards is:  RBI said the objective of the sandbox was
! Param Vir Chakra to foster responsible innovation in financial
services, promote efficiency and bring
! Ashoka Chakra benefit to consumers.
! Mahavir Chakra  The proposed FinTech solution should
! Kirti Chakra highlight an existing gap in the financial
ecosystem and the proposal should
! Vir Chakra demonstrate how it would address the
! Shaurya Chakra problem, and bring benefits to consumers
or the industry and/or perform the same
 Selection Process
work more efficiently.
! Ministry of Defence invites
 RBI will launch the sandbox for entities
recommendations twice in a year
that meet the criteria of minimum net
from the Armed Forces and Union
worth of 25 lakh as per their latest
Ministry of Home Affairs for
gallantry awards. audited balance sheet.

! Recommendations in respect of  The entity should either be a company


civilian citizens (other than Defence incorporated and registered in the country
personnel) are received from the or banks licensed to operate in India.
Union Ministry of Home Affairs  While money transfer services, digital
(MHA). know-your customer, financial inclusion
! Recommendations received from and cyber security products are included,
the Armed Forces and MHA are crypto currency, credit registry and credit
considered by the Central Honours information have been left out.
& Awards Committee (CH&AC)
comprising of Raksha Mantri, three 41. Correct Option: (c)
Service Chiefs & Defence Secretary.
Home Secretary is also member for the Explanation:
cases recommended by the Ministry of  Statement 1 is correct: The Security
Home Affairs. Council consists of fifteen members. The
 Thereafter, recommendations of the body’s presidency rotates monthly among
CH&AC are submitted for approval of its members.
the Prime Minister and the President.  Statement 2 is incorrect: The Security
After approval of the President, awards are Council meets year-round, unlike the
announced on the occasion of the Republic General Assembly.
Day and Independence Day.
Supplementary Notes:
40. Correct Option: (d)
 It is one of the six principal organs of the
Explanation: United Nations (UN).
 Option (d) is correct: The Reserve Bank  Like the UN as a whole, it was created
of India (RBI) issued the final framework following World War II to address the
for regulatory sandbox in order to enable failings of a previous international
innovations in the financial technology organization, the League of Nations, in
space. maintaining world peace.
Supplementary Notes:  The council held its first session in 1946.

Regulatory Sand Box  It is the only body of the UN with the


authority to issue binding resolutions to
 The Reserve Bank of India (RBI) issued member states.
the final framework for regulatory
sandbox in order to enable innovations  The Security Council consists of fifteen
in the financial technology space. members:
 A regulatory sandbox usually refers to ! The great powers that were the victors
live testing of new products or services in of World War II – the Soviet Union
TARGET PT 2020 183

(now represented by Russia), the  It also aimed to promote regional peace and
United Kingdom, France, Republic stability based on the rule of law and the
of China (now represented by the principles of the United Nations Charter.
People’s Republic of China), and the
 It is headquartered in Jakarta, Indonesia.
United States – serve as the body’s
five permanent members. ASEAN Way
! These can veto any substantive  Over the years the ASEAN member
resolution, including those on the countries have developed a form of
admission of new member states or interaction that is informal, non-
nominees for the office of Secretary- confrontationist and cooperative
General. called the ASEAN Way.
! In addition, the council has 10 non-  It is in sync with the modest ideas and
permanent members, elected on a goals of the ASEAN.
regional basis to serve a term of two
years.  Unlike the European Union, there is
little desire in ASEAN for supranational
! The body’s presidency rotates monthly structures and institutions
among its members.
 Resolutions of the Security Council are 43. Correct Option: (c)
typically enforced by UN peacekeepers,
military forces voluntarily provided by
Explanation:
member states and funded independently  Option (c) is correct: Operation Sunrise
of the main UN budget. is a joint operation conducted by India &
 Unlike the General Assembly, the Security Myanmar.
Council meets year-round. Each Security Supplementary Notes:
Council member must have a representative
available at UN Headquarters at all times Operation Sunrise
in case an emergency meeting becomes
necessary.  The armies of India and Myanmar carried
out a three-week-long coordinated
 In news: On a request from China, operation in their respective border areas,
consultations on Kashmir were scheduled targeting several militant groups operating
by United Nations Security Council in Manipur, Nagaland, and Assam.
(UNSC) recently to discuss Kashmir
(India’s abrogation of Article 370).  The first phase of “Operation Sunrise” was
conducted three months ago along the Indo-
42. Correct Option: (b) Myanmar border, during which a number of
camps of north-east-based militant groups
Explanation: were busted.
 Option (b) is correct: ‘ASEAN Way’ is a
form of interaction among ASEAN members 44. Correct Option: (a)
that is informal and cooperative. Explanation:
Supplementary Notes:  Option (a) is correct: Marshall Plan
influenced the establishment of the
Association of Southeast Asian Nations Organisation for European Economic
(ASEAN) Cooperation in 1948.
 ASEAN was established in 1967 by five
countries of this region — Indonesia, Supplementary Notes:
Malaysia, the Philippines, Singapore,
The Marshall Plan
and Thailand — by signing the Bangkok
Declaration. Over the years, Brunei,  Aided by the Cold War, America extended
Vietnam, Laos, Myanmar, and Cambodia massive financial help to Europe for
joined ASEAN taking its strength to ten. reviving Europe’s economy under a plan
called the Marshall Plan.
 The objectives of ASEAN were primarily
to accelerate economic growth and  Under the Marshall Plan, the Organisation
through that ‘social progress and cultural for European Economic Cooperation was
development’ in the south east Asian established in 1948 channeling aid to the
region. western European states.
184 TARGET PT 2020

 Its members were- France, West Germany, Separatism, and Extremism.


Italy, Belgium, the Netherlands, and
 India and Pakistan joined SCO in 2017 at a
Luxembourg.
summit in Astana, Kazakhstan
45. Correct Option: (b)
47. Correct Option: (a)
Explanation:
Explanation:
 Statement 1 is incorrect: The First
Meeting of the Japan-India Space Dialogue  Statement 2 is incorrect: India and the
was held in Delhi. United States recently signed the First
Amendment to the Statement of Guiding
Supplementary Notes: Principles (SGP) on Triangular Cooperation
for Global Development.
Japan-India Space Dialogue
 The Japan-India Space Dialogue, Supplementary Notes:
established in October 2018 aims to keep
Triangular Cooperation for Global
pace with the U.S., China, and Russia in
Development
the area of space.
 India and the United States recently
 The First Meeting of the Japan-India
signed the First Amendment to the
Space Dialogue was held in Delhi on March
8, 2019. Statement of Guiding Principles (SGP)
on Triangular Cooperation for Global
 The dialogue brought together outer space- Development, mainly focused on Asia
related ministries and agencies in two and Africa.
countries and provided an opportunity to
exchange information on the Space Policy  The SGP agreement was signed in
of each country and to hold discussions November 2014. It underscores the
on space security, bilateral cooperation contribution of India-U.S. partnership to
between JAXA-ISRO, their space industries, global stability and prosperity.
global navigation satellite system, space
 It provides a framework for promoting
situational awareness (SSA), space-related
cooperation between the two countries
norms and other areas of mutual interest.
to meet the developmental aspirations
 Both sides shared the view that they of partner countries, particularly in
would deepen the mutual understanding Asia and Africa.
of respective space policies and explore
concrete cooperation between the two  The First Amendment to the SGP
countries. Agreement extends the validity of the SGP
agreement upto 2021.
 Both sides also decided to hold this dialogue
on a regular basis to enhance bilateral  It expands the scope of capacity building
cooperation activities undertaken jointly by India and
the US under this framework and also
46. Correct Option: (c) provides a consultative mechanism for
joint biannual monitoring and review of
Explanation: activities undertaken under the SGP.
 Statements 3 and 5 are incorrect:
 Under the agreement, India and the US
India and Pakistan are not the founding
will continue to offer capacity-building
member of SCO.
assistance to partner countries in multiple
Supplementary Notes: sectors, focusing primarily on agriculture,
regional connectivity, trade and
Shanghai Cooperation Organisation investments, nutrition, health, clean and
(SCO) renewable energy, women empowerment,
 It was founded in June 2001 by China, disaster preparedness, water, sanitation,
Russia, and the Central Asian states of education, and institution building.
Kazakhstan, Kyrgyzstan, Tajikistan,  The agreement helps in fulfilling the
and Uzbekistan. joint commitment of both nations to work
 It aimed to cooperate against non-traditional together and leverage their combined
security challenges, in particular fighting capacities to provide demand-driven
the so-called “three evils” of Terrorism, development partnership.
TARGET PT 2020 185

48. Correct Option: (c) responsible democratic country and a true


friend and neighbour to the Afghanistan.
Explanation:
 Indian is engaged with Afghanistan by
Both statements are correct following ways:
Supplementary notes:  Developing social infrastructure as
hospitals, schools;
goAML
 Public infrastructure such as Salma dam,
 This platform was developed by United
Nations Office on Drugs and Crimes and parliament building ;
(UNODC) as a strategic response  Humanitarian assistance such as medical
against financial crime, as advised by missions;
the Conference of Parties to the United
Nations Convention against Transnational  Training of military officer and soldiers;
Organized Crime.  Military warfare such as military
 It is a one-stop, fully integrated solution helicopters and repairing the old soviet era
that meets the IT needs of most Financial helicopters.
Intelligence Unit (FIUs), with the primary
objective of enhancing the capacity of  India is likely concerned that any deal
member states to detect financial crime and that could introduce the Taliban back into
to meet international standards relating to the Afghan government could dilute its
money laundering and terrorist financing. political influence in Afghanistan.
 The UAE’s Financial Intelligence Unit  India’s hedge toward Iran, by investing in
(FIU) launched the new anti-money the construction of the Chabahar port (the
laundering platform ‘goAML’ (Anti-Money only Iranian port with direct access to the
Laundering) to curb the organized crimes. Indian Ocean), could pave the way for a
 The UAE has become the first country in continuing economic role.
the Gulf to launch a new reporting platform
 But India would probably see a peace
developed by the United Nations Office
agreement, particularly one championed
on Drugs and Crime (UNODC) to curb
organized crimes. All financial institutions by Pakistan, as a short-term setback to its
in the country have to report any suspicious interests in Afghanistan.
transactions through this platform.
Afghan as an elusive benefit for India:
 The platform will help the FIU prevent
money laundering, financing of terrorism  Afghanistan is a gateway for the north-
and other illicit financial activities. south corridor for India.
 Afghanistan has a rich source of oil which
49. Correct Option: (d) can help India to full fill their demand.
Explanation:  India developed Chahbar port to increase
All statements are correct import and export with Afghan and counter
Pakistan in West Sea.
Supplementary notes:
 Elusive peace in afghan can help India
Afghan Peace Process and India project of TAPI.

 With limited options of intervention in  It will help India to overcome China’s One-
Afghanistan, India is playing a role of a Road-One-Belt initiatives.

**********

You might also like